Download as pdf or txt
Download as pdf or txt
You are on page 1of 301

CONTENT

P–I 1 – 59

 ERROR & MEASUREMENT

 KINEMATICS

 LAWS OF MOTION & FRICTION

P – II 60 – 107

 CIRCULAR MOTION

 WORK, POWER & ENERGY

 CENTRE OF MASS

 COLLISION & MOMENTUM

P – III 108 – 156

 ROTATION
&
 SIMPLE HARMONIC MOTION

P – IV 157 – 194

 GRAVITATION

 FLUID MECHANICS
&
 PROPERTIES OF MATTER

P–V 195 – 237

 HEAT & THERMODYNAMICS

P – VI 238 – 300

 MODERN PHYSICS
&
 SEMICONDUCTORS

CENTERS : MUMBAI / DELHI /AKOLA / LUCKNOW / NASHIK / PUNE / NAGPUR / BOKARO / DUBAI
PHYSICAL WORLD
CONCEPT SUMMARY

1. Physical deals with the study of the basic laws of nature and their manifestation in different
phenomena. The basic laws of physics are universal and apply in widely different contexts and
conditions.

2. The scope of physics is wide, covering a tremendous range of magnitude of physical quantities.

3. Physics and technology are related to each other. Sometimes technology gives rise to new physics :
at other times physics generates new technology. Both have direct impact on society.

4. There are four fundamental forces in nature that govern the diverse phenomena of the macroscopic
and the microscopic world. These are the ‘gravitational force’ the ‘electromagnetic force’, the strong
nuclear force’, and the ‘weak nuclear force’. Unification of different forces/ domains in nature is a
basic quest in physics.

5. The physical quantities that remain unchanged in a process are called conserved quantities. Some of
the general conservation laws in nature include the laws of conservation of mass, energy, linear
momentum, angular momentum, charge, parity, etc. Some conservation laws are true for one
fundamental force but not for the other.

6. Conservation laws have a deep connection with symmetries of nature. Symmetries of space and time,
and other types of symmetries play a central role in modern theories of fundamental forces in nature.

UNITS AND MEASUREMENTS


CONCEPT SUMMARY
1. Physics is a quantitative science, based on measurement of physical quantities. Certain physical
quantities have been chosen as fundamental or base quantities (such as length, mass, time, electric
current, thermodynamic temperature, amount of substance, and luminous intensity).

2. Each base quantity is defined in terms of a certain basic, arbitrarily chosen but property standardized
reference standard called unit (such as metre, kilogram, second, ampere, Kelvin, mole and candela).
The units for the fundamental or base quantities are called fundamental or base units.

3. Other physical quantities, derived from the base quantities, derived from the base quantities, can be
expressed as a combination of the base units and are called derived units. A complete set of units,
both fundamental and derived, is called a system of units.

4. The International System of Units (SI) based based on seven base units is at present internationally
accepted unit system and is widely used throughout the world.

5. The SI units are used in all physical measurements, for both the base quantities and the derived
quantities obtained from them. Certain derived units are expressed by means of SI units with special
names (such as joule, newton, watt, etc).

CENTERS : MUMBAI /DELHI /AKOLA /LUCKNOW /NASHIK /PUNE /NAGPUR /BOKARO /DUBAI # 1
6. The SI units have well defined and internationally accepted unit symbols (such as m for metre, kg for
kilogram, s for second, A for ampere, N for newton etc.).

7. Physical measurements are usually expressed for small and large quantities in scientific notation,
with powers of 10. Scientific notation and the prefixes are used to simplify measurement notation
and numerical computation, giving indication to the precision of the numbers.

8. Certain general rules and guidelines must be followed for using notations for physical quantities and
standard symbols for SI units, some other units and SI prefixes for expressing properly the physical
quantities and measurements.

9. In computing any physical quantity, the units for derived quantities involved in the relationship(s)
are treated as though they were algebraic quantities till the desired units are obtained.

10. Direct and indirect methods can be used for the measurement of physical quantities. In measured
quantities, while expressing the result, the accuracy and precision of measuring instruments along
with errors in measurements should be taken into account.

11. In measured and computed quantities proper significant figures only should be retained. Rules for
determining the number of significant figures, carrying out arithmetic operations with them, and
‘rounding off’ the uncertain digits must be followed.

12. The dimensions of base quantities and combination of these dimensions describe the nature of
physical quantities. Dimensional analysis can be used to check the dimensional consistency of
equations, deducing relations among the physical quantities. etc. A dimensionally consistent equation
need not be actually an exact (correct) equation. but a dimensionally wrong or inconsistent equation
must be wrong.
MOTION IN A PLANE
CONCEPT SUMMARY

1. Scalar quantities are quantities with magnitudes only. Examples are distance, speed, mass and
temperature.

2. Vector quantities are quantities with magnitude and direction both. Examples are displacement,
velocity and acceleration. They obey special rules of vector algebra.

3. A vector A multiplied by a real number  is also a vector, whose magnitude is  times the
magnitude of the vector A and whose direction is the same or opposite depending upon whether  is
positive or negative.

4. Two vectors A and B may be added graphically using head-to-tail method or parallelogram method.

5. Vector addition is commutative:


A+B =B+A
It also obeys the associative law:
(A + B) + C = A + (B + C)

6. A null or zero vector is a vector with zero magnitude. Since the magnitude is zero, we don’t have to
specify its direction. It has the properties:
A + 0 = A 0  0

CENTERS : MUMBAI /DELHI /AKOLA /LUCKNOW /NASHIK /PUNE /NAGPUR /BOKARO /DUBAI # 2
7. The subtraction of vector B from A is defined as the sum of A and – B :
A – B = A+ (– B)

8. A vector A can be resolved into component along two given vectors a and b lying in the same plane:
A  a   b
where  and  are real numbers.

9. A unit vector associated with a vector A has magnitude one and is along the vector A:
A
n̂ 
A
The unit vectors ˆi, ˆj, kˆ are vectors of unit magnitude and point in the direction of the x, y-. and
z-axes, respectively in a right-handed coordinate system.
10. A vector A can be expressed as
A  A x ˆi  A y ˆj
where A x ,A y are its components along x-, and y -axes. If vector A makes an angle 
Ay
With the x-axis, then Ax  A cos , A y  Asin  and A  A  A 2x  A 2y , tan  
Ax

11. Vectors can be conveniently added using analytical method. If sum of two vectors A and B, that lie
in x-y plane, is R. then:
R  R x ˆi  R y ˆj , where, R x  Ax  Bx ' and R y  A y  By

12. The position vector of an object in x-y plane is given by r  xiˆ  yjˆ and the displacement from
position r to position r ’ is given by
r  r ' r
  x ' x  ˆi   y ' y  ˆj
 xiˆ  yjˆ

r
13. If an object undergoes a displacement r in time t , its average velocity is given by v  . The
t
velocity of an object at time t is the limiting value of the average velocity as t tends to zero:
 r dr
v  lim  . It can be written in unit vector notation as:
t  0 t dt
dx dy dz
v  x ˆi  y ˆi  z kˆ where  x  , y  , z 
dt dt dt
When position of an object is plotted on a coordinate system. v is always tangent to the curve
representing the path of the object.

14. If the velocity of an object changes from v to v' in time t , then Its average acceleration is given
v  v ' v
by : a  
t t
The acceleration a at any time t is the limiting value of a as t  0 :
v dv
a  lim 
t  0 t dt
In component form, we have: a  a x ˆi  a y ˆj  a z kˆ

CENTERS : MUMBAI /DELHI /AKOLA /LUCKNOW /NASHIK /PUNE /NAGPUR /BOKARO /DUBAI # 3
dv x dv dv
where, a x  .a y  y .a z  z
dt dt dt

15. If an object is moving in a plane with constant acceleration   a  a 2x  a 2y and its position vector
at time t = 0 is r0, then at any other time t. It will be at a point given by :
1
r  r0  v 0 t  at 2
2
and its velocity Is given by:
v  v0  a t
where v0 is the velocity at time t = 0
In component form:
1
x  x 0  v0x t  a y t 2
2
1
y  y0   0y t  a y t 2
2
x  0x  a x t
 y   0y  a y t
Motion in a plane can be treated as superposition of two separate simultaneous one - dimensional
motions along two perpendicular directions

16. An object that is in flight after being projected is called a projectile. If an object is projected with
Initial velocity v0 making an angle 0 with x-axis and If we assume its initial position to coincide
with the origin of the coordinate system, then the position and velocity of the projectile at time t are
given by:
x    0 cos 0  t
y   v 0 sin 0  t  1/ 2  g t 2
x  0x  0 cos 0
y  0 sin 0  g t
The path of a projectile is parabolic and is given by:
gx 2
y   tan 0  x  2
2   0 cos 0 
The maximum height that a projectile attains is :
2

hm
  sin 0 
 0
2g
The time taken to reach this height is:
 sin 0
tm  0
g
The horizontal distance travelled by a projectile from its initial position to the position it passes y = 0
during its fall Is called the range. R of the projectile. It is:
 2 sin 20
R 0
g
17. When an object follows a circular path at constant speed, the motion of the object is called uniform
circular motion. The magnitude of its acceleration is a c  v2 / R . The direction of a c , is always
towards the centre of the circle.

CENTERS : MUMBAI /DELHI /AKOLA /LUCKNOW /NASHIK /PUNE /NAGPUR /BOKARO /DUBAI # 4
The angular speed  , is the rate of change of angular distance. It Is related to velocity v by   R .
The acceleration is a c  2 R .
If T is the time period of revolution of the object in circular motion and v is its frequency, we have
  2 v,   2vR, a c  42 v2 R .

MOTION IN A STRAIGHT LINE


CONCEPT SUMMARY

1. An object is said to be in motion if its position changes with time. The position of the object can be
specified with reference to a conveniently chosen origin. For motion in a straight line, position to the
right of the origin is taken as positive and to the left as negative.

2. Path length is defined as the total length of the path traversed by an object.

3. Displacement is the change in position: x  x 2  x1 . Path length is greater or equal to the magnitude
of the displacement between the same points.

4. An object is said to be in uniform motion in a straight line if its displacement is equal in equal
intervals of time. Otherwise, the motion is said to be non-uniform.

5. Average velocity is the displacement divided by the time interval in which the displacement occurs:
x

t
On an x-t graph, the average velocity over a time interval is the slope of the line connecting the initial
and final positions corresponding to that interval.

6. Average Speed is the ratio of total path length traversed and the corresponding time interval. The
average speed of an object is greater or equal to the magnitude of the average velocity over a given
time interval.

7. Instantaneous velocity or simply velocity is defined as the limit of the average velocity as the time
interval t becomes infinitesimally small:
x dx
  lim   lim 
t  0 t  0  t dt
The velocity at a particular instant is equal to the slope of the tangent drawn on position-time graph
at that instant.

8. Average acceleration is the change in velocity divided by the time interval during which the change
occurs:

a
t
9. Instantaneous acceleration is defined as the limit of the average acceleration as the time interval t
goes to zero:
 d
a  lim a  lim 
t  0 t  0 t dt
The acceleration of an object at a particular time is the slope of the velocity-time graph at that instant
of time. For uniform motion, acceleration is zero and the x-t graph is a straight line inclined to the
time axis and the v-t graph is a straight line parallel to the time axis. For motion with uniform
acceleration, x-t graph is a parabola while the v-t graph is a straight line inclined to the time axis.

CENTERS : MUMBAI /DELHI /AKOLA /LUCKNOW /NASHIK /PUNE /NAGPUR /BOKARO /DUBAI # 5
10. The area under the velocity-time curve between times t1 and t2 is equal to the displacement of the
object during that interval of time.

11. For objects in uniformly accelerated rectilinear motion, the five quantities, displacement x, time
taken t, initial velocity v0 , final velocity v and acceleration a are related by a set of simple
equations called kinematic equations of motion:
v  v0  at
1
x  v 0 t  at 2
2
2 2
v  v0  2ax
If the position of the object at time t = 0 is 0. If the particle starts at x = x0, x in above equations is
replaced by (x – x0).

LAWS OF MOTION
CONCEPT SUMMARY

1. Aristotle’s view that a force is necessary to keep a body in uniform motion is wrong. A force is
necessary in practice to counter the opposing force of friction.

2. Galileo extrapolated simple observations on motion of bodies on inclined planes, and arrived at the
law of inertia. Newton’s first law of motion is the same law rephrased thus: ‘Everybody continues to
be in its state of rest or of uniform motion in a straight line. unless compelled by some external force
to act otherwise’. In simple terms, the First Law is “If external force on a body is zero, its
acceleration is zero”.

3. Momentum (p) of a body is the product of its mass (m) and velocity (v):
p=mv

4. Newton’s second law of motion:


The rate of change of momentum of a body is proportional to the applied force and takes place in the
direction in which the force acts. Thus
dp
Fk k m a
dt
Where F is the external force on the body and a its acceleration. We set the constant of
proportionality k = 1 in SI units. Then
dp
F  ma
dt
(a) The second law is consistent with the First Law (F = 0 implies a = 0)
(b) It is a vector equation
(c) It is applicable to a particle, and also to a body or a system of particles, provided F is the total
external force on the system and a is the acceleration of the system as a whole.
(d) F at a point at a certain instant determines a at the same point at that instant. That is the Second
Law is a local law: a at an Instant does not depend on the history of motion.

5. Impulse is the product of force and time which equals change in momentum. The notion of impulse
is useful when a large force acts for a short time to produce a measurable change in momentum.
Since the time of action of the force is very short, one can assume that there is no appreciable change
in the position of the body during the action of the impulsive force.

CENTERS : MUMBAI /DELHI /AKOLA /LUCKNOW /NASHIK /PUNE /NAGPUR /BOKARO /DUBAI # 6
6. Newton’s third law of motion:
To every action, there is always an equal and opposite reaction in simple terms, the law can be stated
thus :
Forces in nature always occur between pairs of bodies. Force on a body A by body B is equal and
opposite to the force on the body B by A.
Action and reaction forces are simultaneous forces. There is no cause-effect relation between action
and reaction. Any of the two mutual forces can be called action and the other reaction. Action and
reaction act on different bodies and so they cannot be cancelled out. The internal action and reaction
forces between different parts of a body do, however, sum to zero.

7. Law of Conservation of Momentum


The total momentum of an isolated system of particles is conserved. The law follows from the
second and third law of motion.

8. Friction
Frictional force opposes (impending or actual) relative motion between two surfaces in contact. It is
the component of the contact force along the common tangent to the surface in contact. Static
friction f s opposes actual relative motion: kinetic friction f k opposes actual relative motion. They
are independent of the area of contact and satisfy the following approximate laws:
f s   f s max   s R f k  k R
s (co-efficient of static friction) and  k (co-efficient of kinetic friction) are constants characteristic
of the pair of surfaces in contact. It is found experimentally that  k is less than s .

KINEMATICS
FORMULA SUMMARY

MOTION

Uniformly Non-uniformly
Accelerated Accelerated

Straight- Line Motion in a Plane Straight line Motion in a plane


e.g. projectile Motion e.g. circular motion

(1) For Uniformly Accelerated Motion:


      
v = u  at  s  ut  at 2 / 2  v 2  u 2  2a.s
a
sn = u + 2 n  1 sn= displacement in the nth second
2
uv
Vavg =
2
If acceleration is not uniform, above equations are not applicable. Then, use basic differential equation
  2
 dx  dv d x
v a 
dt dt dt 2
dv dv dx dv
[ Imp : a =v is used in many problems i.e.  
dx dt dt dx
This is used especially when acceleration is given as a function of x]
CENTERS : MUMBAI /DELHI /AKOLA /LUCKNOW /NASHIK /PUNE /NAGPUR /BOKARO /DUBAI # 7
T T

 vdt
0
 v.dt
0
Average speed = T
=
T
 dt
0

(2) Relative Velocity :


  
VAB  VA  VB
 2
VAB : velocity of A w. r. t. B VAB = V A  VB2  2V AVB cos 

Note: Velocity of swimmer will always be the velocity relative to water & hence his velocity
relative to ground changes .
  
VS.G  VS.W  Vw
  
Also Vrain ,man  Vman = Vrain
 While finding minimum distance between two objects moving with constant velocity, when their
position at certain instant is given, find relative path using relative velocity & the  ar distance is
the minimum distance

VB,A
x : minimum distance.
x

A0 B0
(3) Projectile Motion :

v = ( u cos  ) î + ( u sin  - gt ) ĵ
2
1 gx
(i) Equation of trajectory : y = x tan  -
2 u cos2 
2

1   1 
(ii) Average Velocity =  u cos t î   u sin t  gt 2 ˆj
t   2 
2u sin 
(iii) Time of flight=
g
u 2 sin 2 
Maximum height =
2g
u 2 sin 2
Range = ,maximum at   450 For a given velocity and range two values of angle of
g
projection are possible.
(iv) 
Minimum Velocity of projectile to pass through pt. (x, y) is = g y  x 2  y 2 
2
v
(v) Radius of curvature = where a r is the component of acceleration perpendicular to
ar
the velocity at that point. v 

u 

CENTERS : MUMBAI /DELHI /AKOLA /LUCKNOW /NASHIK /PUNE /NAGPUR /BOKARO /DUBAI # 8
u 2 cos 2 
Radius of curvature at angle  =
g cos3 
(vi) On an inclined plane at angle ,
Take X & Y axes along & perpendicular to the incline respectively & solve problem from basic
equations.
v = [ u cos (  -  ) – ( g sin  ) t ] î + [ u sin (  -  ) – ( g cos  ) t ] ˆj ]

s = [ u cos (  -  ) t – ½ ( g sin  ) t2 ] î + [ u sin (  -  )t – ½ ( g cos  ) t2 ] ˆj
2u 2 sin(   ) cos
range =
g cos2 
  
for fixed , range is maximum for  =   
4 2

2u sin(   )
time of flight =
g cos 

y u x

 ax = - g sin 
ay = - g cos 

(4) Circular Motion :

For non uniform circular motion,


v2
ar =  r 2 ar : radial acceleration
r
dv
at = r   at : tangential acceleration
dt
2 2
a= ar  at
v2 2
In uniform circular motion , at = 0 ; a =ar = r =
r
2
=
 T      
a  r v   r ar =   v

v2
(i) Banking of tracks : tan  =
Rg
if friction coefficient is , then for particular radius
Rg (tan    ) Rg (tan    )
Vmin = Vmax =
(1   tan  ) (1   tan  )

(ii) condition for skidding and toppling of car :


Rag
Vskidding =  Rg Vtoppling =
h
CENTERS : MUMBAI /DELHI /AKOLA /LUCKNOW /NASHIK /PUNE /NAGPUR /BOKARO /DUBAI # 9
where 2a = separation between inner & outer wheels
h = height of centre of gravity from ground
condition for toppling for motion on banked road ( angle  )
 a cos  h sin  
Vtoppling = Rg  
 h cos  a sin  

(iii) Conical Pendulum:


h
time period = 2
g
for constant angular acceleration  ,
h 
 = 0 +  t
 = 0 t + ½  t2
2 = 02 + 2 r
  0
avg =
2

NEWTON’S LAWS

FORMULA SUMMARY
While solving problems using Newton’s laws, apply the following method:

(1) Draw the free body diagrams. Make sure that all the forces are taken into account. They include:
Normal force & friction (at all points of contact ), gravitational force, spring force, electrostatic
force, Lorentz force or any other given force
(2) Choose axes properly & apply Newton’s 2 nd law along each axis after resolving all the forces.
(3) Look for constraint equations until
number of equations = number of unknowns.
(4) While working in non-inertial frame, consider pseudo forces.
(5) Be sure what your system is

 Work in a non-inertial frame of reference only if the study of motion


becomes very simple compared to that of the ground frame.
Here, acceleration of block of mass ’m’ is along the incline only
When in the frame of the wedge. But, in the ground frame the motion
is complicated. Even direction of acceleration is unclear

Don’t use non-inertial frame for such cases


Study motion of m
in ground frame itself

 For a system of variable mass


 
dv  dM
Fext  M  v M ,dm
dt dt

 Important points :
(1) In a system involving pulleys , work done by tension force on the whole system=0

  T. ds  0

CENTERS : MUMBAI /DELHI /AKOLA /LUCKNOW /NASHIK /PUNE /NAGPUR /BOKARO /DUBAI # 10
 
  T.v  0 &  T.a  0
This gives the required constraint equations.

(2) Similar equations can be applied for internal normal forces


 
i .e.  N.d s  0
  
  N.V  0 &  N.a  0

 Spring force:
 
F   kx , k : spring constant

Equivalent spring constant :


1 1
(i) For series combination 
k eq ki

(ii) for parallel combination keq =  k i

friction
f
fk

Applied force

In question involving friction , be sure that friction is static or kinetic .


Fl = s N Fk = k N
Fs  Fl
N
Fc

 Angle of friction ,  = tan-1()


F Also, angle of repose = 
on a smooth surface :
mg sin  cos m
a1 = a2
(M  m sin 2  )
(M  m) g sin  a1 M
a2 = 
(M  m sin 2  )
 
a m ,g  a 1  a 2

CENTERS : MUMBAI /DELHI /AKOLA /LUCKNOW /NASHIK /PUNE /NAGPUR /BOKARO /DUBAI # 11
UNIT & DIMENSIONS

Sr Physical Quantity Dimensions Unit SI CGS Conversion Factor


No (1 SI /1 CGS )
1 Length [ L1M0T0 ] M* cm 100
2 Mass [ L0M1T0 ] Kg* g 1000
3 Time [ L0M0T1 ] s* s 1
4 Current [ L0M0T0 I1 ] A* CGSE unit 3  10 9
5 Temperature - K* K 1
6 Area [ L2M0T0I0 ] m2 cm2 104
7 Volume [ L3M0T0 I0] m3 cm3 106
8 Wavelength [ LM0T0 ] m cm 100
9 Velocity [ LM0T-1 ] m/s cm/s 102
10 Acceleration [ LM0T-2 ] m/s2 cm/s 102
11 Momentum [M1L1T-1 ] Kg ms – 1 g – cms- 1 105
12 Force [M1 LT-2 ] N dyne 105
13 Work [M1L2T-2 ] J erg 107
14 Energy [M1L2T-2 ] J erg 107
15 Power [M1L2T-3 ] W erg /s 107
16 Density [M1L-3T0 ] Kg/m3 g/cm3 10 – 3
17 Frequency [M0L0T-1 ] Hz Hz 1
18 Angular Displacement - Rad Rad 1
19 Angular Velocity [M0L0T-1 ] rad/s rad/s 1
20 Angular acceleration [M0L0T-2 ] rad/s2 rad/s2 1
21 Angular momentum [M1L2T-1 ] kg – m2/s g – cm2/s 107
22 Torque [ML2T-2 ] N-m Dyne - cm 107
23 Moment of Inertia [ML2T0] Kg-m2 g-cm2 107
24 Pressure [ML-1T-2] Pa dyne/cm2 10
25 Entropy [M1L2T-2 ] J erg 107
26 Charge [M0L0 T1I1 ] coulomb GGSE unit 3  10 9
27 Voltage [M1L2 T-3I-1 ] V GGSE 1/300
28 Resistance [M1L2 T-3I-2 ] Ω GGSE unit 1/ (9  1011)
29 Resistivity [M1L3 T-3I-2 ] Ωm GGSE unit 1/ (9  109)
30 Electric field [ML T-3I-1 ] V/m GGSE unit 1/(3  104)
31 Gravitational Potential [L2 T-2 ] m2s -2 cm2s -2 104
32 Electric Potential [ML2 T-3I-1 ] V GGSE unit 1/300
33 Capacitance [M-1L-2 T4I2 ] F cm 9  10 11
34 Inductance [M1L2 T-2I-2 ] H cm 109
35 Conductivity [M-1L-3 T3I2 ] S m-1 CGSE unit 9  10 9
36 Current Density [ I1L-2 ] A/m2 CGSE unit 3  10 5
37 Electric Dipole Moment [ L1 T1 I1 ] C–m CGSE unit 3  10 11
38 Electric Flux [M1L3 T-3I-1 ] V-m CGSE unit 1/3
39 Magnetic Dipole moment [ L2I1] A-m2 Oe/cm 4π  10-5
40 Magnetic Field strength [ L-1I1] A/m Oe 4π  10-3
41 Magnetic Flux [ M1L2T-2 I-1 ] Wb Mx 108
42 Magnetic Induction [ M1T-2 I-1 ] T G 104
43 Permeability [M1L1 T-2I-1 ] Wb/A – m
44 Permittivity [M-1L-3 T4I2 ] C2 /N – m2
45 Pole Strength [ L1 I1 ] A–m Oe – cm2 4π  101

CENTERS : MUMBAI /DELHI /AKOLA /LUCKNOW /NASHIK /PUNE /NAGPUR /BOKARO /DUBAI # 12
46 Magnetic Potential [M1L1 T-2I-1 ] Wb/m
47 Specific Heat [ L2T– 2 ] J kg – 1 K – 1 Ergg-1K-1 104
48 Stress [M1L-1 T-2 ] Pa dyne/cm2 10
49 Electric potential [M1L T-3I-1 ] V/m CGSE unit 1/( 3  104)
gradient
50 Luminous Intensity Cd* Cd 1
51 Amount of Substance Mole* Mole 1

* Basic SI unit ( total 7 )


 Least Count :
(i) Smallest non – zero reading by an instrument
(ii) Decides the precision of measurement
 Significant Figures :

(I) Rules to identify:


(i) All the non – zero digits are significant
(ii) All the zeroes between two non – zero digits are significant, irrespective of the position of the
decimal point
(iii) If a number is less than 1 , the zeroes on the right of decimal pt. but to the left of the first non-
zero digit are not significant e.g. 0.003746 : 4 significant figures.
(iv) The terminal or trailing zeroes in a number without decimal pt. are not significant; but in a
number with decimal pt. are significant.
(v) In scientific notation, every number is expressed as a  10b where b  I and 1 ≤ | a| < 10 .
The total number of significant figures in the number equals that in a , irrespective of b

(II) Rules for arithmetic operation :


(i) In multiplication or division , the final result should retain as many significant figures as are
there in the original number with the least significant digits.
(ii) In addition or subtraction , the final result should retain as many decimal places as there are
in the number with the least decimal places.

(III) Rounding off the uncertain digits :


(i) The preceding digit is raised by 1 if the insignificant digit to be removed is more than 5 &
is left unchanged if the latter is less than 5.
(ii) In case when uncertain digit is 5 , if the preceding digit is even , the insignificant digit is
simply dropped ; if it is odd , it is raised by 1.

 Error Estimation :

(i) If X = a1  a2  ……  an then absolute error in measurement of x is ,


n
∆x=  a
i 1
i , where ∆ ai is absolute error = | ai |

n ki

(ii) If X =  aii 1
then , the relative error in the measurement of X is given
n
x  a i 
  | k i | 
x i 1  ai 

(iii) For a given instrument , when no other source of error is mentioned


maximum error = least count of instrument .

CENTERS : MUMBAI /DELHI /AKOLA /LUCKNOW /NASHIK /PUNE /NAGPUR /BOKARO /DUBAI # 13
LENGTH MEASUREMENT
(I) Vernier Callipers :

(i) Principle : If ( N + 1 ) vernier divisions are equal to N main sacle division , then
N
V  s where v = smallest division of vernier scale
( N  1)
S = smallest division of main scale
s
(ii) Least Count : L.C of vernier caplliper =
( N  1)

Note : Least count of a vernier calliper in general = ( least count of main scale ) – ( least count of vernier
scale )

(iii) Zero error : is to be subtracted from observed reading

s
(iv) Length = M.S. reading +  vernier scale reading ( line of v. s. coinciding with line
( N  1)
on M.S.)

(II) Micrometer ( Screw gauge )

(i) Pitch : The linear distance moved by the screw forward or backward when one complete rotation is
given to circular cap.
pitch of the screw
(ii) L.C.
number of divisions or circular sacle
(iii) Length = M.S. reading + circular scale reading  L.C.
(iv) Zero error : same as in V.C.

 Experiments

(I) Determination of ‘g’ by simple pendulum


  
g = 4π 2  2  for small amplitude
T 
l : length of pendulum
T : Time period of pendulum
Error Estimation :

g l 2T
 
g l T

(II) Determination of Young’s Modulus by searle’s method :


MgL
Y  2 where
r l
M : Mass of load
L : length of wire
r : Radius of c.s. of wire
l : extension of wire for the given load

CENTERS : MUMBAI /DELHI /AKOLA /LUCKNOW /NASHIK /PUNE /NAGPUR /BOKARO /DUBAI # 14
Error Estimation :

 Y  M  L 2 r 
   
Y M L r 

(III) Specific heat of liquid using calorimeter :

Mc 2 (T2  T ) m1
C=  C 1 where
m 2 (T  T1 ) m2

M = Mass of liquid
m1 = mass of calorimeter & stirrer
m2 = mass of cold liquid
T1 = Temperature of cold liquid
T2 = Temperature of hot solid
T = final temperature of ( liquid + solid + calorimeter)
C1 = Sp. heat of calorimeter
C2 = Sp. heat of solid

Error estimation :

MC 2 (T2  T ) m1
x1  x2  C2
m 2 (T  T1 ) m2
C = x1 – x2 C  x 1  x 2
x 1 M m 2 (T2  T) (T  T1 )  x 
    & x 1   1 x 1
x1 M m2 (T2  T) T  T1  x1 

x 2 m 2 m 1 x 2
   x 2  ( )x 2
x2 m2 m1 x2

(IV) focal length using u – v method :

1 1 1
Formula :   for lens
v u f
1 1 1
  for mirror
v u f
u : object distance
v : Image distance
f : Focal length

[ For sign convention , refer Optics ]

Error formula :

 f  u  v  u  v
  
f u v uv

CENTERS : MUMBAI /DELHI /AKOLA /LUCKNOW /NASHIK /PUNE /NAGPUR /BOKARO /DUBAI # 15
(V) Resonance Tube Experiment :
It l1 & l2 are first two readings ( from water level to top of tube )
V = 2f ( l2 – l1 )
V : velocity of sound in air
f : frequency of tuning fork

l 2  3l 1
e = 0.3 d or e=
2
v f l 1  l 2
Error estimation :  
v f l 2  l1

(VI) Verification of ohm’s law :


+ -
V

+ V -
V
 constant = R where , V : Voltage across R
I
I : Current flowing through R

(VII) Specific resistance of material of wire using meter – bridge :


X R

l 2

l1
X=R where x : unknown resistance , l1 is the balancing length.
l2
Error Estimation :
x R l1 l 2
   Here  1  1cm or 1mm
x R l1 l2

x R    2 
So ,  1   2 ……. Generally     1 
x R   1 2 

CENTERS : MUMBAI /DELHI /AKOLA /LUCKNOW /NASHIK /PUNE /NAGPUR /BOKARO /DUBAI # 16
x
 is minimum when  1  2 is maximum but  1   2  L  1m
x
 When the balancing point ( null point ) is near the centre error is minimum

(VIII) Post office box :

X R1 R2 E
R

G
Rh
K
R  x R R 2 R 1
X  R  2    
 R1  x R R2 R1

 Mathematical Approximation Methods :

Binomial Series :
n ( n  1) 2
( 1 + x) n = 1 + n x + x + …..
2
For |x| < 1 nR
If | x| < < 1 then neglecting second & higher order terms of x ( 1 + x ) n = 1 + n x

bt
( a + b ) t = at ( 1 + ) = at + tat – 1 b, when |a| >> |b| ( first two terms of binomial
a
expansion )
1
e.g.  (1  x ) 1  1  x , if | x | < < 1
1 x

It is especially useful in thermal expansion etc.

x  y ( x  y) y ( x  y) 2 y 2y
 (1  ) 1  2
 (1  )  1 
xy x x x x x
if | y| < < | x| & similarly
xy 2y
1
xy x

CENTERS : MUMBAI /DELHI /AKOLA /LUCKNOW /NASHIK /PUNE /NAGPUR /BOKARO /DUBAI # 17
SINGLE CHOICE QUESTIONS

1. The displacement of a particle moving in a straight line is described by the relation, s= 6 + 12t – 2t2.
Here s is in meters and t in seconds. The distance covered by particle in first 5 seconds is :
(a) 20 m (b) 32 m (c) 24 m (d) 26 m

2. In the figure mA : mB : mC = 1 : 2 : 4. All surface are smooth. The acceleration of wedge c is :

B
C
600

g g g
(a) (b) (c) (d) zero
4 3 2

3. A ball is thrown horizontally with velocity 30 m/s from top of a 60 m high tower. simultaneously,
another ball is thrown vertically upward with velocity 40 m/s from the bottom of same tower. The
shortest distance between the two balls is (g = 10 m/s2)
(a) 36 m (b) 48 m (c) 60 m (d) 45 m

4. Two blocks of masses 10kg and 4kg are hanging from a smooth, massless and movable pulley with
the help of a massless string that passes over the movable pulley. A vertically upward force of 120N
has been applied to the pulley. Acceleration of the pulley is
(a) 0.5 m/s2, upward (b) 0.5 m/s2, downward
2
(c) 2 m/s , downward (d) 5 m/s2 downward

5. In an experiment the angles are required to be measured using an instrument. 29 divisions of the
main scale exactly coincide with the 30 divisions of the vernier scale. If the smallest division of the
main scale is half-a-degree   0.5o  , then the least count of the instrument is:
(a) one degree (b) half degree (c) one minute (d) half minute

6. A Wedge with a rough groove in the shape of a quarter of a circle is kept on a smooth table (see
figure). A disc is placed in the groove with a small clearance. Friction exists between groove and
disc. The wedge is moved with an acceleration 3g . If disc is to remain stationary in the position
shown relative to groove, the coefficient of friction required can be.

30° a  3g

1 1 1 9
(A) (B) (C) (D)
3 4 5 10

7. A box of mass 1 kg is lying on a rough horizontal floor and a horizontal force acting on box has a
magnitude of F = 3t2 N, where t is in seconds. If the box starts from rest, determine its speed (in m/s)
when t = 3 sec. The coefficient of static and kinetic friction between the box and horizontal floor are
s = 0.3 and k = 0.2.
(A) 18 (B) 20 (C) 22 (D)24

CENTERS : MUMBAI /DELHI /AKOLA /LUCKNOW /NASHIK /PUNE /NAGPUR /BOKARO /DUBAI # 18
8. Figure shows N identical blocks connected with identical springs on a smooth horizontal surface. A
constant force is pulling the blocks horizontally. During motion all the springs have acquired their
stable configurations. If stretch in spring connecting 4th and 5th block is two times that between 8th
and 9th block what is total number of blocks.
N N–1 2 1
P
(A) 6 (B) 8 (C) 10 (D) 12

9. A particle starts moving rectilinearly at time t = 0 such that its velocity 'v' changes with time 't'
according to the equation v = t2 – t where t is in seconds and v is in m/s. The time interval for which
the particle retards is
(A) t < 1/2 (B) 1/2 < t < 1 (C) t > 1 (D) t < 1/2 and t > 1

10. A rope of length L and mass M is being pulled on a rough horizontal floor by a constant horizontal
force F = Mg. The force is acting at one end of the rope in the same direction as the length of the
rope. The coefficient of kinetic friction between rope and floor is 1/2. Then, the tension at the mid-
point of the rope is-
(A) Mg/4 (B) 2Mg/5 (C) Mg/8 (D) Mg/2

11. A wedge can slide frictionlessly on a fixed incline of angle 37°. A girl who is strapped on the wedge
[no relative motion between wedge and girl] pushes a small ball on the horizontal smooth upper
surface of wedge as shown in figure. Ball is pushed at the same instant as the motion of wedge starts.
Initial velocity of ball is 24 m/s relative to wedge. The time (in sec) after which ball returns to girl
is
(Assume that horizontal surface of wedge is very long.)

Wedge

37°
(A) 10 (B) 12 (C) 8 (D) 6

12. Two particles are projected from same point on ground in two mutually perpendicular planes with
same initial speed u at same angle 60° above horizontal. If acceleration due to gravity be ‘g’, the time
instant at which their velocities are inclined at 60° with each other is/are -
3u u 
3 1 u 
(A)
2g

(B) 3  1 
2g
(C)
2g
(D) never attained.

13. The ratio of tensions in the string connected to the block of mass m2 in figure-(a) and figure-(b)
respectively is (friction is absent everywhere) : [m1 = 50 kg, m2 = 80 kg and F = 1000 N].

m2
m2
m1
F m F
1

(a) (b)
(A) 7 : 2 (B) 2 : 7 (C) 3 : 4 (D) 4 : 3

CENTERS : MUMBAI /DELHI /AKOLA /LUCKNOW /NASHIK /PUNE /NAGPUR /BOKARO /DUBAI # 19
14. When the gap is closed without placing any object in the screw gauge whose least count is 0.005
mm, the 5th division on its circular scale with the reference line on main scale, and when a small
sphere is placed reading on main scale advances by 4 divisions, whereas circular scale reading
advances by five times to the corresponding reading when no object was placed. There are 200
divisions on the circular scale. The radius of the sphere is
(A) 4.10 mm (B) 4.05 mm (C) 2.10 mm (D) 2.05 mm

15. A ball is projected horizontally from an incline so as to strike a cart sliding on the incline. Neglect
height of cart and point of projection of ball above incline. At the instance the ball is thrown, the
speed of cart is
v0 = 10 m/s

sliding
6m
frictionlessly
on incline
=37°

(A) 4 (B) 6 (C) 8 (D) None of these

16. Find the maximum value of (M/m) in the situation shown in figure so that the system remains at rest.
Friction coefficient of both the contacts is µ, string is massless and pulley is friction less.
m
M

cos  sin 
(A) (B)
sin   µcos  sin    cos 
 cos  
(C) (D)
sin    cos  sin    cos 

17. A small block of mass m (= 1 kg) is pulled on a frictionless horizontal surface by means of a light
rope at a rate of u(=10 m/s) with a pulley situated at a height h(= 9 m) above the level of block. The
block was initially placed at a very large distance from the pulley. The angle  (in degree) of rope
with horizontal, when the block leaves the surface is
u=10 m/s

h= 9 m
v

(A) 30 (B) 37 (C) 53 (D) 60

18. An object of mass m rests on an inclined plane that makes angle   450 with the horizontal floor.
What minimum force (in N), parallel to the incline must be applied to the object in order to move it
along the plane parallel to the floor as shown? The coefficient of static friction between the object
and the plane is 

mg mg
(A) mg (B) 2 mg (C) (D)
2 2

CENTERS : MUMBAI /DELHI /AKOLA /LUCKNOW /NASHIK /PUNE /NAGPUR /BOKARO /DUBAI # 20
19. With two fingers, you hold a cone motionless upside down, as shown in figure. The mass of the cone
is m = 1kg, and the coefficient of static friction between your fingers and the cone is ( = 0.5). What
is the minimum normal force (in Newton) you must apply with each finger in order to hold up the
cone ?
Finger Finger

12cm

10cm
(A) 30 (B) 65 (C) 25 (D) 20

20. If the number of significant figures in 8000,800.00, 80.08x103 & 0.008 are p, q, r & s respectively
then p + q + r + s will be equal to
(A) 10 (B) 11 (C) 12 (D) 13

21. Figure shows a radar screen, with the dots denoting respective positions of Indian SUKHOI. A, B, C
and PAKI F-16-E. All are flying with constant velocity in horizontal plane. SUKHOI-A reports to
ground control that E is moving due north with velocity 160 m/s. At same time SUKHOI-B reports
that PAKI F-16-E is moving due east at 120 m/s. Through what minimum angle SUKHOI-C
(originally moving in north east) turns so that it is alligned in direction of motion of E.
N y

W E

S B Plane Direction
E
SUKHOI A East
45°
x SUKHOI B North
C A SUKHOI C North East
PAKI F-16 E Unknown direction

Radar Screen
(A) 8° clockwise (B) 8° anticlockwise
(C) 16° clockwise (D) 16° anticlockwise

22. Two blocks of equal masses (M) are connected by a string and are kept on rough horizontal surface
as shown in figure. The coefficient of friction between the blocks and the surface is  .
If 0  F1  F2  2Mg , then choose the correct statement.
B A
F2 F1

(a) the direction of friction on block A is towards right.


(b) the direction of friction on block B may be towards left or right
(c) tension in the string must be zero
(d) friction force on block B must be zero

CENTERS : MUMBAI /DELHI /AKOLA /LUCKNOW /NASHIK /PUNE /NAGPUR /BOKARO /DUBAI # 21
23. The vernier of a vernier scale is divided into 10 divisions which coincide with 9 divisions of the main
scale, each main scale division being 0.5 mm. When the two jaws of the instrument are in contact
with each other, the 4th division of the vernier scale coincides with a main scale division and the
zero of the vernier lies to the right of the zero of the main scale. When a sphere is inserted between
the jaws, the zero of vernier scale lies slightly to the right of 1.8 cm and the sixth vernier division
coincides with a main division. The diameter of sphere will be
(a) 1.85 cm (b) 1.81 cm (c) 1.75 cm (d) 1.71 cm

24. Two masses of 5 kg and 15 kg respectively are connected by a massless spring and a constant force
F = 200 N is applied on 15 kg block as shown. If at an instant, the 5 kg mass has acceleration of
magnitude 10m/s2 as shown in figure,find magnitude of acceleration in m/s2 of 15 kg mass at the
same instant. All surfaces are smooth.
10 m/s2

5 kg 15 kg 200N

(A) 5 (B)10 (C) 15 (D) None of these

25. In system shown, all pulleys & strings are massless. Friction is absent everywhere. A block of mass
m = 20 kg is attached to movable pulley. A constant force applied at free end of string, makes the
point of application of force move down with a constant acceleration of 4m/s2. Find applied force.

a=4m/s2

m=20 kg

(A) 40 N (B) 80 N (C) 120 N (D) None of these

26. A projectile is required to hit a target whose coordinates relative to the horizontal and vertical axes
through the point of projection are  ,   . If the projection velocity is 2g , it is impossible to hit
the target if
3 1 3 1
(A)    (B)    (C)    (D)   
4 4 4 4

CENTERS : MUMBAI /DELHI /AKOLA /LUCKNOW /NASHIK /PUNE /NAGPUR /BOKARO /DUBAI # 22
27. The acceleration of B is

a cos 1 a cos  2 a sin 1 a sin 2


(A) (B) (C) (D)
cos 2 cos 1 sin  2 sin 1

28. A block of mass 10 kg is released on a fixed wedge inside a cart which is moving with a constant
velocity of 10 m/s towards right. The initial velocity of the block w.r.t the cart is zero. The work
done by normal reaction (with respect to ground) on the block in two seconds is g  10 m s 2 

(A) Zero (B) 1200 J (C) 1024 J (D) 960 J

29. A particle projected from a horizontal plane (x-z plane) such that its velocity vector at time t is given

by V  ai   b  ct  j . Its range on the horizontal plane is given by
2ba ba 3ba
(a) (b) (c) (d) none of these
c c c

30. A weight W is supported by two cables as shown. The tension in the cable making angle  with
horizontal will be minimum when the value of  is

T2 T1

 60o

(a) 0 (b) 30o (c) 60 o (d) none of these

CENTERS : MUMBAI /DELHI /AKOLA /LUCKNOW /NASHIK /PUNE /NAGPUR /BOKARO /DUBAI # 23
31. If in the arrangement shown, motor runs winding rope at a rate of v m/s the upward speed of plat
form will be

v v v
(a) (b) (c) (d) none of these
3 4 7

32. I am sitting in a car, which is undergoing acceleration in the forward direction of magnitude a. I feel
a force pushing me horizontally backwards, into the back of the car seat. If my mass is M and the
acceleration due to gravity is g, what is the magnitude of this force?
(a) Mg (b) M a 2  g 2 (c) Ma (d) M  a  g 

33. A particle projected with speed u at an angle  from horizontal at t  0 . Its horizontal component of
velocity  vx  varies with time t as shown in the following graph:

(a) (b)

vx
vx
Time Time
T 2 T T2 T
(c) (d)

vx vx

Time Time
T 2 T T2 T

34. A block of mass m is pulled in the direction shown in the figure on a rough horizontal ground with a
constant acceleration of magnitude ‘a’.

(a) Tcos   ma (b) k mg (c) k  T  mg  (d) k Tsin 

CENTERS : MUMBAI /DELHI /AKOLA /LUCKNOW /NASHIK /PUNE /NAGPUR /BOKARO /DUBAI # 24
35. A block B moves with a velocity u relative to the wedge A. If the velocity of the wedge is v as
shown in figure, what is the value of  so that the block B moves vertically as seen from ground.

u
A
v B

u v u v


(a) cos1   (b) cos 1   (c) sin 1   (d) sin 1  
 v u v u

36. In the figure let ‘a’ and T be the acceleration of the blocks and tension in the string respectively. The
string and the pulley are frictionless and massless. Which of the graphs show the correct relationship
between a and T for the system in which sum of the two masses m1 and m2 is constant.

m1  m 2

m1
m2

T T
(a) (b)

a2 a2
(c) T (d) T

1 1
a2 a2

37. A body is projected with a velocity u at an angle  with the horizontal. The velocity of the body will
become perpendicular to the velocity of projection after a time t given by
2u sin  u sin  2u u
(a) (b) (c) (d)
g g gsin  gsin 
38. A ball is thrown from a point on a ground at some angle of projection. At the same time a bird starts
from a point directly above this point of projection at a height h horizontally with speed u. Given that
in its flight ball just touches the bird at one point. Find the distance on ground where the ball strikes.
h 2h 2h h
(a) 2u (b) u (c) 2u (d) u
g g g g

CENTERS : MUMBAI /DELHI /AKOLA /LUCKNOW /NASHIK /PUNE /NAGPUR /BOKARO /DUBAI # 25
39. Particles A and B are moving with constant velocities along x and y axis respectively, the graph of
separation between them with time is

(a) (b) (c) (d)

40. A man is crossing a river flowing with velocity of 5m/s. He reaches a point directly across at a
distance of 60m in 5 sec. His velocity in still water should be
B

V r  5m s 60m

A
(a) 12m/s (b) 13m/s (c) 5m/s (d) 10m/s
o
41. A motor boat is to reach at point 30 upstream (w.r.t. normal) on other side of a river flowing with
velocity 5m/s. Velocity of motorboat w.r.t. water is 5 3 m/s. The driver should steer the boat at an
angle
(a) 120o w.r.t. stream direction
(b) 30o w.r.t. normal to the bank
(c) 30 o w.r.t. the line of destination from starting point
(d) none of these

42. A flag is mounted on a car moving due North with velocity of 20km/hr. Strong winds are blowing
due East with velocity of 20km/hr. The flag will point in a direction
(a) East (b) North –East (c) South-East (d) South-West

43. Three ships A, B and C are in motion. The motion of A as seen by B is v along north-east. The
motion of B as seen by C is speed v towards the north-west. Then as seen by A, C will be moving
towards-
(a) North (b) South (c) East (d) West

44. Find the acceleration of 3kg mass when acceleration of 2kg mass is 2ms2 as shown in figure.

3kg 2 kg 10 N

2ms 2
(a) 3ms 2 (b) 2ms 2 (c) 0.5ms 2 (d) zero

CENTERS : MUMBAI /DELHI /AKOLA /LUCKNOW /NASHIK /PUNE /NAGPUR /BOKARO /DUBAI # 26
45. Block B of mass 100kg rests on rough surface of friction coefficient
  1 3.A rope is tied to block B as shown in figure. The maximum
acceleration with which boy A of 25kg can climbs on rope without 100 kg
making the block move is: 37 0

(a)
4g
(b)
g B
3 3   1/ 3 A
g 3g 25 kg
(c) (d)
2 4
46.  
A block placed on a rough inclined plane of inclination   30o can just be pushed upwards by
applying a force “F” as shown. If the angle of inclination of the inclined plane is increased to
   60  , the same block can just be prevented from sliding down by application of a force of same
o

magnitude. The coefficient of friction between the block and the inclined plane is
3 1 2 3 1 3 1
(a) (b) (c) (d) None of these
3 1 3 1 3 1

47. With what minimum velocity should block be projected from left end A towards end B such that it

reaches the other end B of conveyor belt moving with constant velocity v. Friction coefficient

between block and belt is .

A m v0
B
 V

L
(a) gL (b) 2gL (c) 3gL (d) 2 gL

For Question No 48 to 52 refer figure to given below:

10 kg F
  0.1
5 kg
  0.3
48. When F  2N , the frictional force between 5kg block and ground is
(a) 2N (b) 0 (c) 8N (d) 10 N

49. When F  2N , the frictional force between 10kg block and 5kg block is
(a) 2N (b) 15N (c)10N (d) None

50. The maximum “F” which will not cause motion of any of the blocks.
(a) 10N (b) 15N (c) data insufficient (d) None

CENTERS : MUMBAI /DELHI /AKOLA /LUCKNOW /NASHIK /PUNE /NAGPUR /BOKARO /DUBAI # 27
51. The maximum acceleration of 5kg block
(a) 1m s2 (b) 3m s2 (c) 0 (d) None

52. The acceleration of 10kg block when F  30N


(a) 2m s2 (b) 3m s2 (c) 1m s 2 (d) None

53. A conical pendulum moving in a circle with angular velocity  as shown. If l



tension in the string is T, which of following equations are correct?
(a) T  m21 (b) T sin   m21 m
(c) T  mg cos  (d) T  m21sin 

54. A car travelling on a smooth road passes through a curved portion of the road in the form of an arc of
circle of radius 10m. If the mass of car is 500kg, the reaction on car at lowest point P where its speed
is 20m/s is

P
(a) 35kN (b) 30kN (c) 25kN (d) 20kN

55. A man of mass M stands at one end of a plank of length L which lies at rest on a frictionless surface.
The man walks to other end of the plank. If the mass of the plank is M/3, then the distance that the
man moves relative to ground is:
(a) 3L/4 (b) L/4 (c) 4L/5 (d) L/3

56. A man weighing 80kg is standing at the centre of a flat boat and he is 20m from the shore. He walks
8m on the boat towards the shore and then halts. The boat weighs 200kg. How far is he from the
shore at this instant ?
(a) 11.2m (b) 13.8m (c) 14.3m (d) 15.4m

57. From a disc of radius a, an isosceles right angled triangle with the hypotenuse as the diameter of the
circle is removed. The distance of the centre of gravity of the remaining position from the centre of
the circle is
  1 a a a
(a) 3  1 a (b) (c) (d)
6 3    1 3    1
car
58. A bob is hanging over a pulley inside a car through a string. The second end a
of the string is in the hand of a person standing in the car. The car is moving
a

with constant acceleration ‘a’ directed horizontally as shown in figure. Other
m
end of the string is pulled with constant acceleration ‘a’ vertically. The tension
in the
string is equal to D m  g  a 

(A) m g 2  a 2 (B) m g 2  a 2  ma (C) m g 2  a 2  ma (D) m  g  a 

CENTERS : MUMBAI /DELHI /AKOLA /LUCKNOW /NASHIK /PUNE /NAGPUR /BOKARO /DUBAI # 28
59. A system is shown in the figure. A man standing on the block is pulling the rope. Velocity of the
point of string in contact with the hand of the man is 2 m/s downwards.
The velocity of the block will be
(A) 3 m/s (B) 2 m/s

(C) 1/2 m/s (D) 1 m/s

60. A block of mass 2 kg is given a push horizontally and then the Block
8
starts sliding over a horizontal plane. The figure shows the velocity-time

(velocity in ms)
graph of the motion. The co-efficient of sliding friction between the plane

V
and the block is: (Take g = 10 m / s 2 )
(A) 0.02 (B) 0.20
t 4
(C) 0.04 (D) 0.40  time in sec 

61. A small coin of mass 40 g is placed on the horizontal surface of a rotating


disc. The disk starts from rest and is given a constant angular acceleration
  2 rad / S2 . The coefficient of static friction between the coin and the disc
is s  3 / 4 and coefficient of kinetic friction is k  0.5 . The coin is placed
at a distance r = 1 m from the centre of the disc. The magnitude of the
resultant force on the coin exerted by the disc just before it starts slipping on
the disc is:
(A) 0.2 N (B) 0.3 N (C) 0.4 N (D) 0.5 N

62. The figure below shows the forces that three charged particles exert on each other. Which of the four
situations shown can be correct.

(I) (II)

(III) (IV)
(a) all of the above (b) none of the above (c) II, III (d) II, III & IV

63. A man crosses a river perpendicular to the river flow in t seconds and travels an equal distance
downstream in T seconds. The ratio of man’s speed in still water to the speed of river water will be
t2  T2 T2  t 2 t 2  T2 T2  t 2
(A) (B) (C) (D)
t 2  T2 T2  t2 t 2  T2 T2  t2

CENTERS : MUMBAI /DELHI /AKOLA /LUCKNOW /NASHIK /PUNE /NAGPUR /BOKARO /DUBAI # 29
64. If the coefficient of friction between an insect and a spherical bowl of radius R is  , then the
maximum height above center up to which the insect can crawl in the bowl is
R  1 
(A) (B) R 1   (C) R  1   2  (D) R  1   2  1
1  2  1   2 

65. A particle is moving in a circle of radius R in such a way that at any instant the total acceleration
makes an angle of 450 with the radius vector. If the initial speed of the particle is v0 then the time
taken by the particle to complete the first revolution is
R 2  R 2R 2 2R 
(A) e (B) 1  e 2   (C) e (D) 1  e2 
v0 v0 v0 v0

66. A particle flying horizontally strikes a smooth incline plane and rebounds vertically. If the angle
  600 , then the coefficient of restitution for collision is

1 1 1
(A) (B) (C) (D) 1
3 3 2

MULTIPLE CHOICE QUESTIONS

1. The figure shows the velocity (v) of a particle plotted v


against time (t).
(a) The particle changes its direction of motion at some
T
point. O
2T t
(b) The acceleration of the particle remains constant
(c) The displacement of the particle is zero.
(d) The initial and final speeds of the particle are the
same.

2. A man who can swim at a speed v relative to the water wants to cross a river of width d, flowing with
a speed u. The point opposite him across the river is P.
(a) The minimum time in which he can cross the river is d/v.
(b) He can reach the point P in time d/v.
d
(c) He can reach the point P in time
v  u2
2

(d) He cannot reach P if u > v.

CENTERS : MUMBAI /DELHI /AKOLA /LUCKNOW /NASHIK /PUNE /NAGPUR /BOKARO /DUBAI # 30
3. If no external force acts on a system
(a) Velocity of centre of mass remains constant
(b) Velocity of centre of mass is not constant
(c) Both velocity and acceleration of centre of mass remains same but not zero
(d) Acceleration of centre of mass is zero

4. In the figure, the block B of mass m starts from rest at the top of a
B(m)
wedge W of mass M. All surfaces are without friction. W can slide on
the ground. B slides down onto the ground, moves along it with a
speed v, has an elastic collision with the wall, and climbs back onto W.
(a) B will reach the top of W again
(b) From the beginning, till the collision with the wall, the centre of W(M)
mass of ‘B plus W’ is stationary
2mv
(c) Just after the collision, the centre of mass of ‘B plus W’ moves with the velocity .
mM
2mv
(d) When B reaches its highest position on W, the speed of W is
mM
5. In the system shown in the diagram all surfaces are smooth, pulley and strings are ideal. If and are
the accelerations of the two blocks, then just after the system is released from rest, then choose
correct statement (s) :

B A
 
(A) a A  a B
 
(B) a A  a B
(C) Acceleration of A relative to B is vertically downwards.
(D) Normal force exerted by B on A is zero.

6. Two boats start from mid point of river in case (i) and in mid point of pond of still water in case (ii).
If all velocity shown in figure are relative to water then mark the correct option about time taken to
reach the nearest bank.
v v
A  C 
d d Still
River water
 
B Flow D
v v
Case (i) Case (ii)
(A) Boat A takes least time.
(B) Boat B takes maximum time.
(C) All the four boats A,B,C and D take same time.
(D) Time of reaching bank can not be compared

CENTERS : MUMBAI /DELHI /AKOLA /LUCKNOW /NASHIK /PUNE /NAGPUR /BOKARO /DUBAI # 31
7. Find the acceleration of the three masses A, B and C shown in figure. Friction coefficient between all
surfaces is 0.5. Pulleys are smooth. (Given mA = 1kg, mB = 1kg, mC = 2kg.)
A

B
C

7g 7g 7g 7g
(A) a A  (B) a C  (C) a B  (D) a A 
9 9 18 18
8. A particle moves in a circle of radius R, with a constant speed v. Then, during a time interval
[R/3v], which of the following is true?
(A) | average acceleration | = 3v2/R (B) | average velocity | = 3v/
2
(C) | average acceleration | = 2v /R (D) average speed = 3v/

9. Which of the following statement(s) is/are incorrect :


(A) Friction force always acts opposite to the net applied force
(B) If net force on a body is zero, its acceleration will be same in all frames.
(C) A person is pushing a box. The force exerted by box on person is less than the force by person on
box.
(D) A moving object can move with a constant velocity by a push less than the limiting static friction

10. A projectile is projected as shown in figure. A proper light arrangement makes a shadow on the wall
as well as on the floor ?
Light

u
Light


Which of the following graphs is incorrect.
Acceleration velocity
of shadow of shadow
on wall on wall
t t
(A) (B)
velocity Height
of shadow of shadow
on ground on wall
t t
(C) (D)

CENTERS : MUMBAI /DELHI /AKOLA /LUCKNOW /NASHIK /PUNE /NAGPUR /BOKARO /DUBAI # 32
11. In figure, a block of mass m is released from rest when spring was in its natural length. The pulley
also has mass m but it is frictionless. Suppose the value of m is such that finally it is just able to lift
the block M up after releasing it.
Rod

String
m

M
M
(A) The weight of m required to just lift M is g
2
M
(B) The tension in the rod, when m is in has zero acceleration g
2
M
(C) The normal force acting on M when m has zero acceleration g
2
(D) The tension in the string when displacement of m is maximum possible is Mg.

12. A boy is pushing a box on a horizontal floor from a position of rest to rest while moving on a straight
line. Consider the three phases of motion. Floor is rough with a small friction coefficient.
(i) Initially a constant hard push on the box to get it moving and attain a maximum velocity.
(ii) Mild push to keep the box moving with constant velocity.
(iii) to pull back the box to bring it to stop with same retardation.
Which of the following graphs are correct.
velocity Acceleration

t t
(A) (B)
Force exerted Friction force
by worker

t
t
(C) (D)

13. Position of 2 vehicles A and B with reference to origin O and


their velocities are as shown at time instant t = 0. If they ˆj
move with constant velocities as shown. Choose the correct î
statement (s) 
VA  20 ˆi m s
(A) Distance of closest approach is 100m
(B) Magnitude of relative velocity is 40 m/s
A O origin
3 100 ˆi m
(C) The instant at which they are closest to each other is  20
VB  ˆjm s
t = 7.5 s. 3
(D) All above statements are false. B 100 3 ˆjm

CENTERS : MUMBAI /DELHI /AKOLA /LUCKNOW /NASHIK /PUNE /NAGPUR /BOKARO /DUBAI # 33
14. A particle moves in x – y plane with constant acceleration of 1.5 m / s 2 y
in direction making an angle of 370 with x – axis. At t = 0, particle is
at origin and its velocity is 8 m/s along x – axis. Choose the correct 2

statement (s) a  1.5 m s

(A) At t = 4s, the coordinates of particle is (41.6 m, 7.2 m)


1 37 0
(B) At t = 4s, velocity makes angle tan 1   with x – axis x
 3 u=8 m/s
 9 
(C) At t = 4s, velocity makes an angle tan 1   with x – axis
 32 
(D) All above statements are false

15. A block of mass 1 kg is at rest with respect to a wedge moving towards left with constant
2
acceleration of 5 m s . Let R be normal reaction between block and wedge. All surfaces are smooth
2
g = 10 m s . Then

1 kg

a

1
(A) R  5 5 N (B) R = 15 N (C) tan   (D) tan   2
2
16. A particle moving with speed V changes its direction of motion by angle  without change in speed.
Choose the correct statement (s)

(A) Magnitude of change of velocity is 2 V sin
2
(B) Magnitude of change of velocity is zero
(C) Change in magnitude of velocity is zero
(D) All above statements are false

17. Which of the following statement(s) is/are true?


(a) The magnitude of the sum of two vectors must be greater than the magnitude of their vector
(b) If the speed of a moving particle is constant, the acceleration must be zero
(c) Distance traveled by an object must be greater than or equal to magnitude of displacement vector
(d) If a particle is moving along a circle it must be accelerating.

18. If T is the total time of flight, h is the maximum height & R is the range for horizontal motion, the x
& y co-ordinates of projectile motion and time t are related as:
 t  t  X  X 
(a) y  4h  1   (b) y  4h  1  
 T  T   R  R 
 T  T   R  R 
(c) y  4h  1   (d) y  4h  1  
 t  t  X  X 

CENTERS : MUMBAI /DELHI /AKOLA /LUCKNOW /NASHIK /PUNE /NAGPUR /BOKARO /DUBAI # 34
19. An observer moves with a constant speed along the line joining two stationary objects. He will
observe that the two objects
(a) have the same speed (b) have the same velocity
(c) move in the same direction (d) move in opposite directions

20. A man on a rectilinearly moving cart, facing the direction of motion, throws a ball straight up with
respect to himself
(a) The ball will always return to him
(b) The ball will never return to him
(c) The ball will return to him if the cart moves with constant velocity
(d) The ball will fall behind him if the cart moves with some acceleration

21. A block is pushed with a velocity of 2 ms 1 (relative to ground) on a belt, which is moving with
velocity 4ms 1 in a direction opposite to the initial velocity of block. If the block stop slipping on the
belt after 4 sec of the throwing then choose the correct statement(s)
(a) Displacement with respect to ground is zero after 2.66 sec and magnitude of displacement with
respect to ground is 12m after 4 sec.
(b) Magnitude of displacement with respect to ground in 4 sec is 4m.
(c) Magnitude of displacement with respect to belt in 4 sec is 12m.
(d) Displacement with respect to ground is zero in 8/3 sec.

22. A block A (5 kg) rests over another block B (3 kg) placed over a smooth horizontal surface. There is
friction between A and B. A horizontal force F1 gradually increasing from zero to a maximum is
applied to A so that the blocks move together without relative motion. Instead of this another
horizontal force F2 , gradually increasing from zero to a maximum is applied to B so that the blocks
move together without relative motion. Then

F1
A 5kg A 5kg
F2
B 3kg B 3kg

(A) F1  max   F2  max (B) F1  max   F2  max

(C) F1  max   F2  max  (D) F1  max  : F2  max   5:3

23. Two identical bodies are interconnected with a massless and inextensible thread. The system is in
gravity free space with the thread just taut. Each ball is imparted a velocity v, one towards the other
ball and the other perpendicular to the first, at t  0 . Then,
(a) the thread will become taut at t   L v
(b) the thread will become taut at some time t   L v
(c) the thread will always remain taut for t   L v
(d) the kinetic energy of the system will always remain mv 2

CENTERS : MUMBAI /DELHI /AKOLA /LUCKNOW /NASHIK /PUNE /NAGPUR /BOKARO /DUBAI # 35
INTEGER TYPE

1. A small squirrel jumps from pole 1 to pole 2 in horizontal direction. Squirrels is observed by a very
small observer at origin. If average velocity vector is expressed as v x ˆi  v y ˆj  v z kˆ , then value of
v x  v y  vz  (in unit m/s) is
21

z Pole 2
y Pole 1
15m
50 2 m
20 m
m
50

x
90°
observer
• 50m

2. An old record player of 10 cm radius turns at 10 rad/s while mounted on a 30° incline as shown in
the figure. A particle of mass m can be placed any where on the rotating record. If the least possible
coefficient of friction  that must exist for no slipping to occur is , find 2 3 .

30°

3. A man can just push a box on 37° concrete slope. When he keeps it at the point where the angle
increases to 53°, he can just hold it from sliding back. If the coefficient of friction between the box
1
and the concrete slope is µ, find . Assume that the man is applying same magnitude of force along

the tangent to the curve only.

530

370

4. A projectile is thrown with a velocity of 10 2 m/s at an angle of 45º with horizontal. The square of
time interval between the moments when speed becomes 125 m / s is (g  10m / s2 )

5. All contact surfaces are smooth. The ratio of masses m1 : m2 : m3  7 : 5 : 3 . The force F applied to m1
is finite and constant. Ratio of magnitude of contact force between m2 and m3 to the magnitude of
contact force between m1 & m2 is K. Find the value of 8 K.
F
m1
m2 m3

CENTERS : MUMBAI /DELHI /AKOLA /LUCKNOW /NASHIK /PUNE /NAGPUR /BOKARO /DUBAI # 36
COMPREHENSION TYPE

PARAGRAPH – 1
A lift can move upward or downward. A light inextensible string fixed from ceiling
of lift with a frictionless pulley and tension in string T1 . Two masses of m1 and m2 T1
are connected with inextensible light string and tension in this string T2 as shown in
figure. Read the questions carefully and answer. T2

1. If m1  m2  m and lift moving with constant velocity then value of T1 : m1


m2
(a)  mg (b)  mg (c)  mg (d)  mg

2. If m1 is very small as compared to m2 and lift is moving with constant velocity then value of T2 is
nearly:
(a) m2g (b) 2m1g (c)  m1  m2  g (d) zero
3. If m1  m2 and m1 is moving at a certain instant with velocity v upward with respect to lift is moving
in upward direction with constant acceleration  a  g  then speed of m1 with respect to lift:
(a) increases (b) decreases
(c) remains constant (d) depend upon acceleration on lift

PARAGRAPH – 2
A student having mass 80kg stands on horizontal weighing machine of negligible mass, attached to a
massless platform that slides down a 30 o rough incline plane. The weighing machine read 75kg.
(Take  10m s2 )

1. Calculate the vertical acceleration a y of the student.


5 3 5 4
(a) m sec2 (b) m sec2 (c) m sec 2 (d) m sec 2
8 8 4 5

2. Calculate the acceleration of the student down the slope a s


3 5 5 4
(a) m sec2 (b) m sec 2 (c) m sec2 (d) m sec 2
8 4 8 5

3. Calculate the coefficient of kinetic friction  between the platform and the slope:
3 2
(a) (b) (c) 3 (d) None of these
4 3

CENTERS : MUMBAI /DELHI /AKOLA /LUCKNOW /NASHIK /PUNE /NAGPUR /BOKARO /DUBAI # 37
PARAGRAPH – 3
A projectile is thrown with a velocity of 50ms 1 at an angle of 53o with the horizontal

1. Choose the incorrect statement


(a) It travels vertically with a velocity of 40ms 1
(b) It travels horizontally with a velocity of 30ms 1
(c) The minimum velocity of the projectile is 30ms 1
(d) None of these

2. Determine the instants at which the projectile is at the same height


(a) t  1s and t  7s (b) t  3s and t  5s (c) t  2s and t  6s (d) all the above

3. The equation of the trajectory is given by


(a) 180y  240x  x 2 (b) 180y  x 2  240x (c) 180y  135x  x 2 (d) 180y  x 2  135x

PARAGRAPH – 4

Two projectiles are thrown simultaneously in the same plane from the same point. If their velocities
are v1 and v 2 at angles 1 and  2 respectively from the horizontal, then answer the following
questions.

1. The trajectory of particle 1 with respect to particle 2 will be


(a) a parabola (b) a straight line
(c) a vertical straight line (d) a horizontal straight line

2. If v1 cos 1  v2 cos 2 , then choose the incorrect statement


(a) one particle will remain exactly below or above the other particles
(b) the trajectory of one with respect to other will be a vertical straight line
(c) both will have the same range
(d) none of these

3. If v1 sin 1  v 2 sin 2 , then choose the incorrect statement


(a) the time of flight of both the particles will be same
(b) the maximum height attained by the particles will be same
(c) the trajectory of one with respect to another will be a horizontal straight line
(d) none of these

4. At a given instant, A is moving with velocity of 5m/sec upwards. What is


velocity of B at that instant
(a) 15m / s  (b) 15m / s  (c) 5m / s  (d) 5m / s 

CENTERS : MUMBAI /DELHI /AKOLA /LUCKNOW /NASHIK /PUNE /NAGPUR /BOKARO /DUBAI # 38
ANSWER KEY

SINGLE CHOICE QUESTIONS


1. (D) 2. (D) 3. (A) 4. (A) 5. (C)

6. (D) 7. (C) 8. (D) 9. (B) 10. (D)

11. (A) 12. (B) 13. (C) 14. (D) 15. (A)

16. (D) 17. (D) 18. (B) 19. (B) 20. (B)

21. (B) 22. (B) 23. (B) 24. (B) 25. (C)

26. (A) 27. (C) 28. (B) 29. (A) 30. (B)

31. (B) 32. (C) 33. (B) 34. (A) 35. (C)

36. (B) 37. (D) 38. (D) 39. (D) 40. (B)

41. (C) 42. (C) 43. (B) 44. (B) 45. (B)

46. (B) 47. (B) 48. (A) 49. (A) 50. (A)

51. (C) 52. (A) 53. (A) 54. (C) 55. (B)

56. (C) 57. (C) 58. (C) 59. (B) 60. (B)

61. (D) 62. (C) 63. (C) 64. (B) 65. (B)

66. (A)

MULTIPLE CHOICE QUESTIONS


1. (ABCD) 2. (ACD) 3. (AD) 4. (CD) 5. (A,B,D)

6. (C) 7. (D) 8. (A, B) 9. (A,B,C) 10. (A, B, D)

11. (A,C,D) 12. (C,D) 13. (ABC) 14. (AC) 15. (AC)

16. (AC) 17. (AC) 18. (AB) 19. (ABC) 20. (CD)

21. (BCD) 22. (BD) 23. (AC)

INTEGER TYPE
1. [5] 2. [6] 3. [7] 4. [1] 5. [3]

COMPREHENSION TYPE
1. (C) 2. (B) 3. (C) 4. (A) 5. (B)

6. (A) 7. (A) 8. (D) 9. (A) 10. (B)

11. (C) 12. (D) 13. (A)

CENTERS : MUMBAI /DELHI /AKOLA /LUCKNOW /NASHIK /PUNE /NAGPUR /BOKARO /DUBAI # 39
CRASH COURSE P – I (SOLUTION)

SINGLE CHOICE QUESTIONS

ds
1. v  12  4t
dt
 u = 12 m/ s and
a = –4 m/s 2
u
t0   3s
a
Here t 0 is the ti me when veloci ty becomes zero. Si nce the given time t = 5 s is greater than t 0 . The
distance is greater than di splacement which is,
u2 1 2
d  a  t  t0 
2a 2
144 1
   4 4
2 4 2
= 18 + 8
= 26 m

2. Let acceleration of C i s a (towards right) and relative acceleration of B down the wedge is a r
A bsolute acceleration of A = a r – a (towards left)
Horizontal component of acceleration of B relative to ground
= a r cos 60° + a ...(towards right)
si nce net external force in horizontal directi on is zero.
Therefore.
 ar 
m(a r – a) = 2m   a  + 4m(a)
 2 
or a = 0

3. (1) 30
A
a1 = a2 = g or arel = 0 or vrel = constant = 30 2  40 2 = 50 40
m/s
That means path of one ball will be a straight line parallel to
vrel as seen from the other ball.
Perpendicular distance of vrel from initial position will be the
shortest distance between the balls. C
30 AB = 60 m
Therefore, shortest distance = 60  = 36 m
50 900

4. (1)
120 B
Tension, T = = 60 N
2
60  40
Acceleration of 4 kg block, a1  = 5 m/s2, upwards
4
100  60
Acceleration of 4 kg block, a2  = 4 m/s2, downwards
10

CENTERS : MUMBAI /DELHI /AKOLA /LUCKNOW /NASHIK /PUNE /NAGPUR /BOKARO /DUBAI # 40
a1  a2  5    4 
Therefore, acceleration of pulley, a   = +0.5 m/s2 (+ve sign means upward)
2 2
5. Value of 1 main scale division  0.5 o
30 vernier scale divisions
 29  main scale divisions
 29  0.5o
29
 Value of 1 vernier scale division   0.5o
30
Least count  value of 1 m.s.d  value of 1 v.s.d.
29 0.5o 0.5
 0.5o   0.5o    60 min
30 30 30
 1min
6.

30 30
ma

30

mg f

For along the tangential direction


 
m 3g cos 30  mgsin 30  f
N  m 3g sin 30  mg cos 30
from solving
1

3
7.
Motion will start when F  smg at t = 1s
after that F – kmg = ma
3
dV
3t2 – 2 = a or 3t2 – 2 =  V =  (3t 2  2)dt = 22 m/s
dt 1

( N  4)a
2
8. ( N  8)a
 N  12
 
v. a  0
9.
(t 2  t )(2t  1)  0

10.
B M A
f F F  f  Ma
B f M
T  a
f T 2 2
2
F Mg
T 
2 2

CENTERS : MUMBAI /DELHI /AKOLA /LUCKNOW /NASHIK /PUNE /NAGPUR /BOKARO /DUBAI # 41
1 2
11. x = ut + at
2
1
0 = ut – g sin cos t2
2
gsin
v0

2u 2  24  5  5
 t= = = 10 s ]
g sin  cos  10  3  4

12.

v1  u cos 60o i  (u sin 60o  gt ) j
 
v2  u cos 60o j  (u sin 60o  gt ) k
 
v1 .v2 (u sin 60o  gt )u cos 60 1
cos      0 2 o 2

v1 . v2 (u cos 60 )  (u sin 60  gt ) 2

13. In fig. (a) 1000 – T1 = 50a In fig.(b)


T1 – 800 = 80a 1000 + 500 – T2 = 50a
12, 000
T1 = T2 – 800 = 80a
13
16, 000
T2 =
13

m2
m2
m1
F m F
1

(a) (b)

14. 1msd = 200 × 0.005 = 1mm


2r = 4 × 1 + 25 × 0.005 – 5 × 0.005
= 4.1
r = 2.05 mm]

15. Time taken by the ball to hit the trolley


2u sin 37o
T  1.5sec
g cos 37o
Down the incline plane Vrel  T  Srel as arel  0
(u cos37o  v)  T  6

CENTERS : MUMBAI /DELHI /AKOLA /LUCKNOW /NASHIK /PUNE /NAGPUR /BOKARO /DUBAI # 42
M
16. For maximum value of
m
Mg cos   T  Mg sin 
m
M
Also T   mg 

M   
  
m  sin    cos  

17. By constraint motion


vcos = u
v = u sec 
u=10 m/s

h= 9 m
v

dv  d 
or a = = usectan   
dt  dt 
       
When contact leaves, N=0
i.e. Tsin = mg and Tcos = ma
Tsin T
N

Tcos
a
mg
Tcos =ma and T sin +N = mg
g h
 tan = or a = g cot also = tan
a x
       hcot = x
dx d
So v   h cosec2 
dt dt
d
and a  u sec  tan 
dt
v h cosec2 u sec  h cosec 2
or  or 
a u sec  tan  g cot  u sec  tan 
gh
or tan2 = or tan= 3
u
or  = 60° ]

18. (b)
2 2
Friction force =   s mg cos     mg sin  

CENTERS : MUMBAI /DELHI /AKOLA /LUCKNOW /NASHIK /PUNE /NAGPUR /BOKARO /DUBAI # 43
19. 2N sin   mg  2N cos 

f 

N
12
5

mg
N
2   cos   sin  
= 65

20.
p = 1 q = 5 r = 4 s = 1 so p + q + r + s = 11

21.

vEA  160 j

vEB 120 i

v  120 i  160 j
E

vy 160 4
 tan    
vx 120 3
o
   53
 8o anticlockwise

22. (B)
B A
F2
F1
u u
F1  F2
Fnet on system < Zmg
 Blocks will be at rest
 F1 is greater friction on A will be towards left, the of friction on B depends on magnitudes of
F1 & F2 .
Hence (B)

23. (b)
9
Least court (LC) =1 MSD – 1VSD = 0.05mm – (0.05) mm = 0.005 cm
10
Reading of scale = Diameter = MSR + n ( Least count) – zero error
= [ 1.8 + 6 × (0.005) – 4×(0.005)] cm
D = 1.81 cm

CENTERS : MUMBAI /DELHI /AKOLA /LUCKNOW /NASHIK /PUNE /NAGPUR /BOKARO /DUBAI # 44
24. (B)
2
10 m s

T
5
T  5 10  50
a

200
15 kg
200  50  15  a
2
 a  10 m s
25. (C)

f f

2f

2f
2
20 kg 2ms

200
2
20 kg block will move up with 2 m s
2f  200  20  2
2 f = 240 f = 120

26. (C)
gx 2 y
y = x tan– (1 + tan2)
2u 2
g 2 (, )
  =  tan  – (1+ tan2)
2  2g V

  =  tan – (1+ tan2) 
x
4
 4 
 tan2 – 4 tan +  1    0
 
4  16  4 1  4 /  
tan  =
2
 4 
for real  16 – 4 1    0
 

CENTERS : MUMBAI /DELHI /AKOLA /LUCKNOW /NASHIK /PUNE /NAGPUR /BOKARO /DUBAI # 45
4
 41+

4
3

3
   to hit the target.
4

27. (A)
aB

2 aB
1
1 A
2
a
a
a cos 1 = aB cos 2
a cos 1
aB =
cos  2

28. (D)
Let V be the velocity of the block w.r.t the cart after 2 seconds
1
 V = 6  2 = 12 m /s S=  6  22  12m
2
2 2
 10   2  12  10  cos 37 o w.r.t the car
Vground = 12 
10 kg
= 436
a = g sin 37o = 6 m
1 1 2
ΔK = 10  436   10  10
2 2
3 37 o
ΔK = 1680 = WN + 10  10  12 
5
WN = 960 J

29. (a)
b 2b
Sol. v y  0 at t  T
c c

2ab
Range  aT 
c

30. (b)
Sol. T2 sin   T1 sin 60  w
T1 cos 60o  T2 cos 
T2 sin   T2 cos . tan 60o  w

T2 sin   3 cos   w 
CENTERS : MUMBAI /DELHI /AKOLA /LUCKNOW /NASHIK /PUNE /NAGPUR /BOKARO /DUBAI # 46
2T2  sin 30o sin   cos 30o cos    w
cos  30    in maximum
30    0
  30o

31. (b)
Sol. 4TX A  TX B
4VA  VB
v
VA 
4

32. (c)

33. (b)
Sol. vx  constant No. acceleration.

34. (a)
Sol. The horizontal component of force is Fx  T cos   Ffr . This turn equals ma x . Solve for Ffr to get (a)

35. (c)
  
Sol. VB  VBA  VA

u sin 

 vA  v
u sin  
u cos   v 

36. (b)
Sol. Let m1  m 2  m  constant and m1  x  m2  m  x
x  m  x
a g
m
2x  m  x 
T g
m
m 2
 T
2g
 g  a2 

37. (d)

Sol. u  u cos i  u sin j

v   u cos   i   u sin   gt  j
 
u .v  0

38. (d)

39. (d)

CENTERS : MUMBAI /DELHI /AKOLA /LUCKNOW /NASHIK /PUNE /NAGPUR /BOKARO /DUBAI # 47
40. (b)
6v
Sol. vR 
5
 12m s
2 2
vm  12   5
 13m s

41. (c)
vr v
Sol.  r
sin120 sin 
5 3 5
o

cos 30 sin 
1
sin      30o
2
42. (c) 43. (b)
44. (b)
Sol. 10  fs  2  2  f s  6N  f s  3  a 3kg  a 3kg  2m s2

45. (b)
Sol. For block B,
Taking vertical component
N  Tmax sin 37o  100g N T T
o
N  100g  Tmax sin 37 (1) 100 kg
37 0
Taking horizontal component 25kg
B
Tmax cos37o  N (2) F1   1/3 25g
A

From (1) and (2)


100 g
1000 100g
Tmax   (3)
3 3
For the boy
Tmax  25g  25a max (4)
From (3) and (4)
1000g g
 25g  25a max  a max  m s2 N
3 3
F 0
46. (c) s in
3 0 mg cos30
mg
Sol. When block is being pushed upward F  N
1
mg

F  mgsin 30o   mg cos30o (1)   30 0

When the block is just prevented to slide


F   mg cos 60o  mgsin 60o (2) N FF
1  N
From (1) and (2)
mg sin 30o  mg cos30o  mg cos 60o  mgsin 60o 0

s i n3
0 mg cos30
3 1 mg
 mg
3 1 

CENTERS : MUMBAI /DELHI /AKOLA /LUCKNOW /NASHIK /PUNE /NAGPUR /BOKARO /DUBAI # 48
47. (b)
Sol. 0  v  g t
v
t (1)
g
1
S  ut  at 2 (2)
2
From (1) and (2)
2
v 1  v  v2 v2
Lv   g      L 
g 2  g  g 2g
v2
L  v  2Lg
2g

48. (a)
Sol.   0.1    0.3
Since Fl  mg 10 kg F
  0.1
F11  0.110  g  F1  10N 5 kg
F12  0.3 10  5   10N  F  15N
  0.3
Since F  Fl1 , F  Fl2
Fr2  2N, Fr1  2N

49. (a) Sol. Fr1  2N

50. (a) Sol. F  10N

51. (c) F11


5 kg
Sol. F11  F12 , a max  0
F12
52. (a)
Sol. 30  10  10  a
a  2m s2

53. (a)
Sol. T cos   mg (1)
T  sin   m  lsin  2 (2)  l
T
T  mL2 T cos 
T sin 
54. (c)
mg
2
mv
Sol. N  mg 
R
mv 2
N  mg 
R
 25KN

CENTERS : MUMBAI /DELHI /AKOLA /LUCKNOW /NASHIK /PUNE /NAGPUR /BOKARO /DUBAI # 49
55. (b)
m1x1  m2 x 2 M
Sol. x cm 
m1  m 2
L
m x  m 2 x 2 M/3
x cm  1 1
m1  m2
Let plank be displaced by x towards right to keep
x cm  0 ; Displacement of man will be  L  x  towards left
M
0  M  L  x   x
3
M 4M 3
ML  MX  x  ML  x  x L
3 3 4
 3  L
Displacement of man   L  L  
 4  4

56. (c)
shore
m1x1  m2 x 2 m1x1  m 2 x 2
Sol. x cm  x cm 
m1  m 2 m1  m2 80 kg
8m
200kg

Displacement of C.M. is 0
Displacement of man with respect to boat is 8m. let ward and let x
the displacement of boat is x rightward so displacement man with
respect to ground is  8  x  left ward 20 m

m1x1  m2 x 2  0  80  8  x   200x  0


1600 40
x  x
280 60
16 100
His distance from the sore  20  8    14.3m
7 7

57. (c)
Sol. Centre of mass of triangular lamina is at (h/3, b/3)
Let m1 is the mass of a triangular lamina & m 2 is the mass of
remaining part
m x  m2 x 2
x cm  1 1 0  m1  a 3  m 2 x 2
m1  m 2
ma
x2  1 (1)
3m 2
Since mass is distributed over area a
1  a /3,0  0, x 2 
m1 2  a  2a
 C1  0, x  C2
m2 a 2  a 2
a
1
 (2)
 1
a
x2 
3    1

CENTERS : MUMBAI /DELHI /AKOLA /LUCKNOW /NASHIK /PUNE /NAGPUR /BOKARO /DUBAI # 50
58. (c)
Sol. (Force diagram in the frame of the car)
Applying Newton’s law perpendicular to string
a
mg sin   ma cos  tan  
g
Applying Newton’s law along string
 T  m g 2  a 2  ma T  m g 2  a 2  ma

59. (b)
Sol. l1  l2  l3  l4  C
dl1 dl2 dl3 dl4 l2 l
   0 l3 1
dt dt dt dt
v v0 v2  0 v  2m / s l4 v

60. (b)
2m /s
Sol. at t = 0 b = 8 and at t=4 v=0
v2  u 2  2as
On putting the value 0  64  2  a  s …(1)
But s is area of v-t graph s  1/ 2  4  8  16 …(2)
By equation (1) and (2) 0  64  2  a  16
a=2 but a  g  2
so,  = 2/10 = 0.20

61. (d)
Sol. The friction force on coin just before coin is to slip will be : f   s mg
Normal reaction on the coin : N = mg
The resultant reaction by disk to the coin is
2 2
= N2  f 2   mg     mg 
s
 mg 1   2
9
= 40 103 10  1   0.5 N
16
62. (C)

63. (C)

v speed of man
u
d
u seed of river
v v2  u 2

d
 t=
v  u2
2

2
d t2  v  u 
T= 
vu T 2 v2  u 2
t2 v  u
 
T2 v  u
CENTERS : MUMBAI /DELHI /AKOLA /LUCKNOW /NASHIK /PUNE /NAGPUR /BOKARO /DUBAI # 51
v t 2  T2
 
u t2  T2

64. (B)
N = mg cos N
f 
f = N = mg sin  
 mgcos = mg sin 
 tan =   h
h = R(1 – cos)
 mg
1 
h = R 1  
 1  2 

65. (B)
v2 v
aC = at = at
R
dv v 2
v  aC
ds R
dv 1
  dS
v R
v 2 R
dv 1
   dS
vo
v R 0

v
ln   2
vo
v  vo e 2
dv v 2
 
dt R
v
dv 1 t
  2   dt
vo
v R0
1 1 t
  
 vo v   R
R
t= 1  e 2  
vo

66. (A)
mucos 60o = mv cos 30o
u 60o
 v=
3 30o
u
o u
v cos 60
e= o
2 3
u cos 60 3u
60o
2
1
e=
3
CENTERS : MUMBAI /DELHI /AKOLA /LUCKNOW /NASHIK /PUNE /NAGPUR /BOKARO /DUBAI # 52
MULTIPLE CHOICE QUESTIONS
1. Particle will change its direction at t = T. Slope is constant so, acceleration will be constant.
Total area of graph is zero
v

T
O
2T t

2. For shortest time


v
d d
t u
v
For reaching p
d u
t
v  u2
2
v2  u 2
If u > v then zero drift is not possible v

 F
3. a cm  ext
M
If fext = 0
acm = 0
Vcm = constant

4. By conservation of linear momentum


MV0  mv
mv H
V0 
M
M v
again MV0  mv  (m  M)v' m
V0

mv m
M  mv  (m  M)v' V
M V0

M V M
2mV m
 V'
m M
MV0  mV 2mV
Vcm   .
mM mM

5. From constraint relation magnitude of acceleration of block a & b will be same, block A will move
down ward and B will be move horizontally. ]

6. Time depends only on y-direction (perpendicular to river flow) velocity which is same for all the
four

CENTERS : MUMBAI /DELHI /AKOLA /LUCKNOW /NASHIK /PUNE /NAGPUR /BOKARO /DUBAI # 53
7.

Suppose only block (A) and (C) move


mg
2T – = ma
2
2mg – T = 2m . 2a
----------------------
3.5mg = 9 ma
7 7
a= g & 2a = g
18 9
7g 4 mg
T = 2mg – 2m. = mg < ]
9 9 2

8. magnitude of displacement
R 3v
vavg  
R / 3v 

Magnitude of change of velocity = 2v sin
6
3v 2
a avg 
R


3
v

9. (A,B,C)

10. Velocity of shadow on ground is which will remain constant with time

11. (A,C,D)

12. (C,D)
direction of friction does not change
friction is constant.

13. (A, B, C)
 20 ˆ  400 40
VA  VB  VAB  20iˆ  j VAB  400  
3 3 3

CENTERS : MUMBAI /DELHI /AKOLA /LUCKNOW /NASHIK /PUNE /NAGPUR /BOKARO /DUBAI # 54
30 0 60
30
100 3

1002  3  100 2  200 100

100 3 300
t   7.5 s
40 40
3

14. (A, C)
 
u  8i a  1.2i  0.9j
  
v  u  a t  8i  4.8j  3.6j  12.8iˆ  3.6ˆj
3.6 9   1 2
tan    s  u.t  a t
12.8 32 2
 32iˆ  9.6iˆ  7.2ˆj  41.6 iˆ  7.2ˆj

15. (A, C)
R cos 

R sin 
mg

R sin   1 5 R cos   10
1 5 5
tan    R  5 5
2 sin  1
5

16. (A, C)


v

ˆ
v i  vi v f  v cos  ˆi  v sin  ˆj
   
v  Vf  Vi V  ˆi  v cos   v   vsin  ˆj
 2
 
V   v cos   v   v 2 sin 2  V  2vsin
2
CENTERS : MUMBAI /DELHI /AKOLA /LUCKNOW /NASHIK /PUNE /NAGPUR /BOKARO /DUBAI # 55
17. (a, c) 18. (a,b) 19. (a,b,c) 20. (c,d) 21. (b,c,d)

22. (b, d)
Sol. Case I :
Since, no relative motion :
F  Ff Ff 8
a 1   F1 max   F!
5 3 3
Case II :
F F  Ff 8 F1max  5
a f  2  F2 max  F! Clearly : F1max   F2 max  and 
5 3 5 F2max  3

23. (a, c)
2x 2Lcos 45o x
Sol. t 
2v v
L L x
t v BA  2v
v
A v L B

INTEGER TYPE
2h 25 x 50
1. t= = =1s vx =  = 50 m/s
g 10 t 1
y 50 z
vy =  = 50 m/s vz = = 5 m/s
t 1 t
x y
15

50 20

x
50
 Ans. 105 ]

2.

2
mg sin  + mrw

[Ans. 0006]
f  mg sin  + mr2
µ mg cos  mg sin  + mr2
2 r
or µ  tan  + ]
g cos 

3. [0007]

CENTERS : MUMBAI /DELHI /AKOLA /LUCKNOW /NASHIK /PUNE /NAGPUR /BOKARO /DUBAI # 56
4. (1)

5. (3)
F
m1  7k, m2  5k, m3  3k a , for m3 , N3  m3a
15k
3kF F
 N3  
15k 5
F F
For m2 , N2  N3  5k  
15k 3
8F N3 3
 N2     k,
15 N2 8
8k = 3
COMPREHENSION TYPE

PARAGRAPH – 1

1. (c) 2. (b)
3. (c)
Sol. if lift has acceleration zero, then the acceleration of centre of mass of the two blocks is downwards
 T1   m1  m 2  g
If masses are equal then no acceleration of centre of mass.
 T   m1  m 2  g
T1

a T2
m1 m2

In frame of lift

T2
 T2  m1g  m1a 0  m1a

m1a 0
mg
T2
 m2g  m2a 0  T2  m2a
a

m2g m2a 0
2  m1m2  g  a 0 
Solving  T2 
m1  m2
For m1  m2
If m1  m2  No lift acceleration of blocksurt
 Relative velocity remains constant

CENTERS : MUMBAI /DELHI /AKOLA /LUCKNOW /NASHIK /PUNE /NAGPUR /BOKARO /DUBAI # 57
PARAGRAPH – 2
1. (a)
Sol. mg  N  ma y
80g  75g  80a y
5
a y  m sec2
8
2. (b)
Sol. a s sin 30 o  a y

a s  m sec2
4

3. (a)
5
Sol. mg sin 30o  mg cos30o  m  
 4
5
5   5 3
4
3
  0.43
4

PARAGRAPH – 3
1. (a)

2. (d)
Sol. For same height
1 1
y  u yt  a yt2  y  u y t  gt 2
2 2
2
2u y  4u 2y  8gh
gt  2u y t  2h  0  t
2g
2
u y  u  2gh
y u y  u 2y  2gh
t1  , t2 
h g
o
2u y 2  50  sin 53
t1  t 2    8sec .
g 10

3. (a)
4
Sol. u x  u cos   30m s , u y  u sin   40 m s , tan  
3
x  u cos  t (1)
1
y  sin t  gt 2 (2)
2
1 gx 2
From (1) & (2) y  x tan  
2  u cos  2

CENTERS : MUMBAI /DELHI /AKOLA /LUCKNOW /NASHIK /PUNE /NAGPUR /BOKARO /DUBAI # 58
4 gx 2 4 4 gx 2 4x x 2
y x 2  y x  y 
3 2   30  3 3 1800 3 180
180y  240x  x 2

PARAGRAPH – 4
1. (b)

2. (c)
Sol. V12 x   v1 cos 1  v 2 cos 2
a12 x   0  v12 y  V1 sin 1  V2 sin 2  a12 y  0
x  v12 y t (1)
y 12 y t (2)
v12 y 
From (1) & (2) y
v12 y 
3. (d)
Sol. y  0 x
  90o
For same range U x1 U y1  Vx1Vy1

4. (a)
v 21 y  v  sin  2  sin 1 
Sol. x  y x
v 21 x  v  cos  2  cos 1 
sin 2  sin 1
y x
cos 2  cos 1

CENTERS : MUMBAI /DELHI /AKOLA /LUCKNOW /NASHIK /PUNE /NAGPUR /BOKARO /DUBAI # 59
WORK, ENERGY AND POWER

CONCEPT SUMMARY

 MOTION IN VERTICAL CIRCLE:

Body of mass m tied to a string on a circular path, with velocity V as shown

(i) v  5gr , body completes a vertical circle.


(ii) 2 gr  u  5gr , particle leaves circle between C & B.
(iii) u< 2 gr , particle oscillates about A .
When body is at the end of rod, velocities are 4 gr & 2 gr . in case (i) & (iii) above

 WORK ENERGY & POWER

CONCEPT SUMMARY

1. The work-energy theorem states that the change in kinetic energy of a body is the work done by the
net force on the body.
K f  K i  Wnet

2. A force is conservative if (i) work done by it on an object is path independent and depends only on
the end points x i , x j , or (ii) the work done by the force is zero for an arbitrary closed path taken by
the object such that it returns to its initial position.

3. For a conservative force in one dimension, we may define a potential energy function V(x) such that
dV  x 
Fx  
dx
xf

Or Vi  Vf   F  x  dx
xi

4. The principle of conservation of mechanical energy states that the total mechanical energy of a body
remains constant if the only forces that act on the body are conservative.

5. The gravitational potential energy of a particle of mass m at a height x about the earth’s surface is
V x  m g x
where the variation of g with height is ignored.

CENTERS : MUMBAI /DELHI /AKOLA /LUCKNOW /NASHIK /PUNE /NAGPUR /BOKARO /DUBAI # 60
6. The elastic potential energy of a spring of force constant k and extension x is
1
V x   k x2
2

7. The scalar or dot product of two vectors A and B is written as A.B and is a scalar quantity given by:
A.B = AB cos  , where  is the angle between A and B. It can be positive, negative or zero
depending upon the value of  . The scalar product of two vectors can be interpreted as the product
of magnitude of one vector and component of the other vector along the first vector. For unit vectors:
ˆi.iˆ  ˆj.jˆ  k.k
ˆ ˆ  1 and ˆi.jˆ  ˆj.kˆ  k.i
ˆˆ0
Scalar products obey the commutative and the distributive laws.

WORK, ENERGY, POWER

FORMULA SUMMARY

 Work –Energy Theorem :


w =  F.S
Work done by external forces on a system gives change in KE of system.
Work done in Cartesian coordinate system is
x2 y2 z2
w=   
Fx dx  Fy dy  Fz dz
x1 y1 z1

work done is also area under force – displacement graph


 Conservative Force: -
Work done along a closed path is zero. Work done depends only on initial & final position.

 To find if a given force field is conservative or not:



Given : F( x , y,z )  f1 ( x, y, z) î  f 2 (x, y, z, ) ĵ  f 3 ( x, y, z, ) k̂
It is conservative iff.

f 2 f1 f1 f 3 f 2 f 3
 ,  ; 
x y z x z y

 If in a problem, velocities of bodies are asked, use energy conservation, and other laws which give
velocity directly.
If acceleration is asked , use Newton’s laws.

 Power:

dW  
P  F .V
dt
T
 .dt W
Pav  
0
T

T

water of density , flows out with velocity v and cross sectional area of pipe is A, then
force = Av 2 , power = Av 3

 If a body is in equilibrium, it may have non zero velocity.


CENTERS : MUMBAI /DELHI /AKOLA /LUCKNOW /NASHIK /PUNE /NAGPUR /BOKARO /DUBAI # 61
CONSERVATION LAWS

FORMULA SUMMARY

 Momentum: Absence of external force.


 Mechanical energy: Absence of dissipative force.
 Angular Momentum: Absence of external torque.

In momentum conservation, small forces can be neglected if time interval considered is small.
e.g. In collisions effect of gravity is usually neglected

Centre of Mass :

(1) Semicircular wire : 2R/  from diameter


(2) Semicircular plate : 4R/3  from diameter.
(3) Hollow hemisphere : R/2 from base
(4) Solid Hemisphere : 3R/8 from base.
(5) Hollow cone. H/3 from base
(6) solid cone. H/4 from base
(7) Triangle : Centroid

COLLISIONS

 Momentum is conserved.
 In ‘elastic collisions’, energy is conserved.

Velocityof separation  v1  v 2 


e= =
Velocityof approach u l  u 2 
Resolve velocities of the bodies at the point of collision before &after collision, along the common
normal to the surface of collision & use definition of e.
0  e  1.
e =1 : Perfectly elastic collision
e=0 : Perfectly inelastic collision.
For head on collision between two bodies of mass m1& m2 moving in same direction ( u1 , u2 ), with
co-efficient of restitution ‘e’,
 
u1 u2
m1 m2

 m  em 2   m 1  e  
V1   1  u1   2  u 2
 m1  m 2   m1  m 2  

 m 1  e    m  em 1 
V2   l  u 1   2  u 2
 m1  m 2   m1  m 2 

CENTERS : MUMBAI /DELHI /AKOLA /LUCKNOW /NASHIK /PUNE /NAGPUR /BOKARO /DUBAI # 62
Note: As mentioned before, velocities are to be taken along common normal while using ‘e’. For this, find
point of contact & draw the normal at that point.
For perfectly elastic collision, with e=1

m1  m 2  u  2m 2 u . 2m 1 m  m1 
V1  V2  u1  2 u
m1  m 2  1 m1  m 2 2 m1  m 2 m1  m 2  2

2m 1 m 2 1 m m 
Impulse = u 1  u 2  Loss of kinetic energy =  1 2  ( u 1  u 2 ) 2 (1  e 2 )
m1  m 2 2  m1  m 2 
(1) If bodies are identical, velocities are interchanged
(2) If. m2 .>> m1, its velocity does not change & other body is reflected approximately at the same
velocity
(3) If m1 is massive & m2 is at rest,
v1 = u1 & v2 = 2u1

Note:
(1) While solving simple questions where energy is conserved, use e=1, instead of energy conservation.
(2) But, in case of rotation & complicated situations, conserve energy, rather than using e =1
(3) As momentum & angular momentum are vector quantities they are conserved separately along three
axes & these equations can be used.
(4) For oblique collision of equal masses , when e = 1, they fly off at 90°

SINGLE CHOICE QUESTIONS

1. A ball of mass 1 kg is released from position A inside a wedge with a hemispherical cut of radius 0.5
m as shown in figure. The force exerted by the vertical wall MO on wedge, when the ball is in
position B is : (all surfaces are smooth) (g = 10 m/s2) :
M

A
C

600
B

O N

5 3 15 3
(a) 10 N (b) N (c) N (d) 7.5 N
2 2

2. v-t graph of an object of mass 1 kg is shown. Select the wrong statement :


(a) Work done on the object in 30 s is zero
(b) The average acceleration of the object is zero 20
(c) The average velocity of the object is zero
(d) The average force on the object is zero v(m/s) 10

30
10 20 t (s)
CENTERS : MUMBAI /DELHI /AKOLA /LUCKNOW /NASHIK /PUNE /NAGPUR /BOKARO /DUBAI # 63
3. An observer holds one end of a uniform chain of mass M such that lower end of the chain just
touches a horizontal floor. The observer releases the chain from this position. Just before last link of
the chain strikes the floor, force applied by chain on the floor is
(a) Mg (b) 3Mg (c) 4Mg (d) infinite

4. Two blocks of masses 3kg and 6kg are connected to the ends of a spring of spring constant k = 50
N/m. The whole system is kept at rest on a smooth horizontal floor. Suddenly 3kg block is given a
velocity 10 m/s away from 6kg block. Maximum elongation in spring will be
(a) 6 m (b) 2m (c) 2 3 m (d) 1m

5. The potential energy of a 1kg particle free to move along the x-axis is given by
 x4 x2 
V  x      joule
 4 2 
The total mechanical energy of the particle is 2J.
Then, the maximum speed (in m/s) is
1 3
(a) (b) 2 (c) (d) 2
2 2

6. A circular disc of radius R is removed from a bigger circular disc of radius 2R such that the
circumferences of the disc touch. The centre of mass of the new disc is R from the centre of the
bigger disc. The value of  is
1 1 1 1
(a) (b) (c) (d)
3 2 6 4

7. A block of mass 2kg is free to move along the x-axis. It is initially at rest. From t  0 onwards it is
subjected to a time-dependent force F  t  varies with t as shown in figure. The kinetic energy of the
block after 4.5 seconds is
Ft

4N

4.5s
O 3s t

(a) 4.50J (b) 7.50J (c) 5.06J (d) 14.06J


8. A small block of mass m is lying at rest at point P of a wedge having a smooth semi circular track of
radius R. The minimum value of horizontal acceleration a 0 of wedge so that mass can just reach the
point Q?
Q

a0 P

(a) g/2 (b) g (c) g (d) not possible

CENTERS : MUMBAI /DELHI /AKOLA /LUCKNOW /NASHIK /PUNE /NAGPUR /BOKARO /DUBAI # 64
9. A uniform body of mass M and radius R has a small mass m attached at edge as shown in the figure.
The system is placed on a perfectly rough horizontal surface such that mass m is at the same
horizontal level as the centre of body. It is assumed that there is no slipping at point A. If I A is the
moment of the inertia of combined system about point of contact A then the normal reaction at point
A just after the system is released from rest is (M = 6kg, m = 2kg, IA= 4kg m2, R = 1m, g = 10m/s2)

M
m

A
(a) 60 N (b) 80 N (c) 75 N (d) 70 N

10. A satellite is moving on a circular orbit around a planet. A


sharp impulse is given to the satellite such that it escapes P2
from the gravitational field of the planet. The impulse can P1
be given in two different ways as shown. The ratio of the
P 
impulse given in the two different cases  1  is
 P2 
(a) 2 (b) 2 (c) 2 1 (d) 2 1

11. Two balls A and B having masses 1 kg and 2 kg, moving with speeds 21 m/s and 4 m/s respectively
in opposite directions, collide head on. After collision A moves with a speed of 1 m/s in the same
direction, then the incorrect statement is
(A) The velocity of B after collision is 6 m/s opposite to the direction before collision
(B) The coefficient of restitution is 0.2
(C) The loss in kinetic energy due to collision is 200 J
(D) The impulse of the force between the two balls is 40 Ns

12. A flat car of mass m0 , starts moving on a smooth horizontal plane due to a constant horizontal force
F. Sand spills on the flat car from a stationary hopper and sticks to the car. The rate of loading is
constant and is equal to  kg s . The time dependence of the velocity of car is

Ft Ft 2 F  m0  2Ft
(A) (B) (C) ln (D)
m0  t  m0  t  2   m 0  t  m0  t

CENTERS : MUMBAI /DELHI /AKOLA /LUCKNOW /NASHIK /PUNE /NAGPUR /BOKARO /DUBAI # 65
13. Two objects with masses m1 and m2 are moving along the x- axis in the positive direction with
speed v1 and v2 , respectively, where v1 is less then v2 . The speed of the centre of mass of this
system of two bodies is
(a) less then v1 (b) equal to v1
(c) equal to the average of v1 and v2 (d) greater than v1 and less than v2

14. A 50 kg boy runs at a speed of 10 m s and jumps onto a cart as shown in the figure. The car is
initially at rest. If the speed of the cart with the boy on it is 2.50 m s . What is the mass of the cart?
2.5 m/s

10 m/s
v=0

(a) 150 kg (b) 210 kg (c) 175 kg (d) 260 kg

15. A body of mass m accelerates uniformly from rest to a speed v 0 in time t 0 . The work done on the
body till any time t is
3
1  t2  1 t   t   t 
(a) mv0 2  2  (b) mv0 2  0  2
(c) mv0   (d) mv 0  2

2  t0  2  t   t0   t0 

16. A man who is running has half the kinetic energy of the boy of half his mass. The man increases his
speed up by 1m/s and then has the same kinetic energy as the boy. The original speed of the man was
(a) 2 m s 
(b) 2  1 m s  (c) 2 m s (d) 2  1 m s  
17. A rope of length l and mass ‘m’ is connected to chain of length l and mass 2m and hung vertically as
shown in figure. What is the change in gravitational potential energy if the system is inverted and
hung from same point ?

l
m

2m l

(a) mg l (b) 4mg l (c) 3mg l (d) 2mg l



The work done by force F  x i  y j around the path shown in the figure is-
2 2
18.
Y

C B (a, a)
(0, a)

O A
x
(0, 0) (a, 0)
2 4 3
(a) a 3 (b) zero (c) a 3 (d) a
3 3
CENTERS : MUMBAI /DELHI /AKOLA /LUCKNOW /NASHIK /PUNE /NAGPUR /BOKARO /DUBAI # 66
PARAGRAPH FOR QUESTION NO. 19 TO 21

A small ball B of mass m is suspended with light inelastic string of length L from a block A of same
mass m which can move on smooth horizontal surface as shown in the figure. The ball is displaced
by angle  from equilibrium position & then released.

L

L
u 0
B

19. The displacement of block when ball reaches the equilibrium position is
L sin 
(a) (b) Lsin  (c) L (d) none of these
2

20. Maximum velocity of block during subsequent motion of the system after release of ball is
12 12
(a) gl 1  cos    (b)  2g11  cos   
12
(c)  g1cos  (d) information insufficient to decide

21. The displacement of centre of mass of A  B system till the string becomes vertical is
L L
(a) zero (b) 1  cos   (c) 1  sin  (d) none of these
2 2

22. Three blocks are initially placed as shown in figure. Block A has mass m and initial velocity v to the
right. Block B with mass m and block C with mass 4m are both initially at rest. Neglect friction. All
collisions are elastic. The final velocity of block A is
v
A B C
m m m
(a) 0.6v to the left (b) 1.4v to the left (c) v to the left (d) 0.4v to the right

23. As shown in the figure a body of mass m moving vertically downwards with speed 3m/s hits a
smooth fixed inclined plane and rebounds with a velocity v f in the horizontal direction. If angleof
inclined is 30o , the velocity v f will be

vf
m
30 0

(a) 3m/s (b) 3m s (c) 1 3m s (d) this is not possible

CENTERS : MUMBAI /DELHI /AKOLA /LUCKNOW /NASHIK /PUNE /NAGPUR /BOKARO /DUBAI # 67
24. Two massless string of length 5m hang from the ceiling very near to each other as shown in the
figure. Two balls A and B of masses 0.25kg and 0.5kg are attached to the string. The ball A is
released from rest at a height 0.45 m as shown in the figure. Immediately after the collision, the
kinetic energy of ball B is 1J. The velocity of ball A just after the collision is (g = 10 m/s2)

0.45m A B
1
(a) 5ms to the right (b) 5ms 1 to the left
(c) 1ms  1 to the right (d) 1ms  1 to the left

25. The system of the wedge and the block connected by a massless spring as shown in the figure is
released with the spring in its natural length. Friction is absent, maximum elongation in the spring
will be
k
M

M
37 0

3Mg 6Mg 4Mg 8Mg


(a) (b) (c) (d)
5k 5k 5k 5k

26. The blocks A and B shown in the figure have masses M A  5 kg and
M B  4 kg . The system is released from rest. The speed of B after A
has travelled a distance 1 m along the incline is
A 5m

37 0
B
3 3 g g
(A) g (B) g (C) (D)
2 4 2 3 2

27. The potential energy for a force field F is given by U (x, y) = cos (x + y). The force acting on a
 
particle at a position given by coordinates  0,  is
 4
1   1   1 3  1 3 
(a)  
i j  (b)  
i j (c)  i  j  (d)  i  j
2 2 2 2  2 2 

28. Two masses ‘m’ and ‘2 m’ are placed in fixed horizontal circular Smooth
hollow tube as shown. The mass ‘m’ is moving with speed ‘u’ and the
mass ‘2 m’ is stationary. After their first collision, the time elapsed for 2m
next collision. (co efficient of restitution e = 1/2)
2r 4r 3r 12r
m u
(A) (B) (C) (D)
u u u u

CENTERS : MUMBAI /DELHI /AKOLA /LUCKNOW /NASHIK /PUNE /NAGPUR /BOKARO /DUBAI # 68
29. A bob is attached to one end of a string and other end of which is fixed at Peg
A. The bob is taken to a position were string makes an angle of 30o with the
horizontal. On the circular path of the bob in vertical plane there is a peg ‘B’
at a symmetrical position with respect to the position of release as shown in
the figure. If Vc and Va be the minimum speeds in clockwise and
anticlockwise directions respectively, given to the bob in order to hit the peg
‘B’ then ratio Vc : Va is equal to :
(A) 1 : 1 (B) 1 : 2 (C) 1 : 2 (D) 1 : 4

30. A train of mass M is moving on a circular track of radius ‘R’ with constant speed V. The length of
the train is half of the perimeter of the track. The linear momentum of the train will be

2MV
(A) zero (B) (C) MVR (D) MV

31. A canon shell moving along a straight line bursts in to two parts. Just after the burst one part moves
with momentum 20 Ns making an angle 30o with the original line of motion. The minimum
momentum of the other part of shell just after the burst is :
(A) 0 Ns (B) 5 Ns (C) 10 Ns (D) 17.32 Ns

32. In the system shown, the mass m = 2 kg oscillates in a circular arc of amplitude 60o , the minimum
value of coefficient of friction between block of mass 8 kg and surface of table to avoid slipping is
8 kg 

60o

2 kg

(a) 0.25 (b) 0.50 (c) 0.40 (d) None of these

33. A particle is hanging from a fixed point O by means of a string of length a. There is a small smooth
nail O' in the same horizontal line with O at a distance b (< a) from O. The minimum velocity with
which particle should be projected from its lowest position in order that it may make a complete
revolution round the nail without string becoming slack.
O b O'

(a) 3gl (b) 5gl (c)  5a  3b  g (d)  5b  3a  g

CENTERS : MUMBAI /DELHI /AKOLA /LUCKNOW /NASHIK /PUNE /NAGPUR /BOKARO /DUBAI # 69
34. The system shown is released from rest. Mass of ball is m kg and that of wedge is M kg. When ball
reaches at highest point on other side of the wedge, velocity of ball and wedge, is (initially wedge is
kept at rest against a wall)
m
R
M

smooth

M 2gR M 2gR M m 2gR


(a) (b) (c) 2gR (d)
mM M m mM

35. Two identical spheres A, B are in a smooth horizontal circular groove at opposite ends of a diameter.
A is projected along the groove and impinges on B in a time interval T. Let e be the coefficient of
restitution, then

A B
a a

(a) The second impact will occur at the same time T


2T
(b) The second impact will occur after time
e
(c) The second impact will occur at the time 2T
(d) None of these

36. A smooth track is shown in figure. A block of mass M is pushed against a spring of spring constant
K fixed at the left end and is then released. Find the initial compression of the spring so that the
normal reaction at point P is zero

P Circular part
Radius = R
M

3MgR 2MgR MgR MgR


(a) (b) (c) (d)
K K 4K K

37. A sphere of mass m falls on a smooth hemisphere of mass M resting with its plane face on smooth
horizontal table, so that at the moment of impact line of centres makes an angle  with the vertical.
The velocity of sphere just before impact is u and e is the coefficient of restitution
(a) The velocity of hemisphere after impact will be zero
mu sin 
(b) The hemisphere will move with a velocity v 
m cos2 
mu sin  cos  1  e 
(c) The hemisphere will move with a velocity v 
 M  m sin 2  
(d) None of these

CENTERS : MUMBAI /DELHI /AKOLA /LUCKNOW /NASHIK /PUNE /NAGPUR /BOKARO /DUBAI # 70
38. A heavy particle hanging vertically from a fixed point by a light inextensible cord of length l is
struck by a horizontal blow which imparts to it a velocity 2  gl  . Then
3l
(a) The cord become slack when the particle has risen to a height above the fixed point
4
2l
(b) The cord becomes slack when the particle has risen to a height above the fixed point
3
l
(c) The cord becomes slack when particle is at a height above the fixed point
3
(d) None of these

39. A block of mass 0.5 kg is pulled by 12 N force on a fixed block. Speed of block is constant. Find
contact force applied by lower block on upper block
m = 0.5 kg F = 12 N

(a) 12 N (b) 5 N (c) 13 N (d) 17 N

40. A heavy particle hangs from a point O, by a string of length a; It is projected horizontally with a
velocity v   2  3  ag . The angle with downward vertical, string makes when it becomes stack is
 1   1   1   1 
(a)   sin 1   (b)   cos 1   (c)   cos1   (d)   sin 1  
 3  3  2  2

41. A ball of mass M strikes another ball of mass m at rest. If they separate in mutually perpendicular
directions, then the coefficient of restitution (e) is
M m m
(a) (b) (c) (d) Zero
m M 2M

42. An object of mass 40 kg having velocity 4iˆ m/s collides with another object of mass 40 kg having
velocity 3ˆj . If the collision is perfectly inelastic, then the loss of mechanical energy
(a) 250 J (b) 100 J (c) 125 J (d) 35 J

43. A ball A moving with momentum 2i  4 j collides with ball B moving with momentum 6 ˆj . After
collision momentum of ball B is 10jˆ . Which of the following statement is correct?
(a) After collision momentum of A is 4iˆ
(b) After collision momentum of A is parallel to momentum of B
(c) After collision momentum of A is 2iˆ
 
(d) If PA and PB are momentum of balls A and B, then   p A  p B   0

CENTERS : MUMBAI /DELHI /AKOLA /LUCKNOW /NASHIK /PUNE /NAGPUR /BOKARO /DUBAI # 71
44. Two blocks of same mass (4 kg) are placed according to diagram. Initial velocities of bodies are 4
m/s and 2 m/s and the string is taut. Find the impulse on 4 kg when the string again becomes taut
4 m/s
4 kg

4 kg 2 m/s

(a) 24 N-s (b) 6 N-s (c) 4 N-s (d) 2 N-s

45. A uniform ring, having radius a and mass m is to be rotated in the horizontal plane about its own axis
with constant angular velocity  . What would be the tension in the ring and nature of force?
MR2 MR2
(a) tensile (b) MR 2 tensile (c) compressive (d) MR2 compressive
2 2

46. A loaded spring gun of mass M fires a ‘shot’ of mass m with a velocity v at an angle of elevation  .
The gun is initially at rest on a horizontal frictionless surface. After firing the centre of mass of the
gun-shot system
vm
(a) Moves with a velocity
M
vm
(b) Moves with a velocity in the horizontal direction
M cos 
mvsin 
(c) Moves with a velocity along vertical
Mm
(d) Moves with velocity v  M  m  M  m  in horizontal direction

47. A particle of mass m moves on the x-axis under the influence of a force of attraction towards the
 k
origin is given by F  2 ˆi . If the particle starts from rest at x = a, the speed it will attain to reach the
x
point x = x will be
1 1 1
2k  a  x  2
2k  a  x  2
k  ax  m a  x  2
(a) (b) (c) (d)
m  ax  m  ax  m  a  x  2k  ax 

48. A body of mass M moving with a speed u has a ‘head on’ collision with a body of mass m originally
at rest. If M > > m, the speed of the body of mass m after collision, will be nearly
(collision is perfectly elastic)
um uM u
(a) (b) (c) (d) 2 u
M m 2
49. A man of mass M stands at one end of the plank of length L which lies at rest on a frictionless
M
surface. The man walks to the other end of the plank. If the mass of plank is , the distance that the
3
man moves relative to the ground is
3L L 4L L
(a) (b) (c) (d)
4 4 5 3
CENTERS : MUMBAI /DELHI /AKOLA /LUCKNOW /NASHIK /PUNE /NAGPUR /BOKARO /DUBAI # 72
50. A block of mass m is attached to four unscratched massless spring of spring constant k1 and k 2 as
shown in figure. The block is displaced towards right through distance x and is released. Speed of
block when displacement of block is x/2 from mean position is
k1 k2

k1 k2

x2 3  k1  k 2  x 2  k 1k 2  x 2  k1  k 2  x 2
(a) 2  k1  k 2  (b) (c) (d)
m 2m 3  k1  k 2  m  k1  k 2  m

51. A small body of mass m is located on a horizontal plane at the point O. The body acquires a
horizontal velocity u. the mean power developed by the friction force during the whole time of
motion if the friction coefficient is 
1 1 mug 2mug
(a) mug (b) (c) 2 mug (d)
2 2  

52. A stationary body explodes into two fragments of masses m1 and m 2 . If the linear momentum of one
of the fragments is p, the minimum energy of explosion is
p2 p2 p2  m1  m2  p2
(A) (B) (C) (D)
2  m1  m2  2 m1m 2 2m1m 2 2  m1  m2 

53. In figure shown, ‘car one’ is traveling due to north and ‘car two’ is traveling due east. They collide
and stick to each other. 1,2,3,4 and 5 are shown possible direction of motion of cars. Which is most
likely?
(3)
(2) (4)

(1)
(5)

Two
One

(a) 2 (b) 3 (c) 4 (d) 5

54. A man pulls a bucket of water from a depth of h from a well. If the mass of the rope and that bucket
of water is m and M respectively, the work done by the man is
Mm  m M 
(a)  M  m  gh (b)   gh (c)  M   gh (d)   m  gh
 2   2  2 
55. A man pushes wall and fails to displace it. He does
(a) Negative work (b) Positive but not maximum work
(c) No work at all (d) Maximum work
CENTERS : MUMBAI /DELHI /AKOLA /LUCKNOW /NASHIK /PUNE /NAGPUR /BOKARO /DUBAI # 73
56. The work done in slowly pulling up a block of wood weighing 2 KN for a length of 10 m on a
smooth plane inclined at an angle of 30 0 with the horizontal by a force parallel to the incline is
(a) 20 kJ (b) 15 kJ (c) 10 kJ (d) 16 kJ

MULTIPLE CHOICE QUESTIONS

1. Two particles A and B, of mass m each, are joined by a rigid massless u


A(m)
rod of length l. A particle P of mass m, moving with a speed u normal P(m)
to AB, strikes A and sticks to it. The centre of mass of the ‘A + B + P’
system is C.
(a) The velocity of C before impact is u/3
(b) The velocity of C after impact is u/3 B(m)

(c) The velocity of ‘A + P’ immediately after impact is u/2.


(d) The velocity of B immediately after impact is zero.

2. The potential energy U for a force field F is such that U = -kxy, where k is a constant.
 
(a) F  kyiˆ  kxjˆ (b) F  kxiˆ  kyjˆ
 
(c) The force F is a conservative force (d) The F is a non-conservative force

3. A ball of mass m is attached to the lower end of a light vertical spring of force constant k. The upper
end of the spring is fixed. The ball is released from rest with the spring at its normal (unstretched)
length, and comes to rest again after descending through a distance x.
(a) x = mg/k
(b) x = 2mg/k
(c) The ball will have no acceleration at the position where it has descended through x/2.
(d) The ball will have an upward acceleration equal to g at its lowermost position.

4. The potential energy of a particle of mass 0.1 kg moving along the X-axis is given by
U  5x  x  4 J , when X is in meters. It can be concluded that:
(a) the particle is acted upon by a constant force
(b) the speed of the particle is maximum at x = 2 m
(c) the particle executes simple harmonic motion
(d) the period of oscillation of the particle is  / 5 seconds
2 2
5. The potential energy function of a particle moving in the one dimension is U  kx 2e  x a where a
and k are constant then
(a) equilibrium is at x  a 2
(b) at x  0 , equilibrium is stable
(c) at unstable equilibrium potential energy is ka 2 e
(d) at x  a ,equilibrium is unstable

6. In the following K  kinetic energy, U  potential energy, and W  work. In using the expression
K 2  U 2  K1  U1  Wother , we recognize that
(a) We are assuming no friction is present
(b) Gravitational and elastic potential energies would be included in U1 and U 2
(c) K and U can be either positive or negative, depending on the reference point chosen.
(d) Wother can be positive or negative

CENTERS : MUMBAI /DELHI /AKOLA /LUCKNOW /NASHIK /PUNE /NAGPUR /BOKARO /DUBAI # 74
7. An object of mass 3m , initially at rest on a frictionless horizontal surface, explodes breaking into two
fragments of mass m and 2m, respectively. Which one of the following statements after the
explosion is true?
(a) Velocity of center of mass increases
(b) Speed of smaller fragment will be twice that of larger fragment
(c) Fragments have equal magnitude of momentum in ground frame but different magnitude of
momentum in center of mass frame.
(d) Kinetic energy of system increases

8. A uniform rod of length l is falling down, with a velocity v0 when one of its end hits a fixed edge as
shown

(a) If the collision is elastic, the centre of mass may have upwards velocity just after the collision
(b) If the collision is partially elastic, the centre of mass may come to rest just after the collision
(c) If the collision is perfectly inelastic, the centre of mass has a downward velocity just after the
collision,
(d) If the collision is elastic, the centre of mass has a downward velocity just after the collision.

9. A particle A suffers an oblique elastic collision with a particle B that is at rest initially. If their
masses are the same, then, after the collision
(a) B moves along line of impact
(b) velocities of A and B will be interchange along line of impact
(c) they will move in mutually perpendicular directions
(d) they must move in symmetric directions with respect to the initial line of motion of A

10. The potential energy in joules of a particle of mass1kg moving in a plane is givenby U  3x  4y ,
the position coordinates of the point being x and y, measured in metres. If the particle is initially at
rest at (6, 4), then
(a) its acceleration is of magnitude 5m s2
(b) its speed when it crosses the y-axis is 10 m / s
(c) it crosses the y-axis  x  0 at y  4
(d) it moves in a straight line passing through the origin (0,0)

11. A box of mass m is released from rest at position 1 on the frictionless curved track shown. It slides a
distance d along the track in time t to reach position 2, dropping by a vertical distance h. Let v and a
be the instantaneous speed and instantaneous acceleration respectively, of the box at position 2
Which of the following equations is valid for this situation?
m1

h
d 2

2 2 2
(a) h  vt (b) h  1 2 gt  (c) d  1 2 at (d) mgh  1 2 mv

CENTERS : MUMBAI /DELHI /AKOLA /LUCKNOW /NASHIK /PUNE /NAGPUR /BOKARO /DUBAI # 75
12. In an inelastic collision,
(a) the velocity of both the particles may be same after the collision
(b) kinetic energy is not conserved
(c) linear momentum of the system is conserved
(d) velocity of separation will be less than velocity of approach

13. Two blocks A(5kg) and B(2kg) attached to the ends of a spring of spring constant 1120N/m are
placed on a smooth horizontal plane with the spring undeformed. Simultaneously velocities of 3m/s
and 10m/s along the line of the spring in the same direction are imparted to A and B then
 
3m / s 10 m / s
A 5 2 B
(a) when the extension of the spring is maximum the velocities of A and B are zero.
(b) the maximum extension of the spring is 25cm
(c) maximum extension and maximum compression occur alternately
(d) the maximum compression occur for the first time after  56 sec.

14. Choose the correct alternatives


m
m v
v m
N2 v
N1
N3
The figure shows a block of mass m moving without friction along three tracks with same speed v
mv2
(a) N1  mg (b) N2  mg 
R
2
mv
(c) N3  mg  (d) If fig, the block will leave path, v  gR
R

15. In case of a simple pendulum of length L


(a) The tension at the lowest point for just completing circular motion is 6 mg
(b) The velocity v at the lowest point for just completing circular motion is 5gL
(c) The pendulum will loop if v  5gL
(d) The motion of bob of pendulum is of variable acceleration

16. A particle moves in the xy plane according to the law x = a sin  t  and y  a 1  cos t  where a
and  are constants. Then the particle traces _____
(a) A parabola (b) A straight line equally inclined to x and y axis
(c) A circle (d) A distance proportional to time

CENTERS : MUMBAI /DELHI /AKOLA /LUCKNOW /NASHIK /PUNE /NAGPUR /BOKARO /DUBAI # 76
17. A fly crawls with constant speed v along the radial spoke of a wheel which is rotating with constant
angular speed  . Initially the fly starts from centre of the wheel along x-axis. Then
y

ê êr
v

x fixed
O frame

y

(a) The actual velocity of the fly in fixed frame  is v  veˆ r  reˆ 

(b) The actual velocity of the fly in fixed frame  is v  veˆ r  veˆ 
 v.
(c) The position of fly at any instant r  eˆ r

vt
(d) The position of fly at any instant rˆ  eˆ r

18. A ball of mass m is placed in smooth groove in the disc at the center of the groove as shown and
frame starts to rotate with angular speed  . Which of the following statement is incorrect?

r
m
disc

(a) Net force on ball is towards center


(b) Motion of ball is in radially outward direction w.r.t. centre
(c) Centripetal force is mr2
(d) Motion of ball is circular

19. An imperfectly elastic ball is projected with velocity gh at an angle  with the horizontal, so that
it strikes a vertical wall distant c from the point of projection, and returns to the point of projection.
Then
A
gh


O c N
c c
(a) Time from A to O = (b) Time from A to O =
 gh  cos  e gh cos 
c c
(c) Time from O to A = (d) The coefficient of restitution =
 gh  cos   h sin 2  c 

CENTERS : MUMBAI /DELHI /AKOLA /LUCKNOW /NASHIK /PUNE /NAGPUR /BOKARO /DUBAI # 77
20. A series of n elastic balls whose masses are m, em , e 2 m ,… etc are at rest separated by intervals with
their centers on a straight line. Here, e is coefficient of restitution for the collision. The first is made
to impinge directly on the second with velocity u. Then
(a) The first (n – 1) balls will be moving with the same velocity (1 – e) u
(b) The last one ball will move with velocity u
1
(c) The kinetic energy of the system is mu 2 1  e  en 
2
(d) None of these
1
21. A particle is projected horizontally with a velocity ag from highest point of the outside of a
2
fixed smooth sphere of radius a. Then
(a) Particle will leave the sphere where contact force becomes zero
a
(b) Particle will leave the sphere at a vertical distance below from the point of projection
6
 5
(c) Particle will leave the contact at an angle   cos1   from the vertical
6
(d) None of these

22. Two bars connected by a weightless spring of stiffness k and length (in non-deformed state) l0 rest
on a horizontal plane. A constant horizontal force F starts acting on one of the bars as shown in
figure. Then
m1 k m2
F

F
(a) When m1  m2  m then maximum elongation in the spring is
k
F
(b) When m1  m2  m , then maximum compression in the spring =
k
(c) Maximum compression in the spring is zero
2Fm2
(d) The maximum elongation in the spring 
k  m1  m2 

23. Two perfectly inelastic bodies of masses m1 and m 2 moving with velocities u1 and u 2 in the same
direction impinge directly. Then select the correct alternative
(a) There is no loss in kinetic energy
m1m 2 2
(b) The loss in kinetic energy is  u1  u 2 
2  m1  m 2 
(c) After the collision, they move with common velocity
(d) There is no loss in momentum of the system

CENTERS : MUMBAI /DELHI /AKOLA /LUCKNOW /NASHIK /PUNE /NAGPUR /BOKARO /DUBAI # 78
COMPREHENSION TYPE

PARAGRAPH – 1

A system consist of block A and B each of mass m connected by a light spring as shown with block
B in contact with a wall. The block A compresses the spring by 3mg/K from natural length of spring
and then released from rest. Neglect friction any where.

3mg K
K
B A

1. Acceleration of centre of mass of system comprising A and B just after A is released, is:
3g
(a) 0 (b) (c) 3g (d) None of these
2

2. Velocity of centre of mass of system comprising A and B when block B just looses contact with the
wall:
m 3g m m
(a) 3g (b) (c) 2g (d) None of these
k 2 k k

3. Maximum extension in the spring after system loses contact with wall:
3mg 3mg 3mg
(a) (b) (c) (d) None of these
2k 2k 2k

INTEGER TYPE

1. A block of mass 2 kg is released from rest on a spring A of spring constant 100 N/m as shown in the
figure. The block compresses the spring B through a distance 10 cm. The spring constant of spring B is
x  10 2 N/m. find the value of x.
M
10 cm

10 cm
A
B

2. The block shown in figure is released from rest with the spring initially unstretched. The spring
constant is 500 N/m and mass of the block is 1 kg. Find the maximum extension in spring (in cm)

CENTERS : MUMBAI /DELHI /AKOLA /LUCKNOW /NASHIK /PUNE /NAGPUR /BOKARO /DUBAI # 79
3. A large mass M = 20 kg and a small mass m = 2 kg are at the two
ends of the string that passes through a smooth tube as shown in
figure. The mass m moves around in a circular path, which lies in 
horizontal plane. The length of the string from the mass m to the top m
of the tube is l = 10 cm and  is the angle this length makes with
Horizontal
vertical. What should be the number of revolution per second of Plane
mass m so that M remain stationary  
10   ? M

A
4. Two bodies A and B connected by a light rigid bar 10 m long move in two
frictionless guides as shown in figure. B starts from rest when it is vertically
below A. If the velocity of B is found to be 138  cm/s when x = 6 m find  .
B
Assume mA  mB  200 kg and mC  100 kg (use g = 9.8 m / s 2 ) x

5. A body is projected vertically upwards from the ground with a speed of 20 m/s. after 1 s the body
explodes into two parts in the ratio 1 : 3. After explosion the large mass starts moving horizontally. If
the maximum height reached by smaller mass is 19x in m then find the value of x
(Take g = 10 m / s 2 )

CENTERS : MUMBAI /DELHI /AKOLA /LUCKNOW /NASHIK /PUNE /NAGPUR /BOKARO /DUBAI # 80
ANSWER KEY
SINGLE CHOICE QUESTIONS

1. (c) 2. (c) 3. (b) 4. (b) 5. (c)

6. (a) 7. (c) 8. (c) 9. (d) 10. (d)

11. (d) 12. (a) 13. (d) 14. (a) 15. (a)

16. (d) 17. (a) 18. (b) 19. (a) 20. (a)

21. (b) 22. (a) 23. (b) 24. (d) 25. (b)

26. (c) 27. (b) 28. (b) 29. (c) 30. (b)

31. (c) 32. (b) 33. (c) 34. (d) 35. (b)

36. (b) 37. (c) 38. (b) 39. (c) 40. (b)

41. (a) 42. (a) 43. (c) 44. (c) 45. (a)

46. (c) 47. (a) 48. (d) 49. (a) 50. (b)

51. (a) 52. (B) 53. (c) 54. (c) 55. (c)

56. (c)

MULTIPLE CHOICE QUESTIONS

1. (abcd) 2. (ac) 3. (bcd) 4. (bcd) 5. (bcd)

6. (b,d) 7. (b,d) 8. (c,d) 9. (abc) 10. (abc)

11. (d) 12. (abcd) 13. (bcd) 14. (a, b, c, d) 15. (a, b, c, d)

16. (c, d) 17. (a, c) 18. (a, c, d) 19. (b, c, d) 20. (a, b, c)

21. (a, b, c) 22. (a, c, d) 23. (b, c, d)

COMPREHENSION TYPE

1. (b) 2. (b) 3. (a)

INTEGER TYPE

1. (8) 2. (2) 3. (5) 4. (5) 5. (5)

CENTERS : MUMBAI /DELHI /AKOLA /LUCKNOW /NASHIK /PUNE /NAGPUR /BOKARO /DUBAI # 81
CRASH COURSE P – II (SOLUTION)

SINGLE CHIOCE QUESTIONS


1. In position B, speed of ball would be :
v 2 = 2gh
Where N
h = (R) cos 60°
1

60 0
=  0.5   0
2 mg cos 600 mg mg sin 60
= 0.25 m
 v 2 = (2)(10)(0.25) m 2 /s 2
= 5 m 2 /s 2
Let N be the reaction between the ball and the wedge in
position B. Then,
mv0
N  mg cos 600 
R
2
mv
or N   mg cos 600
R
  2   1 10  1 
1
     
 0.5  2

= 15 N
Now for horizontal equilibrium of wedge Force exerted by vertical wall on wedge
= N sin 60°
15 3
 N
2

2. Initial velocity = final velocity = 0.


But displacement  0
From work-energy theorem
W = KE = 0

3. (B)
When the chain has fallen a distance y<L from rest, v  2 gy
M 
dm  L    vdt  Mv
Rate of increase of mass on the floor,   
dt dt L
dm Mv 2 2Mgy
Force applied by moving part of the chain on the floor, F1  v  
dt L L
M  Mgy
Force applied by that part of chain which is resting on the floor, F2    y  g 
 L  L
3Mgy
Total force applied by the chain on the floor, F  F1  F2 
L
Just before the last link hits the floor, y = L or F = 3Mg

CENTERS: MUMBAI /DELHI /AKOLA /LUCKNOW /NASHIK /PUNE/NAGPUR /BOKARO /DUBAI # 82


4. (B)
1 1  3 6 
Total KE in CM frame   vrel 2   2
 10 = 100 J
2 2  3 6 
1
Therefore, the maximum energy that can be stored in spring is 100 J or Kxmax 2  100
2

x4 x2
5. V 
4 2
dV
  x3  x
dx
dV
V is maximum or minimum if
dx
 
 0 , i.e. if x 3  x  0 or x x 2  1  0 which gives x  0 , 1 and

d2V
 1 . Now  3x 2  1
dx 2
d 2V d 2V d2V
V is maximum if is negative and V is minimum if is positive. For x  0,  1
dx 2 dx 2 dx 2
d2V
Hence for x  0, V is maximum. For x  x  1, 2  2 . Hence for x   1 , V is minimum. Now
dx
x4 x2
V 
4 2
Therefore, the minimum potential energy is (put x   1 )
4 2
 1  1
1 1 1
Vmin      joule 
4 2 4 2 4
Maximum kinetic energy is
1 1 2
2
K max  mmax  max  m  1kg  .
2 2
Given, total energy E  2 joule. Form E  V  K , we have
1 1
2    2max
4 2
3
which gives  max  ms 1
2

6. Let  be the mass per unit area of the disc.


2
Mass of the disc M    4  2R   16R 2
Mass of the removed part m1  4R 2
 Mass of the remaining part m2  M  m1

CENTERS: MUMBAI /DELHI /AKOLA /LUCKNOW /NASHIK /PUNE/NAGPUR /BOKARO /DUBAI # 83


Let G be the centre of mass of the remaining part of the disc. The centre of the complete disc is O
and O ' is the centre of the removed part of the disc. From the definition of the centre of mass, we
have
m  OO ' m 2  OG
x cm  1
m1  m 2
Since xcm  0 , we ger
m1  OO' m2  OG  0
Let O be at the origin  x  0  , then OG   x and OO '  R
Then
M 3M R
R    x   x 
4 4 3
1
Comparing this with x  R , we get  
3

4N 4
7. Slope of graph     Ns 1 . therefore,
3s 3
4
F  t4
3
4.5s
 4 
Now change in momentum     3 t  4 dt
0
4.5s
2
2t
Or 2    4t  4.5
3 0

Which gives   2.25ms 1 . Therefore,


1 1 2
K.E.  m2   2   2.25 
2 2
 5.06J

8. (C)
R
f
m

m
i

Applying w.r.t in wedge frame


w.d pseudo  w gr  k.
maR  mgR  0
ag

9. (D)

M
R/4
M m=2kg
R c.m. m=6kg

CENTERS: MUMBAI /DELHI /AKOLA /LUCKNOW /NASHIK /PUNE/NAGPUR /BOKARO /DUBAI # 84


ab  A  IA  mg  R  I A
mgR R
 ac.m  y   
IA 4
mg  mg  N   M m  a cmy  N  70 N

10. (d)
GM
P1  m ( 2  1)  P2  mv 2
r
2GM GM
v22  v02   P2  m P1  P2 ( 2  1)
r r
P1
  ( 2  1)
P2

11. (D)
1  21 + 2  (– 4) = 1 1 + 2 vB
 vB = 6 m/s
6 1 1
e=   0.2
21  ( 4) 5
1 1 1 1
ΔK =  1  (21) 2   2  4 2   1  12   1  62
2 2 2 2
ΔK = 200 J

12. (A)

m o  t V

at time t

Impulse is
Ft = (mo + t)V
Ft
 V=
mo  t

13. (d)
m1v1  m2 v2
vC 
m1  m 2

14. (a)
50 10   50  m  2.5

15. (a)
v0
Sol. a
t0

CENTERS: MUMBAI /DELHI /AKOLA /LUCKNOW /NASHIK /PUNE/NAGPUR /BOKARO /DUBAI # 85


0 v  at
v
v 0 t
t0
From work energy theorem
1 1 v2
W  K  W mv 2  W  m 20 t 2
2 2 t0
16. (d)
1 1 1
Sol.  M M VM 2   M B VB 2 (1)
2 2 2
MM  2MB (2)
Form (1) & (2)
1
VM  VB (3)
2
1 2 1
When man speed is up by 1 m/s  M M  VM  1  M B VB 2 (4)
2 2
2  VM  1  VB
1
Form (3) and (4) 2  VM  1  2VM  VM 
 2 1
VM   
2 1 m / s

17. (a)
  3  7mg
Sol. U1    mg  2mg   
 2 2 2
  3  5mg
U 2    2mg  mg     U  mg
 2 2 2

18. (b)
Sol. W  WOA  WAB  WBC  WCO
a a 0 0
  x 2 dx   y 2 dy   x 2 dx  y 2 dy
0 0 a a

0

19. (a)
Sol. x cm  0
0    L sin   x  m  mx
L sin   x  x
2x  L sin 
x   L sin   / 2

20. (a)
Sol. KE gain by system  mg L 1  cos  
Half of that will be shared by block

CENTERS: MUMBAI /DELHI /AKOLA /LUCKNOW /NASHIK /PUNE/NAGPUR /BOKARO /DUBAI # 86


1 1
mv 2  mg L 1  cos    v  gL 1  cos  
2 2
21. (b)
mL 1  cos   0
Sol.  1 cm 
mm
L
1  cos  
2

22. (a)
Sol. Block B will have velocity of A after collision & Block A will have velocity of B after collision.
Let VB velocity of B after collision with C
 m  4m  3
VB  v  VB   V
m  4m 5
VB  0.6V & VA  0.6V

23. (b)
Sol. Velocity along surface remain same
3sin 30o  vf sin 60o
1 3
3  vf 
2 2
vf  3m / s

24. (d)
1 1
Sol. KE B  m B vB2 1  0.5  v B 2
2 2
vB  2m s u A  2gh
 2 10  0.45  3m s
From conservation of linear momentum
0.25  3  0  0.25  vA  0.5 2 0.75  0.25vA  1
0.25vA  0.25 vA  1m s

25. (b)
Sol. From energy conservation
PE g  PE spring  0  PEg  PE spring
1 2 1 2
mgh  kx  mg x sin 37 o  kx
2 2
2mg sin 37o 2mg 3 6mg
x  x   x
k k 5 5k
26. (c)
1
Sol. If A moves down the incline by 1 metre, B shall move up by metre. If the speed of B is v then the
2
speed of A is 2v.
From conservation of energy:
1 2 1 3 1
Gain in K.E. = loss in P.E. m A  2v   m B v 2  m A g   m Bg 
2 2 5 2
CENTERS: MUMBAI /DELHI /AKOLA /LUCKNOW /NASHIK /PUNE/NAGPUR /BOKARO /DUBAI # 87
1 g
Solving we get v
2 3
27. (b)
U U
Sol. Fx    sin  x  y  Fy    sin  x  y 
x y
1 1 1  ˆ 
Fx  sin  x  y    Fy  sin  x  y     F i j
0,  /4 
2 0,  /4 
2 2 

28. (b)
Sol. Let the speeds of balls of mass m and 2m after collision be v1 and v2 as shown in figure.
Applying conservation of momentum
U
mv1  2mv 2  mu and  v1  v 2 
2
U
Solving we get v1  0 and v2 
2
U
Hence the ball of mass m comes to rest and ball of mass 2m moves with speed .
2
2r 4r
t 
u/2 u
29. (c)
Sol. For anti-clockwise motion, speed at the highest point should be gR .
Conserving energy at (1) & (2) :

1 R 1
mv 2a  mg  m  gR 
2 2 2
2
 va  gR  gR  2gR  va  2gR
For clock-wise motion, the bob must have atleast speed initially, such that
string is taut & it does not become loose anywhere until it reaches the peg B.
At the initial position :
gR
2
mc c 2 1
T  mg cos 60o  ; Vc / Cs 
R 2gR 2
For VC min : Put T = 0;
gR
gR 2 1
 Vc   Vc / Va 
2 2gR 2
 Vc : Va  1: 2
30. (b)
2R
Sol. If we treat the train as a ring of mass ‘M’ then its COM will be at a distance from the centre of

the circle. Velocity of centre of mass is:
VCM  R CM .
2R 2R  V   V
= .       
  R  R

CENTERS: MUMBAI /DELHI /AKOLA /LUCKNOW /NASHIK /PUNE/NAGPUR /BOKARO /DUBAI # 88


2V 2MV
 VCM   MVCM 
 
As the linear momentum of any system = MVCM
2MV
 the linear momentum of the train =

31. (c)
Sol. As shown in the figure the component of momentum of one shell along initial direction and
perpendicular to initial direction are P1x  10 3Ns and P1y  10Ns

For momentum of the system to be zero in y-direction P2y must be 10 Ns. 2nd part of shell may or
may not have momentum in x-direction
 P2 min  10 Ns .

32. (b)

u=0
v
1 1 mv 2
mu 2  mgl 1  cos 60o   mv 2  0  2 mg 1  cos 60o   mg
2 2 l
Tension will be maximum at the bottom (lower most point)
T
v2

Mg
mv 2
T  mg  T max   2mg
l
T

Tmax  2mg
f(s)

8g
For
Tmax  2mg N  8g f s  2 mg
To ensure no slipping, f s  N
2mg  8g m
4

CENTERS: MUMBAI /DELHI /AKOLA /LUCKNOW /NASHIK /PUNE/NAGPUR /BOKARO /DUBAI # 89


 1  min  1
2 2
33. (c)
v  3g  a  b 
O O'
(a-b)

u
For completing the circle about the nail.
1 1
mu 2  0  m3g  a  b   mg a u 2  3g  a  b   2ag
2 2
u 2  5ga  3gb u  5a  3b  g

34. (d)
u=0
m R
Q
v v1

1
mgR  0  mv 2  0
2
v  2gR
Till then the ball reaches the lowest point, linear momentum will not be conserved. Post that, linear
momentum shall also be conserved.
m 2gR   M  m v1
m 2gR
v1 
 M  m
35. (b)

a
v

Before impact after impact


v1

v v2

Q
T
v
v1  v 2  e  0  v 

CENTERS: MUMBAI /DELHI /AKOLA /LUCKNOW /NASHIK /PUNE/NAGPUR /BOKARO /DUBAI # 90


Vref  v1  v2  ev
2a
Next impact takes place after tence T1 
v ref
2a
T1 
ev
2
T1  T
e
36. (b)
x v
P
M

v
a r  v2 / R

Mg
2
Mv
N 0
r
v0
1
Kx 2  0  MgR  0
2
2MgR
x
K
37. (c)

M  u cos  u sin 

v cos   v1


u sin 
v M 
 u sin 

Using conservation of linear momentum in horizontal direction.


MV  m  v1 sin   u sin  cos    0 …(1)
v1    V sin    e    u cos     0   …(2)
Eliminating v1 from equation (1)
MV  m  e u cos V sin   sin   mu sin  cos 
MV  me u cos  sin   mVsin2   m u sin  cos 
m u sin  cos  1  e 
V
 M  msin 2  
CENTERS: MUMBAI /DELHI /AKOLA /LUCKNOW /NASHIK /PUNE/NAGPUR /BOKARO /DUBAI # 91
38. (b)

T  mg sin 
mg cos 
fixed point  mg

3


T  mg sin 
u  2 gl mg cos 

1 2 1
2

m 2 g 
 O  mg  1  sin    mv 2
2
…..(1)
2
mv
T  mg sin  
l
as T  0
mv2  mgl sin  ….(2)
Using equation (1)
1 1
m 4 gl  mgl = 1  sin    mgl sin 
2 2
mgl sin 
2 mgl = mgl + mglsin 
2
3 mgl sin 
 mgl
2
sin   2 3
2l
h  l sin   above the fixed point
3

39. (c)
N v=constant

F=12 N
fk

mg

f k  F  12 N
N  mg  0.5g  5N
2
Total contact force  12   52  13 N

CENTERS: MUMBAI /DELHI /AKOLA /LUCKNOW /NASHIK /PUNE/NAGPUR /BOKARO /DUBAI # 92


40. (b)
O
a r  v 2
1 

T

Q v1
Q
mg sin 
mg cos 
v mg
1 1
mv12  mgl 1  cos    mv 2   …..(1)
2 2
m1 v12
V  2  3 mg T  mg cos  
l
T0
mv12   mgl cos  ……(2)
using equation (2) in equation (1) 
1 1
 mgl cos   mgl 1  cos    mv2
2 2
 mgl cos  1
2

 mgl 1  cos    m 2  3 mg
2

 cos  1
As l  a  1  cos    2  3 
2 2
3 3 1
 cos   1  1  cos  
2 2 3
 1 
  cos 1  
 3
41. (a)
Before impact After impact
M v sin 
M

 v
m
m
v1

  v1  o 
e
 o  v cos  
Conserving linear momentum Mv  0  Mvsin  sin   mv1 cos 
V1 M
mv1 sin   mvsin  cos  e 
V cos  m

42. (a)

   
40 4iˆ  40 3jˆ  80V perfectly inelastic collision

 40  34iˆ  3jˆ   3 
v  2iˆ  ˆj
 
80  2 

CENTERS: MUMBAI /DELHI /AKOLA /LUCKNOW /NASHIK /PUNE/NAGPUR /BOKARO /DUBAI # 93


1 2 1 2 1  9 
Loss of mechanical energy  KE   40  4   40  3    80  4   
2 2  2  4 
1  25 
 KE  40 16  9   40  
2  4 
 25 
 20  25   40    250 J
 4 
43. (c)
Using conservation of linear momentum,

 
2ˆj  4ˆj  6jˆ  10jˆ  P Af 

PA f   2iˆ
44. (c)
m/5

4 kg

4 kg 2m/sec

4t the string will be taut again after tencet. The hanging mass will be under free fall.
1 1
4t  2t  gt 2 2t  10t 2
2 2
t  4 /10  t  4sec
At this instant the velocities of each mass. 4m/sec
4m/sec

v=2+g(.4)=6m/sec

After the jirk


T
N V1

T
 V1

mg
mg

 Tdt  4  v  4 
1
  Tdt  4  v 1  6   4  2v1  10   0  v1  5m / sec
  Tdt  4 5  4   4N sec
45. (a)


cm
2R / 

T T

CENTERS: MUMBAI /DELHI /AKOLA /LUCKNOW /NASHIK /PUNE/NAGPUR /BOKARO /DUBAI # 94


m 2 2R m 2 R
2T   T (Tensile in nature)
2  2
As  = constant So   0
 Tension throughout the string is same

46. (c)
As the surface is frictionless so cm doesn’t none horizontal where as normal contact force acrs on
the system in the vertically upward direction so cm moues vertically upward.
v sin 

Q
v cos 

mv cos 
M
mvsin 
vcm  vertically upward.
 M  m
47. (a)
x
k 1 2
a x 2 dx  2 mv  0
x
 1  1
 K    mv 2
 x a 2
1 1 1
K     mv 2
x a 2
2K  1 1 
v   
m x a

48. (d)
mu  0  mv1  mv 2 …(1)
 v2  v1   e  0  u  …(2)
 mu  mv 2 
v2     eu
 M 
 m
v 2 1    u  eu
 M
u 1  e 
v2 
 m
1  
 M
As m << M
m
  1
M
v 2  u 1  e 
As e = 1 so v2  2u
CENTERS: MUMBAI /DELHI /AKOLA /LUCKNOW /NASHIK /PUNE/NAGPUR /BOKARO /DUBAI # 95
49. (a)
m
 L  x
3
x
M
 
As  F  ext  on system = 0 so vem  constant
vcm  0 i.e., c.m. remains at Rest.
Initially,
M
Mx  L  x  0
3
L x
x  0
3 3
3x  L  x  0
x  L / 4
Distance moved by man wrt ground  Lx
3L
 LL 
4 4
50. (b)
Two strings in parallel  K1 eq   2K1
K 2  eq   2K 2
2 2
1 1 2 1 x 1  x  1 2
 2 2K1x  2 2K 2   x    0  2 2K1  2   2 2K 2  2   2 mv
2

x2 1
  1 2
K  K x 2
  1 2   mv 2
K  K
4 2
3  K1  K 2  x 2
1 3  K1  K 2  x 2
 mv 2  v 
4 2 2m
51. (a)
v  u  at
0  u  gt
u
t
g
a  g
N
v

mg
f  K   mg
1
mu 2
Mean power developed by the fraction force = 2
t
1
mu 2
2
 u 
 g 
 

CENTERS: MUMBAI /DELHI /AKOLA /LUCKNOW /NASHIK /PUNE/NAGPUR /BOKARO /DUBAI # 96


1
m u g
2

52. (b)
1
2I  2I 
12

 2M L2  L2  I
I
ML2
I
12

53. (c) 54. (b) 55. (c)

56. (b)
  2000 sin 15o g 10  5.17 kJ

MULTIPLE CHOICE QUESTIONS

1. (a, b, c, d)
(Before collision)
(m  m  m)u cm  mu
u
u cm 
3
 No external force working on system so u cm will be constant .
Initial angular momentum = mul
No external torque working on system so final = (m  m)u ' l
mul  2mu ' l
u
u'
2
mu mu
Final linear momentum =   m B u B'  mu
2 2
u 'B  0

2. (a, c)
U   kxy
U
Fx    kyiˆ
x
F  kxjˆ
y

F  kyiˆ  kxjˆ
F is conservative force.

CENTERS: MUMBAI /DELHI /AKOLA /LUCKNOW /NASHIK /PUNE/NAGPUR /BOKARO /DUBAI # 97


3. (b, c, d)
1 2
kx  mgx
2
2mg
x
k
mg
at x 
k

kmg
 mg  ma
k
a=0

2mg
at x 
k
2mg – mg = ma
a g

4. (2,3,4)
U  5x  x  4  J
dU 10x  20
F  10x  20   a  100x  200
dx 0.1
Speed will be maximum at equilibrium a  0 i.e, x  2
the particle executes SHM.
2 2
Time period  
 10

5. (b,c,d)
 dU
F 0
dx

6. (b,d) 7. (b,d)

8. (c,d)
m2 
mv  mv0   Ndt    Ndt
12 2
 
v   ev0   v  ev0 
2 2 v0
m
3  mv  mv 0  0
6 v0
V
4v   e  3 v0  0 v
 3  e  v0
4

9. (a, b, c)

CENTERS: MUMBAI /DELHI /AKOLA /LUCKNOW /NASHIK /PUNE/NAGPUR /BOKARO /DUBAI # 98


10. (a, b, c)
Sol. U  3x  4y
 dU  dU  
F i j  F  3i  4j  F  5N
dx dy
Fx F F
(a) ax    3m s2 , a y  y   4m s2  a  5m s 2
m m m

(c) Taking motion along X-axis & Y-axis


1 3 2
x  x0  u x t  a x t 2  x  6 t (1)
2 2
1 2 4
y  y0  u y t  a y t  y  4  t 2 or y  4  2t 2 (2)
2 2
3
It will cross the y-axis (at x  0 )  0  6  t2  t  2sec
2
at t  2sec, y  4m
dx dy
(b) Vx   3t, Vy   4t  at t  2 sec  Vx  6, Vy  8
dt dt
V  Vx2  Vx2  10m / s  Eliminating t from eq (1) and (2)
(d) path of particle
2
y  4  2 6  x 
3
4
y  48 x
3
4
y  x4
3

11. (d)

12. (a, b, c, d)

13. (b, c, d)
Sol. At maximum extension blocks will be at rest in C frame means both have velocity equal to vcm .
 
 m v1  m v 2  5  3  2  10 35
1 2
v cm   v cm    5m / s
m1  m 2 5 2 7
From conservation of energy
K  U  0  U  K
1 2 1 1 1
kx max  m1 v12  m 2 v 2 2   m1  m 2  v cm2

2 2 2 2
2 2
kx max  5   3  2  10    5  2   52
2
 kx 2max  45  200 175
70 1 1
x 2max   x 2max   x max  m  x max  25cm
1120 16 4
A and B will perform S.H.M. in C-Frame so minimum speed B is 0.

14. (a, b, c, d)
N1  mg

CENTERS: MUMBAI /DELHI /AKOLA /LUCKNOW /NASHIK /PUNE/NAGPUR /BOKARO /DUBAI # 99


mv 2
mg  N 2 
R
 g  v2 
N2  m  
 R 
N2

v2
ar 
R
mg
mv 2
N 3  mg 
R
mv 2
N 3  mg 
R
N3
2
ar  v
R

mg

N 2  0  v  gR
v  gR
block will leave the path.

15. (a, b, c, d)
T v 02 / 

5gl  v 0

mg
2
mv 0
T  mg 

m
T  mg  5g

T  6mg
v  5gl It will loop. In any case, circular motion is of variable acceleration.

16. (c, d)
x  a sin  t 
a  y  a cos  t 
Squaring and adding both the equations
2
x 2   a  y  a 2
2
x2   y  a   a2
As the circular motion has an angular velocity  which is given constant. So
  t
Distance  RQ  Rt proportional to time.
CENTERS: MUMBAI /DELHI /AKOLA /LUCKNOW /NASHIK /PUNE/NAGPUR /BOKARO /DUBAI # 100
17. (a, c)
With reference to a fixed frame 

v  veˆ r  wreˆ q
w

wr
v


The position of fly  r  r cos  ˆi  r sin ˆj  r  vt 
  
 
r  vt cos ˆi  sin ˆj     t 

 v r
 r
 r
 v
 r eˆ r

18. (a, c, d)
As there is no friction between the ball and the groove so there will be no centripetal force i.e., No
net force on the ball towards the center hence the ball moves in radially outward direction w.r.t disc

19. (b, c, d)
After Impact e gh cos 

Before Impact gh cos 

v
A
H
e gh cos 

gh sin 

O gh cos  N

c
t  OA  
gh cos 
gc
v  gh sin  
gh cos 
c
t  AO  
e gh cos 
Due to the impact only horizontal component of velocity gets affected so total time of flight remains
the same
t OA   t  AO   T

CENTERS: MUMBAI /DELHI /AKOLA /LUCKNOW /NASHIK /PUNE/NAGPUR /BOKARO /DUBAI # 101
c c 2 gh sin 
 
gh cos  e gh cos  g
c  1  2 gh sin 
1   
gh cos   e  g
1 h sin 2
1 
e c
1 h sin 2  c

e c
c
e
h sin 2  c

20. (a, b, c)
Before impact
v1
mek mek 1
After impact
v1' v2
mek mek 1
me k v1  0  me k  v1  ev 2  by conservation of linear momentum.
v1  v1'  ev2 …(1)
v2  v  e  0  v1 
'
1

v2  v1'  ev1
v1  ev1  v2  ev2
v 2  v1
v1'  v1 1  e 
After each impact, the next ball will gain velocity of u and the previous one will have velocity
u 1  e  .
So, 1 st (n – 1) ball’s will move with (1 – e)u. and the last ball will move with velocity u .
1 n 1 2 1
K.E.  system    me k 1  1  e  u   me n 1u 2
2 k 1 2
n 1
1  2 
 mu 2   e k 1 1  e   e n 1 
2  k 1 
2 e e  1 n 1 
o n 1
1 
 mu 2 1  e  e 
2   e  1 
1
 mu 2  e  1  e n 1  1  e n 1 
2
1
 mu 2  en  e  e n 1  1  e n 1 
2
1
 mu 2  en  e  1
2

CENTERS: MUMBAI /DELHI /AKOLA /LUCKNOW /NASHIK /PUNE/NAGPUR /BOKARO /DUBAI # 102
21. (a, b, c)
u  ag / 2
h

v

1 1
mu 2  mga 1  cos    mv 2 …(1)
2 2
As N = 0 at angle of rotation 
mV 2
So mg cos   …(2)
a
Using eq. (1)
2
1  ag  1
m    mga 1  cos     mg a cos  
2  2  2
mga mga cos 
 mga  mga cos  
4 2
5mga 3
 mga cos 
4 2
5
cos  
6
  cos 1  5 / 6 
 5 a
h  a 1  cos    a 1   
 6 6

22. (a, c, d)
kx F  kx
a1  a2 
m2 m1
kx F
kx
vdv
a rel   a 2  a 1  
dx
 F  kx   kx  dv
  v
 m1   m 2  dx
F  kx  1 1  v dv
  
m1  m1 m 2  dx
x
F  1 1  0

0  m1  kx  m1  m2 dx  0 v dv
 
At maximum compression or maximum extension relative velocity should become zero.
Fx kx 2  1 1 
    0
m1 2  m1 m 2 
F / m1
 x  0 or x
K 1 1 

2  m1 m 2 

CENTERS: MUMBAI /DELHI /AKOLA /LUCKNOW /NASHIK /PUNE/NAGPUR /BOKARO /DUBAI # 103
2m2 F
 x
K  m1  m2 
When m1  m2  m then x max  F / K
Compression the spring will be zero.

23. (b, c, d)
1 1 1
m1u 12  m 2 u 2 2  KE  loss    m1  m 2  v 2
2 2 2
m1u1  m 2 u 2   m1m 2  v [linear momentum remains consecued]
2
1 1 1  m1u1  m 2 u 2 
m1u12  m 2 u 2 2   m1  m 2  2
 KE  loss
2 2 2  m1  m2 
m12 u12  m 22 u 22  m1m212  m 2 m1 22  m1212  m2 22  2m1m21 2
KE loss  
2  m1  m 2 
2
m1m 2  1   2 
KE  loss  
2  m1  m 2 

COMPREHENSION TYPE

PARAGRAPH – 1

1. b
m1a1  m2a 2 m  0  m  3g 3g
Sol. a cm   
m1  m 2 m1  m2 2

2. (b)

Sol.

2
1  3mg  1 2
k   mv
2  k  2
9mg 2 m
v  3g
k k
m  0  mv v 3g m
vcm   
mm 2 2 k

3. (a)
Sol. By COE in C-frame
1 1
Vref 2  kx 2
2 2

CENTERS: MUMBAI /DELHI /AKOLA /LUCKNOW /NASHIK /PUNE/NAGPUR /BOKARO /DUBAI # 104
2
1 m m 1 2
 3g   kx
2 2 k  2
2
9 2m
g  kx 2
2 k
3mg
x
2k

INTEGER TYPE

1. (8)
M u = 0 released from rest
10 cm

10 cm
K A  100 N / m
2
K B  x 10 N / m 10 cm
B A
ug = 0

using energy consevation principle


1 2 1 2
mg  0.1  0.1  0.1  0  0  0  x  102  0.1  100  0.1  0.1
2 2
1 100
20  0.3   x  0.04 
2 2
x 4
6 
2 2
x
6 2 x 8
2

2. (2)
2x

K = 500 N/m

Initially the spring is relaxed

M
x v

If the block moves down by x, then the spring is extended by 2x as the string connecting the spring
is in extensible. Using energy conservation principle,
1 1 2
0  mgx  0  mv 2  K  2x   0
2 2
The extension in the spring will be maximum when the block comes to rest momentarily.
i.e., v = 0

CENTERS: MUMBAI /DELHI /AKOLA /LUCKNOW /NASHIK /PUNE/NAGPUR /BOKARO /DUBAI # 105
1 2
mgx  0  k  2x 
2
2mgx  k4x 2
mg  2kx  x  mg / 2k
mg
Maximum extension in spring  2x   2cm
k

3. (5)
If M remains stationary
T
T = Mg

Mg
T cos 
 
m2  sin 
T sin 
m
mg
 sin 
T cos   mg
T sin   m2  sin 
mg  m2
Mg

m
200
  10 10
2  .1
  10  rad / sec
10
 rev / sec
2

4. (5)
VA
A

2  x2  y  VB

C x
VC  VB
Using energy conservation,
1 1 1
 
m A    2  x 2  0  m Cgx 
2
m A v A2  m B v B2  m C v C2
2 2

CENTERS: MUMBAI /DELHI /AKOLA /LUCKNOW /NASHIK /PUNE/NAGPUR /BOKARO /DUBAI # 106
2
 10  100  36  1 2 1  138 
200g 
   10096  200 138   100  
 1008  2 2  100 
2
 138   36 
 200  9.8   2  600  9.8     100   1  0.5 
 100   64 
2

 9.8  400  600  


138   132 
 
100  64 
 5

VA sin 

VA VB sin 
VA cos 
y  VB
x 

VB cos 
VA sin   VB cos 
x
VA  VB
l2  x2
6
VA  138 
8

5. (5)
g  10m / s2
v  u  at
v  20  10 1
v  10 m / s
1 2
h  20 1  10 1
2
h  15 m
Just before v = 10 m/s
explosion

h = 15 m
u = 20 m/s

Conservation of linear momentum, Along vertical direction


mv3  4m 10 
v23 1600
v3  40 m / s  Hmax from ground  h   15  95 m
2g 20
 19x  95 for smaller mass
x=5

CENTERS: MUMBAI /DELHI /AKOLA /LUCKNOW /NASHIK /PUNE/NAGPUR /BOKARO /DUBAI # 107
SYSTEMS OF PARTICLES AND ROTATIONAL MOTION

CONCEPT SUMMARY
1. Ideally, a rigid body is one for which the distances between different particles of the body do not
change, even though there are forces on them.

2. A rigid body fixed at one point or along a line can have only rotational motion. A rigid body not
fixed in some way can have either pure translation or a combination of translation and rotation.

3. In rotation about a fixed axis, every particle of the rigid body moves in a circle which lies in a plane
perpendicular to the axis and has its centre on the axis. Every Point in the rotating rigid body has the
same angular velocity at any instant of time.

4. In pure translation, every particle of the body moves with the same velocity at any instant of time.

5. Angular velocity is a vector. Its magnitude is   d / dt and It is directed along the axis of rotation.
For rotation about a fixed axis, this vector o has a fixed direction.

6. The vector or cross product of two vector a and b is a vector written as a  b. The magnitude of this
vector is absin  and its direction is given by the right handed screw or the right hand rule.

7. The linear velocity of a particle of a rigid body rotating about a fixed axis is given by v    r ,
where r is the position vector of the particle with respect to an origin along the fixed axis. The
relation applies even to more general rotation of a rigid body with one point fixed. In that case r is
the position vector of the particle with respect to the fixed point taken as the origin.

8. The centre of mass of a system of particles is defined as the point whose position vector is

R
 miri
M

9. Velocity of the centre of mass of a system of particles is given by V = P/M. where P is the linear
momentum of the system. The centre of mass moves as if all the mass of the system is concentrated
at this point and all the external forces act at it. If the total external force on the system is zero, then
the total linear momentum of the system is constant.

10. The angular momentum of a system of n particles about the origin is


n
L   ri  p i
i 1

The torque or moment of force on a system of n particles about the origin is


n
   ri  Fi
i 1

The force F acting on the i th particle includes the external as well as internal forces. Assuming
Newton’s third law and that forces between any two particles act along the line joining the particles,
dL
we can show int  0 and   ext
dt

11. A rigid body is in mechanical equilibrium if


(1) it is in translational equilibrium. i.e. the total external force on it is zero:  F  0 , and
i

CENTERS: MUMBAI /DELHI /AKOLA /LUCKNOW /NASHIK /PUNE/NAGPUR /BOKARO /DUBAI # 108
(2) it is in rotational equilibrium. i. e. the total external torque on it is zero :    r  F  0 .
i i i

12. The centre of gravity of an extended body is that point where the total gravitational torque on the
body is zero.

13. The moment of inertia of a rigid body about an axis is defined by the formula I   mi ri2 where ri is
the perpendicular distance of the ith point of the body from the axis. The kinetic energy of rotation is
1
K  I2 .
2

14. The theorem of parallel axes: I z '  I z  Ma 2 , allows us to determine the moment of intertia of a rigid
body about an axis as the sum of the moment of inertia of the body about a parallel axis through its
centre of mass and the product of mass and square of the perpendicular distance between these two
axes.

15. Rotation about a fixed axis is directly analogous to linear motion in respect of kinematics and
dynamics.

16. For a rigid body rotating about a fixed axis (say, z-axis) of rotation, Lz  I , where I is the moment
of Inertia about z-axis. In general, the angular momentum L for such a body is not along the axis of
rotation. Only if the body is symmetric about the axis of rotation. L is along the axis of rotation. in
that case. L  L z  I . The angular acceleration of a rigid body is rotating about a fixed axis is
given by I   . If the external torque  acting on the body is zero, the component of angular
momentum about the fixed axis (say. z-axis), L   I of such a rotating body is constant.

17. For rolling motion without slipping cm  R , where cm is the velocity of translation (i.e. of the
centre of mass), R is the radius and m is the mass of the body. The kinetic energy of such a rolling
body is the sum of kinetic energies of translation and rotation:
1 1
K  m2cm  I2 .
2 2

CENTERS: MUMBAI /DELHI /AKOLA /LUCKNOW /NASHIK /PUNE/NAGPUR /BOKARO /DUBAI # 109
ROTATION
FORMULA SUMMARY

Body Mass Dimensions Axis I


(1) Ring/Hollow M Radius: R Longitudinal MR2
Cylinder passing through
center
(2) Disc/Uniform M Radius: R Longitudinal 1
MR 2
solid cylinder passing through 2
center
(3) Solid sphere M Radius: R Through COM 2
MR 2
5
(4) Solid spherical M Radius: R Through COM 2
MR 2
shell 3
(5) Thin Rod M Length:  Passing through 1
M 2
center 12
perpendicular to
longitudinal axis

1 2
Passing through m
3
end
(6) Rectangle/ M Length:  M 2
Rectangular Breadth :b b 12

  b2 
paralleopiped Height :h l
h

(7) Annular disc M Inner radius – R1 Perpendicular to plane M


I= (R12  R 22 )
Outer radius – R2 2
R2
R1

ML2 MR 2
(8) Hollow M Radius R
Through centre I= 
perpendicular to l2 2
cylinder Length L length

(9) solid cylinder M Radius R Through centre ML2 MR 2


Length L perpendicular to I= 
length l2 4
O

CENTERS: MUMBAI /DELHI /AKOLA /LUCKNOW /NASHIK /PUNE/NAGPUR /BOKARO /DUBAI # 110
(10) cylindrical M Inner radius R1 Through centre ML2 M (R12  R 22 )
shell outer radius R2 perpendicular to I= 
l2 4
length
length L
O

(11) spherical M Inner radius R1 Through diameter


I=
2 
R 5  R15
M 23

shell ( thick ) outer radius R2 5 
R 2  R13 
R2

R1

(12) solid cone. M Radius R height H About axis shown I=


3
MR 2
10

(13) Hollow cone. M Radius R About axis shown MR 2


Height H I=
2

Parallel axes theorem : I = Icm +Md2 ( d = distance between axis through centre of mass and
parallel axis considered )
Perpendicular axis theorem : Iz = Ix +Iy ( Only for laminar bodies )

 Angular momentum :
  
Angular momentum of a particle L  m r  v 
Angular Momentum for a system of particles, about a point
    
L  m rcm  v cm   I cm  cm  rcm is frompoint to c. o. m.

    dL
   I  : angular acceleration  
dt
 In case of pure rolling, use constraint equations carefully


r a2
e.g.
a1 Here , a1 = a2 – r

CENTERS: MUMBAI /DELHI /AKOLA /LUCKNOW /NASHIK /PUNE/NAGPUR /BOKARO /DUBAI # 111
 For a disc, sphere, ring, cylinder or spherical shell :

g sin 
(i) Acceleration of body rolling down on inclined plane a
I
1
MR 2

2gh
(ii) Velocity of body at bottom =
I
1
MR 2

  I 
 2 1  
(iii) Time to reach bottom =   MR 2

 g sin  
 
 
Mg sin  tan 
(iv) Frictional Force = ;  min 
 MR 2  MR 2
1   1
 I  I
 

Translational K.E. MR 2
(v)  ,
Rotational K.E. I

 For equilibrium of rigid body :-


(i) Net force in any direction is zero
(ii) Net torque about any point is zero.

Note:
(1)
Vo 
V0 V =2V0cos   [If V0 =  R ]
 2
V

2
(2) Work done by Torque =   d
1

(1) While applying definition of e in collisions of point masses with rigid bodies, velocities should be
that of point of impact.
(2) In collisions, involving two rigid bodies, write impulse momentum equations and solve them.
Some equations are lost when you simply use conservation laws.
 I.A.O.R ( Instantaneous Axis of rotation )

(i) If velocity of two points is given, then draw


perpendiculars to velocities 
v
& the point of intersection is I.A.O.R   0 v

CENTERS: MUMBAI /DELHI /AKOLA /LUCKNOW /NASHIK /PUNE/NAGPUR /BOKARO /DUBAI # 112
 1 v1
(ii) When two parallel or anti-parallel velocities are given, then  &
 2 v2
v 2  v1


 1 V
1
O V2
2 2  V1
O 1
V2

OSCILLATIONS
CONCEPT SUMMARY

1. The motions which repeat themselves are called periodic motions

2. The period T is the time required for one complete oscillation, or cycle. It is related to the frequency
v by.
1
T
v
The frequency v of periodic or oscillatory motion is the number of oscillations per unit time. In the
SI, it is measured in hertz:
1 hertz =1Hz=1 oscillation per second = 1s 1

3. In simple harmonic motion (SHM), the displacement x(t) of a particle from its equilibrium position is
given by,
x(t) = A cos  t    (displacement)
In which A is the amplitude of the displacement, the quantity  t    is the phase of the motion,
and  is the phase constant. The angular frequency  is related to the period and frequency of the
motion by,
2
  2v (angular frequency).
T
4. Simple harmonic motion is the projection of uniform circular motion on the diameter of the circle in
which the latter motion occurs.

5. The particle velocity and acceleration during SHM as functions of time are given by
v(t)  A sin  t    (Velocity)
a  t   2 A cos  t  
 2 x  t  (acceleration).

6. The force acting simple harmonic motion is proportionate to the displacement and is always directed
towards the centre of motion.

CENTERS: MUMBAI /DELHI /AKOLA /LUCKNOW /NASHIK /PUNE/NAGPUR /BOKARO /DUBAI # 113
1
7. A particle executing simple harmonic motion has, at any time, kinetic energy K  mv 2 and
2
1
potential energy U  kx 2 . If no friction is present the mechanical energy of the system. E = K+ U
2
always remains constant even through K and U change with time.

8. A particle of mass m oscillating under the influence of a Hooke’s law restoring force given by F= –k
x exhibits simple harmonic motion with
k
 (angular frequency)
m
m
T  2 (period)
k
Such a system is also called a linear oscillator:

9. The motion of a simple pendulum swinging through small angles is approximately simple harmonic.
The period of oscillation is given by,
L
T  2
g

10. The mechanical energy in a real oscillating system decreases during oscillations because external
forces, such as drag, inhibit the oscillations and transfer mechanical energy to thermal energy. The
real oscillator and its motion are then said to be damped. If the damping force is given by Fd  bv ,
where v is the velocity of the oscillator and b is a damping constant, then the displacement of the
oscillator is given by.
x  t   Ae bt /2m cos   ' t   
Where  ' , the angular frequency of the damped oscillator, is given by
k b2
'  
m 4m 2
If the damping constant is small then `  where  is the angular frequency of the undamped
oscillator. The mechanical energy E of the damped oscillator is given by
1
E  t   kA 2 e  bt/m
2

11. If an external force with angular frequency d acts on an oscillating system with natural angular
frequency  the system oscillates with angular frequency d . The amplitude of oscillations is the
greatest when
d  
A condition called resonance.

S.H.M.
FORMULA SUMMARY

(1) x = A sin wt  


v = wA cos wt   
a =  w 2 A sin wt     w 2 x

CENTERS: MUMBAI /DELHI /AKOLA /LUCKNOW /NASHIK /PUNE/NAGPUR /BOKARO /DUBAI # 114
1
(2) P.E. = mw 2 x 2
2
1
Energy in SHM = mw 2 A 2
2
1
K.E. =
2

mw 2 A 2  x 2 
(3) For spring- mass system:
m
T  2 .
k
If m s : mass of spring,

ms
m
T  2 3
k

For parallel & series combination, apply keq for k.

Parallel : keq = k1 + k2 + … e.g. 1) 2)


k1
k1 k2

k2

1 1 1 1
Series :     ... e.g. 1)
k eq k l k 2 k 3
k1

k2

Two masses m 1 & m2 connected by spring


 mlm 2
T  2 ,
K m1  m 2
l
 For pendulum T = 2 
g
For physical pendulum
I
T = 2 l : distance of center of mass from point of suspension
mgl
I : Moment of inertia about point of suspension

CENTERS: MUMBAI /DELHI /AKOLA /LUCKNOW /NASHIK /PUNE/NAGPUR /BOKARO /DUBAI # 115
 
 In non – inertial frame, replace ‘g’ by g  a .
 Angular S.H.M. :
   w2 
 = angular acceleration and  is angular displacement.

Note :
dU
F  , U : potential energy
dx
For equilibrium
dU
0
dx

Types of equilibria :
d2U
(i) 2  0  Neutral
dx
d2U
(ii) 2  0  Unstable
dx
d2U
(iii) 2  0  Stable
dx

(1) Basic steps to find time period of S.H.M


(i) If the particle undergoes linear motion, use force method
(a) locate equilibrium position
(b) At a displacement x from this position , draw F.B.D. and write Fext = m a
(c) solve and get a = - ()2 x
2
(d) T=

(ii) If particle undergoes rotational motion also, use torque method :


(a) locate equilibrium
(b) At angular displacement  from it, draw F.B.D. and write ext = I 
(c) solve and get  = - ()2 x
2
(d) T=

(iii) Energy method ( valid for linear and rotational motion ) :
(a) locate the equilibrium position
(b) at displacement ( x,  ) write the total energy ( kinetic + potential ) equation.
dE
(c) = 0. solving it leads to either a = - (2)x or  = - (2)
dt

CENTERS: MUMBAI /DELHI /AKOLA /LUCKNOW /NASHIK /PUNE/NAGPUR /BOKARO /DUBAI # 116
SINGLE CHOICE QUESTIONS

1. Two pendulums have time periods T and 5T/4. They start SHM at the same time from the
mean position. After how many oscillations of the smaller pendulum they will be again in the
same phase :
(a) 5 (b) 4 (c) 11 (d) 9

2. A pipe of length 2m is open at both ends. The speed of sound in air is 340 m/s. The air column
can resonate in frequency :
(a) 255 Hz (b) 200 Hz (c) 175 Hz (d) 260 Hz

3. Two particles of equal mass m at A and B are connected by a rigid light rod AB lying on a
smooth horizontal table. An impulse J is applied at A in the plane of the table and perpendicular
at AB. Then the velocity of particle (just after impulse) at A is :
J J 2J
(a) (b) (c) (d) zero
2m m m
a a
4. In an SHM x = a sin t, the minimum time taken by a particle to reach from x   to x   is
2 2
a
0.5 second what is the minimum time taken by same particle to reach from x   to x + a ?
2
(a) 0.5 s (b) 0.4 s (c) 0.2 s (d) 0.1 s

5. A solid metallic sphere of radius R having moment of inertia equal to I about its diameter is melted
and recast into a solid disc of radius r of a uniform thickness. The moment of inertia of the disc about
an axis passing through its edge and perpendicular to its plane is also equal to I. The ratio r/R is
2 2 2 1
(a) (b) (c) (d)
15 10 5 2
6. A uniform thin rod of mass ‘m’ and length ‘l’ is standing on a smooth horizontal surface. A slight
disturbance cause the lower end to slip on the smooth surface and rod starts falling. Velocity of
centre of mass of the rod at the instant when it makes   37 o with horizontal is :
3gl 9gl 96gl 24gl
(a) (b) (c) (d)
10 26 365 260

7. A solid sphere is rolling purely on a rough horizontal surface (coefficient of kinetic friction   )
with speed of centre  u . It collides inelastically with a smooth vertical wall at a certain moment, the
1
coefficient of restitution being . The sphere will begin pure rolling after a time.
2

3u 2u 3u 2u
(a) (b) (c) (d)
7g 7g 5g 5g

CENTERS: MUMBAI /DELHI /AKOLA /LUCKNOW /NASHIK /PUNE/NAGPUR /BOKARO /DUBAI # 117
8. A uniform rod of mass m length l is held horizontally by two vertical strings of negligible mass, as
shown in figure. The tension on the left string immediately after it is cut will be.

mg mg mg mg
(a) (b) (c) (d)
2 8 4 16

9. A solid ball rolls down a parabolic path ABC from a height h as shown in figure. Portion AB of the
path is rough while BC is smooth. How high will the ball climb in BC?
A
C

2 5 B 3 2
(a) h (b) h (c) h (d) h
3 7 5 7

10. A rod of length l forming an angle  with the horizontal strikes a frictionless floor at A with its
centre of mass velocity 0 and no angular velocity. Assuming the impact at A is perfectly elastic.
The angular velocity of the rod immediately after the impact will be:

0
A 

v 0 cos  v 0 cos  v 0 cos  v 0 cos 


(a) 6 (b) 12 (c) 3 (d) 6
l 1  3cos 2  l 1  3cos 2  l l

11. A cylinder is sandwiched between two planks. Two constant horizontal forces F and 2F are applied
on the planks as shown. The acceleration of centre of mass of cylinder and the top plank are in a
ratio: (assume no slipping)
M
F

M
R
2M
2F
smooth
(a) 4:7 (b) 4:21 (c) 1:7 (d) 1:21

CENTERS: MUMBAI /DELHI /AKOLA /LUCKNOW /NASHIK /PUNE/NAGPUR /BOKARO /DUBAI # 118
12. A hoop of radius r and mass m is rolling without slipping with velocity v towards a step height b
  r  on a horizontal surface. Assume that the hoop does not rebound and no slipping occurs at the
point of contact when the hoop rolls up. What is the minimum velocity v needed for the hoop to roll
up the step.
r gb 4r gb 2r gb r gb
(a) (b) (c) (d)
 2r  b   2r  b   2r  b  r  b
13. A uniform cylinder of mass ‘m’ radius ‘r’ rolls on a fixed cylindrical
surface of radius ‘R’. At a certain instant, the line OC has an angular
velocity '  ' and angular acceleration  . Acceleration of point contact p
with respect to the surface is.

2
R  R  r  2 R  r 2 R 2 2 Rr2
(a) (b) (c) (d)
r r R  r R  r
14. A uniform rod of length 2l and weight W is lying across two sharp contacts on the same level distant
d apart. If neither sharp contact can stand a force greater than T, length of the rod which can project
beyond either sharp contact cannot be greater than
d  W  2T  2d  W  T  d  2W  T  d W  T
(a) l - (b) l - (c) l - (d) l -
W W 2W W

15. A uniform rod is placed with one end on the edge of a table a nearly vertical position is then released
from rest. Find the angle it makes with the vertical at the moment it loses contact with the table.
The edge of the table is sufficiently rough so that rod doesn’t slip before loses contact

 2 1  3 2
(a) cos 1   (b) cos 1   (c) cos 1   (d) cos 1  
5 2 5  3. 

16. From circular disc of radius R and mass 9M, a small disc of radius R/3 is removed from the disc. The
moment of inertia of the remaining disc about an axis perpendicular to the plane of the disc and
passing through O is

2R 3

40 37
(a) 4MR 2 (b) MR 2 (c) 10MR 2 (d) MR 2
4 9

CENTERS: MUMBAI /DELHI /AKOLA /LUCKNOW /NASHIK /PUNE/NAGPUR /BOKARO /DUBAI # 119
17. A small solid sphere of radius r can roll without slipping on the inner surface of a fixed large
spherical shell of radius R  R  r  . The period of small oscillations of the solid sphere around its
equilibrium position is given by.
7 R  r 3 R  r  7 R  r  5 R  r 
(a) 2 (b) 2 (c) 2 (d) 2
5g 2g 2g 2g

18. Find the natural frequency of oscillation of the system as shown in figure. Pulleys are massless and
frictionless. Springs and strings are also massless.

1 16k 1 8k 1 21k 1 4k
(a) (b) (c) (d)
2 m 2 m 2 m 2 m

19. A small bob attached to a light inextensible thread of length l has a periodic time T when allowed to
vibrate as a simple pendulum. The thread is now suspended from a fixed end O of a vertical rigid rod
of length 3l 4 . If now the pendulum performs periodic oscillations in this arrangement, the periodic
time will be
O

3l 4 l

(a) 3T/4 (b) 4T/5 (c) 2T/3 (d) 5T/6

20. A thin rod of length L and uniform cross-section is pivoted at its lowest point P
inside a stationary homogeneous and non-viscous liquid. The rod is free to
rotate in a vertical plane about a horizontal axis passing through P. The density
d2
d1 of the material of the rod is smaller than the density d2 of the liquid. The rod
is displaced by small angle  from its equilibrium position and then released.
The motion of the rod is simple harmonic, its angular frequency is  then P

5g  d 2  d1  5g  d 2  d1 
(a)   (b)  
2d1L 3d1L
2g  d 2  d1  3g  d 2  d1 
(c)   (d)  
d1L 2d1L

CENTERS: MUMBAI /DELHI /AKOLA /LUCKNOW /NASHIK /PUNE/NAGPUR /BOKARO /DUBAI # 120
21. A thin rod of mass M and length L is hinged at its upper end, and released from rest from a
horizontal position. The tension at a point located a distance L/3 from the hinge point, when the rod
becomes vertical, will be
(a) 22Mg/27 (b) 11Mg/13 (c) 6Mg/11 (d) 2Mg

22. Two identical simple pendulums A and B are fixed at same point. They are displaced by an angle α
and β (α and β are very small and β > α) and released from rest. Find the time after which B reaches
its initial position for the first time. Collisions are elastic and length of the strings is l.

 

B
A

    2 
(a)  (b) 2  (c) (d)
g g  g  g

23. A child is standing on the edge of a merry- go-around that has the 
shape of a disk, as shown in the figure. The mass of the child is 40
kilograms. The merry-go-around has a mass of 200 kilograms and a
radius of 2.5 meters, and it is rotating with an angular velocity of
  2.0 radians per second. The child then walks slowly toward the
center of the merry-go-round. What will be the final angular velocity
of the merry-go-around when the child reaches the center? (The size
of the child can be neglected)
(a) 2.0 rad s (b) 2.2 rad s (c) 2.4 rad s (d) 2.8 rad s

24. An uniform semi-solid sphere is placed with flat surface on rough inclined plane as shown in figure.
If friction is sufficient to prevent slipping, then the minimum angle  at which toppling occur is


1 8  4
(a) tan 1   (b) 45 (c) tan 1   (d) tan 1  
 2  3  3
25. A solid sphere and a solid cylinder having the same mass and radius roll down the same incline. The
ratio of their acceleration is
(a) 14 :15 (b) 1: 2 (c) 15 :14 (d) 2 :1

26. A wheel 4m in diameter rotates about a fixed frictionless horizontal axis, T


2
about which its moment of inertia is 10 kg m . A constant tension of 40 N is
maintained on a rope wrapped around the rim of the wheel. If the wheel starts
from rest at t  0 s , find the length of rope unwounded till t  3s
(a) 36.0 m (b) 72.0 m (c) 18.0 m (d) 720 m
CENTERS: MUMBAI /DELHI /AKOLA /LUCKNOW /NASHIK /PUNE/NAGPUR /BOKARO /DUBAI # 121
27. A particle of mass m is projected with velocity v making an angle of 45 with the horizontal. The
magnitude of angular momentum of the projectile about the point of projection when the particle is at
its a maximum height is
mv3 mv3
(a) zero (b) (c) (d) none
4 2 g  2g
28. A solid sphere of radius r and coefficient of linear expansion  rotates about an axis passing through
it’s centre in absence of any external torque. What should be the change in temperature so that it’s
angular velocity increases by 1%.
1 1 1 1
(a) increase (b) increase (c) decrease (d) decrease
100 200 100 200

29. A particle of mass m is released from rest at point A in the figure falling freely under gravity parallel
to the vertical Y axis. The magnitude of angular momentum of particle about point O when it reaches
B is: (Where OA  b and AB  h )
O b A


h

Y B

(a) mh  bg (b) mb 2gh (c) mh 2gh (d) None of these

30. The coefficient of friction between block of mass m and 2 m is


  2 tan  . There is no friction between block of mass 2m and inclined
plane. The maximum amplitude of two block system for which there is
no relative motion between both the blocks.
k mg sin 
(A) g sin  (B)
m k
3mg sin 
(C) (D) None of these
k

31. A thin tablecloth covering a horizontal table and a uniform body of round
shape lies on top of it. The tablecloth is pulled from under the body, and
friction causes the body to slide and rotate. What is the body’s final motion on
the table? (Assume that the table is so large that the body does not fall off it.)
(a) Body will finally roll towards left.
(b) Body will finally, roll towards right.
(c) Body will finally come to rest
(d) Any of the above is possible depending on shape of body

32. A small sphere D of mass m and radius r rolls without slipping inside a large
fixed hemispherical radius R   r  as shown in figure. If the sphere starts from
rest at the top point of the hemisphere normal force exerted by the small sphere
on the hemisphere. When its is at the bottom B of the hemisphere, is
10 17 5 7
(a) mg (b) mg (c) mg (d) mg
7 7 7 5
CENTERS: MUMBAI /DELHI /AKOLA /LUCKNOW /NASHIK /PUNE/NAGPUR /BOKARO /DUBAI # 122
33. A hollow smooth uniform sphere A of mass ‘m’ rolls without sliding on a
smooth horizontal surface collides head on elastically with another
stationary smooth solid sphere B of the same mass m and same radius. The
ratio of kinetic energy of ‘B’ to that of ‘A’ just after the collision is
(a) 1: 1 (b) 2 : 3 (c) 3 : 2 (d) None

34. A plank of mass M is placed over smooth inclined plane and a sphere is
placed over the plank. Friction is sufficient to prevent slipping between
sphere and plank. If plank and sphere are released from rest, the frictional
force on sphere is-
(a) up the plane (b) down the plane (c) horizontal (d) zero

35. The moment of inertia of a solid cylinder about its axis is given by 1 2  MR 2 . If this cylinder rolls
without slipping the ratio of its rotational kinetic energy to its translational kinetic energy is-
(a) 1: 1 (b) 2: 3 (c) 1: 2 (d) 1: 2

36. A force F is applied to a dumbbell for a time interval, t first as in (i) and
then as in (ii). In which case does the dumbbell acquire the greater centre
– of – mass speed?
(a) (i)
(b) (ii)
(c) there is no difference
(d) the answer depends on the rotational inertia of the dumbbell

37. A hoop and a solid cylinder have the same mass and radius. They both roll, without slipping, on
horizontal surface. If their kinetic energies are equal-
(a) the hoop has a greater translational speed than the cylinder
(b) the cylinder has a greater translational speed the hoop
(c) the hoop and the cylinder have the same translational speed
(d) the hoop has a greater rotational speed than the cylinder
2m
38. In the pulley system shown, if radii of the bigger and smaller pulley are 2 m and 1 1m
m respectively and the acceleration of block A is 5 m / s 2 in the downward
direction, then the acceleration of block B will be :
(A) 0 m / s 2 (B) 5 m / s 2
(C) 10 m / s 2 (D) 5 / 2 m / s 2 A B

39. A sphere is released on a smooth inclined plane from the top. When it moves down its angular
momentum is:
(A) conserved about every point
(B) conserved about the point of contact only
(C) conserved about the centre of the sphere only
(D) Conserved about any point on a line parallel to the inclined plane and passing through the centre
of the ball.

40. A smooth tube of certain mass is rotated in gravity free space and released. 
The two balls shown in the figure move towards ends of the tube. For the
whole system which of the following quantity is not conserved.
(A) Angular momentum (B) Linear momentum
(C) Kinetic energy (D) Angular speed
CENTERS: MUMBAI /DELHI /AKOLA /LUCKNOW /NASHIK /PUNE/NAGPUR /BOKARO /DUBAI # 123
41. A small block of mass ‘m’ is rigidly attached at ‘P’ to a ring of mass
P
‘3m’ and radius ‘r’. The system is released from rest at   90 o
and rolls without sliding. The angular acceleration of hoop just after 
release is –
g g g g
(A) (B) (C) (D)
4r 8r 3r 2r

42. A uniform rectangular plate of mass m which is free to rotate about the
Smooth vertical hinge passing through the centre and perpendicular to
the plate, is lying on a smooth horizontal surface. A particle of mass m
moving with speed ‘u’ collides with the plate and sticks to it as shown
in figure. The angular velocity of the plate after collision will be:

12 u 12 u 3u 3u
(A) (B) (C) (D)
5 a 19 a 2a 5a

43. Statement -1: The moment of inertia of any body is minimum about axis which passes through its
centre of mass as compared to any other parallel axis.
Statement -2: The entire mass of a body can be assumed to be concentrated at its centre of mass for
applying Newtons force Law.
(a) Statement -1 is true, statement- 2 is true and statement -2 is correct explanation for statement -1
(b) Statement- 1 is true, statement -2 is true and statement -2 is NOT the correct explanation for
statement -1.
(c) Statement-1 is true, statement-2 is false.
(d) Statement-1 is false, statement-2 is true.

44. The circular motion of a particle with constant speed is


(a) periodic but not SHM (b) SHM but not periodic
(c) periodic and SHM (d) neither periodic nor SHM

45. In the figure, the block of mass m, attached to the spring of stiffness k is in
contact with the complete elastic wall, and the compression in the spring is
‘e’. The spring is compressed further by ‘e’ by displacing the block towards
left and is then released. If the collision between the block and the wall is
completely elastic then the time period of oscillations of the block will be –
2 m m
(a) (b) 2
3 k k
 m  m
(c) (d)
3 k 6 k

46. A rod whose ends are A and B of length 25cm is hanged in vertical plane. When hanged from point
A and point B the time periods calculated are 3 sec and 4 sec respectively. Given the moment of
inertia of rod about axis perpendicular to the rod is in the ratio 9: 4 at points A and B. Find the
distance of the centre of mass from point A.
(a) 9cm (b) 5cm (c) 25cm (d) 20cm

CENTERS: MUMBAI /DELHI /AKOLA /LUCKNOW /NASHIK /PUNE/NAGPUR /BOKARO /DUBAI # 124
47. A wire frame in the shape of an equilateral triangle is hinged at one vertex so that it can swing freely
in a vertical plane, with the plane of the  always remaining vertical. The side of the frame is 1 3
m. The time period in seconds of small oscillations of the frame will be –
  
(a) (b)  2 (c) (d)
2 6 

48. A circle disc has a tiny hole in it, at a distance z from its center. Its mass is M and radius R (R > z). A
horizontal shaft is passed through the hole and held fixed so that the disc can freely swing in the
vertical plane. For small disturbance, the disc performs SHM whose time period is minimum for z =
(a) R/2 (b) R/3 (c) R 2 (d) R 3

49. A particle is subjected to two mutually perpendicular simple harmonic motions such that its x and y
 
coordinates are given by x  2sin t; y  2sin  t  
 4
The path of the particle will be:
(a) an ellipse (b) a straight line (c) a parabola (d) a circle

50. In the figure shown, the springs are connected to the rod at one end and at
the midpoint. The rod is hinged at its lower end. Rotational SHM of the rod
(Mass m, length L ) will occur only if-
(a) k  mg 3L (b) k  2mg 3L
(c) k  2mg 5L (d) k  0

51. Two simple harmonic motions y1  Asin t and y2  Acos t are superimposed on a particle of
mass m. The total mechanical energy of the particle is:
1 1
(a) m2 A 2 (b) m2 A 2 (c) m2 A 2 (d) zero
2 4

52. A particle is executing SHM of amplitude A, about the mean position x  0 . Which of the following
cannot be a possible phase difference between the positions of the particle at x  A / 2 and
x  A / 2 .
(a) 75o (b) 165o (c) 135o (d) 195o

53. A particle performs S.H.M. on x-axis with amplitude A and time period T. The time taken by the
particle to travel a distance A/5 starting from rest is :
T T 4 T 1 T 1
(A) (B) cos 1   (C) cos 1   (D) sin 1  
20 2 5 2 5 2 5

54. A uniform rod of mass m and length lis hinged at its mid point in such a
way that it can rotate in the vertical plane about a horizontal axis passing
through the hinge. One of its ends is attached to a spring of spring
constant k which is unstretched when the rod is horizontal. If this end is
now given a small displacement and released angular frequency of the
resulting motion is
K 2K 3K g
(A) (B) (C) (D)
m m m l

CENTERS: MUMBAI /DELHI /AKOLA /LUCKNOW /NASHIK /PUNE/NAGPUR /BOKARO /DUBAI # 125
55. The period of the free oscillations of the system shown here if mass M1
is pulled down a little and force constant of the spring is k and masses
of the fixed pulleys are negligible, is
M1  M 2 M1  4M 2
(A) T  2 (B) T  2
k k
M 2  4M1 M 2  3M1
(C) T  2 (D) T  2
k k

56. A rod of mass M and length is hinged at its one end and carries a block of mass m at its other end. A
spring of force constant k1 is installed at distance a from the hinge and another of force constant k 2
at a distance b as shown in the figure. If the whole arrangement rests on a smooth horizontal table
top, the frequency of vibration is
k2
b
a

k1 M

m
2 2
1 k 1a  k 2 b 1 k 2  k1
(A) (B)
2  M 2 Mm
L2  m  
 3
a2 k 2b2
k 2  k1 k1 
1 b2 1 a2
(C) (D)
2 M 2 4
4 m mM
3 3

57. m1 & m 2 are connected with a light inextensible string with m1 lying on smooth table and m2
hanging as shown in figure. m 1 is also connected to a light spring which is initially unstretched and
the system is released from rest.
k
m1

m2

k  m1  m2 
(A) system performs SHM with angular frequency given by
m1m2
k
(B) system performs SHM with angular frequency given by
m1  m 2
(C) tension in string will be 0 when the system is released
mg
(D) maximum displacement of m1 will be 2
k

CENTERS: MUMBAI /DELHI /AKOLA /LUCKNOW /NASHIK /PUNE/NAGPUR /BOKARO /DUBAI # 126
58. Statement-1: Motion of ball bouncing elastically in vertical direction on a smooth horizontal floor is
a periodic motion but not an SHM.
Statement-2: Motion is SHM when restoring force is proportional to displacement from mean
position.
(a) Statement -1 is true, statement- 2 is true and statement -2 is correct explanation for statement -1
(b) Statement- 1 is true, statement -2 is true and statement -2 is NOT the correct explanation for
statement -1.
(c) Statement-1 is true, statement-2 is false.
(d) Statement-1 is false, statement-2 is true.

59. Statement -1: A particle, simultaneously subjected to two simple harmonic motions of same
frequency and same amplitude, will perform SHM only if two SHM’s are in the same direction.
Statement -2: A particle simultaneously subjected to each other the particle can be in uniform
circular motion.
(a) Statement -1 is true, statement- 2 is true and statement -2 is correct explanation for statement -1
(b) Statement- 1 is true, statement -2 is true and statement -2 is NOT the correct explanation for
statement -1.
(c) Statement-1 is true, statement-2 is false.
(d) Statement-1 is false, statement-2 is true.

60. A tube of given shape has total length 2 and its area of cross-section is S.
Its bottom and upper halves are filled with two non-viscous, non-
compressible liquids of densities 3 and  respectively. If its motion is
simple harmonic then find its angular frequency.

2g 3g 2g g
(a) (b) (c) (d)
3 2  

61. In the given triangular sheet, PQ  QR  l and PQR  90 . If M is the mass of the sheet, then its
moment of inertia about PR is

l
P Q

Ml 2 Ml 2 Ml 2 Ml 2
(a) (b) (c) (d)
24 12 6 3

CENTERS: MUMBAI /DELHI /AKOLA /LUCKNOW /NASHIK /PUNE/NAGPUR /BOKARO /DUBAI # 127
MULTIPLE CHOICE QUESTIONS

1. A man pushes a solid cylinder of mass ' m1 ' with the help of a plank of mass 'm2 ' by applying a
horizontal force ‘F’ as shown. There is no slipping at any contact. Then the ratio of acceleration of
plank to that of centre of mass of cylinder and direction of friction at point 1 & 2 cylinder is given
by.
f
m2
2
m1

ap 1 f1  ap 2 f1 
(a)  (b) (c)  (d)
ac 2 f2  ac 1 f2 

2. A billiard ball rolling without slipping on a horizontal rough surface hits an identical, stationary
billiard ball in a head-on collision. Final speed of the balls after the collision are u1 & u 2 .
u1 : speed of original ball after collision, u 2 : speed of the second ball. Coefficient of friction between
surface and balls are same
2v0 2 2 5
(a) u1  (b) u1  v0 (c) u 2  v0 (d) u 2  v0
7 5 3 7

3. A solid cylinder is rolling down a rough inclined plane of inclination  . Then


(a) the friction force is dissipative
(b) the friction force is necessarily changing
(c) the friction force will aid rotation but hinder translation
(d) the friction force is reduced if  is reduced

4. A rod AC of length l and mass m is kept on a horizontal smooth plane. It is free to rotate and move.
A particle of same m moving on the plane with velocity V strikes the rod at point B making angle
37 o with the rod. The collision is elastic. After collision,

B l 4

A 37 C
V

72 V
(a) the angular velocity of the rod will be
55l
(b) the centre of the rod will travel a distance  3 in the time in which it makes half rotation
24 mV
(c) impulse of the impact force is
55
(d) none of these

CENTERS: MUMBAI /DELHI /AKOLA /LUCKNOW /NASHIK /PUNE/NAGPUR /BOKARO /DUBAI # 128
5. Two horizontal discs of different radii are free to rotate about their central vertical axes. One is given
some angular velocity, the other is stationary. Their rims are now brought in contact. There is friction
between the rims. Then
(a) the force of friction between the rims will disappear when they have equal angular speeds.
(b) the force of friction between the rims will disappear when they have equal linear velocities
(c) the angular momentum of the system will be conserved
(d) the rotational kinetic energy of the system will not be conserved

6. A body of mass m is attached to a spring constant k which hangs from the ceiling of an elevator at
rest in equilibrium. Now the elevator starts accelerating upwards with its acceleration varying with
time as a  pt  q , where p and q are positive constants. In the frame of elevator:
(a) the block will performs S.H.M. for all value of p and q
(b) the block will not performs S.H.M. in general for all value of p and q expect p  0
(c) the block will perform S.H.M. provided for all value of p and q expect p  0
(d) the velocity of the block will vary simple harmonically for all value of p and q

7. A particle is suspended by two ideal strings as shown in the figure. Now mass m is given a small
displacement perpendicular to the plane of triangle formed. Choose
7l

5l
3 2l

M
3 3l
(a) The period of oscillation of the system is 2
g
3l
(b) The period of oscillation of the system is 2
g
(c) The period of oscillation of the system is independent of M
(d) If the distance between the suspension points was kept constant and the length of the strings were
quadrupled then the period of the system will be double

8. For a body executing S.H.M. with amplitudes A, time period T, max velocity max and phase
constant zero, which of the following statements are correct?
(a) At y   A 2  ,    max 2  (b)    max 2 for y   A 2
(c) For t   T 8 , y   A 2  (d) For y   A 2 , t   T 8

9. Function x  A sin 2 t  B cos 2 t  C sin  t cos t represents SHM


(a) For any value of A, B and C (except C  0 )
(b) If A   B, C  2B , amplitude  B 2
(c) If A  B; C  0
(d) If A  B;C  2B , amplitude = B

CENTERS: MUMBAI /DELHI /AKOLA /LUCKNOW /NASHIK /PUNE/NAGPUR /BOKARO /DUBAI # 129
10. A uniform rod of mass m  2kg and length l = 0.5m is sliding along
two manually perpendicular smooth walls with the two ends P and Q
having velocities  p  4m s and  Q  3m s as shown. Then
(a) The angular velocity of rod,   10rad s , counter clockwise
(b) The angular velocity of rod,   5.0rad s , counter clockwise
(c) The velocity of centre of mass of rod, cm  2.5m s
25
(d) The total kinetic energy of rod, K  joule
3

11. Two simple harmonic motion are represented by equations:


Y1  10sin  3t   / 4; Y2  5 sin 3t  3 cos3t 
(a) The amplitude ratio of the two SHM is 1 : 1 (b) The amplitude ratio of the two SHM is 2 : 1
(c) Time period of both the SHMs are equal (d) Time period of two SHMs are different

12. Rod B sticks to rod A after collision. Collision takes placed on a


horizontal plane. Rod –A is hinged at O. friction is absent every where O
m, L
2v 0 A
(a) Angular velocity of system just after collision is
5L
9 v0
(b) Velocity of centre of mass of system just after collision is v0
40
m, L
10L
(c) Centre of mass of system is at a distance of from O at the
4 B
moment of collision.
9
(d) Kinetic energy of system just after collision is mv 20
40

13. A rod of weight w is supported by two parallel knife edges A and B and is in equilibrium in a
horizontal position. The knives are at a distance d from each other. The centre of mass of the rod is at
a distance x from A.
wx w d  x 
(a) the normal reaction at A is (b) the normal reaction at A is
d d
wx w d  x
(c) the normal reaction at B is (d) the normal reaction at B is
d d

14. A block with a square base measuring a and height h, is placed on an incline plane. The coefficient
of friction is  . The angle of inclination   of the plane is gradually increased. The block will
a a
(a) topple before sliding if   (b) topple before sliding if  
h h
a a
(c) Slide before toppling if   (d) slide before toppling  
h h

CENTERS: MUMBAI /DELHI /AKOLA /LUCKNOW /NASHIK /PUNE/NAGPUR /BOKARO /DUBAI # 130
   
15. The torque  on a body about a given point is found to be equal to A  L where A is a constant

vector and L is the angular momentum of the body about the point. From this its follows that –

dL 
(a) is perpendicular to L at all instants of time
dt  
(b) the components of L in the direction of A does not change with time
(c) the

magnitude of L does not change with time
(d) L does not change with time
16. A small ball of mass m suspended from the celling at a point O by a thread of
length l moves along a horizontal circle with a constant angular velocity  .
(a) linear momentum of system (ball + rod) is conserved
(b) angular momentum of system (ball + rod) about hinged point A is conserved
(c) kinetic energy of system (ball + rod) before the collision is equal to kinetic
energy of system just after the collisions
(d) linear momentum of ball is conserved

17. In the given figure a ball strikes a uniform rod of same mass elastically and A
rod is hinged at point A. then which of the statement(s) is/are correct?
(a) linear momentum of system (ball + rod) is conserved.
(b) angular momentum of system (ball + rod) about hinged point A is
conserved
(c) kinetic energy of system (ball + rod) before the collision is equal to
kinetic energy of system just after the collision U
(d) linear momentum of ball is conserved

18. A uniform disc is rolling on a horizontal surface. At a certain instant B is the


point of contact and A is at height 2R from ground, where R is radius of disc –
(a) The magnitude of the angular momentum of the disc about B is thrice that
about A
(b) The angular momentum of the disc about A anticlockwise
(c) The angular momentum of the disc about B is clockwise
(d) The angular momentum of the disc about A is equal to that about B.

19. The figure shows a graph between velocity and displacement (from mean
position) of a particle performing SHM:
(a) the time period of the particle is 1.57s
2
(b) the maximum acceleration will be 40cm s
(c) the velocity of particle is 2 21 cm s when it is at a distance 1cm from the mean position.
(d) none of these

20. Two springs with negligible masses and force constant of K1  200Nm 1 and
K 2  160Nm 1 are attached to the block of mass m  10kg as shown in the
figure. Initially the block is at rest, at the equilibrium position in which both
springs are neither stretched nor compressed. At time t  0 , a sharp impulse
of 50 Ns is given to the block with a hammer.

(a) Period of oscillations for the mass m is s.
3
(b) Maximum velocity of the mass during its oscillation is 5ms1

CENTERS: MUMBAI /DELHI /AKOLA /LUCKNOW /NASHIK /PUNE/NAGPUR /BOKARO /DUBAI # 131
(c) Data is insufficient to determine maximum velocity.
(d) Amplitude of oscillations is 0.42m.

21. A mass of 0.2kg is attached to the lower end of a massless spring of force-constant 200 N/m, the
upper end of which is fixed to a rigid support. Which of the following statements is/are true?
(a) In equilibrium, the spring will be stretched by 1cm.
(b) If the mass is raised till the spring is in its unstreched state and then released, it will go down by
2cm before moving upwards
(c) The frequency of oscillation will be nearly 5 Hz.
(d) If the system is taken to the moon, the frequency of oscillation will be the same as on the earth.

22. The potential energy of a particle of mass 0.1kg, moving along x-axis, is given by U  5x  x  4 J
where x is in metres. It can be concluded that
(a) the particle is acted upon by a constant force
(b) the speed of the particle is maximum at x  2m
(c) the particle executes simple harmonic motion
(d) the period of oscillations of the particle is  5 s

INTEGER TYPE

1. A circular hoop of radius ‘R’ rolls without slipping at constant speed on a horizontal surface. If p is
any point on the rim of this hoop, distance moved by point p in one full rotation of hoop is  R , then
value for is_______.

2. A cone of given vertical angle 2 , rests on a rough plane which is inclined to the horizontal. As the
inclination of the plane is increased, the cone will slide before it topples over, if the coefficient of
friction    tan  . Value of is____. O

3. A rod of mass m and length l is hinged at one end O. A particle of mass m


travelling with speed  collides with the rod at a distance x from the centre
of mass of the rod such that the reaction force at hinge is zero, then x  l  .
Value of is v
x
m

4. A block of mass m is attached to one end of light inextensible string passing over a smooth light
pulley B and under another smooth light A as shown in the figure. The another end of the string is
fixed to a ceiling. A and B are held by springs of springs constant k1 and k 2 . If R1  R 2  R angular
R
frequency of small oscillations of system is   where is.
m

k2
B
5. A spring of force constant k = 300 N/m connects two blocks having masses 2 kg and 3
kg, lying on a smooth horizontal plane. If the spring block system is released from a
A
stretched position, find the number of complete oscillations in 2 seconds. Take   10 .

k1

CENTERS: MUMBAI /DELHI /AKOLA /LUCKNOW /NASHIK /PUNE/NAGPUR /BOKARO /DUBAI # 132
ANSWER KEY

SINGLE CHOICE QUESTIONS

1. (a) 2. (a) 3. (b) 4. (b) 5. (a)

6. (c) 7. (a) 8. (c) 9. (b) 10. (b)

11. (d) 12. (a) 13. (a) 14. (d) 15. (c)

16. (a) 17. (a) 18. (a) 19. (a) 20. (d)

21. (d) 22. (b) 23. (d) 24. (c) 25. (c)

26. (b) 27. (b) 28. (d) 29. (b) 30. (c)

31. (c) 32. (b) 33. (c) 34. (d) 35. (c)

36. (c) 37. (b) 38. (d) 39. (d) 40. (d)

41. (b) 42. (d) 43 (b) 44. (a) 45. (a)

46. (d) 47. (d) 48. (c) 49. (c) 50. (c)

51. (b) 52. (c) 53. (b) 54. (c) 55. (c)

56. (a) 57. (b) 58. (a) 59. (d) 60. (d)

61. (c)

MULTIPLE CHOICE

1. (c,d) 2. (a,d) 3. (c,d) 4. (a,b,c) 5. (b,d)

6. (c,d) 7. (b,c) 8. (a,b,c,d) 9. (b,d) 10. (a,c,d)

11. (1,3) 12. (bc) 13. (bc) 14. (ad) 15. (abc)

16. (bcd) 17. (bc) 18. (ac) 19. (abc) 20. (ab)

21. (abcd) 22. (bcd)

INTEGER TYPE

1. [8] 2. [4] 3. [6] 4. [8] 5. [5]

CENTERS: MUMBAI /DELHI /AKOLA /LUCKNOW /NASHIK /PUNE/NAGPUR /BOKARO /DUBAI # 133
CRASH COUSE P – III (SOLUTION)
SINGLE CHOICE QUESTIONS
1. B eat time peri od
1
1 1

T 5T
4
= 5T
So after 5 oscillati ons of the smaller pendulum they will be agai n in same phase.

2. Frequency of fundamental tone wi ll be


v
f0 
21
340
or f0 
 2  2 
= 85 Hz
Other frequencies will be 2f 0 , 3f 0 , 4f 0 etc. or 170 Hz, 255 Hz, 340 Hz etc.

J
3. v
2m
Jl

2ml2
J

ml
 v A  v  l
J J
 
2m 2m
J

m
4. (2)
a a
Change in phase between x   and x   is 1  750
2 2
a
Change in phase between x   and x = +a is  2  600
2
  2
In an SHM, phase is changed at a constant rate therefore, 1 
t1 t2

5. Let M be the mass of the sphere. The mass of the disc will also be M. The moment inertia of the
sphere about its diameter is
2
Is  MR 2
5
The moment of inertial of the disc about its edge and perpendicular to its plane is (using parallel axes
theorem)
1
Id  Icm  Mh 2  Mr 2  Mr 2
2

CENTERS: MUMBAI /DELHI /AKOLA /LUCKNOW /NASHIK /PUNE/NAGPUR /BOKARO /DUBAI # 134
3
 Mr 2
2
Given I s  Id . Hence, we have
2 3
MR 2  Mr 2
3 2
r 2
Which gives 
R 15
6.

N mg
37


l 37 Vcm
2 37

 1 1  m 2  2 
mg
2
1  sin 37   0   m Vcm
2
   
2 2  12  
Centre of mass of rod will move in vertical direction.
Lower end is moving along the ground. So its velocity in vertical direction will be zero.

VCM  cos 37  0
2
96 g
Solving VCM 
365

7. If speed of centre just after collision is '  '


  eu  
u
 
2

Since wall is smooth impulse was only given horizontally by normal reaction wall. Just after
collision situations is shown in diagram.
u
Where  
r
Kinetic function acts on sphere towards right until pure rolling starts. If  ' is final speed
ft  m     ' 
2 
and  ' is final angular speed ftr   mr 2  1  
5 

CENTERS: MUMBAI /DELHI /AKOLA /LUCKNOW /NASHIK /PUNE/NAGPUR /BOKARO /DUBAI # 135
8.

A C B
l
At this instant, about ‘A’   mg
2
3
Since   I  g l
2
3g
Hence acceleration of C is downwards
4
Since acceleration of A is zero downwards
3g
areal 
4
3g
 ' about C is:  '  areal l 2 
2l
Tl
But about C  ' 
2
2
l  ml  mg
So, T  '  T
2  12  4

9. At B, total kinetic energy  mgh


Here, m  mass of ball
The ratio of rotational to translation kinetic energy would be,
KR 2 5
 mgh and K T  mgh
KT 7 7
In portion BC, friction is absent. Therefore, rotational kinetic energy will remain constant and
translational kinetic energy will convert into potential energy. Hence. If H be the height to which ball
climbs in mgH  K T
5 5
or mgH  mgh or H h
7 7

10. Let  &  are speed of centre and angular speed about centre after collision.
For elastic collision
l
 cos     0 
2 
About centre
l
J  m  0    J cos   I
2
Solving above equations we get
120 cos  f1

l 1  3cos 2  
11. for upper plank a
 m1R
f1  F  m  a  2R 
For lower plank
f2
CENTERS: MUMBAI /DELHI /AKOLA /LUCKNOW /NASHIK /PUNE/NAGPUR /BOKARO /DUBAI # 136
2F  f 2  2m  a  2R 
For cylinder
m
 mR 2  f2
f 2  f1  ma &  f1  f 2  R     f1
 2 
Solving 4 equations we get 1
21F 2F f2
a  F 26m & a  2R  2m
26

12. After the collision the hoop rotates around the collision point A.

In order to know whether the hoop can roll up the step or not we to know the angular velocity
immediately after the collision.
Angular momentum w.r.t. A before collision
L1  IC1  m  r  b   (ii)
Where I C  moment of inertia about the centre of hoop
Angular momentum after collision w.r.t. A
L 2  I C2  mr 2
L 2   I C  mr 2  2 , putting I C  mt (iii)
2   2r  b  V  2r 2 
And kinetic energy after collision
1
2
 
IC  mr 2 22  mgb

2r gb 2r gb
  min 
 2r  b   2r  b 

13. Speed of C:     R  r 
Speed of p w.r.t. C
 2 C 
2  R  r 
a pc  upwards '
  r
But a p  a pc  a c
2 P
2  R  r  2
upwards   R  r  downwards
r
 2 R  R  r 
ap  upwards
r

CENTERS: MUMBAI /DELHI /AKOLA /LUCKNOW /NASHIK /PUNE/NAGPUR /BOKARO /DUBAI # 137
14. MN is the uniform rod of the length 2land weight W lying across the sharp contact O and P.
Let MO be the maximum length of the rod projected beyond the sharp contact O is maximum and
equal to T (given). Let R be the stress on the other sharp contact P.

d
Let O G  x , then GP   d  x  , now T and R are two parallel forces balanced by weight W of
the rod, so we have
TR  W (i)
and T.OG  R .GP (ii)
from Eqs. (i) and (ii), we get
T  x    W  T  d  x   OG  x, GP   d  x  
 T  x    W  T d  x  W  T    W  T  x  Tx   W  T  d
 W  T  T x  W  Td  Wx   W  T  d


W  Td (iii) 
x M O  M G  OG
W
 
W  Td [from (iii)]
MO  l - x MO  l -
W

15. As the edge of the table is now a very small quarter-circle, the normal force N is always directed
along the rod’s axis. The static frictional force Ffr is tangential to this quarter-circle and can have any
arbitrary value because of the rough edge.

The sum of the component of the rod’s weight alone the rod and te normal force of the table gives
the centripetal force:
 3
Mg cos   N  Ma c  M 2  Mg 1  cos  
2 2
We can thus express the normal force as
Mg
N 5cos   3
2
3
The reaction of the table on the rod becomes zero when N  0 , i.e. when   cos1  53o . At larger
5
angles the normal force should be negative which is impossible, and thus the rod loses contact with
CENTERS: MUMBAI /DELHI /AKOLA /LUCKNOW /NASHIK /PUNE/NAGPUR /BOKARO /DUBAI # 138
the table. Because of the rough edge, the static frictional force is always large enough to prevent
slipping except when the normal force becomes zero; consequently, it has no effect on the motion.

16. Let  be the mass per unit area. The total mass of the disc   R 2  9M

M  Mass of the circular disc cut


2
R R 2
      M
3 9
Let us consider the above system as a complete disc of mass 9M and a negative mass M
superimposed on it.
Moment of inertia  I1  of the complete disc 9MR 2 2 about an axis passing through O and
perpendicular to the plane of the disc.
MI of the cutout portion about an axis passing through O ' and perpendicular to the plane of disc is
2
1 R 
M 
2 3
Therefore, MI  I2  of the cutout portion about an axis passing through O and perpendicular to the
plane of disc is
2 2
1 R  2R  
  M   M   
 2 3  3  
[Using perpendicular axis theorem]
Therefore, the total MI of the system about an axis passing through O and perpendicular to the plane
of the disc is
I  I1  I 2
2 2 2
1 2
1 R  2R   1  1 4
 9MR    M     M      9MR 2  MR 2   
2  2 3  3   2 18 9 

17. Let at equilibrium position point A of the spherical shell and point D of sphere are in contract.
Then r  R 

Where  is the angle by which sphere is distributed.

CENTERS: MUMBAI /DELHI /AKOLA /LUCKNOW /NASHIK /PUNE/NAGPUR /BOKARO /DUBAI # 139
Angle of rotation of sphere from vertical direction is  .
R r
So ,     
r
Equation of motion of centre of mass of small sphere in the tangential direction
d 2
m  R  r  2   mg sin   f (i)
dt
For the rotation of sphere
2 2 d2
mr  fr (ii)
5 dt 2
From Eqs. (i) and (ii)
d 2 2 d 2
mR  r 2   mg sin   mr 2
dt 5 dt
2
2 d 
  mg sin   m  R  r  2 (iii)
5 dt
7 d2
 R  r  2   sin 
5 dt
Since,  is small so sin   
7 d2 d2 5g
 R  r  2  g 2
 
5 dt dt 7 R  r 
2 2
2  T
 
7 R  r
T  2
5g

18.

for same displacement two systems shown in figure are identical.


Let block is displaced by x then spring will stretch by 4 x .
Thus force on block will be 16x
Hence 16kx  k eq x
16k 2 1 16k
and    2 
m T 2 m
l
19. T  2
g
T T'
Tnew   T: period for l length
2 2

CENTERS: MUMBAI /DELHI /AKOLA /LUCKNOW /NASHIK /PUNE/NAGPUR /BOKARO /DUBAI # 140
l
T' : period of length
4
l l 3
  Tnew  T
g 4g 4

20. Let S be the area of cross-section of the rod


In the displaced position, as shown in figure, weight (W) and upthrust  FB  both pass through its
centre of gravity G.
Here,
W  (volume) (density of rod) g   SL  d1  g
FB  (Volume) (density of liquid) g   SL  d 2  g
Given that, d1  d 2 . Therefore, W  FB
Therefore, net force acting at F will ne
F  FB  W   SLg  d 2  d1  upwards. Restoring torque of this force about point P is
L 
  F  r   SLg  d 2  d1  QG      SLg  d 2  d1   sin  
Or
2 
Here, negative sign shows the restoring nature of torque.
 SL2 g  d 2  d1  
Or     (i)
 2 
sin    for small values of 
From Eq. (i), we see that   
Hence, motion of the rod will be simple harmonic.
Rewriting Eq. (i) as
2
d 2  SL g  d 2  d1  
I 2    (ii)
dt  2 
Here, I  moment of inertia of rod about an axis passing through P.
2
ML2  SLd1  L
I 
3 3
Substituting this value of I in Eq. (ii), we have
d 2  3 g  d 2  d1  
2
  
dt 2 d1L 
Comparing this equation with standard differential equation of SHM, ie,
d 2
2
 2 
dt
The angular frequency of oscillation is
3g  d 2  d1 
 A
2d1L
21. By energy conservation T
m1l L
P
3
l 1 ml 2
mg    2
2 2 3
B

CENTERS: MUMBAI /DELHI /AKOLA /LUCKNOW /NASHIK /PUNE/NAGPUR /BOKARO /DUBAI # 141
3l

l
Tension supports weight and circular motion of part PB
l
2 m
T  mg   2 xdx
3 l l
3
22. (b)
Since collision 5s elastic & masses are identical velocities & hence these amplitudes exchange.

23. (d)
By conservation of angular momentum
1 2 2 1 2
 MR  mR  w 0  MR 
2  2

24. (c)
3R
For toppling mgsin   mg cos R
8
8
tan  
3
25. (c)
g sin  g sin 
as  
I 7
1 2
mR
2
a c  gsin 
3

26. (b)
40  2  10  ………..(1)
1
  t 2  36
2
I  r  72 m

27. (b)

u cos 

mu sin 2  u
2g cos 

28. (d)
I  Constant
2 2
mR 2  mR '2 '
5 5
2R 
 0
R 
R 1
 4T  
R 200
CENTERS: MUMBAI /DELHI /AKOLA /LUCKNOW /NASHIK /PUNE/NAGPUR /BOKARO /DUBAI # 142
29. (b)
Sol.

Velocity at B  2gh
 angular momentum  m  2gh  b

30. (c)
Sol. The maximum static frictional force is
f  mg cos   2 tan mg cos   2mg sin  means position

A
extreme position
Applying Newton’s second law to block at lower extreme position f
q mg sin q
2 2
f  mg sin   m A  f  m A  mg sin 
3mg sin 
Or 2 A  gsin  or A
k
31. (c)
Sol.

So body will finally come to rest.

32. (b)
Sol. From energy conservation gain of K.E.  loss of P.E.
1  K2  1  2 10
mv 2  1  2   mgR  mv 2  1    mgr  v gR
2  R  2  5 7
For circular motion of bottom
mv 2 m 10 17
N  mg   N  mg   gR  N mg
R R 7 7

33. (c)
Sol. Due to collision only translational K.E. of A will transfer to B.
1 2
K.E.B 2 mv mv2 K.E.B 3
  2  
K.E.A 1 I2 2 2 v K.E.A 2
mR  2
2 3 R

34. (d)
Sol. Case I
we suppose friction is acting as shown in figure.

CENTERS: MUMBAI /DELHI /AKOLA /LUCKNOW /NASHIK /PUNE/NAGPUR /BOKARO /DUBAI # 143
Point of contact must be at rest
ap  ap ' (1)
f f 
a p  g sin    R  a p  g sin   R
m m I
f Rf f  mR 2 
a p  gsin    R  a p  g sin   1  
m I m I 
f
a p '  gsin    for a p  a p '  f  0
m
Case II
mgsin   f Rf
ap  R
m I
f  mR 2 
a p  g sin    1  
m 1 
f
a p '  gsin   for a p  a p '  f  0
m

35. (c)
1 1 1 v2 1 1
Sol. K R  I2  KR   mR 2 2  K R  . mv 2
2 2 2 R 2 2
1 KR 1
KR  KT  
2 KT 2

36. (c)
Ft
Sol. a cm 
2m
Which is same in both cases.
vcm  a cm t
So speed of centre of mass will be same in both cases

37. (b)
Sol. KE hoop  KE cylinder
1 2  1 
 2 mvcm 1  K R   1  K 2 R 2 
2 2 2
  mv 'cm
  hoop  2  cylinder
1 2 1 2  1
mvcm 1  1  mv 'cm 1  
2 2  2
1 3 1 4 2
2  mvcm 2   mv 'cm 2  v'cm2  vcm 2  v 'cm  v cm
2 2 2 3 3

38. (d)
Sol. Given a A  2  5 m / s 2
  5/ 2 rad / s2  a B  1.     5 / 2 m / s 2

CENTERS: MUMBAI /DELHI /AKOLA /LUCKNOW /NASHIK /PUNE/NAGPUR /BOKARO /DUBAI # 144
39. (d)
Sol. As the inclined plane is smooth, the sphere can never roll rather it will just slip down.
Hence, the angular momentum remains conserved about any point on a line parallel to the inclined
plane and passing through the centre of the ball.

40. (d)
Sol. As    0; Angular momentum, linear momentum remains conserved.
As the two balls will move radially out moment of inertia changes. In order to keep the angular
momentum
 L  I Conserved, angular speed    should change.
41. (b)
Sol. f = 4 ma …(1)
 mg  f  r   3mr 2  mr 2  
mg – f = 4 ma …(2)
g g
From (1) and (2)  8ma  mg a 
8 8r

42. (d)
Sol. conserving the angular momentum
 m  a 2  4a 2  5 2  3 u
ua    ma    
 12 4  5 a
43. (b)

44. (a)

45. (a)
Sol. If wall is not there than (spring + block) will oscillate about mean position O with amplitude 2e.
Time taken to strike wall starting from rest.
y  a cos t
2  2
e  2ecos t   tt T 6
T 3 T
2T T 2 m
Time period of oscillation  2t   
6 3 3 k

46. (d)
Sol.  R   mg sin  x , here x is the distance of centre of gravity from end A
I  mg  
mgx mgx
   
I I
I TA I  25  x 
T  2   A
mgx TB IB x
3 9  25  x 
   x  20cm
4 4 x

CENTERS: MUMBAI /DELHI /AKOLA /LUCKNOW /NASHIK /PUNE/NAGPUR /BOKARO /DUBAI # 145
a
47.   3mg sin       3mag
3
 3mag
Sol. I   3mag  
I
3mga
 (1)
I
3
I  ma 2 (2)
2
From (1) and (2)
3mga 2g
  
3 2 3a
ma
2
2 10
    20
1
3
3
2 
T  T sec
20 5

48. (c)
I I G  MZ 2
Sol. T  2  T  2
mgZ mgZ
For T to be minimum
d  I G  MZ2  R
 0  Z
dZ  MgZ  2

49. (c)
Sol. Equation of trajectory
x 2 y 2 2xy
  cos   sin 2 
a 2 b 2 ab

For  
4
Trajectory will be ellipse.

50. (c)
L
Sol. x2   and x1  L
2
 L L 
R    kx1L cos   kx 2 cos   mg sin 
 2 2 
 L L mg 
R    k  L  L cos   k    cos   Lsin 
 2 2 2 
 5 mgL 
R    kL2 cos   sin 
 4 2 
For small , cos   1 & sin   

CENTERS: MUMBAI /DELHI /AKOLA /LUCKNOW /NASHIK /PUNE/NAGPUR /BOKARO /DUBAI # 146
5 mgL 
R    kL2  
4 2 
For rotational SHM
5 2 mgL 2 mg
kL  0  k
4 2 5 L

51. (b)
Sol. A R  2A, R  
1 2 1
T.E.  m2 2A
2
   2 m2 A 2  T.E.  m2 A 2
2

52. (c)
Sol. Possible phase difference
  225o  30o   195o
or   315o  30 o   285o
or   225o  150o   75o
or   315o  150o   165o

53. (b)
Sol. Particle is starting from rest, i.e. from one of its extreme position.
A
As particle moves a distance , we can represent it on a circle as shown.
5
4A / 5 4 4
cos      cos 1  
A 5 5

 4 1 4
t  cos 1   t  cos 1  
5  5
T 4
 cos 1  
2 5
Method II: As starts from rest i.e. from extreme position x = A
sin  t  
 A
At t = 0 ; x = A   A  A cos t
2 5
4 4
 cos t   t  cos 1
5 5
T  4
t cos 1  
2 5
4A / 5 4 4
cos 0   0  cos 1  
A 5 5
4 1 4
t  cos 1   t  cos1  
5  5
T 4
 cos 1  
2 5
CENTERS: MUMBAI /DELHI /AKOLA /LUCKNOW /NASHIK /PUNE/NAGPUR /BOKARO /DUBAI # 147
54. (c)
Sol. Restoring torque is given by.
l
KX.    I 
2
2
 l  l  ml
K      .  
 2  2  12
3k 3K
   
m m

55. (c)
Sol. If the mass is displaced by x and has speed v 1 spring will extend by x/2 & M 2 will have speed v/2
Energy of this system can be written as
2 2
1 x 1 v 1 x
 M1gx  M1v 2  M 2 g  M 2    K    const.
2 2 2 2 2 2
1 M 1 1
M 1gx  M 1 v 2  2 gx  M 2 v 2  Kx 2  const.
2 2 8 8
Differentiate w.r.t. time
dv M 2 g 1 dv 1
 M 1gv  M 1 v  v  M2v  Kx.v  0
dt 2 4 dt 4
dv M 2 g M 2 dv 1
  M 1g  M 1    Kx  0
dt 2 4 dt 4
 M 2  dv K Mg
 M1    x  M1g  2
 4  dt 4 2
dv  K  2  2M1  M 2  g
a   x 
dt  4M1  M 2  4M1  M 2
K 2 4M1  M 2
Here 2  T  2
4M1  M 2  D

56. (a)
Sol. For small angular displacement  , net torque towards mean position is

   k1a  a   k 2 b  b or I   k1a 2  k 2 b 2  
1
Or (mL2  ML2 )  (k 1a 2  k 2 b 2 )
3
k 1a 2  k 2 b 2 k a 2  k 2 b2
Or   2  1
 M  M
L2  m    L2  m  
 3  3 
 1 k 1a 2  k 2 b 2
Hence frequency,  
2  2  M
L2  m  
 3 
57. (b)
Sol. after the system is released, m 2 moves down.
The extension in the spring becomes :

CENTERS: MUMBAI /DELHI /AKOLA /LUCKNOW /NASHIK /PUNE/NAGPUR /BOKARO /DUBAI # 148
m 2g
 m 2 g  kx 0  which is the new equilibrium position of the system. T
k
For small ‘x’ : restoring force on the system is
F = kx
kx
 a (for  m1  m 2  spring  system) m2
m1  m 2
x x  m1  m 2  m1  m 2
 T  2  2  2
a kx k m2g
2 k
 Angular frequency =   
T m1  m 2
F.B.D. of m1 and m 2 just after the system is released :

m1 T
k  0  0
From above : T= 2 g 2
Hence (C) is incorrect.

m 2g
After x  ; m1 moves towards right till the total kinetic energy acquired does not converted to
k
potential energy.
Hence (D) is also incorrect. Hence (B) is answer.

58. (a)
Sol. For SHM Fnet on ball must   x while Fnet on ball is Mg.

59. (d)
Sol. Two perpendiculars SHM of same frequency and same phase result in circular motion. If out of
phase by  2

60. (d)
F = – 2 s(3 +)g x = – 4sgx
m = s + s3 = 4s
4sg
4 s  g
= =

61. (C)
By conservation of linear momentum

MULTIPLE CHOICE QUESTIONS f2

1. f 2 will obviously act towards right on cylinder. m2 m1


Lets take f1 on cylinder to left
f 2  f1  m1a
 m1R 2  f1
 f1  f 2  R    
 2 

CENTERS: MUMBAI /DELHI /AKOLA /LUCKNOW /NASHIK /PUNE/NAGPUR /BOKARO /DUBAI # 149
 F  f 2   m2  a  2R 
But a    2R [no slipping]
Solving these equations we get
4F 8F
a hence a p 
3m1  8m 2 3m1  8m2
3m1F m1F
f2  f1 
3m1  8m 2 3m1  8m2

2. Friction between the two balls is negligible, and so during the collision, they can only exert forces
normal to their surfaces. Thus, the first ball stops after the collision, while the second acquires the
first’s initial speed after the collision, the first ball rotates on one spot and the second slides without
rotation at speed 0 .

The friction between the balls and the table is of course important and affects the motion of the
balls. The first is accelerates forward by the force of kinetic friction Ffr  mg , whilst the second in
reduced by friction, lasts until both balls reach the state of rolling without slipping. After that their
motion is unchanged.
It will be shown that the final motion of the balls depends neither on the frictional coefficient, nor on
the possible variation of it with position. After 0 r . Its angular momentum about its axis is
2 2
therefore I  mr 2  0 r   m0 r . The angular momentum about the point of contact with the
5 5
table P, must be the same, since the centre of mass of the ball is at rest, i.e. the angular momentum
attributable to translation is zero. The angular momentum of the ball about P cannot be changed
friction any more, as the line of action of this force runs through P. (The sum of the gravitational
force and the reaction of the table is zero, and so they can produce no net torque either.)

The angular momentum of a ball, rolling without slipping at speed  , is the sum of its own angular
2
momentum mr and the angular momentum m  r due to the motion of its centre of mass. Figure
3
shows (on the left) the initially moving ball and the forces acting on it shortly after the collision. on
the right of the figure, the ball is shown ultimately rolling without slipping. According to the law of
2 2 2
conservation of momentum m0 r  mr  mr , which yields   0 . Similar, rezoning shows
5 5 7
5
that the final speed of the other ball has to be 0 , regardless of magnitude of the coefficient of
7
friction.

CENTERS: MUMBAI /DELHI /AKOLA /LUCKNOW /NASHIK /PUNE/NAGPUR /BOKARO /DUBAI # 150
3. As shown in the figure, the component of weight Mg sin  tends to slide the point of contact (of the
cylinder with inclined plane) along its direction. The sliding friction acts in the opposite direction to
this relative motion. Because of frictional force the cylinder rolls. Thus frictional force aids rotation
but hinders translational motion.

Applying Fnet  ma along the direction of inclined plane, we get,


Mg sin   f  Ma c
Where a c  acceleration of centre of mass of the cylinder
 f  Mg sin   Ma c (i)
Where a c  acceleration of centre of mass cylinder.
g sin  g sin 
But ac  
Ic MR 2 2
I I 
MR 2 MR 2
2
 gsin  (ii)
3
Mg sin 
From Eqs. (i) and (ii), f 
3
If  is reduced, then frictional force is reduced.

4. The ball has V ' component of its velocity perpendicular to the length of the rod immediately after the
collision. u is the velocity of CM of the rod and  is angular velocity of the rod just after collision.
the ball strikes the rod with a speed of  cos 53o in the perpendicular direction and its component
along the length of the rod after the collision in unchanged.

Using for the point of collision.


Velocity of separation  Velocity of approach
3V  l 
   u  V' (i)
5  4 
Conserving linear momentum (of rod  particle) in the direction perpendicular to the rod,
3
mV  mu  mV ' (ii)
5
Conserving angular momentum about point ‘D’ as shown in the figure.
 l ml 2  l
0  0   mu     u  (iii)
 4 12  3

CENTERS: MUMBAI /DELHI /AKOLA /LUCKNOW /NASHIK /PUNE/NAGPUR /BOKARO /DUBAI # 151
24V 72V
 u ,W 
55 55l

Time taken to rotate by  angle , t 

l
In the same time, distance travelled  u 2 t 
3
Using angular impulse-angular momentum, equation.
l ml 2 72V
 Ndt 4
 .
4 55l

 l 24mV 
  Ndt  
 4 55 
24m
[Using impulse-momentum equation on the rod  Ndt  mu  55
]

5. The force of friction between the two surfaces in contact disappears when there is no relative (linear)
motion between them. Angular momentum will not be conserved as the discs will have the final
angular velocities in the opposite directions.

6. In the frame of elevator


d2x
mg  ma  kx  m
dt 2
d2x k m g  a  
 2
  x  
dt x k 
2
d x k m  g  pt  a  
or, 2
  x  
dt x k 
There is a term involving t on R.H.S., this does not represent S.H.M. unless p  0
Differentiating again w.r.t. time
d 3x k  dx mp  d2 k mp 
     or    
dt 3
m  dt k  dt 2
m k 
Thus the velocity of the block will vary simple harmonically.

2
7.  y 2   5l   x 2
2
  3 2l    7l - x 
2

 x  4l  y = 3l
3l
 T  2
g
2
8. y  Asin t
T
 T  4
y  t    Asin  (c) and (d)
 8 4 2

CENTERS: MUMBAI /DELHI /AKOLA /LUCKNOW /NASHIK /PUNE/NAGPUR /BOKARO /DUBAI # 152
dy 2 2
 A cos t
dt T T
dy  T  2  2 A max
 t    A cos  
dt  8 T t T 2 2

9. For A   B and C  2B
X  B cos 2  t  B sin 2  t
 
 2Bsin  2t  
 4
This is equation of SHM of amplitude 2B
If A  B and C  2B , then X  B  B sin 2  t
This is also equation of SHM about the point X  B . Function oscillates between X  0 and X  2B
with amplitude B.

10. To find instantaneous axis of rotation we can draw lines perpendicular from  & P to meet at O.
 
   
O lcos
At any instant x 2  y 2  l 2 Q
O
So, p   tan 
4 C
Here, tan   y3
3
3 5 Q
So,   10 rad s (a)
0.5  3 x P4
  
And c has two components  downwards & p rightwaords
2 2
11. (1,3)
y10  10sin  3t   / 4  y2  5 sin 3t  3 cos3t 
 
 10sin  3t   A1  A 2  1
 6
2 2
T1  T2   s
3 3
12. (b,c)
Conservation of Angular momentum about hinge 3L/4
mv 0 L  I
3 3v
I  mL2 ,   0
L/4

5 5L r0
1 2
KE  I
2
2 2
 3L   L  L
rc        10
 4  4 4

13. (b, c)
Sol. For N1

CENTERS: MUMBAI /DELHI /AKOLA /LUCKNOW /NASHIK /PUNE/NAGPUR /BOKARO /DUBAI # 153
Taking torque about B
w d  x 
N1d  w  d  x   0  N1 
d
For N 2
wx
wx  N 2 d  0 , N2 
d
14. (a, d)
Sol. For toppling before sliding
For translational equilibrium
f1  mg sin   mg cos   mg sin 
  tan  (1)
Taking rotation torque about N to topple
h a a
mg sin   mg cos   tan   (2)
2 2 h
a
From (1) and (2) 
h
For sliding before toppling
For translational motion
mg sin   mg cos  tan    (1)
For rotational equilibrium
a h a
mg cos   mgsin    tan  (2)
2 2 h
a a
From (1) and (2)   or  
h h
15. (a, b, c)

16. (b, c, d)
Sol. Since ball moves with constant angular velocity in horizontal circle than
a. For angular momentum about A, it will have angular momentum perpendicular to l. That will have
two component one along CO which is constant and another in the plane which changes direction.
b. Angular momentum about C is constant (mvr) along OC

17. (b, c) 18. (a, c)

19. (a, b, c)
Sol.
2 
a. T   T  1.57 sec
 2
b. A max  2 a  16  2.5  40 cm s 2
2 2
c. v   a 2  y2  v4  2.5   1
v  4 5.25  v  2 21cm s
20. (a, b)
Sol.
m 10 
a. T  2  T  2  T  sec
k1  k 2 360 3
b. Impulse   p
CENTERS: MUMBAI /DELHI /AKOLA /LUCKNOW /NASHIK /PUNE/NAGPUR /BOKARO /DUBAI # 154
50  mv max  50  10v max  v max  5 m s
k1  k 2 5
d. v max  a  v max  a  5  a 6  a  m
m 6
21. (a, b, c, d)
Sol.
mg
a. x  x  1cm
k
1
b. from energy conservation Uspring  Ugravitation  kx '2  mgx
2
x '  2mg  2x  x '  2cm
1 k
c. f  5Hz
2 m
k
d. f  2 which is independent of g.
m

22. (b, c, d)
Sol. U  5x  x  4   U  5x 2  20
dU
F  F  10x  20
dx
c. which is the conditions of S.H.M., K  10
m 
d. T  2  T  sec
k 5
b. speed of the particle will be max. when
F  0  x  2m

INTEGER TYPE
1. For pure rolling    R
At a certain instant when p is at position '  ' , net velocity of p can be found using vector addition of
' ' & 'r ' .
R

  
 
P
2
 net   2   R   2 .2R cos     
 2 sin   2 
But   t {  is constant}
T 2 
 t   t 
S   net dt   2 sin  dt  2 R sin   dt
0  2   2 

2. When the cone is on the point of slipping, the inclination of the plane to the
horizon is  , given by   tan  (i)
Let the plane be inclined at an angle  to the horizon when the cone is on
the toppling over. In this case the vertical line through the centre of gravity
CENTERS: MUMBAI /DELHI /AKOLA /LUCKNOW /NASHIK /PUNE/NAGPUR /BOKARO /DUBAI # 155
G of the cone will just fall within the base of the come i.e., will pass through L.
LO
 From  LG O we have tan  
OG
r
Or tan   ,
4h
Where ' r ' is the radius of the base and ‘h’ is the height of the cone.
4r
 tan    4 tan  ,
h
 LO r 
 tan   VO  h 
 
Now, the cone will slide before it topples over if   
i.e. if tan   tan 
i.e., if   4 tan  [from (i) and (ii)]

3. Since masses are same for rod & particle, centre of mass rod starts to move at  after collision &
particle stops.
l  O x ml
About O Li  m   x  1
2 
ml 2
Lf    l12 
3
By conservation of angular momentum 
x
L f  Li m
We get x l 6

4. 2T  Rx {for each pulley}


If pulley move by ‘x’, blocks moves by x '  ' 4x '
Restoring force on ‘m’ is
Rx R  4x  d2x ' Rx '
T  Or m 2 
2 8 dt 8
R R
2  or 
8m 8m

5. 5
µ m1m2 2 k 3 1000
T = 2 µ=  1.2 kg      5 10
k m1  m2 T µ 1.2 4
2 2
T=  = 0.4 s  n = 5 oscillations in 2 sec
5 10 5

CENTERS: MUMBAI /DELHI /AKOLA /LUCKNOW /NASHIK /PUNE/NAGPUR /BOKARO /DUBAI # 156
GRAVITATION

CONCEPTS SUMMARY & FORMULA SUMMARY


1. Newton’s law of universal gravitation states that the gravitational force of attraction between any
two particles of masses m1 and m2 separated by a distance r has the magnitude
mm
F  G 12 2
r
where G is the universal gravitational constant, which has the value 6.672 1011 N m2 kg2 .
2. If we have to find the resultant gravitational force acting on the particle m due to a number of masses
M1 , M2 ,.....Mn , etc. we use the principle of superposition. Let F1 ,F2 ,....Fn be the individual forces
due to M1 , M2 ,.....Mn each given by the law of gravitation. From the principle of superposition each
force acts independently and uninfluenced by the other bodies. The resultant force FR is then found
n
by vector addition where the symbol ‘  ’ stands for summation.
FR = F1 + F2 + ... + Fn =  Fi
i=1

3. Kepler’s laws of planetary motion state that


(a) All planets move in elliptical orbits with the Sun at one of the focal points
(b) The radius vector drawn from the sun to a planet sweeps out equal areas in equal time intervals.
This follows from the fact that the force of gravitation on the planet is central and hence angular
momentum is conserved.
(c) The square of the orbital period of a planet is proportional to the cube of the semi- major axis of
the elliptical orbit of the planet
The period T and radius R of the circular orbit of a planet about the Sun are related by
 4 2  3
T2   R
 G Ms 
where M s , is the mass of the Sun. Most planets have nearly circular orbits about the Sun. For
elliptical orbits, the above equation is valid if R is replaced by the semi-major axis a.

4. The acceleration due to gravity.


(a) at a height it h above the Earth’s surface
G ME
g h   2
RE  h
G ME  2h 
 1   for h  R E
R 2E  RE 
 2h  G ME
g  h   g 0  1   where g  0  
 RE  R 2E
(b) at depth d below the Earth’s surface is
G ME  d   d 
g d  2 1    g  0  1  
RE  RE   RE 

5. The gravitation force is a conservative force, and therefore a potential energy function can be
defined. The gravitational potential energy associated with two particles separated by a distance r is
G m1 m 2
given by V  
r
where V is taken to be zero at r   . The total potential energy for a system of particles is the sum
CENTERS: MUMBAI /DELHI /AKOLA /LUCKNOW /NASHIK /PUNE/NAGPUR /BOKARO /DUBAI # 157
of energies for all pairs of particles, with each pair represent by a term of the form given by above
equation. This prescription follows from the principle of superposition

6. If an isolated system consists of a particle of mass m moving with a speed v in the vicinity of a
massive body of mass M, the total mechanical energy of the particle is given by
1 GMm
E  m v2 
2 r
That is, the total mechanical energy is the sum of the kinetic and potential energies. The total energy
is a constant of motion.

7. If m moves in a circular orbit of radius a about M, where M >> m, the total energy of the System is
GMm
E
2a
with the choice of the arbitrary constant In the potential energy given in the point 5., above. The total
energy is negative for any bound system, that is, one in which the orbit is closed, such as an elliptical
orbit. The kinetic and potential energies are
GMm
K
2a
GMm
v
a
2G M E
8. The escape speed from the surface of the Earth is e   2gR E and has a value of 11.2
RE
km s 1 .

9. If a particle is outside a uniform spherical shell or solid sphere with a spherically symmetric internal
mass distribution, the sphere attracts the particle as though the mass of the sphere or shell were
concentrated at the centre of the sphere.

10. If a particle is inside a uniform spherical shell, the gravitation force on the particle is zero. If a
particle is inside a homogeneous solid sphere, the force on the particle acts toward the centre of the
sphere. This force is exerted by the spherical mass interior to the particle.

11. A geostationary (geosynchronous communication) satellite moves in a circular orbit in the equatorial
plane at a approximate distance of 4.22  10 4 km from the Earth’s centre.

MECHANICAL PROPERTIES OF FLUIDS


CONCEPT SUMMARY

1. The basic property of a fluid is that it can flow. The fluid does not have any resistance to change of
its shape. Thus, the shape of a fluid is governed by the shape of its container.

2. A liquid is incompressible and has a free surface of its own. A gas is compressible and it expands to
occupy all the space available to it.

3. If F is the normal force exerted by a fluid on an area A then the average pressure Pav is defined as
F
the ratio of the force to area Pav 
A

CENTERS: MUMBAI /DELHI /AKOLA /LUCKNOW /NASHIK /PUNE/NAGPUR /BOKARO /DUBAI # 158
4. The unit of the pressure is the pascal (Pa). It is the same as N m-2. Other common units of pressure
are
1 atm = 1.01  10 5 Pa
1 bar = 105 Pa
1 torr = 133 Pa = 0.133 kPa
1 mm of Hg = 1 torr = 133 Pa

5. Pascal’s law states that: Pressure in a fluid at rest is same at all points which are at the same height.
A change in pressure applied to an enclosed fluid is transmitted undiminished to every point of the
fluid and the walls of the containing vessel.

6. The pressure in a fluid varies with depth h according to the expression


P = Pa+ pgh
where p is the density of the fluid, assumed uniform.

7. The volume of an incompressible fluid passing any point every second in a pipe of non uniform
crossection is the same in the steady flow.
v A = constant (v is the velocity and A is the area of crossection)
The equation is due to mass conservation in incompressible fluid flow.

8. Bernoulli’s principle states that as we move along a streamline, the sum of the pressure (P), the
kinetic energy per unit volume ( v2/2) and the potential energy per unit volume (  gy) remains a
constant.
P + v2/2 + gy = constant
The equation is basically the conservation of energy applied to non viscuss fluid motion in steady
state. There is no fluid which have zero viscosity, so the above statement is true only approximately.
The viscosity is like friction and converts the kinetic energy to heat energy.

9. Though shear strain in a fluid does not require shear stress, when a shear stress is applied to a fluid,
the motion is generated which causes a shear strain growing with time. The ratio of the shear stress
to the time rate of shearing strain is known as coefficient of viscosity, 
Where symbols have their usual meaning and are defined in the text.

10. Stokes’ law states that the viscous drag force F on a sphere of radius a moving with velocity v
through a fluid of viscosity is, F= – 6   av.

11. The onset of turbulence in a fluid is determined by a dimensionless parameter is called the Reynolds
number given by Re= vd/ 
Where d is a typical geometrical length associated with the fluid flow and the other symbols have
their usual meaning.

12. Surface tension is a force per unit length (or surface energy per unit area) acting in the plane of
interface between the liquid and the bounding surface. It is the extra energy that the molecules at the
interface have as compared to the Interior.

CENTERS: MUMBAI /DELHI /AKOLA /LUCKNOW /NASHIK /PUNE/NAGPUR /BOKARO /DUBAI # 159
MECHANICAL PROPERTIES OF SOLIDS
CONCEPT SUMMARY

1. Stress is the restoring force per unit area and strain is the fractional change in dimension. In general
there are three types of stresses (a) tensile stress — longitudinal stress (associated with stretching) or
compressive stress (associated with compression), (b) shearing stress, and (c) hydraulic stress.

2. For small deformations, stress is directly proportional to the strain for many materials. This is known
as Hooke’s law. The constant of proportionality is called modulus of elasticity. Three elastic moduli
viz.. Young’s modulus, shear modulus and bulk modulus are used to describe the elastic behaviour of
objects as they respond to deforming forces that act on them. A class of solids called elastomers does
not obey Hooke’s law.

3. When an object is under tension or compression, the Hookes law takes the form F/A = Y  L/L
where L/L is the tensile or compressive strain of the object. F is the magnitude of the applied force
causing the strain, A is the cross-sectional area over which F is applied (perpendicular to A) and Y is
the Young’s modulus for the object. The stress is F/A.

4. A pair of forces when applied parallel to the upper and lower faces, the solid deforms so that the
upper face moves sideways with respect to the lower. The horizontal displacement L of the upper
face is perpendicular to the vertical height L. This type of deformation is called shear and the
corresponding stress is the shearing stress. This type of stress is possible only in solids.
In this kind of deformation the Hooke’s law takes the form F/A=G L/L whereL is the
displacement of one end of object in the direction of the applied force F, and G is the shear modulus.

5. When an object undergoes hydraulic compression due to a stress exerted by a surrounding fluid, the
Hooke’s law takes the form p=B(V/V).
Where p is the pressure (hydraulic stress) on the object due to the fluid. V/V (the volume strain) is
the absolute fractional change in the object’s volume due to that pressure and B is the bulk modulus
of the object.

ELASTICITY
FORMULA SUMMARY

 (i) Young’s Modulus


F/A
Y
l / l
 P
(ii) Bulk modulus B =
V / V
1
Compressibility = F
B
(iii) Rigidity modulus 
F/ A


A
1 1
 Energy per unit volume =  stress  strain = ( Young’s modulus ) × ( strain )2
2 2

CENTERS: MUMBAI /DELHI /AKOLA /LUCKNOW /NASHIK /PUNE/NAGPUR /BOKARO /DUBAI # 160
YA
 string behaves as a spring of Keq =

 d / d
 Poisson’s ratio () = , d : diameter of rod
l / l

 Thermal stress in a rod ( which is not allowed to expand ) when heated by T is


 Y   T 
where  : coefficien t of linear exp ansion

Here l is
l 0 1  T  & l  l 0 T
l  T
   T 1  T    (T)
l 1   T

FLUID MECHANICS
FORMULA SUMMARY

FLUID STATICS
dp
  g , for stationary fluid
dy

ax
 Tan   for accelerated fluid
g  ay
a x , a y are acceleration in x & y direction
  Angle of inclination of fluid surface with horizontal

Imp: In the reference frame of the container, the liquid is at rest and cannot withstand shearing force so,
net acceleration is always perpendicular to fluid surface in this frame

when fluid is rotating with angular velocity  : Height of liquid , when at rest = H.
y 
 2R 2  2 r 2
y=H- 
4g 2g
( r , y) pressure varies radial outwards
x w 2 r 2
R P = Pc +
2
2 gh A  A 1 , V  2gh
 Velocity of efflux = 2
2

 A2  A1 : area of C.S. of container


1   
A
 1 A2 : area of orifice
 Buoyant force = weight of displaced liquid
If  s  densityof solid,  ω  densityof liquid & v  volumeof solid, submerged in liquid
  
F = vg . If a is acceleration of fluid , then Buoyant force =   v g  a

CENTERS: MUMBAI /DELHI /AKOLA /LUCKNOW /NASHIK /PUNE/NAGPUR /BOKARO /DUBAI # 161
2 1  2
 If equal masses of two liquids of densities 1 and 2 are mixed, mean density is  =
1   2
w1  w 2
 If a body weighs w1 and w2 in air and water then volume of body =
(  w g)
w1
specific gravity of solid =
w1  w 2

FLUID DYNAMICS
FORMULA SUMMARY
 Continuity Equation Av 1   Av 2   cons tan t
.
p 1 2
 Bernoulli’s Theorem  g.h  v  cons tan t .
 2
(steady & irrotational flow of in compressible & non-viscous liquid)
p v2
(a) Pressure head = (b) Gravity head = h (c) Velocity head =
g 2g

 Closely placed streamlines indicate larger velocity

SURFACE TENSION
FORMULA SUMMARY
 F = T

 Excess pressure inside :


4T
Bubble : p  ( two spherical surfaces )
R
2T
Drop : p  ( single spherical surface )
R
ghr
 Capillary rise : T =
2 cos 
 : angle of contact
r : radius of capillary
glr
For tube of insufficient length h=l & cos   [  changes ]
2T
Change in energy of surface
 Surface Tension =
change in area of surface
W  T  A 
Imp : In case of drop splitting in to many, conserve volume.
R1R 2
If R1 and R2 be radii of bubbles. Then radius of common surface is R =
R 2  R1

VISCOSITY
FORMULA SUMMARY
dv
F   A
dx
CENTERS: MUMBAI /DELHI /AKOLA /LUCKNOW /NASHIK /PUNE/NAGPUR /BOKARO /DUBAI # 162
Volume of liquid flowing per second from pipe ( Radius R , length L, pressure difference P )
 pR 4
V=
8 L
 Speed of flow in horizontal tube of length L

V (r) =
 2
p R o  r 2
4L
If two tubes lengths l1, l2 and radii r1, r2 are connected in series across pressure P, then P = P1 + P2
 P1r14  P2 r24 P
 V
8  1 8  2   
8  41  42 
 r1 r2 
When connected in parallel
P  r14 r24 
V   
8   1  2 
Meyer’s relation for flow of gas through a capillary tube:-

P1 P2


 r4 2
P1V1 = P2V2 =
16

P1  P22 
Where V is volume of gas through a cross section per second.

 Stoke’s law :
F= 6rV
2 r 2 (   )g
Terminal velocity v =
9
Velocity at time ‘t’
9 t
2r 2 (  )g  2 r 2

v  1  e 
9  
For turbulent Flow :
vD
Reynold’s Number N =

D : diameter of tube No is critical value of steady flow
N< No  steady flow N>No  turbulent flow
Note :- Consider the shaded layer of liquid. Forces of viscosity are exerted upon it by the layer above
(F1) & layer below it (F2)
For steady state, F1 = F2 F1
F2

CENTERS: MUMBAI /DELHI /AKOLA /LUCKNOW /NASHIK /PUNE/NAGPUR /BOKARO /DUBAI # 163
SINGLE CHOICE QUESTIONS

1. A large tank is filled with water (density = 103 kg/m3). A small hole is made at a depth 10 m
below water surface. The range of water issuing out of the hole is R on ground. What extra
pressure must be applied on the water surface so that the range becomes 2R
(take 1 atm = 105 Pa and g = 10 m/s2) :

10 m

R
(a) 9 atm (b) 4 atm (c) 5 atm (d) 3 atm

2. The gravitational field at a distance R/2 from the centre of a uniform solid sphere of mass M
and radius R is :
GM GM GM
(a) zero (b) 2 (c) 2
(d)
R 2R 4R2

3. Three immiscible liquids L1, L2 and L3 of densities , 3 and 6 respectively are floating one above
the other in a container and depth of each liquid is d. A cylinder of radius r and length 2d floats in
d
vertical position. In equilibrium, lengths of cylinder in liquids L1 and L3 are each. Select the
2
wrong statement

(a) Force applied by liquid L2 on the cylinder is zero


(b) Force applied by liquid L1 on the cylinder is in upward direction
(c) Force applied by liquid L3 on the cylinder is in upward direction
13
(d) Density of material of the cylinder is
4
4. If a pin hole is made in a barometric tube below free surface of liquid in the tube then the correct
statement is
(a) Mercury will flow out of the tube
(b) There will be no change in liquid level in the tube
(c) Liquid level will come down to the free surface in container
(d) Liquid level will come down to the hole only

5. An artificial satellite of mass m is moving in a circular orbit at a height equal to the radius R of the
earth. Suddenly due to internal explosion the satellite breaks into two parts of equal masses. One part
of the satellite stops just after the explosion and then falls to the surface of the earth. The increase in
the mechanical energy of the system (satellite + earth) due to explosion will be
(Given: acceleration due to gravity on the surface of earth is g)
mgR mgR 3mgR
(a) mgR (b) (c) (d)
2 4 4
CENTERS: MUMBAI /DELHI /AKOLA /LUCKNOW /NASHIK /PUNE/NAGPUR /BOKARO /DUBAI # 164
6. A liquid stands at the plane level in the U-tube when at rest. If area of
cross-section of both the limbs are equal, what will be the difference in
heights h of the liquid in the two limbs of U-tube, when the system is a
H
given an acceleration a in horizontal direction towards right as shown ?
g L2 La
(a) (b)
a H g L
2
L a Hg
(c) (d)
Hg a
(L = length of horizontal portion of tube, H = initial height of liquid in both limbs)

7. Two soap bubbles of radius 3 cm and 4 cm coalesce to form a simple bubble under isothermal
conditions. Then the radius of bigger bubble is (Taking surrounding pressure equal to zero)
(a) 7 cm (b) 1 cm (c) 12 cm (d) 5 cm

8. A ball of mass 10kg and density 1 gm/cm3 is attached to the base of a container having a liquid
3
of density 1.1 gm/ cm , with the help of a spring as shown in the figure. The container is going
2
up with an acceleration 2 m / s .If the spring constant of the spring is 200 N/m, the elongation
in the spring is
2 m / s2

(a) 2cm (b) 4cm (c) 6cm (d) 8cm

9. The tube shown is of uniform cross-section . Liquid flows through it at constant speed in the
direction shown by the arrows. The liquid exerts on the tube

(a) a net force to the right (b) a net force to the left
(c) a clockwise torque (d) an anticlockwise torque

CENTERS: MUMBAI /DELHI /AKOLA /LUCKNOW /NASHIK /PUNE/NAGPUR /BOKARO /DUBAI # 165
10. A sphere of mass M and radius R2 has concentric cavity of radius R1 as shown
in figure. The force F exerted by the sphere on a particle of mass m located at a
distance r from the centre of sphere varies as  0  r   

F F F F

r r r r
(a) (b) (c) (d)

11. A Satellite is moving in a circular orbit round the earth with a diameter of 2R. At a certain
point, a rocket fixed to the Satellite is fired such that it increases the velocity of the Satellite
tangentially. The resulting orbit of the satellite would be
(a) same as before
(b) Circular orbit with diameter greater than 2R
(c) elliptical orbit with minimum distance from the centre of earth equal to R
(d) elliptical orbit with maximum distance from the centre of earth equal to R

12. The small opening near the bottom of the vessel shown in figure has an area A. A disk is held
against
the opening to keep the liquid, from running out. Let F1 be the net force on the disk applied by
liquid and air in this case. Now the disk is moved away from opening a short distance. The liquid
comes out and strikes the disk in elastically. Let F2 be the force exerted by the liquid in this
condition. Then F1/F2 is

1 2 1
(a) (b) 1 (c) (d)
2 1 4

13. A small hole is made at the bottom of a symmetrical jar as shown in figure. A liquid is filled into
the jar upto a certain height. The rate of descension of liquid is independent of the level of liquid
in the jar. Then the surface of jar is a surface of revolution of the curve.

(a) y = k x4 (b) y = k x2 (c) y = k x3 (d) y = kx5

14. There are two identical small holes of area of cross –section a on the opposite sides of a tank
containing a liquid of density . The difference in height between the two holes is h. Horizontal
force that will have to be applied on the tank to keep it in equilibrium is
 gha
(a)  gha (b) (c) 2 gha (d) 3 gha
2

CENTERS: MUMBAI /DELHI /AKOLA /LUCKNOW /NASHIK /PUNE/NAGPUR /BOKARO /DUBAI # 166
15. A wooden block , with a coin placed on its top, floats in water as shown in figure. The distance l
and h are shown there. After sometime the coin falls into the water. Then

(a) l decreases and h increases (b) l increases and h decreases


(c) both l and h increase (d) both l and h decreases

16. A block is submerged in a vessel filled with water by a spring attached to the bottom of the
vessel. In equilibrium spring is compressed. The vessel now moves downwards with an
acceleration a( < g). The spring length
(a) will come zero (b) may increase, decrease or remains constant
(c) will decrease (d) will increases

17. When a block of iron floats in mercury at 0°C, fraction k1 of its volume is submerged, while at
the temperature 60°C, a fraction k2 is seen to be submerged. If the coefficient of volume
expansion of iron is γFe and that of mercury is γ H , then the ratio k1 and k 2 can be expressed as
g

1+ 60γ Fe 1 - 60γ Fe 1 + 60γ Fe 1 + 60γ Hg


(a) (b) (c) (d)
1+ 60γ Hg 1+ 60γ Hg 1- 60γ Hg 1+ 60γ Fe

18. The masses and radii of the earth and the moon are M1, R1 and M2 , R 2 respectively. Their centres
are at distance d part The minimum speed with which a particle of mass m should be projected
from a point midway the two centres so as to escape to infinity is
2G  M1 + M 2  4G  M1 + M 2 
(a) (b)
d d
4GM 1M 2 G  M1 + M 2 
(c) (d)
d d

19. In order to shift a body of mass m from a circular orbit of radius 3R to a higher orbit of radius 5R
around the earth , the work done is
3GMm GMm 2 GMm GMm
(a) (b) (c) (d)
5R 2R 15 R 5R

20. Figure shows a planet in an elliptical orbit around the-sun S. Where is the kinetic energy of the
planet maximum ?
P3
P1 P2
P4
(a) P1 (b) P2 (c) P3 (d) P4

21. A small steel ball falls through a syrup with a speed of 10cm/s. If the steel ball is given an upward
force equal to its effective weight, it would move upwards with a speed of
(a) 10cm/s (b) 20cm/s (c) 5cm/s (d) 0

CENTERS: MUMBAI /DELHI /AKOLA /LUCKNOW /NASHIK /PUNE/NAGPUR /BOKARO /DUBAI # 167
22. A spherical ball falls through viscous medium with terminal velocity v. If this ball is replaced by
another ball of the same mass but half the radius, then the terminal velocity will be (neglect the
effect of buoyancy)
(a) v (b) 2v (c) 4v (d) 8v

23. A 5kg rod of square cross section with 5cm side and 1 m long is pulled along a smooth horizontal
surface by a force applied at one end. The rod has a constant acceleration of 2 m/s2. Determine the
elongation in the rod. (Young’s modules of the material of the rod is 5 × 109 N/m 2 ).
(a) Zero, as for elongation to be there, equal and opposite forces must act on the rod
(b) Non –zero but can’t be determine from the given situation
(c) 0.4μm
(d) 16μm

24. Two soap bubbles of different radii are in communication with each other. Then
(a) Air flows from the larger bubble into smaller bubble till both bubbles acquire same size
(b) Air flows from the smaller bubble into larger bubble and the larger bubble grows in size with
decrease in size of smaller bubble
(c) Air doesn’t flow but the sizes of the bubbles change
(d) Sizes of the bubbles remain unchanged

25. A glass rod of diameter d = 2 mm is inserted symmetrically into a glass capillary tube of radius r = 2
mm. Then the whole arrangement is vertically dipped into liquid having surface tension 0.072 N/m.
The height to which liquid will rise on capillary is(take g = 10 m/s2 , density of liquid = 1000 kg/m3.
Assume contact angle to be zero and length of capillary tube to be long enough)
(a) 1.44 cm (b) 6 cm (c) 4.86 cm (d) none of these

26. A solid sphere of radius R, made up of a material of bulk modulus K is surrounded by a liquid in a
cylindrical container. A massless piston of area A floats on the surface of the liquid. When a mass M
is placed on the piston to compress the liquid, the fractional change in the radius of the sphere is
(a) Mg/AK (b) Mg/3AK (c) 3Mg/AK (d) Mg/2AK

27. The length of the needle floating on water is 2.5 cm. The minimum force in addition to its weight
needed to lift the needle above the surface of water will be (surface tension of water is 0.072 N/m)
(a) 3.6×10-3 N (b) 10 -2 N (c) 9 ×10 -4 N (d) 6 ×10 -4 N

28. A bar of cross section A is subjected to two equal and opposite tensile forces as shown. Consider a
cross section BB as shown in the figure. The shearing stress at this point is
B

F  F

B
(a) F cos2θ/A (b) F/A (c) F sin2θ/2A (d) zero

29. A vertical capillary tube with inside radius 0.25 mm is submerged into water so that the length of its
part protruding over the water surface is equal to 25 mm. surface tension of water is 73 103 N m
and angle of contact is zero degree for glass and water, acceleration due to gravity is 9.8m s2 . Then
choose correct statement. (R is radius of curvature of meniscus and h rise in height of water column
in the capillary)
(a) R  0.25 mm (b) h  59.6 mm (c) R  0.60 mm (d) None of these
CENTERS: MUMBAI /DELHI /AKOLA /LUCKNOW /NASHIK /PUNE/NAGPUR /BOKARO /DUBAI # 168
30. A ball of relative density 0.8 falls into water from a height of 2m. The depth to which the ball will
sink is (neglect viscous forces)
(a) 8 m (b) 2 m (c) 6 m (d) 4 m

31. A small wooden ball of density  is immersed in water of density to depth h and then released. The
height H above the surface of water up to which the ball jumps out of water is
h  
(a) (b)   1 h (c) h (d) zero
  

32. A tube is attached as shown in closed vessel containing water. The velocity of water coming out
from a small hole is :

(a) 2 m/s (b) 2 m/s


(c) depends on pressure of air inside vessel (d) None of these

33. A cylindrical vessel filled with water upto height of H stands on a horizontal plane. The side wall of
the vessel has a plugged circular hole touching the bottom. The coefficient of friction between the
bottom of vessel and plane is  and total mass of water plus vessel is M. What should be minimum
diameter of hole so that the vessel begins to move on the floor if plug is remove (here density of
water is )
2M M M
(a) (b) (c) (d) none
H 2H H

34. A solid metallic sphere of radius r is allowed to fall freely through air. If the frictional resistance due
to air is proportional to the cross-sectional area and to the square of the velocity, then the terminal
velocity of the sphere is proportional to which of the following ?
(a) r2 (b) r (c) r3/2 (d) r1/2

35. Two drops of same radius are falling through air with a steady speed of v cm/s. If the two drops
coalesce, what would be the terminal velocity ?
(a) 4 v (b) (d)1/3 v (c) 2 v (d) 64 v

36. A cubical block of side ‘a’ and density ‘’ slides over a fixed inclined plane with constant velocity
‘v’. There is a thin film of viscous fluid of thickness ‘t’ between the plane and the block. Then the
coefficient of viscosity of the thin film will be :




3 a g t 4 a g t a g t
(a) (b) (c) (d) none of these
5v 5v v

CENTERS: MUMBAI /DELHI /AKOLA /LUCKNOW /NASHIK /PUNE/NAGPUR /BOKARO /DUBAI # 169
37. Which of the following graphs best represents the motion of a raindrop ?

v v v v

t t t t
(a) (b) (c) (d)

38. Which of the following is the incorrect graph for a sphere falling in a viscous liquid ?
(Given at t = 0, velocity v = 0 and displacement x = 0)
v a a x

t t v t
(a) (b) (c) (d)

39. The displacement of a ball falling from rest in a viscous medium is plotted against time.

S S S S

t t t t
(a) (b) (c) (d)

40. A 1 mm thick layer of glycerin is placed between a flat plate of area 100 cm2 and a big fixed plate. If
the coefficient of viscosity of glycerin is 1. 0 kg/m-s then how much force is required to move the
plate with a velocity of 7 cm/s?
(a) 3.5 N (b) 0.7 N (c) 1.4 N (d) None

41. A container, whose bottom has round holes with diameter 0.1 mm is filled with water. The
maximum height in cm upto which water can be filled without leakage will be
(surface tension = 75  10 –3 N/m and g= 10 m/s2 )
(a) 20 cm (b) 40 cm (c) 30 cm (d) 60 cm

42. A liquid is filled in a spherical container of radius R till a height h. At this positions the liquid
surface at the edges is also horizontal. The contact angle is

Rh   h R   R h 
(a) 0 (b) cos 1   (c) cos 1   (d) sin 1  
 R   R   R 

CENTERS: MUMBAI /DELHI /AKOLA /LUCKNOW /NASHIK /PUNE/NAGPUR /BOKARO /DUBAI # 170
43. A long capillary tube of radius ‘r’ is initially just vertically immerged inside a liquid of angle of
contact 0 o. If the tube is slowly raised then relation between radius of curvature of meniscus inside
the capillary tube and displacement (h) of tube can be represented by

R R R r
R

h h h h

(a) (b) (c) (d)

44. A cylindrical container of radius ‘R’ and height ‘h’ is 2R


completely filled with a liquid. Two horizontal L shaped pipes
of small cross-section area ‘a’ are connected to the cylinder as
shown in the figure. Now the two pipes are opened and fluid h/2 h/2
starts coming out of the pipes horizontally in opposite R R
directions. Then the torque due to ejected liquid on the system
is:
(a) 4aghR (b) 8aghR
(c) 2 aghR (d) none of these

F2
45. In the figure shown water is filled in a symmetrical container. Four
pistons of equal area A are used at the four opening to keep the
water in equilibrium. Now an additional force F is applied at each
piston. The increase in the pressure at the centre of the container F1 F3
due to this addition is
F 2F 4F
(a) (b) (c) (d) 0
A A A
F4

46. A non uniform cylinder of mass m , length  and radius r is having its centre of mass at a distance
 4 from the centre and lying on the axis of the cylinder. The cylinder is kept in a liquid of uniform
density  . The moment of inertia of the rod about the centre of mass is I. The angular acceleration
of point A relative to point B just after the rod is released from the position shown in figure is

A C CM B

l 4

g  2 r 2 g  2 r 2 g  2 r 2 3g  2r 2


(a) (b) (c) (d)
I 4I 2I 4I

CENTERS: MUMBAI /DELHI /AKOLA /LUCKNOW /NASHIK /PUNE/NAGPUR /BOKARO /DUBAI # 171
47. The figure shows a soap film in which a closed elastic Elastic thread
thread is lying. The film inside the thread is pricked. Now
the sliding wire is moved out so that the surface area
increases. The radius of the circle formed by elastic thread sliding wire
will
(a) increase (b) decrease
(c) remains same (d) data sufficient
soap film

48. A capillary of the shape as shown is dipped in a liquid. Contact angle


between the liquid and the capillary is 0 and effect due to liquid inside the h
meniscus is to be neglected. T is surface tension of the liquid, r is radius of
meniscus and  is density of the liquid then height h in equilibrium is
2T 2T
(a) greater than (b) equal to
rg rg
2T
(c) less than (d) of any value
rg
49. Shape of the meniscus formed by two liquids when capillaries are dipped in them are shown. In I it
is hemispherical where as in II it is flat. Pick correct statement regarding contact angles formed by
the liquids in both situations

I II

(a) It is 180 in I and 90 in II (b) It is 0 in I and 90 in II


(c) It is 90 in I and 0 in II (d) It is greater than 90 in I and equal to 90 in II.

50. For a fluid which is flowing steadily, the level in the vertical tubes is best represented by

(a) (b)

(c) (d)

CENTERS: MUMBAI /DELHI /AKOLA /LUCKNOW /NASHIK /PUNE/NAGPUR /BOKARO /DUBAI # 172
MULTIPLE CHOICE QUESTIONS

1. A particle initially at rest is displaced by applying a non conservative force F in a uniform


gravitational field. In the process following physical quantities associated with the particle are
measured
 U = changed in gravitational potential energy
K = changed in kinetic energy
W1 = worked done by the force F
W2 = worked done by the gravitational force
(a) W2  U (b) K  W1  W2 (c) K  U  W1  W2 (d) W1  W2

2. A small solid ball of density  is held inside at point A in a closed cubical container of side L, filled
with an ideal liquid of density 4 as shown in the figure. Now, if the container starts moving with
constant acceleration a horizontally and the ball is released from point A simultaneously, then
P Q

L/2

L/2

R L S

(a) For ball to hit the top of container at end Q, a = 3g


(b) For ball to hit the top of container at end Q, a = 2g
L
(c) Ball hits the top of container at end Q after a time t 
3g
2L
(d) Ball hits the top of container at end Q after a time t 
3g

3. The end of a capillary tube with a radius r is immersed into water. When the water rises in the tube?
(T is the surface tension and is the density of water).
2T
(a) The water rises to a height h 
 gr
(b) The forces of surface tension perform the work = 4π T2 / ρ g
(c) The potential energy of water increases by 2π T2 / ρ g
(d) The heat evolved during the process is 2π T2 / ρ g

4. A planet is revolving round the sun in an elliptical orbit. The work done on the planet by the
gravitational force of sun is zero
(a) in some parts of the orbit (b) in any part of the orbit
(c) in no part of the orbit (d) in one complete revolution

CENTERS: MUMBAI /DELHI /AKOLA /LUCKNOW /NASHIK /PUNE/NAGPUR /BOKARO /DUBAI # 173
5. Two concentric spherical shells are as shown in figure .The magnitude of gravitational potential (V)
and field strength (E) vary with distance (r) from centre as

E E

r r
(a) (b)
V
V

r r
(c) (d)

6. Some pieces of impurity  density   is embedded in ice. This ice is floating in water
 density  w  . When ice melts, level of water will

(a) fall if   w (b) remain unchanged , if   w


(c) fall if   w (d) rise if   w

7. The viscous force acting on a solid ball of surface area A moving with terminal velocity v is
proportional to
(a) A (b) A1/2 (c) v (d) v1/2

8. A tunnel is dug along chord of the earth at a perpendicular distance R/2 from earth’s centre. The
wall of tunnel may be assumed to be frictionless. A particle is released from one end of the
tunnel. The pressing force by the particle on the wall and the acceleration of the particle vary with
x (distance of the particle from the centre) according to
(a) (b)
Pressing Pressing
Force Force

x x
x  R / 2, x  R x  R / 2, x  R

CENTERS: MUMBAI /DELHI /AKOLA /LUCKNOW /NASHIK /PUNE/NAGPUR /BOKARO /DUBAI # 174
(c) (d)
acceleration acceleration

x x
x  R / 2, x  R x  R / 2, x  R
9. A spring balance reads W1 when a ball is suspended from it. A weighing machine reads W2 when a
tank of liquid is kept on it . When the ball is immersed in the liquid, the spring balance reads W 3 and
the weighing machine reads W4.
(a) W1 > W3 (b) W1 < W3 (c) W2 < W4 (d) W2 > W4

10. If a liquid rises to the same height in two capillaries of the same material at the same temperature
(a) the weight of liquid in both capillaries must be equal
(b) the radius of meniscus must be equal
(c) the capillaries must be cylindrical and vertical
(d) the hydrostatic pressure at the base of capillaries must be same

11. n drops of a liquid, each with surface energy E, join to form a single drop. Then
(a) some energy will be released in the process.
(b) some energy will be absorbed in the process.
(c) the energy released or absorbed will be E(n – n2/3)
(d) the energy released or absorbed will be nE(22/3 – 1)

12. A heavy block of mass 150 kg hangs with the help of three vertical wires of equal length and
equal cross sectional area as shown in figure. Wire II is attached to the centre of mass of block. Take
Y2  2Y1 , where Y1 and Y2 are young’s modulus of wire I & II respectively. For this arrangement
mark out the correct statement(s).
I II III
X X
Y1 Y2 Y1
150kg
(a) The wire I and III should have same Young’s modulus.
(b) Tension in I and III would always be equal.
(c) Tension in I and III would be different.
(d) Tension in II is 75 kg.

13.  11 2

Two bars of steel Y  2 10 N m are joined together as
F
shown. The area of cross section of the left bar is 15 cm  and 0.6m 0.4m
the area of right bar is unknown. The extension in both bars is
the same
(a) The area of right bar is 10cm2
(b) The stress in left and right bar are in ratio 3:2
(c) The decrease in thickness of bar is more for the left
(d) The decrease in thickness of bar is more for right bar
CENTERS: MUMBAI /DELHI /AKOLA /LUCKNOW /NASHIK /PUNE/NAGPUR /BOKARO /DUBAI # 175
INTEGER TYPE

1. A long capillary tube of radius r is dipped in water such that (3/4)1/2 of the height it can rise is
available above free level of water. The angle of contact at the top level in the tube is /w. What is
the value of w?

2. A plate of area 100 cm2 is placed on the upper surface of caster oil, 2mm thick. Taking the
coefficient of viscosity to be 15.5 poise, calculate the horizontal force necessary to move the plate
with a velocity of 3 cm/sec.

3. A body of mass 10 kg is connected to a wire of length 0.3 m and of cross sectional area 10-6 m2.
Its breaking strength is 4.8 ×10 7 N/m2. Find the maximum angular velocity with which it can be
rotated in a horizontal circle

4. A sphere falls from rest into water from a height of 2 m. The relative density of the sphere is 0.80.
Find the depth to which the ball will sink (metres).

5. Distance between centres of two stars is 10a. The masses of these stars are M and 16M and their
radii are a and 2a, respectively. The body of mass m is fired straight from the surface of the larger
star towards the smaller star. What should be the minimum speed to reach the surface of the smaller
star (round off to the nearest integer in the unit of (GM/a)1/2 ).

COMPREHENSION TYPE

PARAGRAPH – 1
If the container filled with liquid gets accelerated horizontally or vertically , pressure in liquid gets
a 
changed. In case of horizontally accelerated liquid  a x  , the free surface has the slope tan 1  x  .
 g 
 
In case of vertically accelerated liquid a y for calculation of pressure, effective g is used. A
closed box with horizontal base 6m by 6m and a height 2m is half filled with liquid. It is given a
g
constant horizontal acceleration g 2 and vertical acceleration downward.
2

1. The angle of the free surface with the horizontal is equal to:
2 1
(a) 30 (b) tan 1 (c) tan 1 (d) 45
3 3

2. Length of exposed portion of top of box is equal to-


(a) 2m (b) 3m (c) 4m (d) 2.5m

5 2
3. Water pressure at the bottom of centre of box is equal to – (atmospheric pressure  10 N m ,
3 2
density of water  1000kg m ,g  10m sec )
(a) 1.1MPa (b) 0.11MPa (c) 0.101MPa (c) 0.011MPa

4. Maximum value of water pressure in the box is equal to


(a) 1.4 MPa (b) 0.14 MPa (c) 0.104 MPa (d) 0.014 MPa

CENTERS: MUMBAI /DELHI /AKOLA /LUCKNOW /NASHIK /PUNE/NAGPUR /BOKARO /DUBAI # 176
5. What is the value of vertical acceleration of box for given horizontal acceleration  g 2 , so that no
part of bottom of box is exposed:
(a) g 2 upward (b) g 4 upward (c) g 4 upward (d) none

SUBJECTIVE TYPE

1. An open cubical tank completely filled with water is kept on a horizontal surface. Its acceleration is
then slowly increased to 2m/s2 as shown in the Fig. The side of the tank is 1 m. Find the mass of
water that would spill out of the tank.

2. A large tank is filled with two liquids of specific gravities 2and . Two holes are made on the wall
of the tank as shown. Find the ratio of the distance from O of the points on the ground where the jets
from holes A & B strike.

3. A laminar stream is flowing vertically down from a tap of cross-section area 1 cm2. At a distance
10 cm below the tap, the cross-section area of the stream has reduced to1/2 cm2. Find the volumetric
flow rate of water from the tap.

4. A cylindrical vessel open at the top is 20 cm high and 10 cm in diameter. A circular hole whose
cross – sectional area 1 cm2 is cut at the centre of the bottom of the vessel. Water flows from a tube
above it into the vessel at the rate 100 cm3s–1. Find the height of water in the vessel under steady
state.

5. A spherical ball of density  and radius 0.003m is dropped into a tube


containing a viscous fluid filled up to the 0 cm mark as show in the figure.
Viscosity of the fluid = 1.260 N.m2 and its density L = /2 = 1260 kg.m–3.
Assume the ball reaches a terminal speed by the 10 cm mark. Find the time
taken by the ball to traverse the distance between the 10 cm and 20 cm mark.
(g = acceleration due to gravity = 10 ms–2)

6. A soap bubble has radius R and thickness d (<< R) as shown. It collapses into a spherical drop. The
ratio of excess pressure in the drop to the excess pressure inside the bubble is

R
R>>d

CENTERS: MUMBAI /DELHI /AKOLA /LUCKNOW /NASHIK /PUNE/NAGPUR /BOKARO /DUBAI # 177
ANSWER KEY

SINGLE CHOICE QUESTIONS

1. (d) 2. (c) 3. (b) 4. (b) 5. (c)

6. (b) 7. (d) 8. (c) 9. (c) 10. (b)

11. (c) 12. (a) 13. (a) 14. (c) 15. (d)

16. (d) 17. (a) 18. (b) 19. (c) 20. (d)

21. (d) 22. (b) 23. (c) 24. (b) 25. (a)

26. (b) 27. (a) 28. (c) 29. (c) 30. (a)

31. (b) 32. (b) 33. (a) 34. (d) 35. (b)

36. (a) 37. (c) 38. (c) 39. (d) 40. (b)

41. (c) 42. (b) 43. (b) 44. (a) 45. (a)

46. (b) 47. (c) 48. (c) 49. (b) 50. (a)

MULTIPLE CHOICE QUESTIONS

1. (1,2) 2. (2,3) 3. (ABCD) 4. (a, d) 5. (a, c)

6. (a, b) 7. (b, c) 8. (b, c) 9. (a, c) 10. (a, b)

11. (a, c) 12. (a, b, d) 13. (ad)

INTEGER TYPE

1. (6) 2. (b) 3. (d) 4. (8) 5. (c)

COMPREHENSION TYPE

1. (d) 2. (c) 3. (b) 4. (b) 5. (a)

SUBJECTIVE TYPE

XA 3
1. 100 kg. 2.  3. V = 4. 9 litre/min 4. h = 5 cm
XB 2

1/3
P2  R 
5. t = 5 sec 6.  
P1  24d 

CENTERS: MUMBAI /DELHI /AKOLA /LUCKNOW /NASHIK /PUNE/NAGPUR /BOKARO /DUBAI # 178
CRASH COURSE P – IV (SOLUTION)
SINGLE CHOICE QUESTIONS
1. Range will become twice if veloci ty of efflux becomes twice. Now as,
v  2 gh
therefore h should become 4 times or 40 m.
Thus an extra pressure equivalent to 30 m of water should be applied.
1 atm = 0.76 × 13.6 m of water
= 10.336 m of water
30 m of water  3 .0 atm

GM
2. E r  r ... (r  R)
R3
R
Here r 
2
GM R
Hence E  .
R3 2
GM

2 R2
3. (B)
Force applied by liquid L1 on curved surface of cylinder is zero but
Force applied by liquid L1 on top face of cylinder is in downward direction and it is non – zero
(F = P x A)
Weight of cylinder = Upthrust on the cylinder
d d 13
 r 2  2d   s   r 2      r 2  d  3   r 2   6    s 
2 2 4

4. (B)
Pressure at point of pin hole inside the tube is less than P0 (atmospheric pressure) so liquid wouldn’t
come out from the pin hole. So level of mercury in the tube won’t change.

1 GMm GMm
5. U1  k1   
2 2R 4R

2R

GM
v
2R
By conservation of linear momentum
mv '
mv  0
2
v '  2v
1m
k f    (2v)2  mv2
2 2 
Potential energy will remain same, change in kinetic energy will be
1 1
= mv2  mv2  mv2
2 2

CENTERS: MUMBAI /DELHI /AKOLA /LUCKNOW /NASHIK /PUNE/NAGPUR /BOKARO /DUBAI # 179
GMm 1 GMmR mgR
  
4R 4 R2 4

6. PA  P0
PB  aL  P0
PC  P0  aL  gh  P0
aL  gh
aL
h
g

7. In isothermal condition
4 R
= 
3
3
P1V1 P2 V2 PV
 
RT RT RT
 4T  4r13  4T  4r23  4T  4R 3
P
 0    P
 0    P
 0  
 r1  3  r2  3  R  3
P0 R13  4Tr12  P0 r23  4Tr22  P0 R 3  4TR 2
If P0 is taken zero
r12  r22  R 2
R = 5 cm

8. (C)
In frame of container
B

mg
ma
kx

B  VP  g  a 
 Ball is in equilibrium
B  mg  ma  kx
x  6c.m

9. The force exerted by the liquid at the bends are shown (The liquid undergoes change of momentum
only at these points’ and hence the liquid and tube exert force an each other) The two forces form a
couple exerting a clockwise torque.

10. The whole space can be divided into three regions


(i) 0 < r < R1 F(r) = 0
CENTERS: MUMBAI /DELHI /AKOLA /LUCKNOW /NASHIK /PUNE/NAGPUR /BOKARO /DUBAI # 180
4  R3 
(ii) R1< r < R2 F(r) = Gm  r - 21 
3  r 
4  R - R3 
3
(iii) R2< r <  F(r) = Gm  2 2 1 
3  r 

11 The resulting path is shown in figure V  V0

12. Let  be the density of liquid. Then


F1 = (  P) A = gh A …(1)
In the second case
F2 = rate of change of momentum
= AV 2
2
= A  2gh 
F2= 2 ghA …(2)
From Eqs. (1) and (2) , we get
F1 1
=
F2 2
-dy
13. Let y be the height of liquid at some instant. Then = constant (given)
dt
From equation of continuity
 -dy 
(πx2)   = a 2gy (a =area of hole)
 dt 
 -dy 
Here, ,   , a and g are constants. Hence squaring the equation, we get
 dt 
y = kx4

14. Thrust force


F = F1 - F2 = av 22
 a  2gh1   a  2gh 2 
= 2ag  h1  h 2 
= 2  agh

15. l will decrease because the block moves up, h will decrease because the coin will displace the
volume of water (V1) equal to its own volume, when it is in the water whereas when it is on the
block it will displace the volume of water (V2), whose weight is equal to weight of coin and
since, density of coin is greater than the density of water V1< V2 .

16. When the vessel was at rest. Equilibrium of block gives


Weight – k x + upthrust
Or W=kx+F …(1)
As the spring moves downwards with acceleration a (< g) upthrust is reduced.

CENTERS: MUMBAI /DELHI /AKOLA /LUCKNOW /NASHIK /PUNE/NAGPUR /BOKARO /DUBAI # 181
Not let x’ be the new comparison. Then
W – k x ' – F ' = ma
g - a W
or W–Fx' -  F= a
 g  g
a a
or (W – F) – k x ' +
F=W
g g
a a
or kx–kx' + F=W
g g
Now science , F < W
x '< x
Therefore, length of spring will increase

  
17.
 and k 2   Fe  Here,   density
k1   Fe
 Hg
0 C   Hg 60 C
k       1  60Fe 
 1  Fe 0 C   Hg  
k 2   Hg     Fe 60 C 1  60 Hg 
0 C

Note : In this problem two concepts are used :


(i) When a solid floats in a liquid , then
Fraction of volume submerged (k) =

(ii) =

18. Potential energy of mass m when it is midway between masses M1 and M2 is


= − −
2 2
= − ( + )
Using conservation of energy
1 2
= ( + )
2
Therefore escape velocity,
4 ( + )
=

19. = − − −
2
=
15

20. Angular momentum should be conserved here,


=
= =
=
21. d.
Initially, W = U + F, where W, U and F represent weight, upthrust and viscous forces
respectively.
Now , W – U = F
CENTERS: MUMBAI /DELHI /AKOLA /LUCKNOW /NASHIK /PUNE/NAGPUR /BOKARO /DUBAI # 182
Or given upward forces = F
So, there will be no velocity

22. b.
= where p is the density
Keeping m constant, if halved, p will increased by a factor of 8.
r2
Now , v0  r 2 p v '0   8p  or v'0  2r 2 p
4
v 0 '  2r 2 p
Dividing, we get,
v0'
= 2 or v '0 = 2v 0 = 2v
v0

23. c
Let The force applied be p; then p = ma = 5 2 = 10 N

= ( − )
= (1 − )2
Δ( ) /
Elongation is dx is =

( )
Δ( ) =
( × )

×
Total elongation
( )
∆ = ∫
× ×
-6
= 0.4 × 10 m

24. b. When two bubbles come into contact, they try to minimize the surface which air flows
from bubble having higher pressure (smaller radius) to bubble having lower pressure.

25. a. Here, surface tension force support the weight of liquid.


×2 + = − ℎ
H = 0.0144 m = 1.44cm.

Mg
26. b Change in pressure due to placing of mass on priston is p =
A
-dp
From bulk modulus definition, K =
dV/V
dV Δp Mg
= =
V k AK
4
From V = pr3 .
3

CENTERS: MUMBAI /DELHI /AKOLA /LUCKNOW /NASHIK /PUNE/NAGPUR /BOKARO /DUBAI # 183
dV 3dR dR 1 dv Mg
=  = =
V R R 3 V 3AK

27. a.
Surface tension force.
F = Surface Tension (2 length) = 3.6 × 10-3 N

28. c.
A
Cross – sectional area of the section is A' =
cosθ
F sinθ F sin2θ
Shearing stress = =
A' 2A

Fcos
F sin 

29. (c)
2T cos 
Capillary height h   h  59.6 mm
rg
here 59.6 mm is greater than the protruding part hence water will rise in the capillary of insufficient
height 25 mm.
2T
Now, R   0.6 mm
hrg

30. (a)
Sol. Loss of gravitational potential energy = work done against Buyonancy
mg (h + h’) = Bh’  W(h + h’) = Bh’
B 1
(h + h’) = h’  h + h’ = h’  h’ = 4h  h’ = 8m
W 0.8

31. (b)
Sol. Work done against Buyonancy = gain of gravitational potential energy
(B – W) h = WH
B  σ 
H =   1 h  H =   1 h
W  ρ 

32. (b)
Sol. V= 2gh  V = 2m/s

CENTERS: MUMBAI /DELHI /AKOLA /LUCKNOW /NASHIK /PUNE/NAGPUR /BOKARO /DUBAI # 184
33. (a)
Sol. Force on vessel = rate of change of linear momentum imparted to flow water 2
Fr = Kavr
through hole.
Force on vessel = mv (where m is mass coming out per second)
(V= volume coming out per second)
F = mv  F = Vv  F = .av.v  F = av2
πd 2 2 ρπd2
Mg . v [where v = 2gh ]  Mg  . 2gh  d min =
4 4
2μM W=
4 r3
3
πρH

34. (d)
Sol. At terminal velocity W= Fr
4 3
r g = Kr2vr2
3
VT r1/2

35. (b)

Density
Density
Sol. VBig = 2VSmall  RB = 2 1/3RS
2 πR 2 (ρ  σ)g
VT =  vT = R2  vT(B) =
9 η 4o C
4oC
2 vT(S) vT(B) = 4 1/3vT(S)
2/3
O Temp O Temp

36. (a)
dv 3 v 3ρagt
Sol. mg sin 37o = A a3 g  = a2  =
dx 5 t 5v
F = 6 rv
37. (c)
dv
Sol. mg– 6v = m
dt
t v
dv
0 dt = m 0 mg  6πηrv a

mg
v= 1  e t   so graph will be expontially rising
6πηr mg
38. (c)
Sol. W – B – ma
mg – 6rv = ma
a Vs v will be a straight line constant slope.
v = K (1 – e–t/r)
so graph A, B, D are correct.

39. (d)
Sol. Displacement will increase continuously.

CENTERS: MUMBAI /DELHI /AKOLA /LUCKNOW /NASHIK /PUNE/NAGPUR /BOKARO /DUBAI # 185
40. (b)
dv 1102  7 102
Sol. F = A  F=  F = 0.7 N
dx 103
41. (c)
2T
Sol. hg =  h = 0.3 m  h = 30 cm
R

42. (b)
R
Rh Rh 
Sol. cos =
R 
h
Rh 
 = cos–1  
 R 

43. (b)

44. (a)
h
Sol. Velocity of efflux of water  v   2g    gh h/2
2
force of ejected water  Rate of change of momentum of ejected
water
   av  v    av2 2R

Torque of these forces about central line

 
 av2 2R.2  4 av2R  4 agh R

45. (a)
Sol. The four piston are initially in equilibrium. As additional force F is applied to each piston, the
F
pressure in fluid at each point must be increased by so that each piston retains state of
A
equilibrium
F2 F2  F

 F
P2 A P  A
 2 A

F1 P1A P3A F3 F1  F  F
P1  A
 F
P3  A
 A
F3  F
 A
P4A  F
P4   A
 A

F4  F
F4

F
Thus the increment in pressure at each point is P  (by Pascal’s law)
A
CENTERS: MUMBAI /DELHI /AKOLA /LUCKNOW /NASHIK /PUNE/NAGPUR /BOKARO /DUBAI # 186
46. (b)
Sol. Torque about CM:
Fb

Fb .  I 
4 CM
1 
I
 
    r 2      g  .
4
r 2 2 g mg

4I
'  ' will be same for all points

47. (c)
Sol. The force exerted by film on wire or thread depends only on the nature of material of the film and
not on its surface area. Hence the radius of circle formed by elastic thread does not change.

48. (c)
Sol. As weight of liquid in capillary is balanced by surface tension, then
r
T  2 r  r 2 h1g (for uniform r radius tube)

h1 
2T h1
r g
but weight of liquid in tapered tube is more than uniform tube of radius
r, then in order to balance h  h1 r
T
h
rg
49. (b)
Sol. For hemispherical shape- For flat surface-

tangent on liquid
0=0 surface   90

tangent on liquid
surface
Solid surface
  90

Solid surface
50. (a)
Sol. From continuity equation, velocity at cross-section (1) is more than that at cross-section (2)
Hence ; P1  P2

CENTERS: MUMBAI /DELHI /AKOLA /LUCKNOW /NASHIK /PUNE/NAGPUR /BOKARO /DUBAI # 187
MULTIPLE CHOICE QUESTIONS

1. (a, b)
Theory

2. (b, c)

P 
 L 
Q
L/2
V4pg
V4pa

Vpg
L/2

x
In frame of constant
Fx  max 4p Va  VPa  VP a x
 a x  3a ……(1)
Time to cover PQ (i.e., L distance)
2L
t …..(2)
3a
In y  direction Fy  max
3Vpg  VP
a y  3g ……(3)
L
Time requirement to cover distance
2
L
 ……(4)
3g
3a
From (2) & (4)  3g a  2g
2

3. (a ,b, c, d)
2T
The water rises to a height h 
gr
The P.E. of water column is
mgh 2T 2
Ep  
2 g
4 T 2
The forces of surface tension perform the work W  2rTh 
g
One half of this work goes to increase the P.E., and the other half to evolve heat
2T 2
Hence Q  .
g

CENTERS: MUMBAI /DELHI /AKOLA /LUCKNOW /NASHIK /PUNE/NAGPUR /BOKARO /DUBAI # 188
4. At two positions, when the planet is closet to the sun(perigee) and when it is farthest from the sun
(apogee), velocity vector is perpendicular to force vector i.e., work done is zero .In one complete
revolution work done is zero.

5. Inside the inner sphere field is zero field is zero but potential is constant .
Between two, field is due to inner sphere potential is due to both but it is constant due to outer
shell. Outside the outer shell, field and potential is due to both and it decreases due to both.

6. Level will fall if initially the impurity pieces were floating along will ice and later it sinks. Level
will remain unchanged if initially they were floating and later they keep floating.

7. F = 6πηrv
 F  v and F  r or F  A1/2

8. b., c
Net force towards centre of earth = mg’ = (mgx)/R

Normal force N = mg’ sin 


mgx R
Thus pressing force N =
R 2x
mg
N= is constant and independent of x.
2
Hence (b). Tangential force F = ma = mg’ cos 
R2
- x2
gx 4
Q = g’cos  =
R x
gx
A= R 2 - 4x 2
R
R
Curve is parabolic and at ,a=0
2
Hence (c).

9. W1, W3 are the forces exerted by the ball and the spring balance on each other before and after
immersion.W2, W4 are the forces exerted by the tank and the weighing machine on each before and
after immersion. Let m and M be the masses of the ball and the tank respectively , and N be the force
intersection between the ball and the liquid in the tank.

10. a., b.
T
h=
Rdg

CENTERS: MUMBAI /DELHI /AKOLA /LUCKNOW /NASHIK /PUNE/NAGPUR /BOKARO /DUBAI # 189
Given that h, T, d and g are fixed , hence R must be same . As weight of liquid is balanced by
force due to surface tension (T, and R are fixed for a given liquid, given material of
capillary at a constant temperature. Hence weights of liquids in both the capillaries must be equal.
So, option a and b are correct

11. a.,c
Let S = surface tension
= surface energy per unit area
r = radius of each small drop
R = radius of a single drop
4 3 4 1/3
n× πr = πR 3 or R = r n
3 3
Initial surface energy
E i = n × 4πr 2 S = nE
Final surface Energy
2 2
E f  4R 2S  4r 2 n 3 S  n 3 E
2
 
Energy released  E i  E f  E  n  n 3 
 

12. a., b., d.


Here for all three wires, length l, elongation ∆ and cross-sectional area A are the same. From
stress T/A
Y= =
strain l / l
T Y
Let T1. T2 and T3 be tension in three wires I, I and III respectively .
For vertical equilibrium ,
T1 + T2 + T3 = 150kg
For rotation equilibrium
T1 × x - T3 x = 0
T1 = T3so Y1  Y3
= =2
T2 = 2T1
So, 2T1 + T2 = 150kg
T2 = 75kg

13. (a,d)
F 0  
   0 0
Ay A A'
0.6 0.4
  A '  10 cm 2
15 A '
F  A 10
stress   1  2 
A 2 A1 15
r   A
  r    r  
r   
15 1500 10 1000
rleft   rright  
0.6 0.36 0.4 0.16
CENTERS: MUMBAI /DELHI /AKOLA /LUCKNOW /NASHIK /PUNE/NAGPUR /BOKARO /DUBAI # 190
INTEGER TYPE
1. (6) Let the height rise under case be h.
Then 2πrσ = πr 2 hpg Where  is the density
h
2r cos   r 2 g 3
2
Dividing, we get
1 2 3
=  cosθ =
cosθ 3 2
θ =30° = π/6

2. (2) The force experienced by the plate will be of viscous nature. To compensate, we need to
apply a force
dv
F = -ηA
dx
= - 15.5 × 100 ×
 -3 × 10
2
= 2.325dynes
F = 2.325 × 10-5 N
In 10 -5 N, F = 2.325 = 2.33  2

3. (4) Breaking strength = tension = mrω2


4.8 × 107 × 10-6 = 10 × 0.3ω 2
ω = 4rad/sec .

4. (8) Retardation in water,


upthust-weight
a =
mass
V (1) g  v(0.8) g
= = 8/4
V (0.8)
Now V22 = V12 - 2(a) h0
0 = 2gh – 2ah0
gh (g)(2)
h0 = = = 8m
a (g/4)

5. (3) At point P between the stars, let the gravitational field intensity be zero. So
(16 )
=
(10 − )
= 4 ⇒ = −4 + 40 ⇒ = 8
10 −
It means that if body crosses the point P, it is attracted by the other star. Thus the critical velocity
is the velocity of the body just to reach the point P, which can be given as
m(Δ ) =
2
where ΔV is potential difference between point A and P
(16 ) − (16 ) 45
Δ = − − + =
2 8 8 2 8
= √2Δ = 45 /4 ≈ 3 /

CENTERS: MUMBAI /DELHI /AKOLA /LUCKNOW /NASHIK /PUNE/NAGPUR /BOKARO /DUBAI # 191
COMPREHENSION

PARAGRAPH – 1

1. (d)
dy a g 2
Sol.  x   1 ……. (effective g will be g  a  g 2 )
dx a y  g  g 2  g

  45
(0,4)

2. (c)
(2,2)
Sol. As the slope of free surface is 45 . Thus free surface passes
through centre of box and having co- ordinates (2,2) at top of
box. Thus length of exposed top part- 2m
1m
 6  2  4m 45
(4,0) x
6m

3. (b)
Sol.  
P  Pa  gh  105  1000 10 1  105  104 N m2  0.11MPa

4. (b)
Sol.  
P  105  103 10  4 N m2   0.1  0.04  MPa  0.14 MPa

5. (a)
1
Sol. As maximum slope of free surface is for the condition of
3 ay
non-exposure of bottom of box, then
ax 1
 2
ay  g 3 ax
as a x  g 2 6
3a x  a y  g
a y  g 2 , thus g 2 upward

CENTERS: MUMBAI /DELHI /AKOLA /LUCKNOW /NASHIK /PUNE/NAGPUR /BOKARO /DUBAI # 192
SUBJECTIVE TYPE 1

a
1. tan =  tan = 0.2
g x
x
0.2 =
1 a 
x = 0.2
mass spill out = V 
= A  1  1000 g
geff
1
=  1  0.2  1000 kg
2
= 100 kg

2. Velocity of efflux at a vA
1
P = vA2
2
h
2Δp 2   σg hg
vA =  vA = 4 
σ σ 2
time of flight
3h
2
tA = 4  3h
g 2g
For vB
h h 
2  σg + × 2σg 
vA = 2 4   v  hg
B

h
2
4 h
tB =  tB =
g 2g
X A VA × t A 1 3 X 3
= =   A 
X B VB × t B 2 1 XB 2

3. From equation of continuity


1
V = a1v1 = a2v2 1  v1 = v2
2
a1 v1
v2 = 2v1
From energy conservation
v22 = v12 + 2gh
2 a2 v2
(2v1)2 = v2 + 2gh  v1 = gh
3
2 2 1 2
V = a1v1 V =1 1000 10 cm3/sec V = 100 cm3/sec  V = litre/sec
3 3 10 3
2
V=6 m litre/min  V = 4. 9 litre/min.
3

CENTERS: MUMBAI /DELHI /AKOLA /LUCKNOW /NASHIK /PUNE/NAGPUR /BOKARO /DUBAI # 193
4. V = a. v  V=a 2gh

100 = 1 2 1000  h
10
h=  h = 5 cm
2

v
2 r 2 (ρ  ρ L )g 2 (3 103 )2 1260  9.8
5. VT =   VT = 
9 η 9 1.260
VT = 2  10 –2 m/s  VT = 2 cm/s
10cm
t=  t = 5 sec
VT

4S 2S
6. P1 =  P2 =
R r
Where P1 is excess pressure is soap bubble and P2 is excess pressure in drop. Let r is the radius of
spherical drop.
P2 R
 …. (1)
P1 2r
From equation of volume
4
4R2d = r2 , r = (3R2d)1/3 ….(2)
3
From (1) and (2)
1/3 1/ 3
P2 R P2  R 3  P2  R 
     
P1 2(3R 2d)1/3 P1  24R 2d  P1  24d 

CENTERS: MUMBAI /DELHI /AKOLA /LUCKNOW /NASHIK /PUNE/NAGPUR /BOKARO /DUBAI # 194
HEAT & THERMODYNAMICS

THERMAL PROPERTIES OF MATTER

CONCEPT SUMMARY
1. Heat is a form of energy that flows between a body and its surrounding medium by virtue of
temperature difference between them. The degree of hotness of the body is quantitatively represented
by temperature.

2. A temperature-measuring device (thermometer) makes use of some measurable property (called


thermometric property) that changes with temperature. Different thermometers lead to different
temperature scales. To construct a temperature scale, two fixed points are chosen and assigned some
arbitrary values of temperature. The two numbers fix the origin of the scale and the size of its unit.

3. The Celsius temperature (tC) and the Farenhelt temperare (tF) are related by tF = (9/5)tC + 32

4. The Ideal gas equation connecting pressure (P), volume (V) and absolute temperature (T) is:
PV = RT
Where is the number of moles and R is the universal gas constant.

5. In the absolute temperature scale, the zero of the scale is the absolute zero of temperature the
temperature where every substance in nature has the least possible molecular activity. The Kelvin
absolute temperature scale (T) has the same unit size as the Celsius scale (TC), but differs in the
origin:
T C = T –273.15

6. The coefficient of linear expansion  t  and volume expansion  v  are defined by the relations:
l
  l t
l
V
  V t
V
Where l and V denote the change in length l and volume V for a change of temperature T. The
relation between them is:  v  3  l

7. The specific heat capacity of a substance is defined by


1 Q
s
m T
Where m is the mass of the substance and Q is the heat required to change its temperature by T.
The molar specific heat capacity of a substance is defined by
1 Q
C
 T
Where is the number of moles of the substance.

8. The latent heat of fusion (Lt) is the heat per unit mass required to change a substance from solid into
liquid at the same temperature and pressure. The latent heat of vaporisation (Lv) is the heat per unit
mass required to change a substance from liquid to the vapour state without change In the
temperature and pressure.

9. The three modes of heat transfer are conduction, convection and radiation.

CENTERS: MUMBAI /DELHI /AKOLA /LUCKNOW /NASHIK /PUNE/NAGPUR /BOKARO /DUBAI # 195
10. In conduction, heat is transferred between neighbouring parts of a body through molecular collisions,
without any flow of matter. For a bar of length L and uniform cross section A with its ends
maintained at temperatures TC and TD. the rate of flow of heat H is:
T  TD
HK A C
L
where K is the thermal conductivity of the material of the bar.

11. Newton’s Law of Cooling says that the rate of cooling of a body is proportional to the excess
temperature of the body over the surroundings:
dQ
  k T2  T1 
dt
Where T1 is the temperature of the surrounding medium and T2 is the temperature of the body.

KINETIC THEORY

CONCEPT SUMMARY

1. The ideal gas equation connecting pressure (P), volume (V) and absolute temperature (T) is
PV=  RT = k B NT
where  is the number of moles and N is the number of molecules. R and K B are universal constants.
R
R= 8.314 J mol1K 1 k B   1.38 1023 J K 1
NA
Real gases satisfy the ideal gas equation only approximately, more so at low pressures and high
temperatures.

2. Kinetic theory of an ideal gas gives the relation


1
P  nmv 2
3
Where n is number density of molecules, m the mass of the molecule and v2 is the mean of squared
speed. Combined with the ideal gas equation it yields a kinetic interpretation of temperature.
1 3 12 3k B T
2 2
 
mv 2  k B T, Vrms  v 2 
m
This tells us that the temperature of a gas is a measure of the average kinetic energy of a molecule.
Independent of the nature of the gas or molecule. In a mixture of gases at a fixed temperature the
heavier molecule has the lower average speed.

3. The translational kinetic energy


3
E k B NT
2
This leads to a relation
2
PV  E
3
4. The law of equipartition of energy states that if a system is in equilibrium at absolute temperature T,
the total energy is distributed equally in different energy modes of absorption, the energy in each

CENTERS: MUMBAI /DELHI /AKOLA /LUCKNOW /NASHIK /PUNE/NAGPUR /BOKARO /DUBAI # 196
1
mode being equal to k B T . Each translational and rotational degree of freedom corresponds to one
2
1
energy mode of absorption and has energy k B T T. Each vibrational frequency has two modes of
2
energy (kinetic and potential) with corresponding energy equal to
1
2x k B T  k B T .
2

5. Using the law of equipartition of energy the molar specific heats of gases can be determined and the
values are in agreement with the experimental values of specific heats of several gases. The
agreement can be improved by improved by including vibrational modes of motion.

6. The mean free path l is the average distance covered by a molecule between two successive
collisions:
1

2nd 2
Where n is the number density and d the diameter of the molecule.

THERMODYNAMICS
CONCEPT SUMMARY

I. The zeroth law of thermodynamics states that ‘two systems in thermal equilibrium with a third system
are in thermal equilibrium with each other’. The Zeroth Law leads to the concept of temperature.

2. Internal energy of a system is the sum of kinetic energies and potential energies of the molecular
constituents of the system. It does not include the over-all kinetic energy of the system. Heat and
work are two modes of energy transfer to the system. Heat is the energy transfer arising due to
temperature difference between the system and the surroundings. Work is energy transfer brought
about by other means, such as moving the piston of a cylinder containing the gas, by raising or
lowering some weight connected to it.

3. The first law of thermodynamics is the general law of conservation of energy applied to any system in
which energy transfer from or to the surroundings (through heat and work) is taken into account. It
states that  Q =  U +  W
where  Q is the heat supplied to the system,  W is the work done by the system and  U is the
change in internal energy of the system.

1 Q
4. The specific heat capacity of a substance is defined by s  where m is the mass of the
m T
substance and  Q is the heat required to change its temperature by  T. The molar specific heat
1 Q
capacity of a substance is defined by C  where is the number of moles of the substance. For
 T
a solid, the law of equipartition of energy gives C = 3R which generally agrees with experiment at
ordinary temperatures.
Calorie is the old unit of heat. 1 calorie is the amount of heat required to raise the temperature of 1g
of water from 14.5°C to 15.5°C. 1 cal = 4.186 J.

CENTERS: MUMBAI /DELHI /AKOLA /LUCKNOW /NASHIK /PUNE/NAGPUR /BOKARO /DUBAI # 197
5. For an ideal gas, the molar specific heat capacities at constant pressure and volume satisfy the relation
Cp–Cv =R where R is the universal gas constant.

6. Equilibrium states of a thermodynamic system are described by state variables. The value of a state
variable depends only on the particular state, not on the path used to arrive at that state. Examples of
state variables are pressure (P), volume (V), temperature (T) and mass (m). Heat and work are not
state variables. An Equation of State (like the ideal gas equation PV = RT) is a relation connecting
different state variables.

7. A quasi-static process is an infinitely slow process such that the system remains in thermal and
mechanical equilibrium with the surroundings throughout. In a quasi-static process, the pressure and
temperature of the environment can differ from those of the system only infinitesimally.

8. In an isothermal expansion of an ideal gas from volume V1 to V2 at temperature T the heat absorbed
(Q) equals the work done (W) by the gas, each given by
V 
Q  w   RT In  2 
 V1 
9. In an adiabatic process of an ideal gas
Cp
PV  = constant where  
Cv
Work done by an ideal gas in an adiabatic change of state from (P1, V1, T1) to (P2, V2, T2) is
R T1  T2 
W
 1

10. Heat engine is a device in which a system undergoes a cyclic process resulting in conversion of heat
into work. If Q1 is the heat absorbed from the source, Q2 is the heat released to the sink, and the work
output in one cycle is W. the efficiency of the engine is:
W Q
  1 2
Q1 Q1
11. In a refrigerator or a heat pump, the system extracts heat Q2 from the cold reservoir and releases Q1
amount of heat to the hot reservoir, with work W done on the system. The co-efficient of performance
of a refrigerator is given by
Q Q2
 2 
W Q1  Q 2
12. The second law of thermodynamics disallows some processes consistent with the First Law of
Thermodynamics. It states Kelvin-Planck statement
No process is possible whose sole result is the absorption of heat from a reservoir and complete
conversion of the beat into work.

Clausius statement
No process is possible whose sole result is the transfer of heat from a colder object to a hotter object.
Put simply, the Second Law implies that no heat engine can have efficiency equa1 to 1 or no
refrigerator can have co-efficient of performance  equal to Infinity.

13. A process is reversible if it can be reversed such that both the system and the surroundings return to
their original states, with no other change anywhere else in the universe. Spontaneous processes of
nature are irreversible. The idealised reversible process is a quasi-static process with no dissipative
factors such as friction, viscosity. etc.
CENTERS: MUMBAI /DELHI /AKOLA /LUCKNOW /NASHIK /PUNE/NAGPUR /BOKARO /DUBAI # 198
14. Carnot engine is a reversible engine operating between two temperatures T1 (source) and T2 (sink).
The Carnot cycle consists of two Isothermal processes connected by two adiabatic processes. The
efficiency of a Carnot engine is given by
T
  1 2 (Carnot engine)
T1
No engine operating between two temperatures can have efficiency greater than that of the Carnot
engine.

15. If Q > 0, heat is added to the system


If Q < 0, heat is removed to the system
If W > 0, Work is done by the system
If W < 0, Work is done on the system

THERMAL PHYSICS

FORMULA SUMMARY

 Thermal expansion & temperature:

L  L 0 (1   )
A  A 0 (1  ) where L : length ; A : area ; V : volume;   2 ,   3
V  V0 1  

0
   0 1   when T < < 1
1  

If a scale is calibrated at temp T0 ,then the reading of length  at temperature T0   is

1   / 1  

So, correct length = reading  1    

1
Time lost by pendulum when temperature rises by t    86400 s perday
2
1
t =  Ts per second
2

 Calorimeter :

For increase in temperature by , without changes of state, Heat absorbed = m s  


where s is specific heat of substance.

For change of state of substance heat absorbed = m L


L: Latent heat of substance for given phase change.

CENTERS: MUMBAI /DELHI /AKOLA /LUCKNOW /NASHIK /PUNE/NAGPUR /BOKARO /DUBAI # 199
 Transfer of heat :
(1) Conduction :
Q dT
H   KA
t dx
A : Area of Cross -section
K: Conductivity of material

For a uniform rod of length ,


Q KA (T1  T2 )

t 
where Tl & T2 are the temperature of the hot & cold end of rod respectively.

L
Thermal resistance =
KA

It is similar to electrical resistance & laws of series and parallel connection apply here also.
Q 2kL(T1  T2 )
For radial flow of heat through a cylindrical tube, 
t log e (r2 / r1 )
where r2 & r1 are outer & inner radii respectively

Note: At steady state, H = thermal current = constant


 dt  c f ( x ).dx c  unknown cons tan t

Now, use
x
T
dT c  f ( x ).dx
 T1
 0
, so that the constant is eliminated.
T2 y
 dT c  f ( x ).dx
T
x

(2) Radiation:

Q 4
(i) Net rate of loss of heat of a body  eA ( T 4  T0 )
t
 :Stephen ' s cons tan t  5.67  10 8 W / m 2  k 4
A :Surface area of body
T : Temperature of body
T0 :Temperature of the surroundings (in k )
Q
: Rate of loss of heat
t
e : emmissivity of the body

(ii) Newton’s law of cooling :


dT
  K ( T  T0 ) if T  T 0   T0
dt
where k is constant

CENTERS: MUMBAI /DELHI /AKOLA /LUCKNOW /NASHIK /PUNE/NAGPUR /BOKARO /DUBAI # 200
3
4eAT0
K and
ms
dT
: Rate of change of temperature
dt
T0 : Temperature of the surrounding
m : mass of the body
s : specific heat of the body

(iii) Kirchoff’s law :

Emission power of body


 coefficient of absorption (a ) of body  emissivity (e) of body
Emissive power of black body

(iv) Wien’s displacement law:

 m T  cons tan t  2.88  10 3 m  k


where  m  wavelength correspond ing to max imum spectral int ensity
& T  Temperatur e of the body
Area under e    graph  T 4 at temperatur e T

Note:
While solving problems on heat flow , remember the following equations
dQ dT
  KA
dt dx
dT dm
 eA (T 4  T04 )  ms L
dt dt
Choose the appropriate relation according to problem

 Kinetic Theory of Gases :

RT 3P
(1) VRMS  3 =
M 
8 RT 8 p
Vavg = .  .
 M  
RT p
V most probable = 2 = 2
M 

(2) . Translational K.E. of gases


1
=  mass  V 2 rms
2
3
 RT per mole , where R is8.314 J / K
2
3
 KT per molecule , where k is1.38  10 23 J / K
2

CENTERS: MUMBAI /DELHI /AKOLA /LUCKNOW /NASHIK /PUNE/NAGPUR /BOKARO /DUBAI # 201
1
PV  mv 2rms m :mass of gas
3

 Degrees of freedom (f):

(i) Monoatomic gas (3) ( 3 translational )


(ii) Diatomic gas (5) ( 3 translational + 2 rotational )
(iii) Polyatomic gas (6) ( 3 translational + 2 rotational + 1 vibrational )

Degree of freedom changes with temperature for diatomic and polyatomic gases as vibrational energy
increases.

Each degree of freedom contributes RT/2.per mole to the energy of gas.


1 f 2
Cv  fR ; Cp  Cv  R  .R
2 2
Cp f  2
 
Cv f
5
  for monoatomic =
3
7
diatomic =
5
4
Polyatomic =
3

 I Law Of Thermodynamics :

dQ = dU +dW = nCv dT + PdV

PROCES ∆U W ∆Q
1. Adiabatic nCv∆T p1 v1  p 2 v 2 0
= - nCv∆T
 1
2. Isochoric nCv∆T 0 nCv∆T
(const. V)
3. Isobaric (P) nCv∆T P(V2 –V1) =n R∆ T nCp∆T
4. Isothermal (T) 0 v v
nRT ln 2 nRT ln 2
v1 v1

 Polytropic processes : (Here, n = moles of gas)

PVx = const
 TVx1  const
 Tx P1x  const
x  0  isobaric
x    adiabatic
x  1 isothermal
 nR 
W U  nC v 
x 1
CENTERS: MUMBAI /DELHI /AKOLA /LUCKNOW /NASHIK /PUNE/NAGPUR /BOKARO /DUBAI # 202
Q  nC
R R R
C  Cv   
1  x  1 1  x
Note:-
(1) U  nCv  (for allprocesses)

(2) U is a state function  ( U ) cyclic  0

(3) Ideal gas equation PV = nRT is always valid

Cp R R
(4)   & Cp  Cv  R  Cv  & Cp 
Cv  1  1

(5) In polytropic processes, bulk modulus = xP.


 For adiabatic process B = p and for isothermal process, B = P

(6) For a mixture of gases,


(Cv )eq   x i (Cv) i
i

( Cp )eq  ( Cv) eq  R   x i ( Cp) i ( xi = mole fraction of ith gas )


i

R
 eq  1 
(Cv) eq

(7) While solving problem on I law, remember that if piston of a container is in equilibrium,
mg
Pout   Pi ns
A
Wtotal Qrejected
(8) Efficiency   1 
Q sup plied Q sup plied

(9) Variation of P with h


dp
(i)   g
dy
p RT
(ii) 
 M
 Mgh  Mgh
RT RT
P  P0 e &   0 e

CENTERS: MUMBAI /DELHI /AKOLA /LUCKNOW /NASHIK /PUNE/NAGPUR /BOKARO /DUBAI # 203
SINGLE CHOICE QUESTIONS
1. Three rods made of the same material and having same cross-sectional area but different lengths 10
cm, 20 cm and 30 cm are joined as shown. The temperature of the joint is:
200C
20 cm

30 cm 10 cm

300C 100C

(a) 20°C (b)23.7°C (c) 16.4°C (d) 18.2°C

2. If two rods of length L and 2L having coefficients of linear expansion  and 2 respectively are
connected so that total length becomes 3L, the average coefficient of linear expansion of the
composite rod equals :
3 5 5
(a)  (b)  (c)  (d) none of these
2 2 3

3. A metallic object of coefficient of cubical expansion 3 × 10 -5 /°C has a cavity inside. The cavity is
partially filled by a liquid of coefficient of cubical expansion 5 × 10 –5/°C. What percent of volume of
cavity should be filled by the liquid so that volume of empty space in the cavity does not change with
temperature ?
(a) 60% (b) 40% (c) 36% (d) 64%

4. A monoatomic gas undergoes a thermodynamic process P2 = cT where c is a constant. Molar heat


capacity of the gas for this process is
3R 5R
(a) (b) 2R (c) (d) 3R
2 2

5. Temperature of a body decreases from 28°C to 27.8°C in 10 min when the body is kept in a
surrounding at 27°C. Now temperature of the same body is raised to 329°C and its is kept in a
surrounding at 327°C. Assuming that the body follows Newton's law of cooling, its temperature after
5 min will become
(a) 327.8°C (b) 327.6°C (c) 328.8°C (d) None

6. Two rods A and B of lengths lA = 3l and lB = 2l are connected in series and a third rod C of length
lC = 5l is connected in parallel with the combination of A and B. Their thermal conductivities are
KA = 4K, KB = 3K. Cross sectional area of all the rods are same. What is thermal conductivity of rod
C if rates of heat conducted across rods A and C are in the ratio 3 : 2
40K 40K 90K
(a) (b) (c) (d) 10K
17 9 17

7. Two Solid metallic spheres S1 and S2 are made of the same material and have identical surface finish.
The mass of S1 is three times that of S2 . Both the spheres are heated to the same high temperature and
placed in the same room having lower temperature but are thermally insulated from each other. Then
ratio of the initial rate of cooling of S1 to that of S2 is
13
1 1 3 1
(a) (b) (c) (d)  
3 3 1  3

CENTERS: MUMBAI /DELHI /AKOLA /LUCKNOW /NASHIK /PUNE/NAGPUR /BOKARO /DUBAI # 204
8. Two identical containers A and B fitted with frictionless pistons contain the same ideal gas at the
same temperature and the same volume V. The mass of the gas A is mA and that in B is mB . The gas
in each cylinder is now allowed to expand isothermally to the same volume 2V. The changes in
pressure in A and B are found to be  P and 1.5P respectively. Then
(a) 4mA  9mB (b) 2mA  3mB (c) 3mA  2mB (d) 9mA  4mB

9. The ideal gas is expanding such that PT 2  constant. The coefficient of volume expansion of the gas
is
1 2 3 4
(a) (b) (c) (d)
T T T T

10. An ideal gas is taken through a series of changes represented in figure. The total work done by the
gas at the end of the cycle is equal to:

B(4P,3V)
P

A(P,V) C(P,3V)
V
(a) Zero (b) 2PV (c) 3PV (d) 5PV

11. If the molar heat capacity of a gas in the process PT = constant is 5R, the number of degrees of
freedom of molecules in the gas is
(a) 4 (b) 5 (c) 6 (d) 3

12. The internal energy of an ideal gas is related as U=a + b (PV) (where P is the pressure of gas, V is
the volume of gas, ‘a’ and ‘b’ are the constants.)for an adiabatic process,  will be
(a) a/b (b) 1+(b/a) (c) 1 + (1/a) (d) none of the above.

13. A student records Q,u , w for a thermodynamics cycle A  B C A. certain entries are
missing. Finds correct entry in following option

AB BC CA
W 40J 30J
u 50J
Q 150J 10J
(a) WBC = -70J (b) QCA = 130J (c) UAB = 190J (d) U CA = -160J

14. An ideal gas expands in such a way that PV2 = constant throughout the process.
(a) The graph of the process of T-V diagram is a parabola
(b) The graph of the process of T-V diagram is a straight line.
(c) Such an expansion is possible only with heating
(d) Such an expansion is possible only with cooling

CENTERS: MUMBAI /DELHI /AKOLA /LUCKNOW /NASHIK /PUNE/NAGPUR /BOKARO /DUBAI # 205
15. PV curve for the process whose VT curve is
T

a
C

P
a P
b c

b C
a
(a) V (b) V

P P
P
a c c a

b
b
(c) V (d) V

16. A given mass of gas expands from a state A to the state B by three paths 1,2,3 as shown in T-V
indicator diagram . If w1, w2 , w3 respectively be the work done by the gas along the three paths , then

T
1 B

2
3
A
V
(a) w1 > w2 > w3 (b) w1< w2 < w3 (c) w1 = w2 = w3 (d) w1< w2 > w3

17. A process is shown in diagram which of the following curves may represent the same process
P

A B

V
T P
B
B
A
A
C C
(a) P (b) T

CENTERS: MUMBAI /DELHI /AKOLA /LUCKNOW /NASHIK /PUNE/NAGPUR /BOKARO /DUBAI # 206
V V
C
A
B
B
A

T C
(c) (d) T

18. The ratio of average translational kinetic energy to rotational kinetic energy of a diatomic molecule
at temperature T is
(a) 3 (b) 7/5 (c) 5/3 (d) 3/2

19. A diatomic gas follows equation PV n = C. What should be the value of n such that its molar heat
capacity during process = R
(a) 2/3 (b) 1 (c) 1.5 (d) 5/3

20. An ideal gas undergoes the process 1 2 as shown in figure, the heat supplied and work done in
process is Q and W respectively .The ratio Q :W is
V

1
(a) γ :  γ - 1 (b) γ (c) γ - 1 (d) γ -
γ

21. A diatomic ideal gas is heated at constant volume until the pressure is double and again heated at
constant pressure until volume is doubled. The average molar heat capacity for whole process is
13R 19R 23R 17R
(a) (b) (c) (d)
6 6 6 6

22. A cylindrical tube of cross section area A has two air tight friction less pistons at its two ends. The
pistons are tied with a straight piece of metallic wire as shown. The cube contains a gas at pressure Po
and temperature T0 If temperature of gas is doubled then tension in the wire is.(Atmospheric
.
pressure = Po)
Metallic wire

(a) 4PoA (b) PoA/2 (c) PoA (d) 2PoA

23. An ideal gas mixture filled inside a balloon expands according to the relation PV2/3 = constant. The
temperature inside the balloon is
(a) increasing (b) decreasing (c) constant (d) cant’ be said

CENTERS: MUMBAI /DELHI /AKOLA /LUCKNOW /NASHIK /PUNE/NAGPUR /BOKARO /DUBAI # 207
24. An ideal gas at pressure P and volume V is expanded to volume 2V. Column I represents process and
column II represents the work done during that processes.
Column – I Column - II
PV(1 - 21 -  )
(a) isobaric (p)
 -1
(b) isothermal (q) PV
(c) adiabatic (r) PVln2
The correct matching of column I and column II is given by
(a) a q b  r c  P (b) a q b p c  r
(c) a p b  q c  r (d) a r b  q c  P

25. Two moles of an ideal monatomic gas undergoes a cyclic process ABCA as shown in figure. The ratio
of temp. at B and A is
P
B
3Po

A 60° 30°
Po C
Vo 6Vo

(a) 16: 3 (b) 27:4 (c) 20 3 : 5 (d) 6 : 1

26. A diatomic gas of molecules weight 30gm/mole is filled in a containers at 27°C. It is moving at a
velocity 100m/s. If it is suddenly stopped the rise in temperature of gas is
60 600 90 900
(a) (b) (c) (d)
R R R R

27. Three processes form a thermodynamics cycle as shown on PV diagram for an ideal gas process 1 2
takes place at constant temperature (300K). Process 2 3 taken place at constant volume During this
process 40J of heat leaves the system. Process 3 1 is adiabatic and temperatureT3 = 275 K. work
done by the gas during the process 3 1 is
P

1
2

3
V
(a) -40J (b) -20J (c) 40J (d) 20J

28. n molecules of a gas is filled in a container at temperature T. If the gas is compressed slowly and
isothermally to half of its initial volume the work done by atmosphere on piston is
nRT -nRT  1
(a) (b) (c) nRT  ln 2-  (d) nRTl n 2
2 2  2

29. The molecules of an ideal gas have 6 degree of freedom .The temperature of the gas is T. The average
translational kinetic energy of its molecules is
6 3 KT
(a) KT (b) KT (c) KT (d)
2 2 2
CENTERS: MUMBAI /DELHI /AKOLA /LUCKNOW /NASHIK /PUNE/NAGPUR /BOKARO /DUBAI # 208
30. In the given graph , the process AB is
P
B

A
C

V
(a) isothermal expansion (b) isothermal compression
(c) adiabatic expansion (d) none

31. The co-efficient of thermal expansion of a rod is temperature dependent and given by formula
α = at where a is constant and t is in °C. If the length of the rod is l at temperature 0°C, then the
temperature at which the length will be 2l is .
ln2 ln4 1 2
(a) (b) (c) (d)
α α α α

32. The temperature of blackbody is increased by 1% The amount of radiation emitted by it increased by
(a) 1% (b) 8% (c) 4% (d) 16%

33. The volume of 0.5 kg of liquid water over the temperature range from 0 C to 50 C fits reasonably
well to the polynomial function
  
V  0.500  4 105 T  5 106 T2 
where volume is measured in cubic meters and T is temperature in degree celsius. The volume
expansion coefficient for liquid water at 20 C is
(a) 1.2 104  C (b) 3.2 104  C
(c) 0.9 104  C (d) 7.2 104  C

34. A Centigrade and a Fahrenheit thermometers are dipped in boiling water. The water temperature is
lowered until the Fahrenheit thermometer register 140o. What is fall in temperature as registered by
the centigrade thermometer:
(a) 80 o (b) 60 o (c) 40 o (d) 30o

35. At 40 oC resistance of platinum is 3.14  and at 100 oC its resistance is 3.76  . Its temperature
coefficient is:
(a) 0.00379/oC (b) 3.79×10 –1/oC (c) 3.79×10–2/oC (d) None of these

36. 22 gm of CO2 at 27 oC is mixed with 16 gm of O2 at 37 oC. The temperature of the mixture is:
(a) 32 oC (b) 27 oC (c) 37 oC (d) 30.5 oC

37. A calorimeter of mass 0.2 kg. and specific heat 900 J/kg.K contains 0.5 kg of a liquid of specific heat
2400 J /kg.K. Its temperature falls from 60 oC to 55 oC in one minute. The rate of cooling is:
1
(a) J/s (b) 15 J/s (c) 115 J/s (d) 110 J/s
16
38. 2 litre water kept in a kettle is heated by 1 KW power source. Kettle is open and it loses heat at the
rate of 160 J/s/ The time taken for the temperature of kettle to changes from 27 oC and 77 oC is:
(a) 8 min 20 sec (b) 6 min 20 sec (c) 5 min (d) 7 min

CENTERS: MUMBAI /DELHI /AKOLA /LUCKNOW /NASHIK /PUNE/NAGPUR /BOKARO /DUBAI # 209
39. A fixed amount of nitrogen gas (1 mole) is taken and is subjected to pressure and temperature
variation. The experiment is performed at high pressures as well as high temperatures. The results
obtained are shown in the fig. The correct variation of PV/ RT with P will be exhibited by:
(3) (2)
2.0 (4)

PV/RT 1.0

0 100 200 300 400 500 600


P(Atoms.)

(a) curve (4) (b) curve (3) (c) curve (2) (d) curve (1)

40. A volume V-absolute temperature T diagram was obtained when a V

gas was heated at a constant pressure. During the heating process


from state 1 to state 2, the mass:
(a) remained constant
(b) decreased V
(c) changed erratically 2
(d) increased
1
T
O T

41. The variation of density of water with temperature is represented by the curve in:
Density
Density

4 oC
4 oC
O Temp O Temp
Density
Density

o o
4C 4C
O O
Temp Temp
(c) (d)

(a) curve (1) (b) curve (2) (c) curve (3) (d) curve (4)

CENTERS: MUMBAI /DELHI /AKOLA /LUCKNOW /NASHIK /PUNE/NAGPUR /BOKARO /DUBAI # 210
P
42. Two different isothermal curves representing the relationship between
volume V and pressure P for ideal gas are shown for absolute
temperatures T1 and T2 respectively in the adjoining figure. Then:
(a) T1  T2 T2
(b) T1  T2
T1
(c) T1  T2 V
(0,0)
(d) none of these

43. A system changes from the state  P1,V1  and  P2 , V2  as shown in the
figure. What is the work done by the system?
(a) 7.5 × 105 joule (b)7.5 × 10 5ergs
5
(c) 12 × 10 joule (d)6 × 10 5 joule

44. One mole of an ideal gas undergoes a cyclic change as show in fig. The
process AB is isothermal. The pressure and volume at point C
is1.013×10 s Pa and 22.4 litres respectively. The temperature of gas at C
is:
(a) 0oC (b) 273oC
(c) 0K (d) 27 oC

45. The work in an ideal monatomic gas along the cyclic path LMNO is
P

M N
3P

L
2P O

O V
V 2V

(a) PV (b) 2PV (c) 3PV (d) 4PV

46. The curve which represents an adiabatic change is

(a) (b)
P
P

V
V

CENTERS: MUMBAI /DELHI /AKOLA /LUCKNOW /NASHIK /PUNE/NAGPUR /BOKARO /DUBAI # 211
P P

V V
(c) (d)

(a) (i) (b) (ii) (c) (iii) (d) (iv)

47. In the diagrams (i) to (iv) variation of volume with changing pressure is shown. A gas is taken along
the path ABCD. The change in internal energy of the gas will be:
V V
D
D C
C

A A
B B

P P
(i) (ii)
V
V
D
D C C

A B
A B
P
P

(iii) (iv)

(a) positive all cases (i) and (iv)


(b) positive in cases (i), (ii) and (iii) but zero in case (iv)
(c) negative in cases (i), (ii) and (iii) but zero in case (iv)
(d) zero in all the four cases

48. Carbon monoxide is carried around a closed cycle abc, in which bc is an


isothermal process, as shown in the figure. The gas absorbs 7000 J of heat,
as its temperature increases from 300 K to 1000 K in going from a to b.
The quantity of heat rejected by the gas during the process ca is:
(a) 4200 J (b) 5000 J
(c) 9000 J (d) 9800 J

49. There are three thermometers-one in contact with the skin of the man, other in between the vest and
the shirt and third in between the shirt and coat. The reading of the thermometers are 30 oC, 25oC and
22oC respectively. If the vest and shirt are of the same thickness, the ratio of their thermal
conductivities is:
(a) 9 : 25 (b) 25 : 9 (c) 5 : 3 (d) 3 : 5

CENTERS: MUMBAI /DELHI /AKOLA /LUCKNOW /NASHIK /PUNE/NAGPUR /BOKARO /DUBAI # 212
50. Heat is flowing through two cylindrical rods of the same material. The diameters of the rods are in
the ratio 1 : 2 and the lengths in the ratio 2 : 1. If the temperature difference between the ends be the
same, then the ratio of the rate of flow of flow if heat through them will be:
(a) 1 : 4 (b) 1 : 8 (c) 1 : 1 (d) 2 : 1

51. A block of steel heated to 100oC is left in a room to cool. Which of the curves shown in fig.
represents the correct behavior:

Temperature
c
B
A

Time
(a) A (b) B (c) C (d) none of these

52. The following figure shows two air-filled bulbs connected by a U-tube partly filled with alcohol.
What happens to the levels of alcohol in the limbs X and Y when an electric bulb placed midway
between the bulbs is lighted?
Painted Black Painted White

Bulb

Alcohol

(a) the level of alcohol in limb X falls while that in limb Y rises
(b) the level of alcohol in limb X rises while that in limb Y falls
(c) the level of alcohol falls in both limbs
(d) there is no change in the levels of alcohol in the two limbs

53. A heat flux of 4000 J/s is to be passed through a copper rod of length of 10 cm and area of cross
section 100 sq. cm. The thermal conductivity of copper is 400 W/m. oC. The two ends of this rod
must be kept at a temperature difference of :
(a) 1oC (b) 10 oC (c) 100oC (d) 1000 oC

54. On a new scale of temperature (which is linear) and called the W scale, the freezing and boiling
points of water are 39 W and 239 W respectively. What will be the temperature on the new scale,
corresponding to a temperature of 39 C on the Celsius scale?
(a) 200 W (b) 139 W (c) 78 W (d) 117 W

CENTERS: MUMBAI /DELHI /AKOLA /LUCKNOW /NASHIK /PUNE/NAGPUR /BOKARO /DUBAI # 213
MULTIPLE CHOICE QUESTIONS
1. Let , rms and  P respectively denote the mean speed, root mean square speed, and most probable
speed of the molecules in an ideal monatomic gas at absolute temperature T. The mass of a molecule
is m. Then:
(a) no molecule can have a speed greater than 2  rms
(b) no molecule can have a speed less than P / 2
(c) P    rms
(d) the average kinetic energy of a molecule is ¾ m2P

2. A hot body emits radiation which has maximum intensity at frequency  m. If the temperature of the
body is doubled:
(a) the maximum intensity radiation will be at frequency 2m
(b) the maximum intensity radiation will be at frequency m 2
(c) the total emitted energy per second will increase by a factor of 16
(d) the total emitted energy per second will increase by a factor of 4 and maximum intensity radiation
will be at frequency  m.

3. The molar heat capacity of a diatomic ideal gas can be (R is universal gas constant)
7R 5R
(a) (b) (c) 0 (d) Infinite
2 2

4. The specific heats of gas are Cp = 0.2 cal/gm°c and Cv = 0.15 cal/gm°C (Take R = 2 cal/mol°c)
(a) The molar mass of the gas is 40gm
(b) The molar mass of the gas can not be determined from the data given
(c) The number of degree of freedom of the gas molecules is 6.
(d) The number of degree of freedom of the gas molecule is 8.

5. According to kinetic theory of gases, which of the following statement will be true.
(a) Ideal gas can not be liquefied
(b) The molecules of ideal gas do not obey newtons laws of motion.
(c) Pressure of gas is always inversly proportional to its volume
(d) Molecules of gas never move in straight line.

6. In the given thermodynamics process, the correct statement is


P
B
3P0

A C
2P0
V
v0 2v 0
(a) Heat given in the complete cycle ABCA is zero
(b) work done in the complete cycle ABCA is zero
1
(c) Work done in the complete cycle ABCA is Po Vo
2
(d) None

CENTERS: MUMBAI /DELHI /AKOLA /LUCKNOW /NASHIK /PUNE/NAGPUR /BOKARO /DUBAI # 214
7. For an ideal gas
(a) The change in internal energy in a constant pressure process from temperature T1 to T2 is equal to
nCv (T2 – T1) where Cv is the molar specific heat at constant volume and n = no of moles of gas
(b) The change in internal energy of gas and work done by gas are equal in an adiabatic process
(c) The internal energy does not change in an isothermal process
(d) All of the above options are correct.
P
8. An ideal gas is taken from point A to point C on P-V diagram through two A
process AOC and ABC as shown in figure. Process AOC is isothermal.
(a) Process AOC requires more heat than process ABC B O
(b) Process ABC requires more heat than process AOC C
Po
(c) both process AOC and ABC requires same amount of heat .
(d) Data is insufficient for comparison at heat required for the two processes. Vo 2Vo

9. Two vessels of the same volume contain the same gas at same temperature. If the pressure in vessels
be in the ratio of 1 :2 then
(a) The ratio of average kinetic energy is 1:2
(b) The ratio of the root mean square velocity is 1:1
(c) The ratio of the average velocity is 1:2
(d) The ratio of number molecules is 1:2

10. When the temperature of a copper coin is raised by 80°C , its diameter increased by 0.2%
(a) Percentage rise in the area of a face is 0.4 %
(b) percentage rise in thickness is 0.4%
(c) percentage rise in volume is 0.6%
(d) Coefficient of linear expansion of copper 0.25 × 10-4 /°C

11. For Two different gas X and Y , having degree of freedom f1 and f 2 lnp
and molar heat capacities at constant volume Cv1 and Cv2
respectively, the ln(p) versus ln(v) graph is plotted for adiabatic y
process, as shown x
(a) f1 > f2 (b) f2>f1
(c) Cv2 > Cv1 (d) Cv1 > Cv2 lnv

12. A rigid body of mass m is moving in a circle of radius r with a constant speed v. What is the work
done by the force in moving the body over half the circumference of the circle.
mv 2 mv 2 r 2
(a) (b) (c) Zero (d)
r 2 r2 mv 2

18. One mole of an ideal monatomic gas has initial temperature T0 is P


made to go through the cycle abca shown in fig. If U denotes the
internal energy, then choose the correct alternative. 2P0 c

(a) Uc  Ub  Ua (b) Uc  Ub  3RT0 P0


a b
9RT0 3RT0
(c) U c  U a  (d) U b  U a 
2 2
V0 2V0 V

CENTERS: MUMBAI /DELHI /AKOLA /LUCKNOW /NASHIK /PUNE/NAGPUR /BOKARO /DUBAI # 215
19. A sample of a pure compound is contained in a closed, well-insulated
container. Heat is added at a constant rate and the sample temperature
is recorded. The resulting data is sketched below. Which of the

Temp
following conclusions is justified from the data given?
(a) The sample boiles
(b) The latent heat of fusion is greater than the latent heat of
t (Min.)
vaporization 5 10 15 20 25
(c) At 5 minutes the sample was mixture of solid and liquid.
(d) At 20 minutes the solid was all liquid

20. The temperature of an isotropic cubical solid [of length L, density d and coefficient of expansion 
per degree Kelvin,] is raised by 10 C . Then, at this temperature, to a good approximation:
(a) length is L 1  10  (b) total surface area is L2 1  20 
(c) density is d 1  30  (d) density is d / 1  30 

21. In the isothermal expansion of an ideal gas:


(a) there is no change in the temperature of the gas
(b) there is no change in the internal energy of the gas
(c) the work done by the gas is equal to the heat supplied to the gas
(d) the work done by the gas is equal to the change in its internal energy

22. An ideal gas is taken from state A (pressure P, volume V) to the state B (pressure P/2, volume 2V)
along a straight line path in the P – V diagram. Select the correct statement (s) from the following:
(a) Work done by the gas in the process A to B exceeds the work that would be done by it if the
system were taken from A to B along an isotherm
(b) In the T – V diagram, the path AB becomes part of parabola
(c) In the P – T diagram, the path AB becomes the part of the hyperbola
(d) In going from A to B, the temperature T of the gas first increases to a maximum value and then
decreases

23. A wall has two layers A and B, each made of different materials. The
thickness of both the layers is the same. The thermal conductivity of A,
1 0 2
KA  3KB .The temperature across the wall is 20o C . In thermal
equilibrium
(a) the temperature difference across A  15o C A B
(b) rate of heat transfer across A is more than across B
(c) rate of heat transfer across both is same
(d) temperature difference across A is 5o C

24. Two metallic sphere A and B are made of same material and have got identical surface finish. The
mass of sphere A is four times that of B. Both the spheres are heated to the same temperature and
placed in a room having lower temperature but thermally insulated from each other.
(A) The ratio of heat loss of A to that of B is 24/3.
(B) The ratio of heat loss of A to that of B is 22/3.
(C) The ratio of the initial rate of cooling of A to that of B is 2-2/3.
(D) The ratio of the initial rate of cooling of A to that of B is 2-4/3.

CENTERS: MUMBAI /DELHI /AKOLA /LUCKNOW /NASHIK /PUNE/NAGPUR /BOKARO /DUBAI # 216
25. A body of mass M is attached to the lower end of a metal wire, whose upper end is fixed. The
elongation of the wire is l.
(A) Loss in gravitational potential energy of M is Mgl
(B) The elastic potential energy stored in the wire is Mgl
(C) The elastic potential energy stored in the wire is 1/2 Mgl
(D) Heat produced is 1/2 Mgl

FOR QUESTIONS 26 TO 27
The figure shows a radiant energy spectrum graph for a black body at a
temperature T.
26. Choose the correct statement(s)
(A) The radiant energy is not equally distributed among all the possible
wavelengths
(B) For a particular wavelength the spectral intensity is maximum
(C) The area under the curve is equal to the total rate at which heat is radiated by the body at that
temperature
(D) None of these

27. If the temperature of the body is raised to a higher temperature T', then choose the correct
statement(s)
(A) The intensity of radiation for every wavelength increases
(B) The maximum intensity occurs at a shorter wavelength
(C) The area under the graph increases
(D) The area under the graph is proportional to the fourth power of temperature

28. An ideal gas has molar heat capacity at constant pressure Cp = 5R/2. The gas is
kept in a cylindrical vessel fitted with a piston which is free to move. Side walls
of the container and the piston are non-conducting. Mass of the frictionless
piston is 9 kg. Initial volume of the gas is 0.0027 m3 and cross-section area of
the piston is 0.09 m2. The initial temperature of the gas is 300 K. Atmospheric
pressure P0 = 1.05 × 105 N/m2. An amount of 2.5 × 104 J of heat energy is
supplied to the gas by a heating coil, then
(A) Initial pressure of the gas is 1.06 × 105 N/m2
(B) Final temperature of the gas is 1000 K
(C) Final pressure of the gas is 1.06 × 105 N/m2
(D) Work done by gas is 9.94 × 103 J

29. Consider the shown case of a freezing lake due to negative environmental
temperature (–°C). Thickness (x) of ice layer is small in comparison to depth
of lake. Rate of increase in x will be greater
(A) if environmental temperature increases
(B) for larger thickness of ice layer
(C) if environmental temperature decreases
(D) for smaller thickness of ice layer

CENTERS: MUMBAI /DELHI /AKOLA /LUCKNOW /NASHIK /PUNE/NAGPUR /BOKARO /DUBAI # 217
INTEGER TYPE QUESTIONS
1. One mole of a diatomic gas is taken through the process P/Tn = C, where n and C are constant. If the
heat capacity of gas is C = – R / 2, then the value of n is

2. Three identical metal rods A, B and C are connected end to end. A temperature differences is
maintained between the free ends of A and C. If the thermal conductivity of B(KB) is three times that
of C (KC) and one third that of A(KA) , the effective thermal conductivity of the system will be
J
________ , if K A  13  .
sm c

3. 5 gm of helium having r.m.s speed of molecules 1000m/s and 24 gm of oxygen having r.m.s speed of
1000m/s are introduced in a thermally isolated vessel. Find the r.m.s speed in m/s of oxygen when
thermal equilibrium is attained.

4. To heat a certain amount of gas by 50°C at constant pressure,160 calorie is required . if the same gas
is heated by 100° at constant volume ,240 calories is required. Degree of freedom of the molecule of
the gas is equal to

5. A body cools in 7 minutes from 60°C to 40°C, What will be its temperature in °c after next 7
minutes .The temperature of surrounding is 10°C

6. It is known that the temperature in the room is +20°C when the outdoor temp is – 20°C and + 10°C
when the outdoor temp is -40°C Assuring newtons law of cooling to be valid, the temperature of
radiator in the room is _________ °C.

7. A ball moving with constant velocity collides with another identical stationary ball. At the moment of
impact the angle between the straight line passing through the centers of the balls and the direction of
initial velocity of the striking ball is 45o . The balls are smooth and the fraction of kinetic energy
which converts to potential energy at the moment of maximum deformation is 4n. Find n.

8. A uniform chain of mass m and length l hangs on a thread and touches the surface of a table. If the
fallen part does not form a heap the force exerted by the table on the chain when half of its length has
fallen on the table is n mg/2. Find n.

SUBJECTIVE TYPE

1. One mole of an ideal monoatomic gas undergoes the cyclic process as shown. Find out efficiency (in
percent) of the cycle.
[Take ln 2 = 0.7]

CENTERS: MUMBAI /DELHI /AKOLA /LUCKNOW /NASHIK /PUNE/NAGPUR /BOKARO /DUBAI # 218
2. A heat engine uses an ideal gas ( = 1.40) that undergoes the reversible cycle shown in figure. Obtain
the thermodynamic percentage efficiency of the engine.

3. The apparatus shown in Figure consists of four glass columns connected by horizontal sections. The
height of two central columns B and C is h0  49cm each. The two outer columns A and D are open
to atmosphere. A and C are maintained at a temperature of 95 C while columns B and D are
maintained at temperature of 5 C . The height of liquid A and D measured from the base line is
h1  52.8cm and h 2  51cm respectively. Calculate co-efficient of thermal expansion of the liquid.

D
A 50 C
B C
950 C 50 C 950 C

CENTERS: MUMBAI /DELHI /AKOLA /LUCKNOW /NASHIK /PUNE/NAGPUR /BOKARO /DUBAI # 219
ANSWER KEY

1. (c) 2. (c) 3. (a) 4. (b) 5. (c)

6. (a) 7. (d) 8. (c) 9. (c) 10. (c)

11. (c) 12. (d) 13. (d) 14. (d) 15. (a)

16. (a) 17. (c) 18. (d) 19. (d) 20. (a)

21. (b) 22. (c) 23. (a) 24. (a) 25. (b)

26. (a) 27. (a) 28. (b) 29. (b) 30. (d)

31. (b) 32. (c) 33. (b) 34. (c) 35. (a)

36. (a) 37. (c) 38. (a) 39. (d) 40. (b)

41. (a) 42. (c) 43. (c) 44. (a) 45. (a)

46. (c) 47. (d) 48. (d) 49. (d) 50. (b)

51. (a) 52. (a) 53. (c) 54. (d)

MULTIPLE CHOICE QUESTIONS

1. (c, d) 2. (a, c) 3. (a,b,c,d) 4. (a,c) 5. (c)

6. (c) 7. (a,c) 8. (a) 9. (b,d) 10. (a,c,d)

11. (b,c) 12. (c) 13. (a, b, c, d) 14. (a, c) 15. (a, b, d)

16. (a, b, c) 17. (a, b, d ) 18. (c, d) 19. (AC) 20. (ACD)

21. (ABC) 22. (ABCD) 23. (ACD) 24. (CD)

INTEGER TYPE

1. (4) 2. (3) 3. (750 m/s) 4. (6) 5. (28)

6.  60  
7. (1) 8. (3)

SUBJECTIVE TYPE
1.
700
2. 24 3.
 h1  h 2  1
 %  17.95%  
39 90  h1  h 0  5090

CENTERS: MUMBAI /DELHI /AKOLA /LUCKNOW /NASHIK /PUNE/NAGPUR /BOKARO /DUBAI # 220
CRASH COURSE P – V (SOLUTION)

SINGLE CHOICE QUESTIONS

1. H 1 = H 2 + H3

200C

H1

00 C

H2 H3

300C 100C
 


20     30   10
   
20 30 10
KA KA KA
3(20 – ) = 2( – 30) + 6( – 10)
Solving, we get
 = 16.4°C

2. (3L)  e ff  = L + 2L(2)()


5
   eff  
3

3. (1)
VL 
VL  VC  VL L   VC  C   100  C 100 = 60 %
VC L

4. (2)
From the relations P2 = cT and PV = nRT we get P = kV where k is some constant.
V2
kV 2  kV12 PV  PV nRT
dW = kV dV or W   kVdV  2  2 2 1 1
V1
2 2 2
nRT  3R 
Q  W  U  nC T   n   T  C  2R
2  2 

5. (3)
Rate of cooling in the first case r1 = 0.02 0C/min.
d 4eA T03 r T 3 T 3003 1 1
r  T  1  013 1  or r2 = 16r1 = 0.32 0C/min
dt ms r2 T02T2 6003  2 16
Temperature after 5 minute will be 329 – 0.32x5 = 327.4 0C

CENTERS: MUMBAI /DELHI /AKOLA /LUCKNOW /NASHIK /PUNE/NAGPUR /BOKARO /DUBAI # 221
6. (A)
3L 2L 5L
Thermal resistance of rod A is R1  similarly, R2  and R3 
4 KA 3 KA KC A
R1  R2 2
If resistance of rod C is R3 then 
R3 3

7. The rate of loss heat is given by


d

ms  4r 2 T 4
dt

d r 2
 Initial rate of cooling is  . Since
dt m
4 3
m r , r  m1 3
3
dQ m 2 3 1
Hence   13
dt m m
Therefore,
d
for S1  13 13
dt m of S2  1
   
d
for S2  m of S1  3
dt

8. The equation of state for an ideal gas of mass m and molecular mass M is
m
PV  RT (i)
M
For an isothermal process, T  constant. Differentiating (i) partially at constant T, we get
PV  VP  0
V
Or P  P (ii)
V
mRT
From (i), P  . Using this in (ii), we get
MV
mRT
P    V  2V  V  V 
MV
m RT m RT
 PA   A and PB   B
MV MV
PA mA
Hence 
PB m B
1 mA
Given PB  1.5PA . Therefore, 
1.5 m B

9. Given PT 2  k (constant). From PV  nRT , we have


nRT
P . Hence
V
nRT3  kv
Differentiating we have

CENTERS: MUMBAI /DELHI /AKOLA /LUCKNOW /NASHIK /PUNE/NAGPUR /BOKARO /DUBAI # 222
V 3nRT 2
3nRT 2 T  kV  
T k
Coefficient of volume expansion is
V 3nRT 2
  (1)
VT kV
nRT
Using V  and PT 2  k in Eq. (1),
P
3
We get  
T
10. Work done in P–V graph is equal to area of P–V graph
1 1
Area  base  height   (2V) (3P) = 3PV
2 2

11. (c)
PT  constant
 PV1/2  constant   u  
f nRT
ncT  nRT 
2 1  1/ 2
f
 nRT  2nRT f 6
2

12. (d)
U  a  b  pv  u  bpv
pv
  u  u  b pv 
1 
  1  1/ b

13. (d)
For AB Q  du  W for BC Q  du  W
Du = 110 W  40
Net u  0 as T  0
u AB  u BC  u CA  0
110  50  u CA  0
VCA  160 J

14. (d)
nRT
PV 2  C PV  nRT  P 
V
CENTERS: MUMBAI /DELHI /AKOLA /LUCKNOW /NASHIK /PUNE/NAGPUR /BOKARO /DUBAI # 223
nRTV 2 C
 C  TV  C  V  as V , T 
V T
So on expansion temp. of gas will decrease
 expansion will possible only with cooling.

15. (a)
T

a
C

From b to c
T  mv
PV  nRT
PV  nRmv  (P = constant)
From c to a (T = C)
PV  nRT
(PV = constant)  c to a will not be a straight line

16. (a)
Area under the curve  w 1  w 2  w 3 
17. (c)

18. (d)
1 5
Total energy = KT  KT
2 2
3
Translational  KT
2
5 3
Rotational = KT  KT  KT
2 2
3
KT
3
ratio  2 
KT 2
19. (d)
P
20. (a)
V

V
T
f 
V= mT Q  npdT  n   1 RdT
2 
PV = nRT w  nRdT
Q f 8
P = constant  1 
w 2 8 1

CENTERS: MUMBAI /DELHI /AKOLA /LUCKNOW /NASHIK /PUNE/NAGPUR /BOKARO /DUBAI # 224
21. (b)
5
Q1  n Cv dT  nRdT
2
P
2T 4T
2p

P T
V
V 2V

Work = Area under the curve  2PV  2nRT


5
du  n C v dt  n  3T 
2
Q  du  w
15 19nRT
Q  nRT  2nRT 
2 2
19nRT
nC3T 
2
19R
C
6

22. (c)
A A
p0 p0 2 p0 p0
T0

2P0 A
P0 A
T

T  P0 A  2P0 A T  P0A

23. (a)
2 nRT
PV 3 = constant PV = nRT P
V
nRT 2 3
V = constant
V
1 1
 TV 3 = constant  Tv 3 T  on v 

24. (a)

CENTERS: MUMBAI /DELHI /AKOLA /LUCKNOW /NASHIK /PUNE/NAGPUR /BOKARO /DUBAI # 225
25. (b)
P
B
3P0 x y

A 60  30
P0 60 30
a b
V 5v 0
V0 6V0

x 1
x sin 60  ysin 30 
y 13
a  x cos60 b ycos 30
a x 1 1
  a : b  1: 3
b y 13 3
5V 9V
VB  V0  0  0
4 4
PV P V T 27
Now at A and B 1 1  2 2  2 
T1 T2 T1 4

26. (a)
30gm mole
300 K 100 m s

f 1  f
nRT   mv2  n RT
2  2  2
1 30 2 60
5RT  100  T 
2 1000 R

27. (a)

28. (b)
nRT
PV  nRT  P 
v
nRT  v  nRT
w  pdv   
V  2 2

29. (b)
3
KT
2

30. (d)

31. (b)
  at

CENTERS: MUMBAI /DELHI /AKOLA /LUCKNOW /NASHIK /PUNE/NAGPUR /BOKARO /DUBAI # 226
2 t

  T    at dt

 0
2 t
at 2
n      

2 0

at 2
n2 
2
n4
n4  at 2  t 
a

32. (c)
R 4T R
R  T4    4%
R T R

33. (b)
1 dv
Coefficient of volume expansion  
v dt
dv
 4  105  10 106 T
dt
 dv 
at T  20 C,    4 105  20 105  16 105 m3  C
 dt 
v  0.5m3 , v  3.2 104  C

34. (c)
Sol. F  212 140  72o F
5 5  72
C  F   40 o
9 9

35. (a)
Sol. 3.14  R 0 1  4  , 3.76  R 0 1  100 

314 1  40
 or  =0.00379/ oC
376 1  100

36. (a)
Sol. Let  be the temperature of mixture then, heat given by O2
 7R  7R  37  
 diatomic, Cp    16  
 4  2 32
22  9R  0  27 
Heat taken by CO2  C p  4R  
2  44
7R 9R
 37         
4 4
259  7     or 16   or    C

CENTERS: MUMBAI /DELHI /AKOLA /LUCKNOW /NASHIK /PUNE/NAGPUR /BOKARO /DUBAI # 227
37. (c)
Sol. Rate of heat lost from calorimeter = 0.2 × 900 × 5 = 900Joule
Heat lost by liquid = 0.5 × 2400 × 5 = 6000 Joule
6900
Total heat lost =  115 Joule/sec.
60
38. (a)
Sol. Rate of heat production =1000 J/s
Rate of heat loss = 160 J/s
Net heat retained in the vessel per second = (1000 – 160) = 840 J/s
Let t be the time taken for temperature of the kettle to changes from 27 oC to 77oC, then
8400 t = [2000 × 1 ×(77 – 27)]
4.2 105
t  8 min .20sec .
840
39. (d)
Sol. At high temperature there is no possibility of liquefaction.
PV
Hence   (no. of moles) = 1 always.
RT

40. (b)
m PV.M
Sol. PV  RT , m 
M RT
V V
or m  , as we go from state 1 to 2, the ratio decreases. Therefore m decreases.
T T

41. (a)

42. (c)
Sol. PV  RT , for same value of V
PT
P2  P1  T2  T1

43. (c)

Sol. W
110 5
 5 105   5  1

6  105  4
 12 105 Joule
2 2

44. (a)
Sol. Pc  1.013  105 Pa , Vc  22.4 lit.
Pc Vc  RTc or 1.013  105  22.4  10 3  RTc
Tc  273K or 0oC

45. (a)
Sol. Work done = Area of cycle LMNO
= (3P – 2P)(2V – V) = PV
CENTERS: MUMBAI /DELHI /AKOLA /LUCKNOW /NASHIK /PUNE/NAGPUR /BOKARO /DUBAI # 228
46. (c)

47. (d)
Sol. For a cyclic process  U = 0

48. (d)
Sol. dQab  7000J, dWab ,  0 , as the vol. is constant.
 dU ab  dQab  7000J
5
7000    R  700 or R  4
2
Ta  300K , Tb  1000K
Tb  Tc  1000K
Ub  Uc
 dQ ac  dWac  7000  P1  V2  V1   7000
   RT2  RT1   7000  4 1000  300  7000
dQ  9800J

49. (d)
K1  30  25 K 2  25  22 
Sol. H1   A , H2  .A
t t
30o C 25o C 22o C

S V S
K E H
I S I
N T R
T

In steady state H1  H 2
K1 3
 5K1  3K 2 or 
K2 5
50. (b)
K    r 2   K    2r  
2

Sol. H1  , H2 
l1 l2
H1  k. r 2 .    K.4 r 2  
 / 
H2  l1   l2 
H1 l2 1
  
H 2 4l1 8
51. (a)
CENTERS: MUMBAI /DELHI /AKOLA /LUCKNOW /NASHIK /PUNE/NAGPUR /BOKARO /DUBAI # 229
52. (a)
Sol. Painted bulb will absorb more heat, its air will expand more  level in X will fall and that in Y will
increase.

53. (c)
KA  Qd
Sol. Q or  
d KA
4000 102 10
   100o C
400 100 104

54. (d)
New scale
100C 239 W

100divisions 200 divisions

0 C 39 C

 39 C  39  2  39   78  39  W  117  W

MULTIPLE CHOICE QUESTIONS


1. (c, d)
3RT 8RT 2RT
vrms  , vav  , vm 
M M M
So, v mp  vav  v rms

3 3 mv02 3 2
K.E.  RT =  mv p
2 2 2 4
2. (a, c)
 mT = constt. E  T4

3. (a, b, c, d)
Q
Q  nCdT  C 
ndT

4. (a, c)
1  1
CP  0.2    1 nRdT C V  0.15 
nRdT
2  2
CP 0.2  1  f 4 2 1 2
    1  1    f  6
C V 0.15  2  2 3 f 3 f

5. (a)

CENTERS: MUMBAI /DELHI /AKOLA /LUCKNOW /NASHIK /PUNE/NAGPUR /BOKARO /DUBAI # 230
6. (c)

3P0

2P0
V
v0 2v 0
Work done = Area under the curve
1 1
 V0  3P0  2P0   P0V0
2 2
7. (a, c)

8. (a)
A
O
P0 B
C

 for AOC   for ABC


v will be same for both as  T  0 
QAOC  Q ABC

9. (b, d)

10. (a, c, d)
L
 T
L
0.2
  0.25 104
100  80

11. (b, c)
pv   c
ln  p    ln v
 1   2  f1  f 2

12. (b)
Since force is always perpendicular to velocity.

13. (a, b, c, d)
One mole of an ideal monatomic gas (initial temperature T0 ) is made to go through the cycle abca
shown in fig. U denotes the internal energy.

CENTERS: MUMBAI /DELHI /AKOLA /LUCKNOW /NASHIK /PUNE/NAGPUR /BOKARO /DUBAI # 231
P

2P0 c

P0
T0 a b

V0 2V0 V
P0 V0 2P0 V0
For the process ab, 
T0 T0
Tb  2T0
Tb  Ta  U b  U a
3R 3RT
U b  U a  Cv T   2T0  T0   0
2 2
For the process bc,
P0  2V0  2P0  2V0 
  Tc  4T0
2T0 Tc
Tc  T0
3R
Uc  U b   4T0  2T0   3RT0
2
For the process ca,
3R 9RT0
Uc  Ua   4T0  T0  
2 2

14. (a,c)

15. (a, b, d)
Sol. (a) L t  L0 1  10 t 
L t  L0 1  10  
Option (a) is correct.
2 2
(b) A t   Lt   L20 1  10    L20 1  20   ,
Expanding binomially and leaving higher powers of  .
Then (b) is also correct.
(c) Density d 0  d t 1  t 
d0 d0
Or dt  
1  3s 10  1  30
Option (d) is correct.

16. (a, b, c)

17. (a, b, d)
Sol. (a) The area under path AB (1) is more than the area under path 2. Therefore option (a) is
correct.

CENTERS: MUMBAI /DELHI /AKOLA /LUCKNOW /NASHIK /PUNE/NAGPUR /BOKARO /DUBAI # 232
Press.
A

1
2
P
B
2

V 2V
vol.

(b) For straight path P  V or P  KV, in PV = R


KV 2  RT or V2  T
 T  V curve is a parabola
 Option (b) is also correct.
(c) Option (c) is not correct because P is not inversely proportional to T.
(d) Since T – V graph between A and B is a parabola, the temperature first increases and then
decreases
Thus option (d) is also correct.

18. (c, d)
2KA  1    KA    2 
Sol. In equilibrium 
d d
21  2    2

d d

 A B 
2K K

1  21  3  1  2 or 1    12o

19. (AC) 20. (ACD) 21. (ABC) 22. (ABCD)

23. (ACD) 24. (CD)

INTEGER TYPE
1. (4)
PT–n = constant
P (Pv)–n = const
n

1n
PV  const
R  R 5R R
C = CV +   Solve to get n = 4.
1 x 2 2 1 n
1 n

CENTERS: MUMBAI /DELHI /AKOLA /LUCKNOW /NASHIK /PUNE/NAGPUR /BOKARO /DUBAI # 233
2. (3)
KA
 K B  3K C
3
1 1 1 1 1 1 3
     
K K A k B K C 3K B K B K B
1 1 1  3
   1  3 K  K
K KB  2  13

3. (750 m/s)

n1c v 1 T1  n 2 cv 2 T2  n1c v1  n 2 cv 2 T 
3RT
vrms 
M

4. (6)
n  C P  T  160 n C v T '  240
CP 50 2 CP 4
  
CV 100 3 CV 3
2 4 2 1
 1   
f 3 f 3

5. (28)
dT
 K  Tavg  Ts 
dt
20 40  T  40  T 
  K  50  10   K  10 
7 7  2 
40  T 40  T  20
  160  4T  20  T
20 2  40
 140  5T  T  28

6.  60 C 

20
40 C
K2
20 10 C
K1
T T

1  2
K 1  T  20   K 2  40  ……(1)
K1  T  10   K 2  50  ……(2)
From (1) and (2)
T  20 4
  5T  100  4T  40
T  10 5
T  100  40
T  60 C

CENTERS: MUMBAI /DELHI /AKOLA /LUCKNOW /NASHIK /PUNE/NAGPUR /BOKARO /DUBAI # 234
7. (1)

8. (3)

dx

When the element falls on table


m m dv
F  gx  2g 
  dt
3mgx


3mg
When x   / 2  F 
2

SUBJECTIVE TYPE
1.

RT0 3R 5R
P0 = [from 1 – 2], CV = , CP =
2V0 2 2
5
1 – 2, Q(1) = nCPT = – RT0
2
2 – 3, Q(2) = W = –RT0ln2
5
3 – 4, Q (3) = nCPT = RT0
2
4 – 1, Q(4) = W = R(2T0)ln2 = 2RT0 ln2
5 
  Qinput = Q(3) + Q(4) = RT0   2 ln 2 
2 
   Qtotal = RT0ln2   
QTotal ln 2 0.7
    =  100% = ×100% = × 100%
Q Input 5 2.5  1.4
2 2 ln 2
700
   = % = 17.95%
39

CENTERS: MUMBAI /DELHI /AKOLA /LUCKNOW /NASHIK /PUNE/NAGPUR /BOKARO /DUBAI # 235
2.

R 5R
v = 1.4 CV = 
0.4 2

PROCESS U W Q
 5R  45
n 9T0  nRT0
1-2  2  0 2
 5R 
n 90T0 
2-3  2  90PoVo 315 PoVo
 5R 
 n 90T0 
3-4  2  0 – 405 PoVo
 5R 
 n 9T0  
63
Po Vo
4-1  2  – 9 Po Vo 2

Total 0 81 PoVo
use PoVo = nRTo
81Po Vo 81
efficiency =   0.24  24%
315Po Vo  22.5Po Vo 337.5

3. Let atmospheric pressure be P0 , and let density of liquid at 5 C be  and co-efficient of thermal
expansion of liquid be  .
Density of liquid at 95 C ,  '   1   95  5      1  90   …..(1)
Pressure at bottom of column A, P1   P0  h1 'g 
Let pressure at top of column B be P2 .
Then pressure at bottom of this column  P2  h 0g
But pressure at bottom of column A and B is same therefore, pressure at bottom of column B is also
equal to P1 .
 P2  h 0g  P0  h1 'g
or P2  P0  h1 'g  h 0g
Pressure at top of column B and C are same. Therefore, pressure at top of column C is also equal to
P2 .
 Pressure at bottom of column C, P3  P2  n 0 'g

CENTERS: MUMBAI /DELHI /AKOLA /LUCKNOW /NASHIK /PUNE/NAGPUR /BOKARO /DUBAI # 236
or P3   P0  h1 'g  h 0g   h 0 'g  P0   h1  h 0   'g  h 0g
Pressure at bottom of column D  P0  h 2g
But pressure at bottom of column C and D is equal
 P0  h 2g  P0   h1  h 0   'g  h 0g
or  h 2  h 0  g   h1  h 0   'g …..(2)
From equations (1) and (2),
 h 2  h 0  g   h1  h 0  1  90  g
or 
 h1  h 2   1 per degree celcius
90  h1  h 0  5090

CENTERS: MUMBAI /DELHI /AKOLA /LUCKNOW /NASHIK /PUNE/NAGPUR /BOKARO /DUBAI # 237
MODERN PHYSICS SEMICONDUCTORS

DUAL NATURE OF RADIATION AND MATTER

CONCEPTS SUMMARY
1. The minimum energy needed by an electron to come out from a metal surface is called the work
function of the metal. Energy (greater than the work function ( 0 ) required for electron emission
from the metal surface can be supplied by suitably heating or applying strong electric field or
Irradiating it by light of suitable frequency.

2. Photoelectric effect is the phenomenon of emission of electrons by metals when illuminated by light
of suitable frequency. Certain metals respond to ultraviolet light while others are sensitive even to
the visible light. Photoelectric effect involves conversion of light energy into electrical energy. It
follows the law of conservation of energy. The photoelectric emission is an instantaneous process
and possesses certain special features.

3. Photoelectric current depends on (i) the intensity of incident light, (ii) the potential difference applied
between the two electrodes, and (iii) the nature of the emitter material.

4. The stopping potential (V0) depends on the frequency of incident light, and (ii) the nature of the
emitter material. For a given frequency of incident light, it is independent of its intensity. The
stopping potential directly related to the maximum kinetic energy of electrons emitted:
eV0  1/ 2  mv max
2
 K max .

5. Below a certain frequency (threshold frequency) v0, characteristic of the metal, no photoelectric
emission takes place, no matter how large the intensity may be.

6. The classical wave theory could not explain the main features of photoelectric effect. Its picture of
continuous absorption of energy from radiation could not explain the independence of Kmax on
intensity, the existence of v0 and the instantaneous nature of the process. Einstein explained these
features on the basis of photon picture of light. According to this, light is composed of discrete
packets of energy called quanta or photons. Each photon carries an energy E (= hv) and momentum p
(= h/  ), which depend on the frequency (v) of incident light and not on its intensity. Photoelectric
emission from the metal surface occurs due to absorption of a photon by an electron.

7. Einstein’s photoelectric equation is in accordance with the energy conservation law as applied to the
photon absorption by an electron in the metal. The maximum kinetic energy (l/2) m v2max is equal to
the photon energy (hv) minus the work function 0max   hv 0  of the target metal:
1
mv 2max  v 0 e  hv  0  h  v  v 0 
2
This photoelectric equation explains all the features of the photoelectric effect. Millikan’s first
precise measurements confirmed the Einstein’s photoelectric equation and obtained an accurate
value of Planck’s constant h. This led to the acceptance of particle or photon description (nature) of
electromagnetic radiation, introduced by Einstein.

8. Radiation has dual nature: wave and particle. The nature of experiment determines whether a wave
or particle description is best suited for understanding the experimental result. Reasoning that
radiation and matter should be symmetrical in nature, Louis Victor de Broglie attributed a wave-like
character to matter (material particles). The waves associated with the moving material particles are
called matter waves or de Broglie waves.
CENTERS: MUMBAI /DELHI /AKOLA /LUCKNOW /NASHIK /PUNE/NAGPUR /BOKARO /DUBAI # 238
9. The de Broglie wavelength (  ) associated with a moving particle is related to its momentum p as:
 = h/p. The dualism of matter is inherent in the de Broglie relation which contains a wave concept
(  ) and a particle concept (p). The de Broglie wavelength is independent of the charge and nature of
the material particle. It is significantly measurable (of the order d atomic-planes spacing in crystals)
only in case of sub-atomic particles like electrons, protons. etc. (due to smallness of their masses and
hence, momenta). However, it is indeed very small, quite beyond measurement, in case of
macroscopic objects, commonly encountered in everyday life.

10. Electron diffraction experiments by Davisson and Germer, and by G. P. Thomson, as well as many
later experiments, have verified and the wave-nature of electrons. The de Broglie hypothesis of
matter waves supports the Bohr’s concept of stationary orbits.

FORMULA SUMMARY
 Photoelectric Effect :
1 2
h  W0  mv max  W0  | e  Vs |
2
W0 (work function) =h0
0 : threshold frequency
Vs : stopping potential
mV 2
Vs =
2e
If intensity of incident light increased, then photocurrent increases

i i
  cons tan t
I2
I1  2   1 
I 2  I1

 Vs V  VS2  VS1 V

 De –Broglie wavelength :
h h h
  
p mv 2m K. E.
For an electron accelerated across P. D. of ‘V’ volts
150
 A0
V
m e  9.11 10 31 kg
rest of mass of photon is zero
E
m = 2 , E  pc
C
power of a light source, P = Nh
where n = no. of photons incident / sec / Area.
 p 
 4 r 2  A 
Photons flux is the number of photons incident on plate per second, N =  
 h 
 
 

CENTERS: MUMBAI /DELHI /AKOLA /LUCKNOW /NASHIK /PUNE/NAGPUR /BOKARO /DUBAI # 239
p I
Photons flow density is the number of photons incident on unit area = 2

4 r h h

Photon concentration : No. of photons per unit volume of space =
h c
Radiation pressure ( normal incidence ) : -

I
(i) perfectly absorbing surface, R.P. =
C
2I
(ii) for perfectly reflecting surface , R.P. =
C
I
(iii) for surface having reflection coefficient , R.P. = ( 1 +  )
C

It light of power P is incident on a surface, ( normal incidence ) then force exerted is


P
(i) F = ( perfectly absorbing )
C
2P
(ii) F = ( perfectly reflecting )
C
P
(iii)F = ( 1 +  ) ( reflection coefficient  )
C
P
If point source power P is placed at centre of perfectly reflecting hemisphere , then F =
2C
If perfectly reflecting solid sphere ( radius r ) is kept in path of parallel beam intensity I, then
 r2I
F=
C
Impulse applied by photons on surface, where E is the total energy of light incident normally
E
Impulse, I = ( perfectly absorbing )
C
2E E
I= ( perfectly reflecting ) I = ( 1 +  ) (reflection coefficient )
C C
ATOMS
CONCEPT SUMMARY
1. Atom, as a whole, is electrically neutral and therefore contains equal amount of positive and negative
charges.

2. In Thomson’s model, an atom is a spherical cloud of positive charges with electrons embedded in it.

3. In Rutherford’s model, most of the mass of the atom and all its positive charge are concentrated in a
tiny nucleus (typically one by ten thousand the size of an atom), and the electrons revolve around it.

4. Rutherford nuclear model has two main difficulties in explaining the structure of atom: (a) it predicts
that atoms are unstable because the accelerated electrons revolving around the nucleus must spiral
into the nucleus. This contradicts the stability of matter. (b) It cannot explain the characteristic line
spectra of atoms of different elements.

5. Atoms of each element are stable and emit characteristic spectrum. The spectrum consists of a set of
isolated parallel lines termed as line spectrum. It provides useful information about the atomic
structure.

CENTERS: MUMBAI /DELHI /AKOLA /LUCKNOW /NASHIK /PUNE/NAGPUR /BOKARO /DUBAI # 240
6. The atomic hydrogen emits a line spectrum consisting of various series. The frequency of any line in
a series can be expressed as a difference of two terms;
1 1 
Lyman series: v  Rc  2  2  ; n  2, 3, 4.....
1 n 
 1 1 
Balmer series: v  Rc  2  2  ; n  2, 4,5.....
2 n 
1 1 
Paschen series: v  Rc  2  2  ; n  4,5, 6...
3 n 
 1 1 
Brackett series: v  Rc  2  2  ; n  5, 6, 7.....
4 n 
 1 1 
Pfund series: c  Rc  2  2  ; n  6, 7,8....
5 n 
7. To explain the line spectra emitted by atoms, as well as the stability atoms, Niel’s Bohr proposed a
model for hydrogenic (single electron) atoms. He introduced three postulates and laid the
foundations of quantum mechanics:
(a) In a hydrogen atom, an electron revolves in certain stable orbits (called stationary orbits) without
the emission of radiant energy.
h
(b) The stationary orbits are those for which the angular momentum is same integral multiple of .
2
(Bohr’s quantisation condition.). That is L = nh/2  , where n is an integer called a quantum number.
(c) The third postulate states that an electron might make a transition from one of its specified non-
radiating orbits to another of lower energy. When it does so, a photon is emitted having energy equal
to the energy difference between the initial and final states. The frequency (v) of the emitted photon
is then given by
hv  Ei  Ef
An atom absorbs radiation of the same frequency the atom emits, in which case the electron is
transferred to an orbit with a higher value of n.
Ei  hv  Ef

8. As a result of the quantisation condition of angular momentum, the electron orbits the nucleus at
only specific radii. For a hydrogen atom it is given by
2
 n 2   h  4e0
rn      2
 m   2  e
The total energy is also quantised:
me4
E n   2 2 2  13.6eV / n 2
8n 0 h
The n = 1 state is called ground state. In hydrogen atom the state energy is —13.6 eV. Higher values
of n correspond to states (n > 1). Atoms are excited to these higher states by collisions with other
atoms or electrons or by absorption of a photon of right frequency.

9. de Brogue’s hypothesis that electrons have a wavelength   h / mv gave an explanation for Bohr’s
quantised orbits by bringing in the wave particle duality. The orbits correspond to circular standing
which the circumference of the orbit equals a whole number of wavelengths.

10. Bohr’s model is applicable only to hydrogenic (single electron) atoms. It cannot be extended to even
two electron atoms such as helium. This model is also unable to explain for the relative intensities
frequencies emitted even by hydrogenic atoms.

CENTERS: MUMBAI /DELHI /AKOLA /LUCKNOW /NASHIK /PUNE/NAGPUR /BOKARO /DUBAI # 241
FORMULA SUMMARY
(I) Physics Of The Atom
Bohr’s Theory : (Applicable for monoelectronic species)
Me :mass of electron in nth Bohr orbit
v : Velocity of electron in nth Bohr orbit
r : Radius of electron in nth Bohr orbit
h: Planck’s constant = 6.626  10 34 J  s
z : Atomic number

e2 mv 2
(1) 
4 0 r 2 r
nh
(2) mvr 
2
n 2 h 2 0  n2  0
 Radius of orbit (r) =   0.529  A
Zme  2
Z 
Ze 2  Z
 Velocity of electron (v)    2.18  10 6  m / s
2nh 0  n
 Z 2 me 4  Z2 
 Energy of electron (E) = =   13 .6   eV
8n 2 h 2 02  n2 
m Z2 e4  Z 2 
 Frequency  = 2 3 3
  3   6.6  1015 Hz
4 0 n h n 
3
n 
Time period T =  2   1.52  10 16 sec
Z 
 Z3 
Magnetic field, B =  5   12.5 Tesla
n 
  e 
Magnetic moment ,    L
 2m 
  eh 
  n   = n Bohr magneton.
 4 m 

 Energy of emitted photon due to electron transition


h = E n 2  E n1
 1 1  1  1 1 
 13.6  z 2  2  2  eV

or    R Z 2  2  2 

 n1 n 2    n1 n2 

R = 1.097 107 m 1

Note : (1) hcR =13.6eV


1
(2)  9.11  10 8 m
R
(3)hc = 1242 eV-nm

CENTERS: MUMBAI /DELHI /AKOLA /LUCKNOW /NASHIK /PUNE/NAGPUR /BOKARO /DUBAI # 242
 Spectral series of hydrogen atom

Sr.No Series n1 n2 Position


1 Lyman 1 2,3,4… U.V.
2 Balmer 2 3,4,5… Visible & U.V.
3 Paschen 3 4,5,6… Infrared
4 Brackett 4 5,6,7… Infrared
5 Pfund 5 6,7,8… Infrared
6 Humphrey 6 7,8,9… Infrared
 -line : Minimum energy (n1  n1+1 transition ) series limit :(n1   Transition)
n ( n  1)
for energy levels ( 1 to n ) , no. of emission spectral lines = , no. of absorption lines
2
= ( n – 1)
(i) Bohr’s theory when nucleus is not stationary :
If M and m are masses of nucleus and electron respectively V1 , V2 their velocities , then
nh nh
V1 = V2 =
2 Mr 2 mr

mM
If reduced mass m =
mM
m' Z 2 e 4  Z2   m' 
Total energy E = - 2
= - 13.6  2   
8 0 n 2 h 2 n  m 
 n 2h 2  m  n 2  0
radius r = 0 2     (0.529) A
 Ze m'  m'   Z 
 m' 
new Rydberg constant , R =   R
m
1 1 1
  R'  2  2 
'  n1 n 2 

(ii) If joint kinetic energy of colliding particles is less than 20.4 eV ( considering particles as H – atoms
or one neutron and one H - atom ) then the nature of collision will be necessarily elastic.

(iii) Doppler effect in photon emission :


V V
for approaching bodies  = -   ,  =  
c c
V
for receding bodies  =  
c
V
 = -  
c

0
(iv) If recoil of atom is considered, then new frequency .   where 0 is the original
 h 0 
1  2 
 2mc 
frequency when atom is stationary.

CENTERS: MUMBAI /DELHI /AKOLA /LUCKNOW /NASHIK /PUNE/NAGPUR /BOKARO /DUBAI # 243
NUCLEI
CONCEPT SUMMARY

1. An atom has a nucleus. The nucleus is positively charged. The radius of the nucleus is smaller than
the radius of an atom by a factor of l04. More than 99.9% mass of the atom is concentrated in the
nucleus.

2. On the atomic scale, mass is measured in atomic mass units (u). By definition, 1 atomic mass unit
(1u) is 1/12th mass of one atom of 12C; 1u = 1.6605631027 kg

3. A nucleus contains a neutral particle called neutron. Its mass is almost the same as that of proton.

4. The atomic number Z is the number of protons in the atomic nucleus of an element. The mass
number A is the total number of protons and neutrons in the atomic nucleus, A = Z+N; Here N
denotes the number neutrons in the nucleus.
A nuclear species or a nuclide is represented as AZ X , where X is the chemical symbol of the species.
Nuclides with the same atomic number Z but different neutron number N are called isotopes.
Nuclides with the same A are isobars and those with the same N are isotones. Most elements are
mixtures of two or more isotopes. The atomic mass of an element is a weighted average of the
masses of Its isotopes The masses are the relative abundances of the Isotopes.

5. A nucleus can be considered to be spherical in shape and assigned a radius Electron scattering
experiments allow determination of the nuclear radius It is found that radii of nuclei fit the formula
R= R0 A1/3 where R0 = a constant = 1.2 fm. This implies that the nuclear density is independent of A.
It is of the order of 1017 kg/m3.

6. Neutrons and protons are bound in a nucleus by the short-range strong nuclear force. The nuclear
force does not distinguish between neutron and proton.

7. The nuclear mass M is always less than the total mass,  m , of its constituents. The difference in
mass of a nucleus and its constituents is called the mass defect, M=(Zmp+(A–Z)mn)–M
Using Einstein s mass energy relation we express this mass difference in terms of energy
Eb = M c2
The energy Eb represents the binding energy of the nucleus. In the mass number range A = 30 to
170, the binding energy per nucleon is nearly constant about 8 MeV nucleon.

8. Energies associated with nuclear processes are about A million times larger than chemical process.

9. The Q-value of a nuclear process is Q = final kinetic energy  initial kinetic energy
Due to conservation of mass-energy this is also,Q (sum of initial masses  sum of final masses) c2

10. Radioactivity is the phenomenon which nuclei of a given species transform by giving out  or  or
 rays;  -rays are helium nuclei;  -rays are electrons.  -rays are electromagnetic radiation of
wavelengths shorter than X-rays:

11. Law of radioactive decay : N (t) = N(0) e t where  is the decay constant or disintegration constant.
The half-life T1 2 of a radionuclide is the time in which N has been reduced to one-half of its initial
value. The mean life  is the time at which N has been reduced to e–1 of its initial value

CENTERS: MUMBAI /DELHI /AKOLA /LUCKNOW /NASHIK /PUNE/NAGPUR /BOKARO /DUBAI # 244
ln 2
T1/2    ln 2

12. Energy is released when less tightly bound nuclei are transmuted into more tightly bound nuclei. In
235
fission, a heavy nucleus like 92 U breaks into two smaller fragments.
235
e.g., 92 U  10 n 133 99 1
51 Sb  41 Nb  4 0 n

13. The fact that more neutrons are produced in fission than are consumed gives the possibility of a
chain reaction with each neutron that is produced triggering another fission. The chain reaction is
uncontrolled and rapid in a nuclear bomb explosion, it is controlled and steady in a nuclear reactor.
In a reactor, the value of the neutron multiplication factor k is maintained at 1.

14. In fusion, lighter nuclei combine to form a larger nucleus. Fusion of hydrogen nuclei into helium
nuclei is the source of energy of all stars including our sun.

FORMULA SUMMARY

(II) Nuclear Physics

 Radioactivity : N=N0e-t

2.303 N  1n 2 0.693
= log  0  t1/ 2  
t  N   
1
Average life or Mean life =

Activity A = A0e-t

Types of decay :
(a) decay with continuous formation :
a decays
A B
formation 
At t = 0 , NA = N0 , NB = 0
1
NA,t =


q  (  N 0  q ) e  t 
(b) successive disintegration :
A 1
B 2
C
t = 0 N0 0 0
time, t N1 N2 N3

N1 = No e  1t
N 0 1  1t
N2 =
 2  1

e  e  2 t 
  e  2 t   2 e  1t 
N3 = N0  1  1
  2  1 

CENTERS: MUMBAI /DELHI /AKOLA /LUCKNOW /NASHIK /PUNE/NAGPUR /BOKARO /DUBAI # 245
(c) simultaneous disintegration
1 B
A then eff = 1 + 2
2 C

(d) Radioactive equilibrium : -


in a radioactive series like 92
238 206
U  82 Pb 
for all intermediate elements 1N1 = 2N2 = …. = n Nn

(e) Disintegration of isotopes :


If 1 , 2 and 3 are decay constants of A, B and C and N1 , N2 and N3 are their respective
 N  2 N 2  3 N 3
initial counts, then net = 1 1
N1  N 2  N 3

 Units of activity :
(1) 1 Becquerel = 1 disintegration per second
(2) 1 rutherford =106 Bq = 106 d.p.s
(3) 1 Curie = 3.7 1010 dps

 Mass Energy equivalence:


E = mc2
931 MeV
1amu = 2
 1.66 10 27 Kg
C
Binding energy of nucleus = (m) c2
where m is mass defect equal to difference between sum of masses of particles composing nucleus
&
mass of nucleus. If m is mass defect in a.m.u. , then
E= (931  m) Mev
For AXz, m = Zmp +(A-Z) Mn-M

 
Zm 11 H  A  Z m n  m AZ X  
We neglect the small difference due to binding energy of ‘e’ s with the nucleus

Nuclear collisions :

Ei = m1C2 + m2C2 + K1 + K2
Ef = m3C2 + m4C2 + K3 + K4
Disintegration energy or Q value of reaction = Q = (m1  m 2 )  (m3  m 4 )  931 MeV.
For  decays :
Q= m   X   m
A
Z
A4
Z 2   
Y  m 42 He C2
For  decay :
Q= m   X  m 
A
Z
A
Z1 
Y C2
+
For  decay,
 
Q = m  AZ X   m  AZ1 Y   2M e C 2
For K capture ,
CENTERS: MUMBAI /DELHI /AKOLA /LUCKNOW /NASHIK /PUNE/NAGPUR /BOKARO /DUBAI # 246
Q= m   X   m
A
Z
A
Z1  
Y C2
A
Note : m ( X) is atomic mass ( including electrons )
Z

A  4
In  dacay, K.E. =   Q.
 A 
Threshold energy : - It is minimum energy needed by bombarding particle to initiate endoergic
reaction :
 m 
Eth = - Q 1  1 
 m2 

 Fission and Fusion Process:


In these process , the energy released = m c2 where
m =(sum of masses of reactants) –(sum of masses of products)

3
Note : In nuclear fusion, high temperature is required , s.t. 2 kT = work required to bring
2
two particles together from infinity

For nuclei of different element the density still remains same


R  R 0 A1/ 3 A : Mass Number
R 0  1.1 10 15 m density,  = 2.3 × 1017 kg/m3

SEMICONDUCTOR ELECTRONICS MATERIALS, DEVICES AND SIMPLE CIRCUITS

CONCEPT SUMMARY

1. Semiconductors are the basic materials used in the present solid state electronic devices like diode,
transistor, ICs. etc.

2. Lattice structure and the atomic structure of constituent elements decide whether a particular material
will be insulator, metal or semiconductor.

3. Metals have low resistivity (10 –2 to 10 –8 m), insulators have very high resistivity (>10 8 m), while
semiconductors have intermediate values of resistivity.

4. Semiconductors are elemental (Si, Ge) as well as compound (GaAs, CdS, etc.).

5. Pure semiconductors are called ‘Intrinsic semiconductors’. The presence of charge carriers (electrons
and holes) is an ‘intrinsic’ property of the material and these are obtained as a result of thermal
excitation. The number of electrons (ne) is equal to the number of holes (nh) in intrinsic conductors.
Holes are essentially electron vacancies with an effective positive charge.

6. The number of charge carriers can be changed by ‘doping’ of a suitable impurity in pure
semiconductors. Such semiconductors are known as extrinsic semiconductors. These are of two types
(n-type and p-type).

7. In n-type semiconductors, ne >> nh while in p-type semiconductors nh >> ne

CENTERS: MUMBAI /DELHI /AKOLA /LUCKNOW /NASHIK /PUNE/NAGPUR /BOKARO /DUBAI # 247
8. n-type semiconducting Si or Ge is obtained by doping with pentavalent atoms (donors) like As, Sb,
P. etc., while p-type Si or Ge can be obtained by doping with trivalent atom acceptors) like B, Al, In
etc.
9. nenh = n 12 in all cases. Further the material possesses an overall charge neutrality.

10. There are two distinct band of energies (called valence band and conduction band) in which the
electrons in a material lie. Valence band energies are low as compared to conduction band energies.
All energy levels in the valence band are filled while energy levels in the conduction band may be
fully empty or partially filled. The electrons in the conduction band are free to move in a solid and
are responsible for the conductivity. The extent of conductivity depends upon the energy gap (Eg)
between the top of valence band (Ev) and the bottom of the conduction band EC. The electrons from
valence band can be excited by heat, light or electrical energy to the conduction band and thus
produce a change in the current flowing in a semiconductor.

11. For insulators Eg> 3 eV, for semiconductors Eg is 0.2 eV to 3 eV, while for metals Eg= 0.

12. p-n Junction is the ‘key’ to all semiconductor devices. When such a Junction is made, a ‘depletion
layer’ is formed consisting of immobile ion-cores devoid of their electrons or holes. This is
responsible for a junction potential barrier.

13. By changing the external applied voltage, junction barriers can be changed. In forward bias (n-side is
connected to negative terminal of the battery and p-side is connected to the positive). The barrier is
decreased while the barrier increases in reverse bias. Hence, forward bias current is more (mA) while
it is very small (A) In a p-n junction diode:

14. Diodes can be used for rectifying an ac voltage (restricting the ac voltage to one direction). With the
help of capacitor or a suitable filter, a dc voltage can be obtained.

15. There are some special purpose diodes.

16. Zener diode is one such special purpose diode. In reverse bias, after a certain voltage, the current
suddenly increases (breakdown voltage) in a Zener diode. This property has been used to obtain
voltage regulation.

17. p-n junctions have also been used to obtain many photonic or optoelectronic devices where one of
the participating entity is ‘photon’: (a) Photodiodes in which photon excitation results in a change of
reverse saturation current which helps us to measure light intensity: (b) Solar cells which convert
photon energy into electricity; (c) Light Emitting Diode and Diode Laser in which electron excitation
by a bias voltage results In the generation of light.

18. Transistor is an n-p-n or p-n-p Junction device. The central block (thin and lightly doped) is called
‘Base’ while the other electrodes are ‘Emitter’ and ‘Collectors’. The emitter-base Junction is forward
biased while collector-base junction is reverse biased.

19. The transistors can be connected in such a manner that either C or E or B is common to both the
input and output. This gives the three configurations in which a transistor is used: Common Emitter
(CE), Common Collector (CC) and Common Base (CB). The plot between IC and VCE for fixed IB is
called output characteristics while the plot between IB and VBE with fixed VCE is called input
characteristics. The Important transistor parameters for CE-configuration are:

CENTERS: MUMBAI /DELHI /AKOLA /LUCKNOW /NASHIK /PUNE/NAGPUR /BOKARO /DUBAI # 248
 V 
input resistance r1   BE 
 I B  VCE
 V 
output resistance ro   CE 
 I C  IB
 I 
current amplification factor    C 
 I B  VCE
20. Transistor can be used as an amplifier and oscillator. In fact, ah oscillator can also be considered as a
self-sustained amplifier in which a part of output is fed-back to the input in the same phase (positive
feedback). The voltage gain of a transistor amplifier in common emitter configuration is:
v  RC
O 
=   v    R , where RC and RB are respectively the resistances in collector and base sides of
 I B

the circuit.

21. When the transistor is used in the cutoff or saturation state, it acts as a switch.

22. There are some special circuits which handle the digital data consisting of 0 and 1 levels. This forms
the subject of Digital Electronics.

23. The important digital circuits performing special logic operations are called logic gates. These are:
OR. AND, NOT, NAND. and NOR gates.

24. In modem day circuit, many logical gates or circuits are integrated in one single ‘Chip’. These are
known as integrated circuits (IC).

COMMUNICATION SYSTEM

1. Electronic communication refers to the faithful transfer of information or message (available in the
form of electrical voltage and current) from one point to another point.

2. Transmitter, transmission channel and receiver are three basic units of a communication system.

3. Two important forms of communication system are: Analog and Digital. The information to be
transmitted is generally in continuous waveform for the former while for the latter it has only
discrete or quantised levels.

4. Every message signal occupies a range of frequencies. The bandwidth of a message signal refers to
the band of frequencies. Which are necessary for satisfactory transmission of the information
contained in the signal. Similarly, any practical communication system permits transmission of a
range of frequencies only, which is referred to as the bandwidth of the system.

5. Low frequencies cannot be transmitted to long distances. Therefore, they are superimposed on a high
frequency carrier signal by a process known as modulation.

6. In modulation, some characteristic of the carrier signal like amplitude, frequency or phase varies in
accordance with the modulating or message signal. Correspondingly, they are called Amplitude
Modulated (AM). Frequency Modulated (FM) or Phase Modulated (PM) waves.

CENTERS: MUMBAI /DELHI /AKOLA /LUCKNOW /NASHIK /PUNE/NAGPUR /BOKARO /DUBAI # 249
7. Pulse modulation could be classified as: Pulse Amplitude Modulation (PAM), Pulse Duration
Modulation (PDM) or Pulse Width Modulation (PWM) and Pulse Position Modulation (PPM).

8. For transmission over long distances, signals are radiated into space using devices called antennas.
The radiated signals propagate as electromagnetic waves and the mode of propagation is influenced
by the presence of the earth and its atmosphere. Near the surface of the earth, electromagnetic waves
propagate as surface waves. Surface wave propagation is useful up to a few MHz frequencies.

9. Long distance communication between two points on the earth is achieved through reflection of
electromagnetic waves by ionosphere. Such waves are called sky waves. Sky wave propagation takes
place up to frequency of about 30 MHz. Above this frequency, electromagnetic waves essentially
propagate as space waves. Space waves are used for line-of-sight communication and satellite
communication.

10. If an antenna radiates electromagnetic waves from a height h T then the range dT .. is given by
2Rh T where R is the radius of the earth.

11. Amplitude modulated signal contains frequencies ( c m ) and c

12. Amplitude modulated waves can be produced by application of the message signal and the carrier
wave to a non-linear device, followed by a band pass filter.

13. AM detection, which is the process of recovering the modulating signal from an AM waveform, is
carried out using a rectifier and an envelope detector.

SINGLE CHOICE QUESTIONS


1. Of the following third period elements Mg, Al, Si, S, Ar the elements that are diamagnetic in
ground state, are :
(A) Si and S (B) Al, Si and S (C) Mg and Ar (D) Mg, Si, S, Ar

2. An alternating current flows through a circuit consisting of inductance L and resistance R.


Periodicity of the supply is /2. Which of the following is true?
(A) The limiting value of impedance is L for low frequency
(B) The limiting value of impedance for high frequency is R.
(C) Limiting value of impedance for high frequency is L
(D) The limiting value of impedance for low frequency is L

3. An electron and photon have same wavelength. If P is the momentum of electron and E the
energy of photon. Then magnitude of P/E in Sl unit is :
(A) 3.0 × 108 (B) 3.33 × 10–9 (C) 9.1 × 10–31 (D) 6.64 × 10–34

4. The combination of the gates shown in fig.


A

produces
(A) NOR gate (B) OR gate (C) AND gate (D) XOR gate
CENTERS: MUMBAI /DELHI /AKOLA /LUCKNOW /NASHIK /PUNE/NAGPUR /BOKARO /DUBAI # 250
5. In hydrogen and hydrogen like atoms, the ratio of difference of energies E4n – E2n and E2n – En
varies with its atomic number z and n as :
(A) z2/n2 (B) z4/n4 (C) z/n (D) z0n0

6. If the surface of a metal is successively exposed to radiations of wavelengths λ1 = 350 nm and λ2 =


450 nm, the maximum velocity of photoelectrons differs by a factor of 2. Work function of the metal
is:
(A) 2.8 × 10-19 J (B) 4 × 10 -19 J (C) 3.2 × 10-19 J (D) None of these

7. Energy levels A, B, C of a certain atom correspond to increasing values of C


energy i.e., EA< EB< EC. If λ1, λ2, λ3 are the wavelengths of radiations λ1
corresponding to transitions C to B, B to A and C to A respectively, which B
of the following statements is correct?
12
(A) λ3 = λ1 + λ2 (B) 3  λ2 λ3
1  2
(C) λ1 + λ2 + λ3 = 0 (D) 3  12  22
2 A

8. Initial number of undecayed atoms in a radioactive sample is 1000. It is found that 100 atoms disintegrated in
2 hours. The number of undecayed atoms in the sample after next 4 hours will be
(A) 700 (B) 1 (C) 810 (D) 729

9. In the given circuit, D1 and D2 are diodes

(A) D1 and D2 both are forward biased (B) D1 is forward biased but D2 is reverse biased
(C) D1 is reverse biased but D2 is forward biased (D) D1 and D2 both are reverse biased

10. When a certain photosensitive surface is illuminated with monochromic light of frequency v, the
stopping potential for photoelectric current is V0 2 . When the same surface is illuminated by
monochromatic light of frequency 3 v 2 , the stopping potential is V0 . The threshold frequency for
photoelectric emission is
2v v 3v 5v
(A) (B) (C) (D)
3 2 5 3
11. The distance of the closest approach of an alpha particle fired towards a fixed nucleus with kinetic
energy K is r0 . The distance of the closest approach when the alpha particle is fired at the same
nucleus with kinetic energy 2K will be
r r
(A) 2r0 (B) 4r0 (C) 0 (D) 0
2 4
12. In an n-p-n transistor circuit, the collector current is 10mA. If 80% of the electrons emitted from
emitter reach the collector, then
(A) the emitter current will be 7.5mA (B) the emitter current will be 12.5mA
(C) the base current will be 3.5mA (D) the base current will be 2.0mA

CENTERS: MUMBAI /DELHI /AKOLA /LUCKNOW /NASHIK /PUNE/NAGPUR /BOKARO /DUBAI # 251
4
13. In a semiconductor, it is found that th of the total current is carried by electrons and the remaining
5
1
th by the holes. If at this temperature, the drift speed of electrons is 2.5 times that of holes, the
5
ratio of the number densities of electrons and holes is
8 5 25 1
(A) (B) (C) (D)
5 8 8 2
14. The circuit shown in figure contains two diodes D1 and D 2 each with a forward resistance of 50
ohms and with infinite backward resistance. If the battery voltage is 6V, the current through the 100
ohm resistance (in amperes) is
150

D1
50

D2 100

6v
(A) zero (B) 0.02 (C) 0.03 (D) 0.036

15. The potential energy U of a moving particle of mass m varies with x shown in figure. The de-Brogile
wavelengths of the particle in the regions 0  x  1 and x  1 are 1 and  2 respectively. If the total
energy of the particle is nE , the ratio 1  2 is

U
E

0 1 x
n n n2 n2
(A) (B) (C) (D)
 n  1  n  1 n 2

1 n 2

1
k
16. Suppose an electrons is attracted towards the origin by a force where ‘k’ is a constant and ‘r’ is
r
the distance of the electrons from the origin. By applying Bohr model to this system, the radius of the
nth orbital of the electron is found to be 'rn ' and the kinetic energy of the electron to be 'Tn ' . Then
which of the following is true?
1 1
(A) Tn  , rn  n 2 (B) Tn  2 , rn  n 2
n n
1
(C) Tn independent of n,rn  n (D) Tn  , rn  n
n

17. A silver sphere (work function 4.6eV) is suspended in a vacuum chamber by an insulating thread.
Ultraviolet light of wavelength 0.2 µm strike on the sphere. The maximum electric potential of the
o
sphere will be (hc = 12400eV A )
(A) 4.6V (B) 6.2V (C) 1.6V (D) 3.2V

CENTERS: MUMBAI /DELHI /AKOLA /LUCKNOW /NASHIK /PUNE/NAGPUR /BOKARO /DUBAI # 252
18. A radioactive isotope is being produced at a constant rate X. Half-life of the radioactive substance is
Y. After some time the number of radioactive nuclei become constant. The value of this constant is
XY X
(A) (B) XY (C) XY log 2 (D)
ln 2 Y

19. When the electron in a hydrogen atom jumps from the second orbit to the first orbit, the wavelength
of the radiation emitted is  . When the electron jumps from the third to the first orbit, the
wavelength of the radiation emitted as
9 4 27 32
(A)  (B)  (C)  (D) 
4 9 32 27
20. The logic circuit shown in figure has the input waveforms ‘A’ and ‘B’ as shown, pick out the correct
output wave from figure.

A
Y
B

Input A

Input B

(A) (B)

(C) (D)

21. A particle starts from rest with uniform acceleration a. Its velocity after n seconds is v. The
displacement of the body in the last two seconds is
2v  n  1 2v  n  1 v  n 1 v  n  1
(A) (B) (C) (D)
n n n n

22. If a beam consisting of ,  and  radiations is passed through an electric field perpendicular to the
beam, the deflections suffered by the components, in decreasing order, are
(a) , ,  (b) , ,  (c) , ,  (d) , , 

23. A sample of radioactive material has mass m , decay constant  , and molecular weight M. Avogadro
constant  NA . The initial activity of the sample is
m mN A
(a)  m (b) (c) (d) mN A e 
M M

CENTERS: MUMBAI /DELHI /AKOLA /LUCKNOW /NASHIK /PUNE/NAGPUR /BOKARO /DUBAI # 253
24. In the previous question, the activity of the sample after time t will be
 mN A  t  mN A   t  mN A  t m
(a) 
 M 
e (b) 
 M 
e (c) 
 M 
e (d)


1  e t 

25. The activity of a sample of radioactive material is A1 at time t1 and A 2 at time t 2  t 2  t1  . Its mean
life is T.
A  A2
(a) A1t1  A2 t 2 (b) 1  constant
t 2  t1
(c) A 2  A1e  t1  t 2 T
Al (d) A 2  A1e t1 Tt 2 

26. 90% of the active nuclei present in a radioactive sample are found to remain undecayed after 1 day.
The percentage of undecayed nuclei left after two days will be
(a) 85% (b) 81% (c) 80% (d) 79%

27. A radioactive nuclide can decay simultaneously by two different processes which have decay
constants 1 and 2 . The effective decay constant of the nuclide is  .
1 1 1 1
(a)   1   2 (b)    1   2  (c)   (d)   1 2
2  1  2
28. A sample of radioactive material is used to provide desired doses of radiation for medical purposes.
The total time for which the sample can be used will depend
(a) only on the number of times radiation is drawn from it
(b) only on the intensity of doses drawn from it
(c) on both (a) and (b)
(d) neither on (a) nor on (b)

29. An orbital electron in the ground state of hydrogen has an angular momentum L1 , and an orbital
electron in the first orbit in the ground state of lithium has an angular momentum L2 .
(a) L1  L2 (b) L1  3L2 (c) L2  3L1 (d) L2  9L1

30. When white light (violet to red) is passed through hydrogen gas at room temperature, absorption
lines will be observed in the
(a) Lyman series (b) Balmer series (c) both (a) and (b) (d) neither (a) nor (b)
31. The minimum wavelength of X- ray that can be produced in a Coolidge tube depends on
(a) the metal used as the target
(b) the intensity of the electron beam striking the target
(c) the current flowing through the filament
(d) the potential difference between the cathode and the anode.

32. An electron is in an excited state in a hydrogen-like atom. It has a total energy of 3.4eV . The
kinetic energy of the electron is E and its de Broglie wavelength is  .
(a) E  6.8eV,  ~ 6.6 1010 m (b) E  3.4eV,  ~ 6.6 1010 m
(c) E  3.4eV,  ~ 6.6 1011 m (d) E  6.8eV,  ~ 6.6 1011 m

33. In n  p  n transistor circuit, the collector current is 10 mA. If 90% of the electrons emitted reach
the collector
(a) the emitter current will be 9 mA (b) the emitter current will be 11 mA
(c) the emitter current will be 1 mA (d) the emitter current will be 0.1 mA
CENTERS: MUMBAI /DELHI /AKOLA /LUCKNOW /NASHIK /PUNE/NAGPUR /BOKARO /DUBAI # 254
34. When n  p  n transistor is used as an amplifier then
(a) electrons move from base to collector (b) electrons move from emitter to base
(c) electrons move from base to emitter (d) holes move from base to emitter

35. In a common base amplifier circuit, calculate the change in base current if that in the collector
current is 2mA and   0.98
(a) 0.04 mA (b) 1.96 mA (c) 980mA (d) 2 mA
36. In a transistor amplifier,   62, R L  5000  and internal resistance of the transistor is 500 . The
ratio of power amplification to voltage amplification will be
(a) 60 (b) 61 (c) 62 (d) 100

37. In the figure shown, the potential difference between A and B is

10K
A

30 V 10K 10K

(a) 0 (b) 5 V (c) 10 V (d) 15 V

38. The circuit shown in figure contains two diodes each with a forward resistance of 50 and with
infinite backward resistance. If the battery is 6V, the current through the 100 resistance (in
ampere)
50 150

50

6V 100

(a) zero (b) 0.02 (c) 0.03 (d) 0.36

39. The current voltage characteristics of a p  n junction diode is represented by the graph
Y Y Y Y

I I I I

O X O X O X O X
V V V V
(a) (b) (c) (d)

CENTERS: MUMBAI /DELHI /AKOLA /LUCKNOW /NASHIK /PUNE/NAGPUR /BOKARO /DUBAI # 255
40. In the circuit shown in figure, transistor used has a current gain   100 , load resistor R c  1k bias
resistor R B  200 k , voltage applied Vcc  10V . If VCE  5V and VBE is negligible, then what are
the values of collector current I c and base current IB ?

RC
RB

VCC  10 V

VCE  5V

(a) IC  5mA, IB  500 mA (b) IC  5mA ,I B  50 A


(c) IC  2mA,I B  200mA (d) IC  2mA,I B  20A

41. In a photoelectric experiment, two metal plates A and B are used for a given light intensity I and
frequency f. Work function of metal B is more than that of A. The correct variation of stopping
potential difference versus frequency is given by :
V A B
V B A

O f O f
(a) (b)
V V B
A
B
A

O f O f
(c) (d)
215 215
42. The half life of At At is 100 s . The time taken for the radioactivity of a sample of At to decay
1
to th of its initial value is
16
(a) 400 s (b) 6.3 s (c) 40 s (d) 300 s

43. The activity of a sample of radioactive material is R 1 at time t1 and R 2 at time t 2  t 2  t1  . If mean
life of the radioactive sample is T, then
R  R2
(a) R 1t1  R 2 t 2 (b) 1  constant
t 2  t1
t t   t 
(c) R 2  R1 exp  1 2  (d) R 2  R 1 exp  1 
 T   Tt 2 
44. An electron in the ground state of hydrogen has an angular momentum L1 , and an electron in the
first excited state of lithium has an angular momentum L 2 .
(a) L1  L2 (b) L1  4L 2 (c) L 2  2L1 (d) L1  2L 2
CENTERS: MUMBAI /DELHI /AKOLA /LUCKNOW /NASHIK /PUNE/NAGPUR /BOKARO /DUBAI # 256
45. The radioactivity of a substance is measured in terms of disintegration per second. 3  1010
disintegration per second is
(a) 1 eV (b) 1 MeV (c) 1 rutherford (d) 1 curie

46. A radioactive nuclide can decay simultaneously by two different processes which have individual
decay constant 1 and 2 , respectively. The effective decay constant of the nuclide is  given by
1 1 1 1
(a)   12 (b)   (c)    1  2  (d)   1   2
 1  2 2
47. A star initially has 10 40 deuterons. It produces energy via the processes 1 H 2  1H 2  1H3  p and
1H2  1H3  2 He4  n . If the average power radiated by the star is 1016 W, the deuteron supply of
the star is exhausted in a time of the order of (mass of H 2  2.0140 , m  p   1.007 u , m  G   1.008 ,

 
u, m He4  4.001u 
6
(a) 10 s (b) 108 s (c) 1012 s (d) 1016 s
48. The radius R of a nucleus changes with the nucleon number A of nucleus as
2 1
(a) R  A 3 (b) R  A 3 (c) R  A 0 (d) R  A

236
49. Consider the fission reaction 92 U  X117  Y117  n  n i.e. two nuclei of same mass number 117
are found plus two neutrons. The binding energy per nucleon of X and Y is 8.5 MeV whereas of
U 236 is 7.6 MeV. The total energy liberated is
(a) 2000 MeV (b) 200 MeV (c) 20 MeV (d) 2 MeV

50. Energy levels A, B, C of a certain atom correspond to increasing values of energy i.e., E A  E B  E C
If 1 ,  2 ,  3 are the wavelengths of radiations corresponding to the transitions C to B, B to A and C
to A respectively, which of the following statements is correct?
C
1
B

A
2 3
1 2
(a)  3  1   2 (b)  3  (c) 1   2   3  0 (d)  32  12   22
1   2
51. The spin angular momentum of an electron is equal to
h h h 2h
(a) (b) (c) (d)
 2 2 2

52. The half life of the radioactive substance is 40 days. The substance will disintegrate completely in
(a) 40 days (b) 400 days (c) 4000 days (d) infinite time

53. Which of the following is best nuclear fuel


(a) Theorium 236 (b) Plutonium 239 (c) Uranium 236 (d) Neptanium 239

54. A radioactive nuclide emits an ,  and a  ray in close succession, the atomic mass of the end
product is reduced by
(a) 8 a.m.u. (b) about 4 a.m.u. (c) 2 a.m.u. (d) 1 a.m.u.
CENTERS: MUMBAI /DELHI /AKOLA /LUCKNOW /NASHIK /PUNE/NAGPUR /BOKARO /DUBAI # 257
55. The first nuclear transmutation (artificial disintegration) was achieved by
(a) J.J. Thomson (b) Chadwick (c) Bequerel (d) Rutherforda

56. If u denotes 1 atomic mass unit. One atom of an element has mass exactly equal to Au, where A is
mass number of element.
(a) A = 1 (b) A = 12
(c) A = 16 (d) A can take up any integral value from 1 to 110

57. The shortest wavelength in the Lyman series is 912 A . Then the longest wavelength in this series is
  o 
(a) 1216 A (b) 1824 A (c) 2434 A (d) 3648 A

58. The frequency of X-rays is in the order of


(a) 300 Hz (b) 3  108 Hz (c) 3 1010 Hz (d) 3 1018 Hz

59. The half life of radium is 1600 years. The fraction of the sample of radium that would remain after
6400 years is
1 1 1 1
(a) (b) (c) (d)
2 4 8 16

60. A hydrogen atom and a Li++ ion are both in the second excited state. If L H and L Li are their
respective electronic angular momenta and E H and ELi their respective energies then,
(a) L H  L li and E H  E Li (b) L H  L li and E H  E Li
(c) L H  L li and E H  E Li (d) L H  L Li and E H  E Li

61. Consider the following reaction


2 2 4
1H  1H  2 He  Q

If m  1 H 2   2.0141u; m  2 He 4   4.0024u
The energy Q released (in MeV) in this fusion reaction is
(a) 12 (b) 6 (c) 24 (d) 48

62. A radioactive nuclide is produced at the constant rate of n per second (say, by bombarding a target
with neutrons). The expected number N of nuclei in existence t seconds after the number is N 0 is
given by
n
(a) N  N 0 e t (b) N   N 0e t

n  n  t n  n
(c) N    N 0   e (d) N    N 0   e t
     
a

(where is the decay constant of the sample)


O
x

63. The maximum kinetic energy  E max  of photoelectrons emitted in a photoelectric cell varies with
frequency (v) as shown in the graph. The slope of the graph is equal to
e
(a) charge of the electron (b) of the electron
m
(c) work function of the emitter (d) Planck’s constant

CENTERS: MUMBAI /DELHI /AKOLA /LUCKNOW /NASHIK /PUNE/NAGPUR /BOKARO /DUBAI # 258
64. An electron at rest is accelerated through a potential difference of 200 V. If the specific charge of the
electron is 1.76 1011 Ckg 1 , the velocity acquired by the electron is
(a) 8.4 105 ms 1 (b) 8.4 106 ms 1 (c) 4.2 105 ms 1 (d) 4.2  106 ms 1

65. In a photoemissive cell with exciting wavelength  , the fastest electron has a speed v. If the exciting
3
wavelength is changed to , the speed of the fastest emitted electrons will be
4
3 4 4 4
(a) v (b) v (c) less than v (d) greater than v
4 3 3 3

66. A moving particle is associated with a wave packet or group of waves. The group velocity is equal to
(a) velocity of light (b) velocity of sound
1
(c) velocity of particle (d)
particle velocity
67. A particle of mass 10 31 kg is moving with a speed of 105 ms 1 . The de Broglie wavelength of the
particle is
o o
(a) 6.63 108 m (b) 6.63 A (c) 66.3 A (d) 6.63  10 7 m

68. A neutron is confined to a nucleus of size 10 14 m. The minimum momentum of the electron may be
(a) 6.6  1020 kgms 1 (b) 3.3 1020 kgms 1
(c) 3.3 1048 kgms 1 (d) 6.6  1048 kgms 1

69. An electron of mass m, when accelerated through a potential difference V has de-Broglie wavelength
 . The de-Broglie wavelength associated with a proton of mass M accelerated through the same
potential difference will be
m m M M
(a)    (b)  (c)    (d) 
M M m m

70. Cathode rays are made to pass between the poles of a magnet as
shown. The effect of the magnetic field is N
(a) to increase the velocity of rays.
o
(b) to deflect them towards the north pole.
(c) to deflect them towards the south pole. S
(d) to deflect them out of the plane of the paper.

71. In an electron microscope if the potential is increased from 20 kV to 80 kV, the resolving power R of
the microscope will become
R
(a) R (b) 2R (c) 4R (d)
2
o o
72. A photon of wavelength 1000 A has energy 12.3 eV. If light of wavelength 5000 A , having
intensity I, falls on a metal surface, the saturation current is 0.40 A and the stopping potential is
1.36 V. The work function of the metal is
(a) 2.47 eV (b) 1.36 eV (c) 1.10 eV (d) 0.43 eV

73. In PROBLEM 11, If the intensity of light is made 4I, the stopping potential will become
(a) 1.36 V (b) 2.72 V (c) 5.44 V (d) 21.76 V
CENTERS: MUMBAI /DELHI /AKOLA /LUCKNOW /NASHIK /PUNE/NAGPUR /BOKARO /DUBAI # 259
74. In PROBLEM 11, If the intensity of light is made 4I, the saturation current will become
(a) 0.4 A (b) 0.8 A (c) 1.6 A (d) 6.4 A

75. The momentum of a photon of frequency v is


hv hv
(a) (b) (c) hvc (d) hvc 2
c2 c

76. Which of the following graphs represents the variation of particle momentum and the associated de
Broglie wavelength?
(a) (b)
p
p

 

(c) (d)
p

 

77. An electron is accelerated through a potential difference of 100 V. Its kinetic energy will be
(a) 100 J (b) 100 erg (c) 100 eV (d) 100 MeV

78. When electrons are accelerated through potential difference of V volt, the de-Broglie wavelength
associated is given by
150 o 150 150 o 150 o
(a)   A (b)   m (c)   A (d)   A
V V V V

79. A photoelectric cell is illuminated by a small bright source of light is placed at 1 m. If the same
source of light is placed 2 m away, the electrons emitted by the cathode
(a) each carries one quarter of its previous momentum.
(b) each carries one quarter of its previous energy.
(c) are half the previous number
(d) are one quarter of the previous number.

80. According to Bohr’s theory, the electron in orbits have definite energy values, then according to
uncertainty principle, the life time of an excited state will be
(a) ZERO (b) finite (c) 10 8 s (d) infinite

81. Which of the following arrangements corresponds to decreasing order of specific charge?
(a) Electron, proton,  -particle (b) Proton,  -particle, electron
(c)  -particle, electron, proton (d) Electron,  -particle, proton

CENTERS: MUMBAI /DELHI /AKOLA /LUCKNOW /NASHIK /PUNE/NAGPUR /BOKARO /DUBAI # 260
82. The energy of a photon corresponding to the visible light of maximum wavelength is approximately
(a) 1 eV (b) 1.6 eV (c) 3.2 eV (d) 7 eV

83. Which one of the following graphs represents correctly the variation of photoelectric current (i) with
intensity (I) of incident radiations

current(i)
current(i)

(a) (b)

Intensity(I)
Intensity(I)

(c) (d)

current(i)
current(i)

Intensity(I) Intensity(I)

84. A material particle with a rest mass m 0 is moving with speed of light c. The de-Broglie wavelength
associated is given by
h m0 c
(a) (b) (c) ZERO (d) 
m 0c h

85. When X-rays are scattered by electrons their wavelength increases. This phenomenon is called
(a) Compton effect (b) Thomson effect (c) Hall effect (d) None of the above

86. An electron, accelerated by a potential difference V, has de Broglie wavelength  . If the electron is
accelerated by a potential difference 4V, its de Broglie wavelength will be
 
(a) 2 (b) 4 (c) (d)
2 4
87. The energy of a photon of wavelength  is
hc  h
(a) hc (b) (c) (d)
 hc c

88. Light of two different frequencies whose photons have energies 1 eV and 2.5 eV successively
illuminate a metal of work function 0.5 eV. The ratio of the maximum speeds of the emitted
electrons will be
(a) 1 : 5 (b) 1 : 4 (c) 1 : 2 (d) 1 : 1

89. A photon of frequency v is incident on a metal surface whose threshold frequency is 0 . The
maximum kinetic energy of the emitted electron will be
1 1
(a) h    0  (b) h    0  (c) h   0  (d) h   0 
2 2

CENTERS: MUMBAI /DELHI /AKOLA /LUCKNOW /NASHIK /PUNE/NAGPUR /BOKARO /DUBAI # 261
90. A beam of  -particles moving with velocity 105 ms 1 enters a region of electric and magnetic fields.
  
All the three vectors v , E and B are mutually perpendicular and the strength of electric field is
2 104 Vm1 . If the beam passes undeflected, then the strength of magnetic field must be
(a) 2 T (b) 0.2 T (c) 5 T (d) 2  108 T

91. The output from a logic gate is 1 when inputs A, B and C are such that
A
B Y
C

(a) A = 1, B = 0, C = 1 (b) A = 1, B = 1, C = 0
(c) A = B = C = 0 (d) A = B = C = 1

92. The symbol represents

A
Y
B
(a) NOT gate (b) OR gate (c) AND gate (d) NOR gate

93. NAND and NOR gates are called universal gates primarily because they
(a) are available universally
(b) can be combined to produce OR, AND and NOT gates
(c) are widely used in Integrated circuit packages
(d) are easiest to manufacture

94. A crystal that is transparent to light and possesses low melting point has
(a) ionic bonding (b) metallic bonding (c) covalent bonding (d) vander Waal’s bonding

95. An n-type semiconductor has resistivity 0.1 m . The number of donor atoms which must be added
to achieve this  e  0.05m 2 V 1s 1 
(a) 1.25  1017 (b) 1.25  10 23 (c) 1.25  1021 (d) 1.25  10 22

96. When pure germanium is doped with trivalent impurity like aluminium; the conduction is due to
(a) electrons (b) holes (c) protons (d) positrons

97. Assuming that the silicondiode having resistance of 20  , the current through the diode is (knee
voltage 0.7 V)
R  180 

2V 0V

(a) 0 mA (b) 10 mA (c) 6.5 mA (d) 13.5 mA

CENTERS: MUMBAI /DELHI /AKOLA /LUCKNOW /NASHIK /PUNE/NAGPUR /BOKARO /DUBAI # 262
98. The following configuration of gate is equivalent to
A OR
B
Y
AND
NAND

(a) NAND (b) XOR (c) OR (d) NOR

99. For circuit shown in the figure I E  4mA, I B  40 A .


What are the values of  and IC ?

E C

(a) 0.99, 3.96 mA (b) 1.01, 4.04 mA (c) 0.97, 4.04 mA (d) 0.99, 4.04 mA

100. Assuming that the junction diode is ideal, the current in the arrangement shown in the figure is
R  100 

1V 2V

(a) 0 mA (b) 2 mA (c) 10 mA (d) 30 mA

101. In case of forward biasing of pn junction diode, which one of the following figures correctly depicts
the direction of flow of current
+
+ p + n
p + n
(a) (b) +
+

+
+

V1 V1

+ +
p p + n
+ n
(c) +
(d) +

+ +

V1 V1

CENTERS: MUMBAI /DELHI /AKOLA /LUCKNOW /NASHIK /PUNE/NAGPUR /BOKARO /DUBAI # 263
MULTIPLE CHOICE QUESTIONS
1. H+, He+ and O++ all having the same kinetic energy pass through a region in which there is a uniform
magnetic field perpendicular to their velocity. The masses of H+, He+ and O++ are 1 amu, 4 amu and
16 amu respectively, then:
(a) H+ will be deflected most (b) O++ will be deflected most
(c) He+ and O++ will be deflected equally (d) All will be deflected equally

2. If En and Jn are the magnitude of total energy and angular momentum of electron in the nth Bohr
orbit respectively, then
1 1
(a) En  J2n (b) En  2 (c) Jn  n (d) Jn 
Jn n
3. Figure shows velocity (v) of a particle plotted against time (t). Choose the correct statement(s)
v

v0

t
T 2T
v0

(A) The particle changes its direction of motion at some point


(B) Acceleration of particle remains constant
(C) Displacement of particle is zero (for time interval t = 0 to t = 2T)
(D) Acceleration is always in direction of velocity

4. In arrangement shown, S is a spring balance hanging from ceiling of a lift. Pulley is light and
 
frictionless. Thread is light and inextensible. g  10 m s 2 Choose correct statement (s)

5 kg

3 kg

(A) When lift is moving up or down with uniform velocity, reading of balance is 80 N
2
(B) When lift moves down with uniform retardation of 4 m s , reading is 105 N
2
(C) When lift moves up with uniform acceleration of 4 m s , reading is 120 N
2
(D) Lift moves down with uniform acceleration of 4 m s , reading is 45 N

CENTERS: MUMBAI /DELHI /AKOLA /LUCKNOW /NASHIK /PUNE/NAGPUR /BOKARO /DUBAI # 264
5. A particle of mass 5 kg, moving in the x-y plane has its potential energy given by U  7x  24y
joules, where x and y are in metre. The particle is initially at the origin and has a velocity

u  14.4i  4.2j  m s .
(A) The particle has a speed of 25 m/s at t = 4 s.
(B) The particle has an acceleration of 5 m s 2 .
(C) The acceleration of the particle is perpendicular to its initial velocity.
(D) None of the above is correct.

6. A gas expands such that its initial and final temperatures are equal. Also, the process followed by the
gas traces a straight line on the P-V diagram.
(A) The temperature of the gas remains constant throughout.
(B) The temperature of the gas first increases and then decreases.
(C) The temperature of the gas first decreases and then increases.
(D) The straight line has negative slope.

7. In which of the following cases the centre of mass of a rod is certainly not at its centre?
(A) The density continuously increases from left to right.
(B) The density continuously decreases from left to right.
(C) The density decreases from left to right up to the centre and then increases.
(D) The density increases from left to right up to the centre and then decreases.

8. A heavy particle is tied to an end of a string of length 1.6 m. Its other end is fixed. It revolves as a
conical pendulum with the string making an angle of 600 with the vertical. Then
4
(A) Its period of revolution is sec
7
(B) The tension in the string is double the weight of the particle
(C) The velocity of the particle is 2.8 3 m s .
(D) The centripetal acceleration of the particle is 9.8 3 m s 2 .

9. When a nucleus with atomic number Z and mass number A undergoes a radioactive decay process,
(a) both Z and A will decrease, if the process is  decay
(b) Z will decrease but A will not changes, if the process is  decay
(c) Z will increase but A will not change, if the process is  decay
(d) Z and A will remain unchanged, if the process is  decay

10. When the nucleus of an electrically neutral atom undergoes a radioactive decay process, it will
remain neutral after the decay if the process is
(a) an  decay (b) a   decay (c) a  decay (d) a K- capture process

11. Which of the following assertions are correct?


(a) A neutron can decay to a proton only inside a nucleus.
(b) A proton can change to a neutron only inside a nucleus.
(c) An isolated neutron can change into a proton.
(d) An isolated proton can change into a neutron.
CENTERS: MUMBAI /DELHI /AKOLA /LUCKNOW /NASHIK /PUNE/NAGPUR /BOKARO /DUBAI # 265
12. Two identical nuclei A and B of the same radioactive element undergo  decay. A emits a  
particle and changes to A’. B emits a   particle and then a   ray photon immediately afterwards,
and changes to B’.
(a) A’ and B’ have the same atomic number and mass number.
(b) A’ and B’ have the same atomic number but different mass numbers
(c) A’ and B’ have different atomic numbers but the same mass number.
(d) A’ and B’ are isotopes

13. A nuclei A undergoes  decay and another nuclide B undergoes  decay


(a) All the   particles emitted by A will have almost the same speed.
(b) The  - particles emitted by A may have widely different speeds
(c) All the   particles emitted by B will have almost the same speed
(d) The   particles emitted by B may have widely different speeds.

14. In the Bohr model of the hydrogen atom, let R,V and E represent the radius of the orbit, speed of the
electron and the total energy of the electron respectively. Which of the following quantities are
proportional to the quantum number n ?
V R
(a) VR (b) RE (c) (d)
E E
15. An electron in a hydrogen atom makes a transition from n  n1 to n  n 2 . The time period of the
electron in the initial state is eight times that in the final state. The possible values of n1 and n2 are
(a) n1  4, n 2  2 (b) n1  8, n 2  2 (c) n1  8,n 2  1 (d) n1  6, n 2  3

16. A beam of ultraviolet light of all wavelengths passes through hydrogen gas at room temperature, in
the x  direction. Assume that all photons emitted due to electron transitions inside the gas emerge in
the y  direction. Let A and B denote the lights emerging from the gas in the x  and y  directions
respectively.
(a) Some of the incident wavelengths will be absent in A.
(b) Only those wavelengths will be present in B which are absent in A.
(c) B will contain some visible light.
(d) B will contain some infrared light.

17. Whenever a hydrogen atom emits a photon in the Balmer series,


(a) it may emit another photon in the Balmer series
(b) it must emit another photon in the Lyman series
(c) the second photon, if emitted, will have a wavelength of about 122 nm.
(d) it may emit a second photon, but the wavelength of this photon cannot be predicted.

18. When an electron moving at a high speed strikes a metal surface, which of the following are
possible?
(a) The entire energy of the electron may be converted into an X- ray photon.
(b) Any fraction of the energy of the electron may be converted into an X- ray photon
(c) The entire energy of the electron may get converted to heat.
(d) The electron may undergo elastic collision with the metal surface.
CENTERS: MUMBAI /DELHI /AKOLA /LUCKNOW /NASHIK /PUNE/NAGPUR /BOKARO /DUBAI # 266
19. A metallic surface ejects electrons when exposed to green light of intensity I but no photoelectrons
are emitted when exposed to yellow light of intensity I. It is possible to eject electrons from the same
surface by
(a) yellow light of some intensity which is more than I.
(b) green light of any intensity.
(c) red light of any intensity.
(d) violet light of any intensity.

INTEGER TYPE
13.6 2
1. Consider a hydrogen like atom whose energy in nth excited state is given by E n   Z . When
n2
this excited atom makes a transition from an excited state to ground state, the most energetic photons
have energy E max = 52.224 eV and the least energetic photons have energy E min = 1.224 eV. The
atomic number of atom is

1
2. A block of mass m = 4 kg is projected with a speed u  m s on a long platform of mass M = 17 kg
2
as shown in the figure. Due to friction between the block and the platform, the block slows down and
after certain a moment comes to rest w.r.t the platform. Find the average power (in watts) developed
by the force of friction during the entire process? g  10 m s 2 

1 m u

2 M

smooth
21
3. All the surfaces shown in the figure are smooth. The block of mass m  kg is projected with a
20
speed u  3gR , where R = 20 cm is the radius of the circular track. Find the normal reaction (in
newton) between the block and the track at the instant when the block is at point P? g  10 m s 2 

R
P
m u
M  2m

4. A uniform rod of length l and mass m  4kg rests on supports at its l


ends. The right support is quickly removed (see figure). The force
 in N  on the left support immediately thereafter is 10 k. The value of mg
k is

5. Seven coins are arranged in a hexagonal, planar pattern so as to touch each other, as
shown in the figure below. Each coin is a uniform disk of mass m  2 kg and radius

 
r  1m . The moment of inertia of the system kg  m 2 of seven coins about an axis
that passes through the center of the central coin and is normal to the plane of the coins is 11 . Find
the value of  .
CENTERS: MUMBAI /DELHI /AKOLA /LUCKNOW /NASHIK /PUNE/NAGPUR /BOKARO /DUBAI # 267
SUBJECTIVE TYPE

1. Assuming that the solar energy is due to the thermonuclear reaction


41 H1  2 He 4  2 1 e   1 e     photon  .
Calculate the rate at which hydrogen is consumed in the sun. Given that energy reaching the earth is
1400 W m2 on surface normal to suns rays. The earth-to-sun distance is 1.5 108 km . The rest mass
of 1 H ,2 He 4 and that of positron or electron are 1.0078, 4.0026 and 0.0005 amu respectively.

2. A charged droplet of mass m  6.4 104 kg is in horizontally arranged parallel-plate capacitor with
plate separation d  10mm . In the absence of voltage between the plates, the droplet falls at a
constant speed v1  0.78mm sec . After a voltage V  95V is applied across the capacitor, the droplet
moves uniformly upwards at a speed of v2  0.016mm sec .Determine the charge q on the droplet.

3. Find the number of waves made by a Bohr electron in one complete revolution in its 3rd orbit of
hydrogen atom. Given: mass of electron  9.108 1031 kg . Planck’s constant  6.62  1034 Joules/
sec. Speed of electron in first Bohr orbit  2.19 108 cm sec

4. A Bi 210 radio nuclide decays according to chain


Bi 210 
1
 P0 210 
2
 Pb 206  stable 
Decay constant 1  1.6  10 6 S1 and  2  5.8  108 sec 1 Calculate the  and  activities of 1mg
of Bi 210 one month after its preparation

5. An electron with angular momentum L  6 1035 N  S and total energy  4 1018 J is in orbit
around a proton. What are the distances of nearest approach and farthest separation.

6. The atomic mass of 8 O16 is 16 amu. Mass of electron  0.00055amu , mass of proton 1.007593 =
amu and mass of neutron  1.008982amu . Calculate the binding energy per nucleon for 8 O16 .

7. The source of energy within the sun is the proton-proton cycle


1 H 1 H 1 H 1 H 4 He  Q
The surface temperature of the sun is 5800 K and its radius is 700000 km . Find the rate at which
hydrogen is consumed in the core of the sun, assuming that all the energy from the sun arises from
the above cycle, Stefan’s constant ,   5.67  10 8 SI units

 
m 1 H  1.007825amu

m  4 He   4.002603amu

8. The kinetic energy of an   particle which flies out of the nucleus of radium atom 88 Ra 226 in
radioactive disintegration in 4.78 MeV
(i) Find the velocity of the   particle
(ii) The total energy evolved during the escape of the   particle

CENTERS: MUMBAI /DELHI /AKOLA /LUCKNOW /NASHIK /PUNE/NAGPUR /BOKARO /DUBAI # 268
9. Given two radioactive elements, a parent having a half life T1 and a daughter product having a half
life T2 . If T1   T2 and if initially there is only the parent element present, how long will it take in
terms of T2 until these elements are within one percent of their equilibrium value.

10. A radio nuclide with half life T  69.31 seconds emits a  particle average kinetic energy
E  11.25 eV . At an instant concentration of  particle at r  2m from the nuclide is
  3  1013 m3
(i) Calculate the number of nuclei in the nuclide at that instant
(ii) If a small circular plate is placed at distance r from nuclide such that   particle strike the plate
normally and come to rest. Calculate pressure experienced by the plate due to collision of  


particle . mass of   9  10 3 kg 
11. A radio nuclide consists of two isotopes. One of the isotopes decays by   emission and other by
  emission with half lives T1  405sec, T2  1620 sec, respectively. At , t  0 probabilities of
getting ,  from the radio nuclide are equal. Calculate their respective probabilities at t  1620 sec .
if at t  0 , total no nuclei in the radio nuclide are N 0 . Calculate the time ‘t’ when total no. nuclei
remained undecayed becomes equal to N 0 2
Log 2  0.3010, log 5.94  0.7742
10 10
And x 4  4x  2.5  0, x  0.594

12. (a) An isotropic point source of radiation power P is placed on the axis of mirror( ideal). The
distance between the source and the mirror is 10 times the radius if mirror. Find the net force that
light exerts on the mirror.
(b) A laser emits a light pulse of duration T  0.10 ms and energy 10 J . Find the mean pressure
exerted by such a pulse when its is focused on a spot of diameter d  10 m on a surface
perpendicular to the beam and with reflection coefficients   0.5 .

13. A small ideally absorbing plate of mass 10 mg is suspended a practically weightless quartz fibre 2cm
long. A light flash from a laser falls on its surface perpendicular to it. Causing the fibre with the plate
to deflect from the vertical by an angle of 0.6 . Calculate the energy of laser flash.

COMPREHENSION TYPE

PASSAGE - I
A single electron orbits arounds a stationary nucleus of charge +Ze, where Z is a constant and e is the
magnitude of the electronic charge. It requires 47.2 eV to excite the electron from the second Bohr
orbit to the third Bohr orbit.
(The ionization energy of hydrogen atom = 13.6 eV. Bohr radius = 5.3 1011 m , velocity of light =
3 108 ms1 . Planck’s constant = 6.6 10 34 J  s ). Based on the above facts, answer the following
questions.
1. The value of Z is
(a) 1 (b) 2 (c) 3 (d) 5

2. The energy required to excite the electron from the second excited state to the third excited state is
(a) 47.2 eV (b) 14.53 eV (c) 16.53 eV (d) 18.53 eV
CENTERS: MUMBAI /DELHI /AKOLA /LUCKNOW /NASHIK /PUNE/NAGPUR /BOKARO /DUBAI # 269
3. The energy required to excite the electron from the first excited state to the second excited state is
(a) 47.2 eV (b) 16.53 eV (c) 255 eV (d) None of these is correct

4. The minimum wavelength of the electromagnetic radiation required to deport the electron first orbit
to an upper orbit is
o o o o
(a) 48.5 A (b) 36.4 A (c) 45.8 A (d) 34.6 A

5. The kinetic energy, potential energy, total energy and the angular momentum of the electron in the
first orbit have respective values given by
(a) 340 eV, 340 eV, 680 eV,1.05 1034 kgm 2s 1
(b) 340eV, 680 eV, 340 eV,1.05 10 34 kgm 2s 1
(c) 680 eV, 340 eV, 680 eV, 2.05 1034 kgm 2s 1
(d) 680 eV, 1360eV, 680 eV, 2.05 1034 kgm2s 1

6. The radius of the first Bohr’s orbit is


o o o o
(a) 0.53 A (b) 0.106 A (c) 5.3 A (d) 1.06 A

PASSAGE – II
Light from a discharge tube containing hydrogen atoms falls on the surface of a piece of sodium. The
kinetic energy of the fastest photoelectrons emitted from sodium is 0.73 eV. The work function for
sodium is 1.82 eV. (Ionization potential of hydrogen in 13.6 eV). Based on the above facts, answer
the following questions.

1. The energy of the photons causing photoelectric emission is


(a) 1.09 eV (b) 13.6 eV (c) 2.55 eV (d) None of these

2. The quantum numbers of the two levels involved in the emission of of these photons are
(a) from n = 2 to n = 4 (b) from n = 4 to n = 2
(c) from n = 1 to n =4 (d) from n = 4 to n = 1

3. The change in angular momentum of the electron in the hydrogen atom for the transition calculated
above is
3h 3h h h
(a) (b)  (c) (d) 
2 2  

4. The recoil speed of the emitting atom, assuming it to be at rest before the transition is
(a) ZERO (b) 0.814 ms 1 (c) 0.418 ms 1 (d) None of these

PASSAGE – III
The resistivity of pure silicon is 3000 ohm m and the mobilities of the electrons and holes are 0.12
m 2 V 1s 1 and 0.025 m2 V1s1 respectively. Now 1019 atoms of phosphorus are added per m 3 of the
specimen and the resistivity becomes 1 . Further 2  1019 atoms of boron are also added per m 3 and
the resistivity of the specimen becomes 2 . Based on these facts, answer the following questions.

1. The intrinsic concentration of charge carries in the sample was


(a) 1.44 1016 cm 3 (b) 1.44 1010 cm 3 (c) 1.44 1014 cm 3 (d) 1.44  108 cm 3

CENTERS: MUMBAI /DELHI /AKOLA /LUCKNOW /NASHIK /PUNE/NAGPUR /BOKARO /DUBAI # 270
2. The value of 1 is
(a) 2.51 ohm m (b) 52.1 ohm m (c) 5.21 ohm m (d) None of these

3. The value of 2 is
(a) 5 ohm m (b) 10 ohm m (c) 15 ohm m (d) 25 ohm m

PASSAGE – IV
In the circuit shown, Si and Ge diodes are used. The Silicon diode conducts at 0.7 V and the
Germanium diode conducts at 0.3 V.
Ge

I
V0

12 V Si 5k

Based on the above facts and diagram provided, answer the following questions
1. The value of V0 is
(a) 11.3 V (b) 11.7 V (c) 12 V (d) None of these

2. The value of I is
(a) 2.26 mA (b) 22.6 mA (c) 2.34 mA (d) 23.4 mA

3. Now, if the germanium diode connection is reversed, then V0 and I change V0' and I ' . Then
(a) V0'  11.7 V (b) V0'  11.3 V (c) I '  2.36 A (d) I '  2.26 mA

PASSAGE – V
In figure, the rms value of input 20 V. The transformation ratio is 1:2.
D1

R L  2 k
~

D2
1. The peak value of the voltage applied to the diodes is
(a) 20 V (b) 20 2 V (c) 40 2 V (d) 80 2 V
2. The d.c. component of the rectified output voltage is
2 2 2
(a) .V (b)  20 V (c)  40  2 V (d) 7 V
  

3. The a.c. component of the rectified output is


2 2
(a) 0.48   40  2 V (b) 0.48   40 V
 
(c) 1.21 7 V (d) 1.21 V

CENTERS: MUMBAI /DELHI /AKOLA /LUCKNOW /NASHIK /PUNE/NAGPUR /BOKARO /DUBAI # 271
ANSWER KEY

SINGLE CHOICE QUESTIONS

1. (C) 2. (C) 3. (B) 4. (B) 5. (D)

6. (B) 7. (B) 8. (D) 9. (B) 10. (B)

11. (C) 12. (B) 13. (A) 14. (B) 15. (B)

16. (C) 17. (C) 18. (A) 19. (C) 20. (C)

21. (A) 22. (C) 23. (C) 24. (B) 25. (C)

26. (B) 27. (A) 28. (D) 29. (A) 30. (D)

31. (D) 32. (B) 33. (B) 34. (A) 35. (A)

36. (C) 37. (C) 38. (B) 39. (A) 40. (B)

41. (A) 42. (A) 43. (C) 44. (C) 45. (D)

46. (D) 47. (C) 48. (B) 49. (B) 50. (B)

51. (C) 52. (D) 53. (B) 54. (B) 55. (D)

56. (B) 57. (A) 58. (D) 59. (D) 60. (B)

61. (C) 62. (C) 63. (D) 64. (B) 65. (D)

66. (C) 67. (A) 68. (B) 69. (B) 70. (D)

71. (B) 72. (C) 73. (A) 74. (C) 75. (B)

76. (C) 77. (C) 78. (A) 79. (D) 80. (D)

81. (A) 82. (B) 83. (C) 84. (C) 85. (A)

86. (C) 87. (B) 88. (C) 89. (A) 90. (D)

91. (A) 92. (B) 93. (D) 94. (C) 95. (B)

96. (C) 97. (B) 98. (A) 99. (A) 100. (D)

101. (B)

CENTERS: MUMBAI /DELHI /AKOLA /LUCKNOW /NASHIK /PUNE/NAGPUR /BOKARO /DUBAI # 272
MULTIPLE CHOICE QUESTIONS

1. (AC) 2. (BC) 3. (ABC) 4. (BD) 5. (ABC)

6. (BD) 7. (AB) 8. (ABCD) 9. (ABCD) 10. (CD)

11. (BC) 12. (A) 13. (AD) 14. (AC) 15. (AD)

16. (ACD) 17. (BC) 18. (ABC) 19. (BD)

INTEGER TYPE

1. (B) 2. (5) 3. (0) 4. (A) 5. (5)

SUBJECTIVE

1. 5.76 10 11
kg sec  2.  7.9 10 19
C 3. (3)

4. 1.46 10 11
sec 5.  4.7110 11
m 6. (7.97 MeV)

7. 6.2 1011 kg s 8. 4.87 MeV 9. 6.64 T2

10. (i) 9.6  10 22 ; (ii) 1.08  10 4 N m 2 11. (1215)

P
12. (a) N (b) 6.37  106 N m 2 13. (13.9 J)
202C

COMPREHENSION

1. (d) 2. (c) 3. (a) 4. (b) 5. (b)

6. (b) 7. (c) 8. (b) 9. (d) 10. (b)

11. (a) 12. (c) 13. (d) 14. (b) 15. (c)

16. (b, d) 17. (b) 18. (c) 19. (a)

CENTERS: MUMBAI /DELHI /AKOLA /LUCKNOW /NASHIK /PUNE/NAGPUR /BOKARO /DUBAI # 273
SOLUTION

SINGEL CHOICE QUESTIONS


1. Al(3s 2 3p 1 ), Si(3s 2 3p 2 ), S(3s2 3p 4 ) all have unpaired electron spin and therefore they are
paramagnetic whereas Mg(3s 2 ) and Ar(3s 2 3p 6 ) have no unpaired electron, hence are
diamagnetic.

h
3.  ...(for electron)
P
h
or P

hc
and E ...(for photon)

P 1
hence 
E c
1

3 108 m / s
= 3.33 × 10 –9 s/m

4. The input of NAND gate (at last) is A .A and B.B

  
So, Y  A.A . B. B  A.B  A  B

E1 E
 1
E 4n  E 2n 16n 2 4n 2
5. 
E 2n  E n E1 E1

4n 2 n 2
1

4

Which is independent of n and z

6. (b)
 hc   hc 
Speed is doubled therefore KE will become 4 times or   W   4   W 
 1   2 
7. (b)
VPB 3 3VPQ
Photon energies of the transitions are related as E1 + E2 = E3 or   VPA 
VPQ 3  5 8
8. (d)
Initial number of undecayed atoms (t = 0), N0 = 1000
Number of undecayed atoms after 2 hours (t = 2 hr), N1 = 1000-100 = 900 = (0.9)x1000
Number of undecayed atoms after next 2 hours (t = 4 hr) N2 =(0.9)2x1000
Number of undecayed atoms after next 2 hours (t = 6 hr) N3 =(0.9)3x1000 = 729

CENTERS: MUMBAI /DELHI /AKOLA /LUCKNOW /NASHIK /PUNE/NAGPUR /BOKARO /DUBAI # 274
9. (b)
P
I

3 4

B A
5
8

VPA 4 VPQ VPB 3 3VPQ


  VPA  similarly,   VPB 
VPQ 4  8 3 VPQ 3  5 8
Clearly, potential at A is more than potential at B. Therefore, D2 is reverse biased.

10. hv  hv 0  eV , where V is the stopping potential. For frequency v, we have


eV
hv  hv0  0 (i)
2
and for frequency v 2 , we have
hv
 hv0  eV0 (ii)
2
From (i) and (ii) on eliminating V0 , we get v0  3v 2 . Hence the correct choice is (b)

11. The distance of the closest approach is given by


1 2Ze 2
r0  .
40 K
1 1
Where K  m2 . Thus r0  . Where K is doubled, r0 becomes half. Hence the correct choice is
2 K
(c)

12. For transistor: I E  I B  IC


We are given that Ic  80% of I E  0.8I E . But I c  10mA
Therefore
Ic 10mA
IE    12.5mA
0.8 0.8
Also IB  I E  IC  12.5  10  2.5mA
Hence the correct choice is (b)

13. n e ee  n h eh  I . given

CENTERS: MUMBAI /DELHI /AKOLA /LUCKNOW /NASHIK /PUNE/NAGPUR /BOKARO /DUBAI # 275
n e ee 4I 5
 4
n h eh I5
n e e n  4 8
or  4 or e  4 h  
n h h nh e 2.5 5
hence the correct choice is (a)

14. In the given circuit only diode D1 will allow the current to pass through as it is forward-biased.
Hence, the current through the 100 ohm resistance is
( resistance of D1  50 )
6V 6
I  A
150  50  100   300
 0.02A
(b)

15. Total energy  KE  PE  K  U


In the region 0  x  1; U1  E , therefore, kinetic energy is
K1  total energy  U1  nE  E   n  1 E
h h
 1   (1)
2mK1 2m  n  1 E
In the region x  1; U 2  0 . Therefore, kinetic energy is
K 2  nE  0  nE
h h
 2   (2)
2mK 2 2mnE
Dividing (1) by (2), we get,
1 n

2  n  1
16. If n is the speed of the electron in the nth orbit, then
m2n k

rn rn
k
 2n 
m
1 k
 Tn  mn2  , which is independent of n.
2 2
nh
Also, angular momentum, L n 
2
nh
Or mn rn  which gives
2
nh nh m n
rn   
2mn 2 m k 2 mk
Thus rn  n . Hence the correct choice is (c)

CENTERS: MUMBAI /DELHI /AKOLA /LUCKNOW /NASHIK /PUNE/NAGPUR /BOKARO /DUBAI # 276
17. (3)
K.E max  hr  
hc
    1.6eV

Vmax of sphere after a sufficient time
 1.6 V
18. (1)
dN n2
 X  N  0 x N0
dt y
xy
N
n2

19. (3)
1  1
 R 1  2 
  2 
1  1
 RZ 2 1  2 
'  3 
27
 
32

20. from above figure, it follows that when A  0 and B  0,Y  0 , when A  0 and B  1;Y  1, when
A  1 and B  0 , Y  1 and when A  B  1 then Y  1 . This is the truth table of OR gate

21. (A)
v = an
v
 a=
n
1 1 v 2 vn
Sn = an2 = n 
2 2 n 2
1 2 1 v 2
Sn – 2 = a  n  2    n  2
2 2 n
2
vn v  n  2 
Slast2 seconds = 
2 2n
v
=  n 2  n 2  4  4n 
2n 
2v(n  1)
=
n
22. (C)
P2
K=
2m
P2 P2 P 2 (m1  m 2 )
Energy =  
2m1 2m 2 2m1m 2

23. (c)
Activity = number of disintegrations per unit time

CENTERS: MUMBAI /DELHI /AKOLA /LUCKNOW /NASHIK /PUNE/NAGPUR /BOKARO /DUBAI # 277
dN
 .N , where N  the total number of unclei
dt
m
Also, N  Number of moles N A    N A .
M

24. (b)
Activity after time t  initial activity et

25. (c)
Let A 0  initial activity
1
Then, A1  A 0 e t and A 2  A 0 e t , Also,  
T
A2 et 2
 t1  e 2 1 
t t

A1 e
or A 2  A1e t1  t 2   A1e t1  t 2  T

26. (b)
Equal fractions decay in equal times
2
if a friction 0.9 remains undecayed after 1 day, a fraction  0.9   0.81 will remain undecayed
after 2 days

27. (a)

28. (d)

29. (a)

30. (d)
At room temperature, all the atoms are in the ground state. The minimum photon energy required for
absorption is 10.2 eV. White light has photon energies less that this and hence is not absorbed.

31. (d)

32. (b)
The potential energy  2  kinetic energy  2E
total energy  2E  E  E  3.4eV
or E  3.4eV
Let p  momentum and m  mass of the electron
p2
 E or p  2mE
2m
h h
de Broglie wavelength ,    ~ 6.6  10 10 m
p 2mE
33. (b)
9
I C  10 mA and IC  IE
10

CENTERS: MUMBAI /DELHI /AKOLA /LUCKNOW /NASHIK /PUNE/NAGPUR /BOKARO /DUBAI # 278
9 100
10  IE or IE   11mA
10 9

34. (a)
In n  p  n transistor, electrons are majority carriers.

35. (a)
 0.98
   49
1   1  0.98
Now IC Ib  49 or I b  IC 49
 I b  2 mA 49  0.04 mA

36. (c)
 RL
Voltage amplification A v 
RC
62  5000
AV   620
500
2 R L
Power amplification A P 
RC
2

AP 
 62  5000
 38440
500
A 38440
 P   62
AV 620

37. (c)
It is clear form the figure that the diode is forward bias. So, it offers low resistance to the flow of
current. Here, we consider that the resistance of diode is zero. The net resistance across AB is 5K .
The current in the circuit is given by
30
I  2mA
10  5
Potential difference across AB
 5k   2mA  10V

38. (b)
Here the lower diode junction is reversed biased and hence there will be no conduction across this
diode. Of course, the above diode is forward biased and there will be conduction through this diode.
The total resistance of the circuit.
 100  150  50  300 
6
current in 100    0.02 amp
300
39. (a) 40. (b) 41. (a)

42. (a)
 Time lapsed 
4  
1  1   1  T1/ 2 
 
16  2   2 

CENTERS: MUMBAI /DELHI /AKOLA /LUCKNOW /NASHIK /PUNE/NAGPUR /BOKARO /DUBAI # 279
Time lapsed
 4
T1/ 2
 Time lapsed = 400 s

43. (c)
If R be the initial activity of the sample, then R 1  R 0et1 and R 2  R 0et 2
0
1 1
Where   {  Mean life = T  }
T 
t 2
R2 e
 t1  e  1 2 
 t t

R1 e
t t 
 R 2  R 1 exp  1 2 
 T 
44. (c)
h
L1  1 …(1)
2
{Using Bohr’s Quantisation Rule}
In the first excited state of Li
h
L2   2  …(2)
2
L2
 2
L1

45. (d) 46. (d)

47. (c)
2 2 3
1 H  1H  1H  p

1 H 2  1H 3  2 He 4  n
 3 1H 2  2 He 4  p  n
m  m  2  
He4  m  p   m  n   3m 1 H 2 
 m   4.001  1.007  1.008  3  2.014   u
 m  0.026 u
 E  c 2 m
  
E  9  1016 0.026  1.67  1027 
 E   931.5 0.026  MeV
 E  3.87 1012 J
1040
As each reaction involves 3 deutrons, so total number of reactions involved in the process  . If
3
each reaction produces an energy, E then
1040
E total  E  1.29 1028 J
3
E total  Pt
Time of exhaustion of the star

CENTERS: MUMBAI /DELHI /AKOLA /LUCKNOW /NASHIK /PUNE/NAGPUR /BOKARO /DUBAI # 280
1.29 10 28
t
1016
 t  1.29  1012 s

48. (b)
4
R 3  A
3
1
 R  A3

49. (b)
Total initial binding energy is 236  7.6
 U i  1793.6 MeV
Total final binding energy is 2 117  8.5
 U f  1989 MeV
 U  195.4 MeV

50. (b)
According to Ritz Combination Principle
E CA  E CB  E BA
hc hc hc
  
 3 1  2

 3  1 2
1   2
51. (c) 52. (d) 53. (b) 54. (b)

55. (d) 56. (b)

57. (a)
The longest wavelength in a series is obtained when a transition takes place between the lowest
consecutive levels.
Here transition must take place from n = 2 to n = 1.

58. (d)

59. (d)
4
N 1 N 1
   
N0  2  N 0 16

60. (b)
Second excited state implies n = 3
 h   h 
 LH  n    3 
 2   2 
 h   h  Z2
 L Li  n    3   EH  13.6  eV
 2   2  n2

CENTERS: MUMBAI /DELHI /AKOLA /LUCKNOW /NASHIK /PUNE/NAGPUR /BOKARO /DUBAI # 281
12 32
 EH  13.6  eV  E Li  13.6  eV
9 32
 E Li  13.6eV  E Li  9 EH
 ELi  EH

61. (c)
2 2 4
1 H  1H  2 He  Q

 m  m  2  
He4  2m 1 H 2   m  4.0024  2  2.0141  m  0.258 u
Since, Q  c2 m
 Q   0.0258 931.5  MeV  Q  24 MeV

62. (c)
dN
 n  N
dt
Because the population N is simultaneously increasing at rate n because and decreasing due to decay
at rate  N.
N t
dN 1  n - N 0  n  n
 N n  N 0 dt
  ln   t  N    N 0   e t
0
  n  N    
TRICK
At t = 0, N = N0 which is satisfied by (c) only.

63. (d)
E K  hv  hv 0
Is the equation of a straight line with slope h.

64. (b)
1 2
mv  eV
2
2eV
 v
m
 v  8.4  106 ms 1

65. (d)
1 hc
mv 2   W (1)
2 
1 hc
mv '2  W (2)
2 3 / 4
4hc
2 W
v '
Dividing,    3
v hc
W

4
v'  v
3
66. (c)
CENTERS: MUMBAI /DELHI /AKOLA /LUCKNOW /NASHIK /PUNE/NAGPUR /BOKARO /DUBAI # 282
67. (a)
h

mv
6.626 10 34
 
1031 105 
   6.63  108 m

68. (b)
Since xp ~ h
  2 10  p  6.6 10
14 34
 p  3.3  1020 kgms 1

69. (b)
h

2meV
h
'
2MeV
m
 '  
M

70. (d)
By Flemings Left Hand Rule.

71. (b)
1 h 1
R and   
 2mqV V
 R V
R2 80 kV
  2
R1 20 kV
 R 2  2R 1  2R

72. (c)
Stopping potential of 1.36 V implies E K  1.36 eV .
Since hv  0  E K


 4.14 10  3 10   
15 8

 1.36  2.46  0  1.36


0
5000 1010
 0  1.1 eV
73. (a)
Stopping potential is independent of intensity and depends upon frequency.

74. (c)
Saturation current  (Intensity)
 Saturation current = 4  0.4 A 
 Saturation current = 1.6 A

CENTERS: MUMBAI /DELHI /AKOLA /LUCKNOW /NASHIK /PUNE/NAGPUR /BOKARO /DUBAI # 283
75. (b)
For photon
E  pc  hv
hv
 p
c
76. (c) 77. (c)

78. (a)
h

2meV
Putting values of h, m, e, we get
150 o 12.27 o
  A   A
V V

79. (d)
1
I
d2
On doubling the distance the intensity becomes one fourth i.e. only one fourth of photons now strike
the target in comparison to the previous number. Since photoelectric effect is a one photon-one
electron phenomena, so only one-fourth photoelectrons are emitted out of the target hence reducing
the current to one fourth the previous value.

80. (d)
According to Bohr’s Theory E  0 . Since Et  h
 t  

81. (a)
e
Se 
me
e
Sp 
mp
2e e 1
S    Sp
4m p 2m p 2
Se is maximum, then comes S p and then S

82. (b)
o
 max  7500 A

83. (c)
Photoelectric current  Intensity
 iI  i  kI

84. (c)
h
 where,
mv

CENTERS: MUMBAI /DELHI /AKOLA /LUCKNOW /NASHIK /PUNE/NAGPUR /BOKARO /DUBAI # 284
m0
m
v2
1
c2
As vc
m
 0

85. (a)

86. (c)
h 
 So, when V becomes 4 V,  becomes
2meV 2
87. (b)

88. (c)
E K  hv  0
  E K 1  1  0.5  0.5 eV
Similarly  E K  2  1  0.5  0.5 eV
 E K 1 1 v12 1 v1 1
     
 E K 2 4 v 22 4 v2 2
89. (a)
E K  hv  hv0

90. (d) 91. (a) 92. (b) 93. (d)

94. (c) 95. (b)

96. (c)
VF  VB 2  0.7 1.3
i  
R 20  180 200
3
i  6.5 10 A  6.5 mA

97. (b)
Output of OR gate = A + B
Output of NAND gate = A.B
 Output Y from AND gate =  A  B .AB

 
  A  B  . A  B  AA  BB  AB  BA
 0  0  AB  BA
This is Boolean expression for XOR gate.

98. (a)
I C  I E  I B  4 mA  40 A   4  0.04   3.96 mA
I 3.96
 C   0.99
IE 4

CENTERS: MUMBAI /DELHI /AKOLA /LUCKNOW /NASHIK /PUNE/NAGPUR /BOKARO /DUBAI # 285
99. (a)
Junction diode is reverse biased, so current is zero.

100. (d) 101. (b)

MULTIPLE CHOICE QUESTIONS


1. (a, c)
mv p 2km
r  
qB qB qB

m  2k 
rH  A  A  
q  B 

1
= AA
1
rHe  2A

4
r0  A  2A
2
H+ will be deflected most because rH  is shorter than rHe & r0 

 rHe  r0 1 He+ and O++ will be deflected equally.


2. (b, c)
1
Em 
n2
nh
Jm 
2
1
So, E m 
J 2n
3. (ABC)

4. (B, D)
2m1m 2
T geff
m1  m 2
(a) uniform velocity geff = g
2  3  5 10 300
T 
8 8
300
2T   75
4
2  3  5 14
(b) geff = 14  T
8
2 T = 105

CENTERS: MUMBAI /DELHI /AKOLA /LUCKNOW /NASHIK /PUNE/NAGPUR /BOKARO /DUBAI # 286
2  3  5 14
(c) geff = 14  T
8
2 T = 105
(d) geff =6
2  3 5  6
T
8
360
2T   45
8

5. (A, B, C)
u = – 7x + 24y
u u ˆ
F = – ˆi  j
x y
F = 7iˆ  24ˆj
7 24
 ax = m/s2 ay =  m/s2
5 5
7
vx = 14.4 +  4 = 20 m/s
5
24
vy = 4.2   4 = 15 m/s
5
v = v 2x  v 2y = 25 m/s

a= a 2x  a 2y = 5 m/s2
 
7 24 

a  v   ˆi  ˆj   14.4iˆ  4.2jˆ
5 5 

   
a  v  0  a  as to v

6. (B, D)
 p p 
p = p1 –  1 2  v p
 v1  v 2  p1v1
Pv = nRT
  p1  p 2   v p1v
  p1    v T
  v1  v 2   nR p2v2
p1  p1  p2  2
 T= v v v
nR  nR  v1  v2  
dT p1 2  p1  p 2 
  v
dv nR nR  v1  v 2 
dT
as v  ↓
dv
 Temperature first increases and then decreases.

7. (A, B)

CENTERS: MUMBAI /DELHI /AKOLA /LUCKNOW /NASHIK /PUNE/NAGPUR /BOKARO /DUBAI # 287
8. (A, B, C, D )
T cos 60 o = mg
mv o
T sin 60o =
 sin 60 o 
60o
v2
tan 60 o = T
g sin 60 o
60o
2 3
v = 9.8  1.6   3
2
r
 v = 2.8 3 m/s
2r 2 1.6 3 4
t=    mg
v 2.8  3 2 7
T = 2 mg
2

a=
v2

 2.8   3
 9.8 3
r 1.6  3 / 2

9. (a,b,c,d) 10. (c,d) 11. (b,c) 12. (a)

13. (a,d)
In  decay, the entire energy is carried away by the   particle as its kinetic energy. In  decay,
the energy is shared between the   particle and the antineutrino. Hence, the speed of the  
particle will vary, depending on the energy of the antineutrino.

14. (a,c)
R  n 2 , V  n 1 , E  n 2

15. (a,d)
In the nth orbit, let rn  radius and n  speed of electron
2rn r
Time period Tn   n
n n
1
Now rn  n 2 and n 
n
rn
  n3 or Tn  n 3
n
3
n  n
Here, 8   1  or 1  2 or n 1  2n 2
 n2  n2

16. (a,c,d)

17. (b,c)
Any transition causing a photon to be emitted in the Balmer series must end at n  2 . This must be
followed by the transition from n  2 to n  1 , emitting a photon of energy 10.2 eV, which
corresponds to a wavelength of about 122 nm. This belongs to the Lyman series

18. (a,b,c)

CENTERS: MUMBAI /DELHI /AKOLA /LUCKNOW /NASHIK /PUNE/NAGPUR /BOKARO /DUBAI # 288
19. (b, d)
 red   violet
VIBGYOR pattern shows that VIBG all have  less than that of yellow colour and hence can
initiate photoelectric effect irrespective of intensity.

INTEGER TYPE

1. 2
1 1
2
Given 52.224 = 13.6  2  2  Z
1 n 
 1 1
1.224 = 13.6  2
 2 Z 2
 (n  1) n 

2. (5)
mu = (m + M) v m v
mu
 v= M
mM
mM
Wf  K  u2
2m  M
also mv – mu = – mg t
Mu
 t=
g(m  M)
mM  u 2 g(m  M)
Pavg = 
2(m  M) Mu
2
u
Pavg = mg = 5W
2

3. (7)
mu = (m + M) v1
Mu
v1 = v
mM
v1
1 1 2 1
also mu 2   m  M  v1  mv2 + mg R
2 2 2
M
mv 2 mu 2  m 
  1    mgR
2 2  mM
mv 2 mu 2  m 
   2mg
R R  m  M 
FBD of the blocks

v N1

N
N
m a
ma M

mg
mg

CENTERS: MUMBAI /DELHI /AKOLA /LUCKNOW /NASHIK /PUNE/NAGPUR /BOKARO /DUBAI # 289
N = Ma
mv 2
 N  ma
R
mv 2 m
 N N
R M
 m  mu 2 M
 N 1   =  2mg
 M R mM
Solving N = 7 newton

4. (1)
L mL2 3g L mL2 mL2 3g
mg  , a  , N  ,
2 3 2L 2 12 12 2L
mg
N  10
4

5. (5)
mr 2  mr 2 2 55 2
Isys    m  2r    6  mr  55
2  2  2

SUBJECTIVE
1. Let m kg of hydrogen be consumed in 1 sec
6.03  10 26  m
The number of H- atoms used 
1
 6.03 1026 m
When four atoms are consumed, energy released is given by E  mc 2
  4 1.0078  4.0026   931MeV
Two positrons released combine with two electrons producing two photons
 Photons energy released
 0.0005  4amu
 0.002  931Mev
 1.862MeV
 Total energy released by burning of 4  H atoms
 26.63  1.862  28.5MeV
 Total energy released by burning of m kg
28.5
   6.03 1026 m  MeV
4
28.5
  6.03 1026 m 1.6  1019 106 J
4
 6.87 1014 m.J
 Energy received on earth per unit area
6.87 1014 m
 2
 1400  given 
4 1.5 1011 
4 1.5  1011 
or, m  14
 5.76 1011 kg sec
6.87 10
CENTERS: MUMBAI /DELHI /AKOLA /LUCKNOW /NASHIK /PUNE/NAGPUR /BOKARO /DUBAI # 290
2. By strokes law
mg  6 rv1
When field is upwards
qE  mg  6  rv 2
qE v
 1  2
mg v1
qV v
 1  2
mg d v1
mgd  v 2 
q 1  
V  v1 
6.4  1018  9.8 10 103  0.016 
  1  
95  0.078 
 7.9 1019 C

3. rn  r0  n 2
 r3  0.529  1010 cm  32  9  0.529  1010 cm
Velocity in n th orbit is
u 2.19  108
un  0  cm sec
n 3
No. of waves in one round
2r3


h h
But   
p mu 3
2r3 2 r3mu 3
 No. of waves  
h  mu 3  h
2  3.14  9  0.529 1018 cm sec  9.108 1028 gm  2.19 108 cm / sec

3  6.62  1027 ergs see
3

4. 1 mg Bi 210 contains
1mg
N 0  6 1023 
210gm
6  10 23  10 3 2
   1019 atoms
210 7
dN B
A1   1N
dt
 1 N 0 e 1t
2  1.6106 30 2460 60 
 1.6  106  1019  e
7
10
 7.2 10 / sec
Rate of decay of P0 is

CENTERS: MUMBAI /DELHI /AKOLA /LUCKNOW /NASHIK /PUNE/NAGPUR /BOKARO /DUBAI # 291
 dN P0
A2   2 N
dt
N

 2 1 0 e1t  e 2 t
 2  1

5.8  10 8  1.6  10 6  1.6106 30 246060 6 11
 6 
e  e 5.810 30243600   1.46 10 / sec
8
5.8  10  1.6  10 

1 m2 v2R 2 L2
5. Kinetic energy KE  mv 2  
2 2mR  2mR 2
When L  mvR  angular momentum
Total energy of electron in orbit is given by
E  KE  U
L2 e2 1
  k [ where k  ]
2mR 2 R 4 0
L2
 R 2E   ke 2 R
2m
L2 ke 2 R
 R2  
2mE E
2
ke R L2
 R2   0
E 2mE
Solving for R
12
ke2  k 2 e4 2L2 
  I 2 
E  E mE 
R
2
12
ke2  k 2e4 2L2 
 2R     
E  E 2 mE 
ke 2 9  10 9  2.56  10 36
Now    5.76  10 11 m
E 4  10 18
2L2 2  36 1070
  1.98 1021 m2
mE 9.110   4 10 
31 18

 2R  5.76 1011   3.32 10 21  1.98 10 21 m 2 


 5.76 1011  3.66 1011
 1.05 1011 m or 4.711011 m
11
 Nearest distance  1.05 10 m
Farthest distance  4.711011 m

6. O16 has 8 protons and 8 neutrons


8
Mass of 8 protons  8  1.00753  8.060744 amu
Mass of 8 neutrons  8 1.008982  8.071856amu
Total mass of protons + neutrons  8.060744  8.071856
 16.132600amu
Mass of O nucleus = mass of atom  mass of 8 electrons
 16  8  0.00055

CENTERS: MUMBAI /DELHI /AKOLA /LUCKNOW /NASHIK /PUNE/NAGPUR /BOKARO /DUBAI # 292
 15.9956amu
 Mass defect  16.1326  15.9956
 0.1370amu
Energy equivalent  0.1370  931  127.547 MeV
127.547
B.E/ nucleon   7.97 MeV
16

7.
    
Q   4m 1H  m 4 He  c2   4 1.007825   4.002603 931.5  26.2MeV

This energy is released for every four protons consumed.
26.2 MeV
 Energy release/ proton   6.6
4 proton
MeV 6.6  1.6  10  13
Now 6.6   6.3  1014 J kg
proton 1.67  10 27

T 4  4R sun
2 3.9  1026
 Hydrogen consumption rate    6.2  1011 kg s
6.3  1014 6.3  1014

8. m  4 1a.m.u  4 1.66 1024


1
m v2  4.78MeV  4.78 1.6 1019 J
2
 v  1.52  107 m s
Total energy evolved in the reaction    KE   Ke residual nucelus
Using momentum conservation law,
m  V  MV where M and V are mass and velocity of residual nucleus.
 m 2 v2  M 2V 2

1 2  m 1 m v 2   1 m v 2   M  m    4.78MeV  226   4.87 MeV
   m  v
2 M 2
         
2  M   222 

9. 1 2
A B C
No zero zero
Initial

N1 N2
At any time t
dN 2
 1N1   2 N 2
dt
After solving, we get
N 
N 2  0 1 e 1t  e  2 t 
 2  1  
if  2   1 , then
 N
N 2  1 1 1  e  2 t 
 2  

CENTERS: MUMBAI /DELHI /AKOLA /LUCKNOW /NASHIK /PUNE/NAGPUR /BOKARO /DUBAI # 293
0.693 0.693
1  and  2 
T1 T2
Where T1 and T2 be the half life of the parent and daughter nuclei.
At equilibrium N 2 2  N11
N11
At equilibrium value of the daughter nuclei is N '2 
2
Thus, N2  N'2 1  e 2 t 
 
N2
According to the given condition,  0.99
N'2
Or e  2 t  1  0.99  0.01
Or  2 t  ln 0.01  4.6
4.6 4.6
t  T2  6.64 T2
 2 0.693

10. Let activity (rate of decay) of the nuclide be A nuclei per second. It means A particles are emitted
per second. If a spherical surface of radius r with centre at position of nuclide be considered, then
A particles crosses this area per second. It means during elemental time internal dt a no. of  A.dt 
of   particle cross this surface. If velocity of   particle be v then above, calculated (A.dt) 
particles are in space having shape of a spherical shell of radius r . and radial thickness  v.dt 

 
volume of this space  4r 2 vdt
Concentration of   particles at distance r from nuclide is
A.dt

4r 2 vdt
 
Activity of the nuclide A  4r 2 v
But A  N
4r 2 v  log 2 
 N ;  but  
  T 
4r 2 vnT
 N
0.6931
1 2E
Kinetic energy of   particle E  mv2 v
2 m
Using above equation
4r 2vT 2E
N  9.61022
0.6931 m

 
(ii) At distance r from the nuclei A 4 r 2  particles cross unit area vdt

per second, let area of small circular plate be S then. number of   r


particles striking the plate per second.

CENTERS: MUMBAI /DELHI /AKOLA /LUCKNOW /NASHIK /PUNE/NAGPUR /BOKARO /DUBAI # 294
A N 0.6931NS
 .S  S
4 r 2 4r 2 4r 2T

Momentum of each particle just before collision is mV and after collision it comes to rest,
momentum becomes zero.
Momentum transferred to the plate
P  mv  0  mv
Force experienced by plate
F  P  no. of particles string/ second
0.6931 NS
 mv 
4r 2 T
P  force per unit area
F 0.6931NS  2E 
P  mv,  But v  
S 4 r 2T  m 
0.6931NS
P 2Em  1.08  104 N m 2
2
4r T

11. Since at t  0 , probabilities of getting ,  from radio nuclide are equal, therefore initial activity is
equal. let it be A0
Activity of first isotope at t  1620sec
t
1
  A
A1  A 0   T1  0
2 16
t
 1  T2 A 0
And of second be, A 2  A0   
 2 2
Total activity at t  1620sec
9
A  A1  A 2  A0
16
A 1
Probability of getting  particle, P1  1  ans (1)
A 9
A 8
Probability of getting  particle, P2  2 
A 9
Let at t  0 , number of two isotopes be N0 and N0 respectively
1 2
log 2
Initial activity of first isotope A1  N0 1  N 0
1 1 T
1
log 2
Initial activity of second isotope A 2  N 0 1  N 0
2 2 T
2
N0 N0
Since, A1  A 2 1  2

T1 T2
N0 T 1
1  1 
N0 T2 4
2
Initially, total no of nuclei, N0  N0  N0
1 2
CENTERS: MUMBAI /DELHI /AKOLA /LUCKNOW /NASHIK /PUNE/NAGPUR /BOKARO /DUBAI # 295
1 4
 N0  N0 and N0  N0
1 5 2 5
At time t , no of nuclei of first isotope that remains undecayed
t t
 1  T1 1  1  405
N1  N 0    N 0  
1 2 5  2
t t
 1  T2 4  1 1620
N2  N0    N0  
2 2 5 2
Total no of nuclei remain undecayed at time t
t t
1  1  405 4  1 1620 N 0 4
N  N1  N 2  N 0  
5  2
 N0  
5 2
 
5
x  4x 
t
 1 1620
where x   
 2
But it equal to N0 2 , hence
N0 4 N

5  
x  4x  0
2
4
 x  4x  2.5  0
 x  0.594 (given)
t
1
or  1620  0.594
2
1620 log  0.594 
t   1215sec
log 2
A
12. (a) Energy flux though the angular space between two cones of heat-angles 
and   d
d
 P 
   2 sin  d 
 4    O
Psin  10 R
Momentum of flux 
2C
P sin  d 
Rate of change of momentum along the normal  2   cos 
 2C  B
Psin  cos  d

C
P 
Force on the mirror   sin  cos  d
C 0
1
When   half angle subtends by mirror  tan 1  
n
Psin 2  P
 F  sin 2 
2C 2C
1
tan  
n

CENTERS: MUMBAI /DELHI /AKOLA /LUCKNOW /NASHIK /PUNE/NAGPUR /BOKARO /DUBAI # 296
P 1 P
F  N
 
2C n 2  1 202C

hv E
(b) P momentum of a photon  
C C
momentum of reflected photon  e  E C 
Change of momentum
 E C    CE C 
 1  e  E C
Force
Pressure 
Area
4 1  e  E
Pressure 
CTd 2
4  1  0.5  10

 3  108 104 1010
 6.27 106 N m 2
 6.37  106 N m 2

13. Let   energy of laser flash O



momentum 
C 

Momentum transfer to the mirror 
C
 B
Let velocity acquired by the mirror, then mv 
C
By conservation of energy to A  B
1 2 A
mv  mgc 1  cos  
2
v2  2gc 1  cos 
 
2  
 2g  2 sin 2  2   2sin  2  g   2g   mc  g
2
m c 2 mc mc
 10 106  3 108   0.6  180  9.8  0.02
 13.9 J
COMPREHENSION
1. (d)
Given E 3  E 2  47.2 eV
Z2
Since E n  (for hydrogen like atoms)
n2
 Z2    Z2  
Or  13.6     13.6      47.2
 9    4 
Solving this equation, we get Z = 5

CENTERS: MUMBAI /DELHI /AKOLA /LUCKNOW /NASHIK /PUNE/NAGPUR /BOKARO /DUBAI # 297
2. (c)
Energy required to excite the electron from 3rd to 4 th orbit:
E 3 4  E 4  E 3
25    25  
 13.6     13.6      16.53 eV
 16    9 

3. (a)
Energy required to excite the electron from first excited state (n = 2) to second excited state (n = 3),
is from n = 2 to n = 3 which is 47.2 eV

4. (b)
Energy required to remove the electron from first orbit to infinity (or the ionization energy) will be
2
E  13.6  5   340 eV
The corresponding wavelength would be,
hc 6.6 10 34  3  108
 
E 340 1.6  10 19
o
  0.0364 10 7 m  36.4 A
5. (b)
In first orbit,
Total energy = -340 eV
Kinetic energy = +340 eV
Potential energy = - 2  340eV = -680 eV
And angular momentum
h 6.6 10 34
L   1.05 10 34 kgm 2s 1
2 2
6. (b)
n2 o
rn  0.53A
Z
Radius of first Bohr orbit (n = 1, Z = 5) is
0.53 1010 o
r1   0.106 A
5
7. (c)
From Einstein’s equation of photoelectric effect,
Energy of photons causing the photoelectric emission = maximum kinetic energy of emitted photons
+ work function.
Or E  K max  W   0.73  1.82  eV
Or E  2.55 eV

8. (b)
In case of hydrogen atom,
E1  13.6 eV, E 2  3.4 eV, E 3  1.5 eV, E 4  0.85 eV
Since, E 4  E 2  2.55 eV
Therefore, quantum numbers of the two levels involved in the emission of these photons are 4 and
2 4  2 .

CENTERS: MUMBAI /DELHI /AKOLA /LUCKNOW /NASHIK /PUNE/NAGPUR /BOKARO /DUBAI # 298
9. (d)
Change in angular momentum in transition from 4 to 2 will be
 h   h  h
L  L 2  L 4  2    4  L  
 2   2  
10. (b)
From conservation of linear momentum
Momentum of hydrogen atoms = Momentum of emitted photon
E
Or mv  (m = mass of hydrogen atom)
c

Or v
E

 2.55  1.6  10 19 J 
mc 1.67  10 27 kg  3.0  108 ms 1 
v  0.814 ms 1s
11. (a)
1 1
The resistivity of pure silicon i  
i n i e   e   h 
When  e and  h are mobilities of electrons and holes respectively.
1
 ni 
i e   e   h 
1
ni  19
n i  1.347  1016 charge carriers/ m3
3000  1.6 10   0.12  0.025 
12. (c)
When 1019 atoms of phosphorous (donor atoms) are added per m 3 , we have
n e  n i or n e  n h
And so n e  1019
1 1
     5.21 ohm  metre
n e e e 10 1.6 1019  0.12
19

13. (d)
When 2 1019 boron (acceptor) atoms are also added per m3 , we get n h  n e  n A  n D
 2 1019  1019
n h  1019 since n e  n h
1 1
    19   25 ohm-metre
n e e h 10 1.6 10 19  0.025
14. (b)
As Ge diode conducts at 0.3 V; both Ge and Si diodes are in parallel, so the p.d. across both diodes
will remain 0.3 V
A 0.3 V, Si diode offers infinite resistance and hence it will not conduct.
 Potential difference across R  5 k is
12  0.3  117 V
 V0  11.7 V
11.7
15. (c) Current I  A  2.26 mA
5 103

CENTERS: MUMBAI /DELHI /AKOLA /LUCKNOW /NASHIK /PUNE/NAGPUR /BOKARO /DUBAI # 299
16. (b, d)
When the connections of Ge diode are reversed, it will be reverse biased and for conduction, the p.d
across the silicon diode will be 0.7 V.
 p.d. across R  5 will be
12 - 0.7 = 11.3 V
 V0  11.3 V
11.3
And Current I  A  2.26 mA
5 103

17. (b) 18. (c)


19. (a)
Let v0 be the peak value of the input voltage and v rms its rms value.
v
Then, Vrms  0
2
 v 0  2 v rms  20 2 V
Since the input is applied of the diodes via a step-up transformation ratio 1 : 2 therefore the peak
value V0 of the voltage applied of the diodes is
V0  20 2  2 V  40 2 V
2 2
Now, vdc  V0   40 2 V
 
2
And v ac  0.48   40 2 V

For a full wave rectifier, the ripple factor is 0.48.

CENTERS: MUMBAI /DELHI /AKOLA /LUCKNOW /NASHIK /PUNE/NAGPUR /BOKARO /DUBAI # 300

You might also like